Download as pdf or txt
Download as pdf or txt
You are on page 1of 481

PRACTICAL VALUATION

(i)
INTERNAL
CIRCULATION ONLY

MULTIPLE CHOICE QUESTIONS FOR

REGISTERED VALUER EXAMINATION


(on certain topics related to Real Estate Valuation mentioned in the
syllabus for Registered Valuer Examination - Land & Building)

PRACTICAL VALUATION - VOLUME 18


(All subsequent revisions and additions as on 25.11.2018 are incorporated)

B. KANAGA SABAPATHY
BE, FIV, FIE, FICA, C. Engg.,...

No.1, Prestige Flats Off : 0431 - 2462232


No.6, Reynolds Road, Mob & Whatsapp : 97918 - 74829
Cantonment Email : bkvaluer@gmail.com
Tiruchirappalli - 620 001 (TN) Web : www.bkanagasabapathy.com

Price : Rs. 1,300/-


Soft copy : Rs. 650/-
(ii)

March 2018
July 2018
Revised Edition August 2018
Second Revised Edition November 2018

C
Anybody can copy anything
from this book and use it anywhere,
provided their intention is :
“LET KNOWLEDGE SPREAD”

Price : Hard copy : Rs. 1,300/- + 100 (postage) - NO STOCK


Soft copy : Rs. 650/-

Publisher & Seller :


Mrs. EZHILARASI KANAGA SABAPATHY
1, Prestige Flats, 6, Reynolds Road
Tiruchirappalli - 620 001 (TN)
Phone : 0431 - 2462232 ;
Mobile & Whatsapp : 97918 - 74829
E.mail : bkvaluer@gmail.com
Web : www.bkanagasabapathy.com

Typeset : M. PREETHI

Printers : RAJKUMAR PRINTERS


Puthur, Tiruchirappalli - 620 017
Phone : 0431 - 2771845

Note :

1) All the 20 books on valuation written by this


author are available only in the above address.

2) Those who are interested in purchasing the


soft copy of the book can credit the amount
Rs. 650/- in the following account and SMS
or whatsapp complete details including their
Email id to 97918 - 74829 and also email to :
bkvaluer@gmail.com

K. EZHILARASI
City Union Bank, Cantonment
Tiruchirappalli - 620 001. (TN)
Savings account no. 500 101010 597268
IFS Code: CIUB 0000 153

3) The website www.bkanagasabapathy.com


may be referred for the contents of the book
and for more details.
(iii)

DEDICATED TO

Mr. R. Jayaraman
who motivated me to write this book
(iv)

DISCLAIMER

While every effort is taken to avoid errors or omissions in this publication, any
mistake or omission that might have crept in is not intentional. It may be taken note
of that neither the publisher nor the author will be responsible for any damage or
loss of any kind arising to any one in any manner of account of such errors and
omissions.
(v)

B. Kanaga sabapathy 1, Prestige flats


Author of 20 books on valuation 6, Reynolds road,
Tiruchirappalli - 620 001

14.03.2018
PREFACE - I

Preliminary coaching for Registered valuer examination as prescribed by IBBI

1.0. From the year 2014, I have been conducting training programmes on valuation at
Tiruchirappalli in ten levels mainly for the sake of beginners and youngsters in the
profession.
Level I - Fundamental principles of valuation
Level II - Valuation for banks - Part I (Preliminary learning)
Level III - Valuation for taxation - Part I (Preliminary learning)
Level IV - Fixation of fair rent in Tamilnadu
Level V - Valuation for banks - Part II (Advanced learning)
Level VI - Valuation for taxation - Part II (Advanced learning)
Level VII - Court Judgements
Level VIII - Leasehold properties
Level IX - Valuation of apartments
Level X - Any advanced topic in valuation

Number of valuers who have been benefitted through these programmes have
crossed 2000.

2.0. For the purpose of conducting Registered valuer examination, Insolvency and
Bankruptcy Board of India (IBBI), the authority authorised by Ministry of Corporate
Affairs (MCA) has prescribed syllabus which consists of 33 marks for non -
technical topics and 67 marks for subjects related to Real estate valuation. A
candidate is expected to attempt 100 questions in two hours. Every question will be
of multiple choice, having four alteratives. Only one alternative will be correct and
unique. The candidate should tick the correct answer out of four. Examination will
be by on-line.

3.0. Pursuant to the request from many participants who have already undergone
(vi)

training under me, I have come forward to conduct an exclusive programme under
Level X as “Preliminary Coaching for Registered valuer examination.

4.0. Initially, I have taken up giving coaching for subjects related to real estate valuation,
i.e., for questions 34 to 94. With the little experience I have in the field of valuation,
I have prepared a set of multiple choice questions related to certain topics
indicated in the syllabus prescribed by IBBI. I intend to add some more questions in
future for the other topics left out. A set of mock test questions are also provided
alongwith the respective answers. The valuers must understand that these
questions are only indicative and they are given here mainly for the purpose for
giving preliminary coaching to the youngsters. Whether one is going to appear for
the examination or not, he must understand clearly that an opportunity has been
given to him to learn something more.

5.0. This booklet is a part of the training programme materials and they are meant purely
for internal circulation only.

All the questions and answers have been prepared with due care and diligence.
In the process of preparing the above questions, I have referred the books of Annamalai
University, Mr. R.K. Gandhi, Dr. Ashok Nain and others. I have taken a few case
studies from those books. I am thankful to them.

It must be borne in mind that these questions are only indicative. Suggestions for
further improvements are certainly welcome. Mistakes / errors can be pointed out.

6.0. I thank Mr. R. Jayaraman and N. Ravindran their assistance in preparing the
questions.

7.0. AIM of the Level - X program : LET US LEARN SOMETHING MORE

bkvaluer@gmail.com B. Kanaga sabapathy


www.bkanagasabapathy.com 14.03.2018
(vii)

B. Kanaga sabapathy 25.11.2018

PREFACE - II

Preliminary coaching for Registered valuer examination

1.0. Based upon the request from the aspirants of IBBI examination who have completed
their 50 hours mandatory education programme, I have been conducting regular
refresher courses for the sake of those who are yet to pass the examination.

2.0. For the purpose of giving coaching for the participants, I have prepared a few lessons
on twelve mark case studies, one mark theory questions, frequently asked questions
in the examination and mock tests. Certain revisions have also been made.

3.0. I have consolidated these revisions and additions and incorporated in the main book
of Practical valuation - Vol. 18 so that a reader can have access through the entire
materials. The ultimate aim is everyone must pass the examination.

4.0. I convey my thanks to Mr. R. Jayaraman, Tiruchirappalli who has furnished more
questions alongwith the appropriate answers. His interest and willingness to share
the knowledge with others are highly appreciated.

5.0. Suggestions for perfection are welcome.

Tiruchirappalli - 620 001 B. Kanaga sabapathy


25.11.2018
(viii)

Attention Readers

1. Please note these study materials are mainly for the purpose of giving an overall
guidance for the purpose of examination. Depending upon the prevalent conditions,
one must take a judicious approach and answer the questions accordingly.

2. a) Go through the study materials not only for the purpose of preparing for the
examination but also to enrich your knowledge.

b) Share these materials to others so that they can also pass the examination.

c) Ignore if there are any repetitions.

d) Please give feedback if there are any mistakes / errors.

3. I advise you to refer the books of the following authors to enrich your knowledge in the
field of valuation.

i) Mr. R.K. Gandhi


ii) Dr. Ashok Nain
iii) Mr. Kirit Budhbhatti
iv) Mr. Shyamales Datta

* * *
(ix)

PRACTICAL VALUATION - Volume 18


(All subsequent revisions and additions as on 25.11.2018 are incorporated)

REGISTERED VALUER EXAMINATION


(Technical portions - Q34 to 94)

CONTENTS

Part I - Syllabus as prescribed by IBBI - 001 - 012

Part II - Indicative multiple choice questions related - 013 - 118


to Q34 to 88 - 616 nos.

Part III - Twelve marks case studies - Q89 to 94 - 119 - 240


- 96 nos.

Part IV - One mark case studies - 52 nos. - 241 - 266

Part V - A few more one mark theory questions - 267 - 300


- 156 nos.

Part VI - Frequently asked questions in the - 301 - 392


examination - 445 nos.

Part VII - Model question paper prescribed by IBBI - 393 - 410

Part VIII - Mock tests - 3 nos. - 411 - 470

* * *

Soft copy : Rs. 650/-


(x)

B. KANAGA SABAPATHY

BOOKS AUTHORED (as on November 2018)

01. Practical Valuation - Volume 1 (1994)

02. Practical Valuation - Volume 2 (1996)

03. Practical Valuation - Volume 3 (1998)

04. Practical Valuation - Volume 4 (2000)

05. Practical Valuation - Volume 5 (2002)

06. Practical Valuation - Volume 6 (2004)

07. Practical Valuation - Volume 7 (2006)

08. Practical Valuation - Volume 8 (2008)

09. Practical Valuation - Volume 9 (2015) - Valuation of Apartments

10. Practical Valuation - Volume 10 (2016) - Valuation of Leasehold properties

11. Practical Valuation - Volume 11 (2016) - Court judgement related to valuation

12. Practical Valuation - Volume 12 (2002) - Valuation for banks – Part I

13. Practical Valuation - Volume 13 (1999) - Fixation of fair rent in Tamilnadu

14. Practical Valuation - Volume 14 (2017) - Valuation for banks – Part II

15. Practical Valuation - Volume 15 (2017) - Valuation for Taxation

16. Practical Valuation - Volume 16 (2018) - 1000 Questions & Opinions

17. Practical Valuation - Volume 17 (2002) - Valuation formats for banks

18. Practical Valuation – Volume 18 (2018) - Indicative multiple choice questions for
Registered valuer examination

19. Practical Valuation – Volume 19 (2018) - Model valuation reports

20. Practical Valuation – Volume 20 (2018) - 750 Questions & Opinoins


1

PART - I

SYLLABUS AS PRESCRIBED BY IBBI


2

This page is kept vacant intentionally.


3

Part - I
REGISTERED VALUER EXAMINATION (L & B)

SYLLABUS AS PRESCRIBED BY
INSOLVENCY AND BANKRUPTCY BOARD OF INDIA (IBBI)

SI. no. Topic Marks

I. NON - TECHNICAL SUBJECTS (25 marks)

a. Principles of Economics 10
Micro economics 5
Macro economic 5
b. Book keeping and accountancy 5
c. Laws - general 6
d. Introduction to statistics 4

II. VALUATION RELATED SUBJECTS (75 marks)

e. Environmental issue in valuation 4


f. Professional ethics and standards 4
g. Law - Real estate 10
Land acquisition 4
The transfer of property act 4
laws relating to inheritance / succession 2
h. Valuation of Real estate 38
Cost, price and value, etc. 13
Income approach to value 7
Market approach to value 5
Cost approach to value 5
Various purposes of valuation 5
Important case laws on principles 3
of valuation of Real estate
i. Principles of insurance and loss assessment 5
j. Report writing 2
k. Case study 12

TOTAL MARKS - 25 + 75 100


4

LAND & BUILDING

a Principles of Economics (10 marks)

Micro-Economics (5 marks)

1 Consumption: Indifference Curve, Consumer’s Surplus, Elasticity.

2 Price Mechanism: Determinants of Price Mechanism, Individual and Market Demand


Schedules, Law of Demand & its Conditions, Exceptions and Limitations of Law of
Demand, Individual and Market Supply Schedules, Conditions and Limitations, Highest,
Lowest and Equilibrium Price, Importance of Time Element.

3 Pricing of Products under different market conditions: Perfect and Imperfect Competition,
Monopoly.

4 Factors of Production and their pricing – Land, Labour, Capital, Entrepreneur and other
factors

5 Theory of Rent, Theory of Wages

Capital and Interest - Types of Capital, Gross Interest, Net Interest

Organisation and Profit - Functions of Entrepreneur, Meaning of Profit and Theories of


Profit.

Macro-Economics (5 marks)

6 Functions & Role of Mone

7 Inflation: Types of Inflation, Causes, Effects, Inflationary Gap, Control of Inflation,


Monetary, Fiscal and Direct Measures

Deflation: Causes, Effects, Deflationary Gap, Measures to Control Deflation, Deficit


Financing.
5

8 Savings and Investment: Savings and Types of Savings, Determinants of Savings,


Investment, Types of Investment, Determinants of Investment, Relationship between
Savings and Investment.

9 Components of Economy: Primary Sector, Secondary Sector, Tertiary Sector, Informal


Sector in Urban Economy, Parasitic Components in Urban Economy.

10 Concepts of GDP and GNP, Capital Formation


Parallel Economy: Definition of Parallel Economy, Causes and Effects of Parallel
Economy on Use of Land and its Valuation - Its Impact on Real Estate Market -
Construction Industry and Parallel Economy.

b. Book Keeping and Accountancy (5 marks)

11 The meaning and objects of Book Keeping, Double Entry Book Keeping.

12 Books of Prime Entry and Subsidiary Books: Cash Book, Bank Book, Journal, Ledger,
Purchase and Sale Books, Debit and Credit Notes Register, Writing of Books, Posting
and Closing of Accounts.

13 Trading Account, Profit and Loss Account, Income and Expenditure Account,

14 Preparation of Balance Sheet for Individuals and Companies and Disclosure


Requirements.

15 Cost, Costing and Elements of Cost, Fixed Expenses, Variable Expenses, Break-Even
Point.

c. Laws-General (6 marks)

16. Indian Legal System: Salient Features of the Indian Constitution, Fundamental Rights,
Directive Principles of the State Policy.

Government: Executive, Legislature and Judiciary

17,18 Laws of Contract: Formation of a Contract, Parties, Void, Voidable and Unenforceable
Contract, Contingent Contract, Misrepresentation and Fraud and Effect thereof,
Termination of Contract, Remedies for Breach, Performance of Contract, Indemnity
and Guarantee, Law of Agency.

19 Tort: General Principles of Tort, Tort affecting Valuation.

Law of Arbitration and Conciliation: Salient Features

Auction: Authority of Auctioneer, Duties of Vendor, Purchaser and Public, Mis-description


6

and Misrepresentation, Advertisements, Particulars and Catalogues, Statements on


the Rostrum, Conduct of Sale, Reservation of Price and Right to Bid, Bidding
Agreements. Memorandum of the Sale. The Deposit, Rights of Auctioneer against
Vendor and Purchaser

Laws of Evidence: Burden of Proof, Presumptions, Conclusive Proof.

20 Salient Features of the Insolvency and Bankruptcy Code, 2016 concerning Valuation.

21 Salient Features of the Companies (Registered Valuers And Valuation) Rules, 2017

Salient Features of the Securitization and Reconstruction of Financial Assets and


Enforcement of Security Interest Act, 2002 (SARFAESI Act) concerning Valuation

Section 5(n) of the Banking Regulation Act, 1949 on “Secured Loan or Advance”

The Companies Act 2013: Sections 192(2), 230 (1,2,3), 231, 232, 247(1), 281(1)

d. Introduction to Statistics (4 marks)

22 Data Classifications and Processing, Graphical Representation of Data, Frequency


Distributions.

23 Measures of Central Tendency, Dispersion and Skewness.

24 Elementary Theory of Probability and Probability Distributions, Sampling and Sampling


Distributions, Estimation

Simple Test of Significance, Regression and Co-relation, Multiple Correlation Coefficient,

25 Time Series

Index Numbers

e. Environmental Issue in Valuation (4 marks)

26 Environment and Valuation - Differences between the ‘Market Price and the Negative
Value Consequent on Environmental Impact.

27 Environmental Issues of Air Pollution, Water Pollution, Environmental Factors and their
effects, Measures to Restore the Damage, Cost to Cure.

28 Outlines of Environmental Legislations: The Forest Act, 1927, Laws related to Industrial
Health & Safety.
7

29 The Water (Prevention and Control of Pollution) Act, 1974, The Air (Prevention and
Control of Pollution) Act,1981, The Environment (Protection) Act,1986

f. Professional Ethics and Standards (4 marks)

30-32 Model Code of Conduct as notified by MCA under the Companies (Registered valuers
and valuation) Rules 2017

33 Other Engagement Considerations

g. Law-Real Estate (10 marks)

34 Land Acquisition (4 marks) - The Right to Fair Compensation and Transparency in the
Land Acquisition, Rehabilitation and Resettlement Act, 2013.

Provisions for Acquisition of Land under the Municipal Laws

35 Building Rules and Regulations of Local Bodies as well as Development Control Rules
& Regulations of different urban development authorities for feasibility of Development
/ Redevelopment on the Land – Rules for Open Space, FSI and Plinth Area Restrictions.

Transferable Development Rights.

36 Rent Control Laws: Sections pertaining to Occupancy Rights of Tenants, Freezing of


Rent and Protection against Eviction of Tenant and its effect on value of property.

37 Right of Way, Section 52 - Licenses under the Indian Easements Act, 1882.

Salient features of the Real Estate (Regulation and Development) Act, 2016 and Real
Estate Regulating Authorities established under the Act.

The Transfer of Property Act, 1882 (4 marks)

38 Transfer of Immovable Property: Sale, Mortgage, Gift, Exchange,

39 Assignment, Charge, Lien, Tenancies / Sub-Tenancies.

40 Lease of Immovable Property, Lease granted by Private and Statutory Bodies - Impact
of each on Valuation.

41 Sections: 3, 5, 6, 7, 25, 53 and 53A

Laws Relating to Inheritance/Succession (2 marks)

42 Mohmedan - Personal Law


8

The Hindu Succession Act, 1956, the Hindu Succession (Amendment) Act, 2005 (39 of
2005)

43 The Indian Succession Act, 1925: Law of succession for person other than Hindu and
Mohmedan

Will & Testament; Succession Certificate

h. Valuation of Real Estate (Total marks - 57)


General (13 marks)
44 Cost, Price and Value

45 Types of Value

46 Basic elements of Value - Marketability, Utility, Scarcity, and Transferability

47,48 Factors affecting Valuation-Physical, Economic, Legal and Social

49 Highest and Best Use, Value in Use, Value in Exchange

50 Real Property: Rights and Interests in Real Estate, Types of ownerships and Types of
occupancy in Real Estate

51 Annuities, Capitalization, Rate of Capitalization, Years Purchase, Sinking Fund,


Redemption of Capital, Reversionary Value

52 Construction and use of Valuation Tables


• Simple Interest amount working
• Compound Interest amount working
• Present Value of Rupee working
• Amount of Rs. 1 / year working

53 Construction and use of Valuation Tables


• Annual Sinking fund working
• Present Value of Future Income of Rs. 1 / year. (Single rate basis)
• Present Value of Future Income of Rs. 1 / year ( Duel rate basis)

54 Urban Infrastructure and its influence on Value of Real Estate

55 Real Estate Market and its characteristics, Investment in Real Estate

56 Factors influencing Demand and Supply Schedule in Real Estate

Income Approach to Value (7 marks)


9

Remunerative Rate of Interest and Accumulative Rate of Interest

Types of rent: Outgoings, Income, Yield, Years’ Purchase

Determination of Market Rent and Standard Rent

57 Relation between Income and Value

58 Valuation of Property affected by the Rent Control Act, Licensed property under the
Easement Act, 1882 and Leasehold properties under the Transfer of Property Act,
1882

59 Derivation of Yield Rate from Market Derived Data.

60 Lease: lessor and lessee: Types of Lease, Lease provisions and Covenants.

61 Valuation of Lessor’s Interest, Lessee’s Interest including Sub-Lease in Leased Property.


Premature Termination of Lease or Surrender of Lease.

62 Real Estate as an Investment, Yield from Real Estate vis-à-vis other forms of
Investments- Sound Investment Comparison.

Investment Decisions: Discounted Cash Flow Techniques-Internal Rate of Return (IRR)


and Net Present Value (NPV)

63 Profit Method: Valuation of Special Properties: Hotels, Cinema, Mall, Petrol Pump, Hill
resorts

Market Approach to Value (5 marks)

64 Types of Market, Demand and Supply Curve, Bell Curve for Overall Sales Performance
(Probability Distribution),

Market Survey & Data Collection, Sources of Sale Transactions,

65 Comparison of Sale Instances – Factors of comparison and weightages for adjustment


in value

66 Hedonic Model and Adjustment Grid Model under Sales comparison Method.

67 Land characteristics and its effect on Land Values

68 Hypothetical Plotting Scheme for value of large size land

Residue Technique and other development methods


10

Valuation for Joint Venture Development of property

Cost Approach to Value (5 marks)

69 Methods of Cost Estimates for Buildings

70 Life of Building: Economic/Physical/Legal

Factors affecting life of the building.

Total Life, Age, Estimating Future Life

71 Various methods of Computation of Depreciation, Functional, Technological and


Economic Obsolescence

72 Reproduction Cost / Replacement cost, Depreciated Replacement Cost (DRC) working,


adopting DRC as Value subject to Demand and Supply aspect

73 Land Value by Market Approach and Building Value by Cost Estimation Method for
Owner Occupied Bungalows, Factories, Public Buildings.

Various purposes of Valuation (5 marks)

74 Valuation of properties for purposes such as:Bank Finance, Auction Reserve, Building
Insurance, Sale, Purchase, Valuation Disputes in Court, Probate, Partition,

75 Rent Fixation, Stamp Duty, Capital Gain Tax, Lease and Mortgage of Property. Any
other purposes not referred above.

76 Asset Valuation under the SARFAESI Act 2002, the LARAR Act 2013, the Companies
Act 2013, the Insolvency and Bankruptcy Code, 2016

77 Concept of Transferable Development Rights (TDR), Concept of Time Share Interest


in Real Property. Valuation of TDR, Time Share Interest and Easement Rights.

78 Study of Indian Accounting Standards (Ind AS) as applicable to Valuation of Real Estate.
Study of International Valuation Standards (IVS) as applicable to Valuation of Real
Estate

Important Case Laws on principles of valuation of Real Estate (3 marks)

79 1. K P Varghese vs ITO (1981) 131 ITR 597(SC)

2. Gold Coast Trust Ltd. vs Humphray (1949) 17 ITR 19


11

3. Rustam C Cooper vs Union of India AIR 1970 SC 564

4. Hays Will Trust vs Hays and Others (1971) 1WLR 758

5. V C Ramchandran vs CWT (1979) 126 ITR 157 Karnataka HC

6. Subh Karan Choudhury vs IAC (1979) 118 ITR 777 Kotkatta HC (Special Value/
FMV)

80 7. Wenger & Co. vs DVO (1978) 115 ITR 648 Delhi HC (Combination of Methods)

8. Sorab Talati vs Josheph Michem Appeal 101 0f 1949 - Vol. - 2 of SOC - page 162
(Bombay) (Invest Theory of Rent)

9. CWT vs P N Sikand (1977) 107 ITR 922 SC

10. SLAO (Eluru) vs Jasti Rohini (1995) 1SCC 717 SC

11. Shubh Ram and Others vs State of Haryana (2010) 1SCC 444

12. Jawaji Nagnathan vs REV. DIV. officer(1994) SCC -4 Page 595 SC

13. Chimanlal Hargovinddas vs SLAO- Pune, AIR 1988 SC 1652

81 Valuer as an Expert witness in Court.

Valuers’ functions & responsibilities, Error of judgement and Professional negligence

Code of conduct for valuers and Professional Ethics for valuers.

i Principles of Insurance and Loss Assessment (5 marks)

82 Principles and legal concepts in relation to Insurance of buildings. The Contract of


Insurance. Insurable Interests and Liability to Insure. Duties of the Insurer and the
Insured.

83 The types of Fire Policies, Reinstatement Cost Policy and policies for other perils,
Terms and Conditions, Perils, Beneficial and Restrictive Clauses.

84 Value at Risk, Sum Insured and Condition of Average, Over and Under Insurance,
Inflation Provisions, other contents, Depreciation, Obsolescence and Betterment.

85 Preparation of Claim for Damages due to Insured Perils.

86 Obligations and Rights of Insurer and Insured.


12

j Report writing (2 marks)

87 Reports - Quality, Structure and Style

Report writing for various purposes of valuation-Sale, Purchase, Purchase, Mortgage,


Taxation, Insurance, Liquidation

88 Contents of the report: Instruction of Clients, Valuation Date, Site Inspection, Location,
Ownership History, Data Collection and Analysis, Type of Construction, Valuation Method,
Value Estimation, Conclusion

k Case Study (12 marks)

This section will have case study for application of valuation techniques. There will be
comprehension(s) narrating the transaction based on which questions will be asked
from the case.

89 Case study (2)


90 Case study (2)
91 Case study (2)
92 Case study (2)
93 Case study (2)
94 Case study (2)

* * *
13

PART - II

MULTIPLE CHOICE QUESTIONS (Q34 TO 88)


14

This page is kept vacant intentionally.


15

INDICATIVE MULTIPLE CHOICE QUESTIONS (Q 34 to 88)

g. LAW - REAL ESTATE (Q34 TO 37)

34.1. How the market value of the property is determined in LARAR?

a) 5 years average GLV b) 3 years average GLV


c) GLV as on the date d) Property on going market value

34.2. What is the basis of fixing market value in LARAR?

a) Value nearer to that place b) 3 Kms surrounding that place


or village
c) Property value on current d) Mutual agreement value
market rate

34.3. Who has the power of taking possession of acquired land?

a) District Collector b) Village Administration officer


c) The District Court d) Central Government

34.4. As per Section 23 of LARAR, the market value is determined as on which date?

a) Date of publication b) Date of Notification


c) Date of possession d) Date of compensation of award

34.5. As per Section 23, what is the additional percentage value given for compulsory
acquisition by court?

a) 15% b) 20% c) 30% d) 40%

34.6. As per Section 49, who has the power to acquire a part or full area of the building?

a) District Collector b) Village Administrative officer


c) The District Court d) Central Government

34.7. What is the minimum period for return of the unutilized acquired property?
16

a) 5 years b) 3 years
c) 7 years d) 2 years

34.8. Can an assigned government / patta land can be taken for acquisition?

a) It can be taken b) It cannot be taken


c) Depending on the value of land d) Only a part of land is taken

34.9. Limit of acquisition of land is restricted for

a) Railways b) Highways
c) Power lines d) Agricultural lands

34.10. What is the power given in Section 51 of LARAR?

a) Exemption of stamp duty b) Award more value for acquisition


c) Release of property to owner d) Acquiring additional area

35.1. Basement as per Building bylaws is excluded in floor area ratio if

a) Used as shop b) Used as car park


c) Used as residence d) Used as godown

35.2. For group housing, the density pattern is taken per dwelling unit as

a) 4.50 persons per DU b) 6.00 persons per DU


c) 2.50 persons per DU d) 3.50 persons per DU

35.3. Is it right to say that size of the plot determines the number of dwelling unit?

a) Correct b) Not correct


c) May be d) Irrelevant

35.4. As per the building bye-laws, the minimum open car space as per parking standards
is

a) 18.00 square metre b) 28.00 square metre


c) 13.00 square metre d) 23.00 square metre

35.5. As per the building bye-laws, the minimum covered car space as per parking
standards, is
17

a) 18.00 square metre b) 28.00 square metre


c) 13.00 square metre d) 23.00 square metre

35.6. As per the building bye-laws, the minimum basement car space as per parking
standards, is

a) 18.00 square metre b) 28.00 square metre


c) 13.00 square metre d) 23.00 square metre

36.1. Rent Control Act is applicable for properties like

a) Public building b) Agricultural


c) Owner occupied d) Tenanted property

36.2. As per which Act, the tenancy right is only occupancy right and the eviction right is
to the landlord?

a) Succession Act b) SARFEASI Act


c) Easement Act d) Rent control Act

36.3. Ownership rights in rent controlled properties

a) Can be transferred b) Can not be transferred


c) Has pledging rights d) Power to make improvements

36.4. The value of the Rent Controlled properties will have a

a) High market value b) Both low and high


c) No difference d) Low market value

36.5. In rent control act, the fair rent is to be determined on which date?

a) Date of Petition b) Date of inspection


c) Date of valuation d) Date of award

36.6. In rent control act, the depreciation is determined by

a) W D V Method b) Any method can be followed


c) Linear Method d) Straight line method
18

36.7. What is an appurtenant land, as defined in rent control act (Tamilnadu)?

a) 50% over building plinth area b) Total extent of property land


c) 70% over building plinth area d) 60% over building plinth area

36.8. In case of a rented building in third floor, how the land is proportioned (Tamilnadu)?

a) 1/3 share of the land b) Total extent of property land


c) 30% over building plinth area d) 50% over building plinth area

37.1. Ownership rights under licensed properties

a) Transferred b) Cannot be transferred


c) Pledging rights d) Power to make improvements

37.2. Tenancy period under licensed properties is

a) Limited to 5 years b) As per the license agreement


c) Cannot be stipulated d) Rest only with the licensee

37.3. The value of the licensed property can be done by

a) Cost approach b) Market approach


c) Any other method d) Income approach

37.4. The revocation of licensed property is not applicable

a) Granted for a limited period b) When licensee is not releasing


c) Licensee becomes absolute d) License is unbroken for 20 years
owner

* * *

Answers :

34.1 - b 34.9 - d 36.1 - d 37.1 - b


34.2 - a 34.10 - a 36.2 - d 37.2 - b
34.3 - a 35.1 - b 36.3 - b 37.3 - d
34.4 - a 35.2 - a 36.4 - d 37.4 - c

34.5 - a 35.3 - a 36.5 - a


34.6 - c 35.4 - a 36.6 - c
34.7 - a 35.5 - d 36.7 - a
34.8 - a 35.6 - b 36.8 - a
19

Q38 to 41. THE TRANSFER OF PROPERTY ACT, 1882

38.1. What is the law by which the property acquisition cannot be made?

a) Government Grants act 1895 b) Indian succession Act, 1925


c) Indian Limitation act 1963 d) Indian Easement Act, 1882

38.2. What is the name of the act which governs grant?

a) Government Grants act 1895 b) Indian succession Act, 1925


c) Indian Limitation act 1963 c) Transfer of Property Act, 1882

38.3. How the Government land is transferred?

a) By assignment b) By Will
c) By adverse possession d) By Gift

38.4. What is the Method of assignment?

a) Assignment by issue of patta b) Will


c) Adverse possession d) Gift

38.5. What is the property type which is not assigned by the Government normally?

a) Mining lands b) Quarries


c) Settlement lands d) Town property

38.6. What is the act which governs succession?

a) Government Grants act 1895 b) Indian succession Act, 1925


c) Indian Limitation act 1963 d) Transfer of Property act, 1882

38.7. What is the law which is not governing succession?

a) Indian succession Act, 1925 b) Hindu Succession act


c) Mohammedan law of Succession d) Transfer of Property act, 1882

38.8. Which person can acquire ownership by succession?


20

a) Property owner b) Owner’s legal heir


c) Under adverse possession d) Trespasser

38.9. Which is not the way for property transfer to owner’s legal heir?

a) Will b) Lease c) Gift d) Settlement

38.10. An adverse possession is governed by which law?

a) Indian succession Act, 1925 b) Government Grants act 1895


c) Laws of prescription of Indian d) Transfer of Property act, 1882
limitation act 1963

38.11. Which act governs the Conveyance of property?

a) Indian succession Act, 1925 b) Government Grants act 1895,


c) Laws of prescription of Indian d) Transfer of Property act, 1882
limitation act 1963

38.12. What is an Immoveable property as per transfer of Property Act?

a) Standing timber b) Growing crops or grass


c) Transport vehicle d) Imbedded to earth –building

38.13. Which is not required for registration of property in transfer?

a) A Registered document b) Physical delivery of property


c) Attestation of Witnesses for d) Property ownership possession
registration

38.14. Who are not eligible for registration of property in transfer?

a) Companies b) Associations of persons


c) One or more living persons d) Mentally retarded / legally
disqualified

38.15. Which are not corporeal rights?

a) Right on tangible assets b) Right on material objects


c) Right on movable & immovable d) Rights is on intangible assets
properties
21

38.16. A sole ownership means

a) A person owning a thing b) Two persons owning a thing


c) Exclusive ownership of a c) A trust owning a property
company

38.17. A co-ownership or concurrent ownership means

a) A person owning a thing b) Two persons owning at the same


time
c) Exclusive ownership of a d) A trust owning a property company

38.18. A trust and beneficial ownership is

a) Solely for the benefit of the trust b) 2 persons owning at the same time
c) Exclusive ownership of a company d) A person owning a thing

38.19. An absolute ownership means

a) Trustee on land ownership b) Clear, perfect title, vested ownership


c) Legal ownership of the property d) Exclusive ownership

38.20. A contingent ownership means

a) Trustee’s land ownership b) Conditional right, with limitations


c) Legal ownership of the property d) Exclusive ownership

38.21. A corporeal Possession or a direct and primary Possession is

a) Owner of a building, car, factory b) Copyrights, patents, trademarks


c) Leasehold rights d) Firm goodwill

38.22. What is a Representative Possession?

a) Owner of a building, car, factory b) Copyrights, patents, trademarks


c) Servant holding owner’s money d) Leasehold rights / Firm goodwill

38.23.What is a Concurrent Possession?

a) Sole owner of a building, car, b) Copyrights, patents, trademarks


factory
22

c) Leasehold rights / Firm goodwill d) By 2 persons jointly at the same time

38.24. What is an Adverse Possession?

a) Sole owner of a building, car, b) Copyrights, patents, trademarks


factory
c) Leasehold rights / Firm goodwill d) Undue ownership rights claim by
other person not having the right

38.25. Transfer of property act, 1882 applies to?

a) Transfer by government b) Non-living person


c) By will d) Conveyance of property

38.26. As per Transfer of property act, 1882, the property that can be transferred is

a) Succession by legal heirship b) Tenancy rights of occupancy


c) Right of entry or breach of d) Clear perfect title ownership
conditions

38.27. Who is called a mortgagor?

a) Account holder b) Banker


c) Sundry Creditor d) Person pledging an interest

38.28. Who is called a mortgagee?

a) Account holder b) Person securing loan advanced


c) Sundry Creditor d) Person pledging an interest

38.29. What is called a mortgage money?

a) Sundry Credits b) Loan advanced


c) Principal + interest on loan d) Interest security
secured

38.30. What is called a mortgage deed?

a) Transfer deed for loan secured b) Loan statement


c) Lien document on deposits d) Interest statement
23

38.31. When does a mortgage by conditional sale become non-effective?

a) On default of mortgage-money b) Sale is void if the mortgagor pays


payment
c) Mortgaged property buyer has to d) If sale conditions are not there in
retransfer the property when loan the mortgage deed
is fore closed

38.32. The Local body authorities for nonpayment of statutory taxes, has taken land of a
company as security and mortgaged towards the taxes due to them. Till the time of
repayment of taxes it enjoys the land and derived the benefits and adjusted towards
the company’s dues to them. What is the type of mortgage?

a) Simple mortgage b) Conditional Sale


c) Usufructuary mortgage d) English mortgage

38.33. As per Section 96, mortgage done by deposit of title-deeds is called as?

a) Simple mortgage b) Conditional Sale


c) Usufructuary mortgage d) English mortgage

39.1. What are rights conferred in assignment?

a) Transfer of rights held by the b) Assignee involvement in a contract


assignor
c) Transfer of an equitable interest d) Non-consent of non-assigning party

39.2. When revocability of a gifted assignment cannot be carried out?

a) If the obligor has already b) If the assignee is in custody of


performed investments
c) Assignor’s death or bankruptcy d) Contract containing
non assignment clause

39.3. When a lien is not called as common-law lien?

a) A passive right to retain & b) No power of sale


cannot be transferred
c) If law conferring sale power d) If surrendered & contract sale
24

40.1. Lease requires registration as per the

a) Government Grants act 1895 b) Indian succession Act, 1925


c) Indian Limitation act 1963 d) Transfer of Property Act, 1882

41.1. Conditional transfer as per Sec. 25 is possible if?

a) Forbidden by law or immoral b) Fraudulent conditions


conditions
c) Implies injury to person or property d) The condition is possible & can be
fulfilled

41.2. Transfer as per Sec. 53 does not mean fraudulent if?

a) Intention to delay the transferor b) With fraudulent conditions


c) Transfer is done with d) Intention to defraud by reason
consideration done

* * *

Answers :

38.1 - d 38.9 - b 38.17 - b 38.25 - d 38.33 - a


38.2 - a 38.10 - c 38.18 - a 38.26 - d 39.1 - d
38.3 - a 38.11 - d 38.19 - b 38.27 - d 39.2 - d
38.4 - a 38.12 - d 38.20 - b 38.28 - b 39.3 - c

38.5 - d 38.13 - b 38.21 - a 38.29 - c 40.1 - d


38.6 - b 38.14 - d 38.22 - c 38.30 - a 41.1 - d
38.7 - d 38.15 - d 38.23 - d 38.31 - d 41.2 - c
38.8 - b 38.16 - a 38.24 - d 38.32 - c
25

Q40. LEASE OF IMMOVABLE PROPERTY

40.1. While transfer of property to lessee under lease, the lessor is not required to pay
capital gains

a) False b) True c) May or may not d) Not relevant

40.2. If both land and building are given on lease, it is called occupational lease.

a) False b) True c) May or may not d) Not relevant

40.3. A leased property is normally valued by income approach.

a) False b) True c) May or may not d) Not relevant

40.4. Capital value is net income multiplied by years purchase.

a) False b) True c) May or may not d) Not relevant

40.5. 99 years lease with renemal clause is called a perpetual lease.

a) False b) True c) May or may not d) Not relevant

40.6. Lease for life is not common in India.

a) False b) True c) May or may not d) Not relevant

40.7. Net profit rent for lessee = Rack rent minus ground rent minus outgoings.

a) False b) True c) May or may not d) Not relevant

40.8. The rights of lessor / lessee depends upon the conditions stipulated in the lease
deed.

a) False b) True c) May or may not d) Not relevant


26

40.9. Even in a perpetual lease, the lessee’s right will be low, if the unexpired period is
less.

a) False b) True c) May or may not d) Not relevant

40.10. The handing over of open land back to the lessor is called Reversion.

a) False b) True c) May or may not d) Not relevant

40.11. A property is a bundle of rights.

a) False b) True c) May or may not d) Not relevant

40.12. The word “covenant” indicates the terms and conditions stipulated in any deed.

a) False b) True c) May or may not d) Not relevant

40.13. A man cannot grant a lease to himself.

a) False b) True c) May or may not d) Not relevant

40.14. Perusal of lease deed is the first duty of a valuer if he wants to value a leasehold
property.

a) False b) True c) May or may not d) Not relevant

40.15. A “deposit” is the amount which is returnable on fulfillment of certain conditions.

a) False b) True c) May or may not d) Not relevant

40.16. The amount of Re. 1 per annum is the reciprocal of the sinking fund.

a) False b) True c) May or may not d) Not relevant

40.17. The rate of capitalisation in leasehold depends on the money market from time to
time.

a) False b) True c) May or may not d) Not relevant


27

40.18. The capital value of income can be detemined from the net income and the
percentage return required on investment.

a) False b) True c) May or may not d) Not relevant

40.19. The rate of capitalisation for a leasehold interest in general is 1% more than the
freehold interest.

a) False b) True c) May or may not d) Not relevant

40.20. A token amount or a nominal rent is called as a Acknowledgement rent.

a) False b) True c) May or may not d) Not relevant

40.21. If the unexpired period of lease is short, the value of lessor’s share will be more.

a) False b) True c) May or may not d) Not relevant

40.22. If the unexpired period of lease is very long, the value of lessee’s share will be
more.

a) False b) True c) May or may not d) Not relevant

40.23. A leaseholder’s interest in a property will normally decrease with the passage of
time and ultimately extinguish with the expiry of lease.

a) False b) True c) May or may not d) Not relevant

40.24. Net Rent = Gross rent - Outgoings.

a) False b) True c) May or may not d) Not relevant

40.25. Longer period of lease enables the lessee to recover his capital invested in the
improvement of the land.

a) False b) True c) May or may not d) Not relevant

40.26. Ground rent is well secured when improvement is done on the land given on lease.

a) False b) True c) May or may not d) Not relevant


28

40.27. In the case of sub - lease, if the proposed ground rent is higher han the original
ground rent, then it is known as improved ground rent.

a) False b) True c) May or may not d) Not relevant

40.28. Valuation procedure for a freehold property and a leasehold property with a
perpetual lease is not same.

a) False b) True c) May or may not d) Not relevant

40.29. The provisions or terms of lease would decide the share value of lessor and
lessee.

a) False b) True c) May or may not d) Not relevant

40.30. When lessor’s interest is valued, single rate table is normally to be used.

a) False b) True c) May or may not d) Not relevant

40.31. A lease where lessee has undertaken to carryout all the repairs and to bear all
outgoings is called as “Full Repairing Lease”.

a) False b) True c) May or may not d) Not relevant

40.32. In the case of perpetual lease with covenant of renewal, the lessor cannot
terminate the lease or refuse to renew the same as long as the lessee do not
violate any terms specified in the lease agreement.

a) False b) True c) May or may not d) Not relevant

40.33. Higher the rate of capitalisation, lower is the value of the asset.

a) False b) True c) May or may not d) Not relevant

40.34. Lower the rate of capitalisation, higher is the value of the asset.

a) False b) True c) May or may not d) Not relevant


29

40.35. If the document creates an interest in the property, it is a lease. But, if it only
permits another person to make use of the property for a temporary period, then it
is a licence.

a) False b) True c) May or may not d) Not relevant

40.36. The owner of a freehold property can do anything with his property.

a) False b) True c) May or may not d) Not relevant

40.37. A freehold property is the highest form of ownership.

a) False b) True c) May or may not d) Not relevant

40.38. Licence and lease are not same.

a) False b) True c) May or may not d) Not relevant

40.39. The leaseholds are less attractive than freeholds from the investment point of view.

a) False b) True c) May or may not d) Not relevant

40.40. The sub - lease can be granted only for a period which is less than the original
lease period.

a) False b) True c) May or may not d) Not relevant

40.41. A rent is governed by the Rent control act. A licence is governed by the Easement
act. A lease is governed by the Transfer of Property act.

a) False b) True c) May or may not d) Not relevant

40.42. Schedule III can not be adopted for valuation of leasehold rights.

a) False b) True c) May or may not d) Not relevant

* * *
Answers :

40.1 to 40.42 - b
30

Q41. THE TRANSFER OF PROPERTY ACT, 1882

1. Which one of the following sections of the transfer of property act, defines notice?

a) Section 2 b) Section 3
c) Section 5 d) Explanantion II of section 3

2. Transfer of property is defined in

a) Section 4 b) Section 5
c) Section 6 d) Section 10

3. Which section lays down that property of any kind may be transferred, except as
otherwise provided by this act or by any other law for the time being in force

a) Section 6 b) Section 7
c) Section 8 d) Section 9

4. Fraudulent transfer is contained in

a) Section 53 b) Section 54
c) Section 55 d) Section 56

5. Plaintiff seeks specific performance of an oral contract and alternatively pleads


benefit of section 53 - A of the transfer of property Act - whether alternative relief

a) Can be granted b) Can be granted subject to


conditions
c) Depends on proof by plaintiff d) Cannot be granted

Ans : d

* * *

Answers :

1 - b, 2 - b, 3 - a, 4 - a, 5 - d

Courtesy : Mr. K.S. Nagarajaiah


31

Q42. MOHMEDAN - PERSONAL LAW, THE HINDU SUCCESSION ACT, 1956, THE HINDU
SUCCESSION (AMENDMENT) ACT, 2005 (39 OF 2005)
Q43. THE INDIAN SUCCESSION ACT, 1925, LAW OF SUCCESSION FOR PERSON
OTHER THAN HINDU AND MOHMEDAN, WILL & TESTAMENT; SUCCESSION
CERTIFICATE

1. Property acquired by a male under Hindu succession act is his

a) Separate property
b) Ancestral property
c) Separate property in relation to existing members
d) None of these (Depends on facts and circumstances)

2. A Hindu dies intestate leaving behind two sons one daughter and window. His
property shall devolve to

a) Sons only b) Widow only


c) Sons and daughter only d) Sons, daughter and widow only

3. As per Muslim law the estate of a deceased person devolves upon his hers

a) After his death b) Before his death


c) At the moment of this death d) None of the above

4. In Muslim law, a gift consideration is called

a) Hibba b) Hiba - bil - iwaz


c) Ariyat d) Sadaque

5. Under Muslim law, the only natural guardian is

a) Father b) Mother
c) Grand father d) Grand mother

* * *
Answers :

1 - d, 2 - d, 3 - a, 4 - b, 5 - a

Courtesy : Mr. K.S. Nagarajaiah


32

Q44. VALUATION OF REAL ESTATE - COST, PRICE AND VALUE

44.1. The actual expenditure in manufacturing an asset is called as

a) Cost b) Price c) Value d) Worth

44.2. The amount paid for acquiring ownership is called

a) Cost b) Price c) Value d) Worth

44.3. Cost plus profit is called as

a) Value b) Increase cost c) Price d) Worth

44.4. Price is always more than the cost.

a) Need not be b) Always more c) Always less d) Definitely more

44.5. An estimate of the price as it ought to be, is called as

a) Cost b) Value c) Price d) Worth

44.6. Mr. X has spent Rs. 1 crore in constructing a residential building and he offers
Rs. 1.15 crores to sell it. The cost is

a) Rs. 1.075 crores b) Rs. 1.15 crores


c) Rs. 1 crore d) More than 1.5 crores

44.7. Mr. Y purchases a house for Rs. 1.15 crores as against the cost of 1.00 crore as
incurred by Mr. X. What is the cost in the hands of Mr. Y.

a) Rs. 1.15 crores b) Rs. 1.00 crore


c) Rs. 1.075 crores d) above Rs. 1.15 crores

44.8. The tag attached to a product in a shop for the purpose of selling is called as

a) Cost tag b) Price tag c) Value tag d) Worth tag


33

44.9. For the purpose of giving loan to a property under mortgage, the bank is directing
its valuer to certify

a) Price b) Cost c) Value d) Worth

44.10. To construct a new building, Mr. X has applied loan. After the construction is
completed, the bank directs the valuer to certify the

a) Cost b) Value c) Price d) Worth

44.11. After the construction of his new building, the assessee for the purpose of income
tax approaches the valuer and request him to certify the

a) Value b) Cost c) Price d) Worth

* * *

Answers :
44.1 - a 44.5 - b 44.9 - c
44.2 - b 44.6 - c 44.10 - a
44.3 - c 44.7 - a 44.11 - b
44.4 - a 44.8 - b
34

Q45. VALUATION OF REAL ESTATE - TYPES OF VALUE

45.1. The rental value of the property assessed by the local authority for levy of property
tax is called as

a) Market value b) Annual letting value


c) Book value d) Distress value

45.2 The value which can be defined as an estimate of the price the property would
realise in the open market under private or public auction is called as

a) Market value b) Auction value


b) Distress value d) Forced sale value

45.3. The written down value of an asset as shown in the books of account is called as

a) Market value b) Depreciated value


c) Book value d) Fair value

45.4. When a property is sold by the owner under distress condition, the sale price is
called as

a) Distress value b) Forced sale value


c) Fair value d) Market value

45.5. When a property is sold in the open market under normal conditions, such value is
called as

a) Auction value b) Market value


c) Distress sale value d) Forced sale value

45.6. The value of a running business of an industrial or commercial establishment with


all its tangible and intangible assets is called as

a) Investment value b) Fair market value


c) Liquidation value d) Going concern value

45.7 When the auction is carried out under order of the court and is also supervised by
the court, such value is usually called as
35

a) Auction value b) Liquidation value


c) Distress sale value d) Fair market value

45.8. When sufficient time is given for auction to liquidate the assets, it is called as

a) Forced liquidation value b) Orderly liquidation value


c) Distressed sale value d) Forced sale value

45.9. When the assets are liquidated as quickly as possible with a very little market
exposure and short time, it is called as

a) Orderly liquidation value b) Distressed sale value


c) Forced liquidation value d) Forced sale value

45.10. The estimate value of mortgage loan amount that could be safely advanced by the
bank is called as

a) Forced sale value b) Mortgage value


c) Liquidation value d) Distressed sale value

45.11. Replacement cost minus depreciation is called as

a) Depreciated replacement cost (DRC) b) Present market value


c) Depreciation value d) Market value

45.12. It is an estimated value of the property worked out on notional concepts for special
purpose, say for purpose of taxation. The value is called as

a) Notional value b) Market value


c) Tax value d) Fair value

45.13. This term in common parlance would mean net money likely to be realised by the
owner after the sale of the property. The value is called as

a) Fair value b) Net value


c) Market value d) Realisable value

45.14. The minimum price mentioned in the advertisement for the purpose of auctioning is
called as

a) Advertisement value b) Upset value (Reserve value)


36

c) Auction value d) Fair value

45.15. It is the price expected for a building whose useful span of life is over but is still
continued use. It is the value at the end of utility period of the asset without being
dismantled.

a) Salvage value b) Scrap value


c) Junk value d) Depreciated value

45.16. It is the value of dismantled materials which has become completely useless for
any further use. The value is called as

a) Depreciation value b) Salvage value


c) Scrap value (Junk value) d) Useless value

45.17. It is the value of a property to a speculator who invests in the property with the sale
motive of selling the property at a profit within a short time.

a) Speculative value b) Desired value


c) Future value d) Potential value

45.18. Mr. ‘X’ would like to buy a car with the special registration number 1 or 786 by giving
extra amount. This value is called as

a) Potential value b) Desired value


c) Fair value d) Special value

45.19. It is the true value of the property as distinct from the agreement value. A property
is purchased for Rs. 50 lakhs but the sale agreement is made for Rs. 25 lakhs.
Rs. 50 lakhs is called as

a) Intrinsic value b) Deed value


c) Agreement value d) Stamp duty value

45.20. It is a value of the property estimated in accordance with the provisions of the
concerned statute like wealth tax schedule III. The value is called as

a) Statutory value b) Tax value


c) Market value d) Book value

* * *
Answers :
45.1 - b 45.5 - b 45.9 - c 45.13 - d 45.17 - a
45.2 - b 45.6 - d 45.10 - b 45.14 - b 45.18 - d
45.3 - c 45.7 - b 45.11 - a 45.15 - a 45.19 - a
45.4 - a 45.8 - b 45.12 - a 45.16 - c 45.20 - a
37

Q46. VALUATION OF REAL ESTATE - BASIC ELEMENTS OF VALUE -


MARKETABILITY, UTILITY, SCARCITY AND TRANSFERABILITY

46.1. The four essential elements of value are

a) Utility, scarcity, demand and transferability


b) Size, area, shape, access
c) Location, amenities, facilities, services
d) Engineering aspect, architectural aspect, design, occupation

46.2. No one would like to buy a property if it has no

a) Utility b) Drawing approval


c) Legal opinion d) Tax assessment

46.3. The property has value only if it can be

a) Transferable b) Non - transferable


c) Non - marketable d) Temple property

46.4. A temple property is

a) Non - transferable b) Marketable


c) Transferable d) Non - marketable property

46.5. A temple property is not marketable, yet it has got value. Instead of market value,
we may call it as

a) Notional value b) Special value


c) Religion value d) Fair value

46.6. A temple property can be valued by assuming

a) Hypothetical sale (not actual sale) b) Comparable sale


c) Guideline rate d) Prevailing market trend
38

46.7. If any commodity is available in abundance, it will

a) Lose its original value b) Increase its value


c) Have no market at all d) Not be sold at all

46.8. If there is no demand for a product or no paying capacity to buy a product,

a) It gains more value b) It loses value


c) The value is stable d) It cannot be sold

46.9. The property has a value only if it can

a) Be transferable b) Have drawing approval


c) Have a title deed d) Have proper approach

46.10. In order to possess value, an asset ought to have

a) Title deed b) Utility


c) Tax assessment d) Rent yielding

46.11. The properties which are available in limited quantity and are not in abundance
command

a) Low value b) High value


c) No value d) Stable value

46.12. The ownership and possession of the property can be transferred by way of sale,
lease, mortgage, will, etc. and hence it has a value. This is called

a) Transferability b) Scarcity
c) Marketability d) Utility

* * *

Answers :
46.1 - a 46.5 - a 46.9 - a
46.2 - a 46.6 - a 46.10 - b
46.3 - a 46.7 - a 46.11 - b
46.4 - d 46.8 - b 46.12 - a
39

Q47. FACTORS AFFECTING VALUATION - PHYSICAL AND ECONOMIC

47.1. The factors such as local population, employment opportunities, changes in


services, trade and commerce, per capital income at district / state level, paying
capacity of local residents, trend of city growth are

a) Micro economic factors b) Macro economic factors


c) Physical factors d) Social factors

47.2. Domestic savings, fixed capital formation in construction and real estate sector,
flow of capital investments in bank, fixed deposits, shares, debentures,
government securities are

a) Macro economic factors b) Micro economic factors


c) Physical factors d) Social factors

47.3. Demand and supply of properties and income fetching properties are called as

a) Economic factors b) Legal factors


c) Physical factors d) Social factors

47.4. State and Central governments, policies of land development, economic &
Taxation policies of government, money market situation, recession period in real
estate are

a) Economic factors b) Legal factors


c) Physical factors d) Social factors

47.5. Inflation or deflation in nation’s economy, availability of money on credit from bank,
burden of property tax, employment opportunity are

a) Economic factors b) Legal factors


c) Physical factors d) Social factors
40

47.6. Land characteristics like size, shape, plot area, frontage, orientations, soil type,
topography are

a) Legal factors b) Physical (Technical) factors


c) Economic factors d) Social factors

47.7. Infrastructure facility like roads, water supply, drainage, power supply,
telecommunication links are

a) Legal factors b) Physical (Technical) factors


c) Economic factors d) Social factors

47.8. Prominence and placement like main road, by - lane, dead end road, remote area
location are

a) Legal factors b) Physical (Technical) factors


c) Economic factors d) Social factors

47.9. Building characteristics like RCC framed structure, further life, age of structure,
deterioration, specification of building, workmanship quality, intelligent building &
green building concepts, obsolescence, maintenance are

a) Legal factors b) Physical (Technical) factors


c) Economic factors d) Social factors

47.10. Functional aspects like optimum use of inner space with minimum wastage,
amenities like swimming pool, garden, lift, security system, intercom facility, health
club, children’s play area are

a) Legal factors b) Physical (Technical) factors


c) Economic factors d) Social factors

47.11. Environmental aspects like noise, smoke pollution level, sea front, nuisance due to
railway track, industries, air port, climatic conditions are

a) Economic factors b) Legal factors


c) Physical factors d) Social factors
41

47.12. Natural calamity like earth quake prone areas, flooding and cyclone hazards,
Tsunami prone area are

a) Economic factors b) Legal factors


c) Physical factors d) Social factors

47.13. Soil condition - Rocky soil, hard muram, black cotton soil, reclaimed soil, filled up
ground are

a) Economic factors b) Legal factors


c) Physical factors d) Social factors

* * *

Answers :
47.1 - a 47.5 - a 47.9 - b 47.13 - c
47.2 - a 47.6 - b 47.10 - b
47.3 - a 47.7 - b 47.11 - c
47.4 - a 47.8 - b 47.12 - c
42

Q48. FACTORS AFFECTING VALUATION - LEGAL AND SOCIAL

48.1. Rent control act, Urban land ceiling act, Coastal regulations, Ecological restriction
are examples for

a) Legal factors b) Economic factors


c) Social factors d) Physical factors

48.2. Transfer of property act with lease provisions, Covenants under lease or
conveyance deed, Easement act for licenses, Civil procedure code are examples
for

a) Legal factors b) Economic factors


c) Social factors d) Physical factors

48.3. Land acquisition act, Building Bye-laws, Town planning acts, Zoning regulations,
Laws governing building construction like development control rules, FSI norms,
Open space regulations, etc, Wealth tax, Income tax act are examples for

a) Legal factors b) Economic factors


c) Social factors d) Physical factors

48.4. Law on earth quake resistant building, Reservation under different acts are
examples for

a) Legal factors b) Economic factors


c) Social factors d) Physical factors

48.5. Safety distance from industrial belt, hazardous zone, etc., height restriction rules
near airport area, safety distance from high tension lines, railway tracks, highways,
water courses, are examples for

a) Legal factors b) Economic factors


c) Social factors d) Physical factors
43

48.6. Locality (like poor class, middle class, posh areas); Neighbourhood (like well
developed, less developed, slum, cremation ground, dumping ground, nuisance
due to community hall); Civic amenities (like proximity of shops, mall, market,
hospital, bus stand, railway station) are examples for

a) Economic factors b) Physical factors


c) Social factors d) Legal factors

48.7. Populations (density in area and population growth), Means of communication


(railway, roadway or waterways) are

a) Economic factors b) Physical factors


c) Social factors d) Legal factors

48.8. Prestige aspect (prominent location, renowned personality, well known celebrity,
famous sport champion, industrialist, politician); Political factor (Linguistic or
religious communal unrest) are examples for

a) Economic factors b) Physical factors


c) Social factors d) Legal factors

48.9. Racial habitation (like parsi colony, mohamedan colony, hindu colony, catholic
colony), Religious factors (like proximity of temple, church, mosque) are the
examples for

a) Economic factors b) Physical factors


c) Social factors d) Legal factors

48.10. Personal factors like Sentimental, Considerations, Belief in vaastu, Liking for
specific neighbourhood are the examples for

a) Economic factors b) Physical factors


c) Social factors d) Legal factors

* * *

Answers :
48.1 to 48.5 - a 48.6 to 48.10 - c
44

Q49. HIGHEST AND BEST USE, VALUE IN USE AND VALUE IN EXCHANGE

49.1. It is worth of an asset to specific user and therefore it is subjective and is usually
measured by cost of replacing the property less depreciation. It is non market based
value. It is called as

a) Value in exchange b) Desired value


c) Going concern value d) Value in use

49.2. It is the price that would tend to prevail in a free, open and competitive market on
the basis of an equilibrium, set by forces of demand and supply. Highest and best
use or alternative use of the property is also considered while estimating value. We
may say it is also Fair market value

a) Forced sale value b) Value in use


c) Going concern value d) Value in exchange

* * *

Answers :
49.1 - d 49.2 - d
45

Q50. REAL PROPERTY - RIGHTS AND INTERESTS IN REAL ESTATE


TYPES OF OWNERSHIP IN REAL ESTATE

50.1. The things which can be physically touched or felt like land, furniture, jewelry are
called as

a) Tangible asset b) Intangible asset


c) Physical asset d) Imaginary asset

50.2. The things which cannot be touched but have the right of ownership of
non - material things are called

a) Tangible asset b) Intangible asset


c) Physical asset d) Imaginary asset

50.3. Land, building are the examples of

a) Imaginary asset b) Intangible asset


c) Physical asset d) Tangible asset

50.4. Copyrights, goodwill are the examples of

a) Physical asset b) Tangible asset


c) Intangible asset d) Intellectual right

50.5. Property consisting of land and objects and substances permanently attached to
the ground is called

a) Real property b) Unreal property


c) Imaginary property d) Physical property

50.6. Plant and machinery, jewellery are considered as

a) Immovable property b) Movable property


c) Stable property d) Imaginary property
46

50.7. A property is a

a) Bundle of rights b) Bundle of ownership


c) Bundle of encumbrance d) Bundle of legalities

50.8. If a thing can be owned only by one person at a time, it is called as

a) Partnership b) Sole ownership


c) Association of persons d) Trust

50.9. If two or more persons own a thing as in the case of a house or land. It is called

a) Cooperative society b) Proprietorship


c) Trust d) Co-ownership

50.10. It is an example of duplicate ownership which allows the separation of the powers
of the management and the rights of management. It is called as

a) Trust b) Association of persons


c) Partnership d) Sole - ownership

50.11. The legal relation between a person and an object denoted as

a) Ownership b) Lease c) Tenant d) Lessee

50.12. ............. is one in which a property is conveyed to a person only for the terms of his
life. It is

a) Leasehold b) Life estate


c) Freehold d) Easement

50.13. It is the most complete ownership in real property. It implies absolute ownership.
The owner’s right is unrestricted in time (till perpetuity). This is called as

a) Easement b) Leasehold
c) Rented d) Freehold

50.14. A freeholder gives out to someone for use for a fixed duration under certain terms
and conditions. This is called as

a) Sale b) Rent c) Lease d) Mortgage


47

50.15. The use of someone’s land without obtaining the title is called as

a) Lease b) Easement
c) Sub - lease d) Ground rent

50.16. The owner of the adjacent premises may use the land owned by his neighbour on
a temporary basis is called as

a) Ground rent b) Leasehold


c) Sub - lease d) Easement right

* * *

Answers :
50.1 - a 50.5 - a 50.9 - d 50.13 - d
50.2 - b 50.6 - b 50.10 - a 50.14 - c
50.3 - d 50.7 - a 50.11 - a 50.15 - b
50.4 - c 50.8 - b 50.12 - b 50.16 - d
48

Q51. VALUATION OF REAL ESTATE - ANNUITIES, CAPITALISATION, RATE OF


CAPITALISATION, YEARS PURCHASE, SINKING FUND, REDEMPTION OF
CAPITAL, REVERSIONARY VALUE

51.1. It is a rate of interest at which the investor is willing to invest his capital to get
benefit. It is called

a) Rate of capitalisation b) Capital value


c) Years purchase d) Sinking fund

51.2. If a person deposits Rs. 10,00,000, in a bank as fixed deposit, the bank offers
interest at 8% on Fixed deposit. This 8% is called as

a) Yield rate b) Rate of capitalisation


c) Years purchase d) Rate of redemption

51.3. The rate of return expected by the investor for recoupment of capital invested in the
property is called as

a) Rate of sinking fund b) Rate of capitalisation


c) Rate of return d) Rate of redemption of capital

51.4. Rate of capitalisation is known as

a) Years purchase b) Discount rate


c) Remunerative rate of interest d) Reversion

51.5. Rate of redemption is also called as

a) Accumulative rate of interest b) Remunerative rate of interest


c) Years purchase d) Reversion

Capital value x Re. 1


51.6. is called as
Rate of capitalisation

a) Years purchase b) Reversion


c) Accumulative rate d) Remunerative rate

51.7. The year purchase to receive annuity of Re. 1 at 8% rate of return is

a) 9 b) 8 c) 10 d) 12.5

51.8. Net Income x Years Purchase gives

a) Annual value b) Reversionary value


49

c) Redemption value d) Capital value

51.9. The amount that has to be set aside annually by building owner at given rate of
interest for the period equal to past age of the building is called

a) Capital value b) Redemption value


c) Sinking fund d) Reversion value
R
51.10. The formula indicates for
(1 + R)n - 1

a) Simple interest b) Compound interest


c) Annual sinking fund d) Annual value

51.11. It is defined as the net annual payment (return on investment) for the capital
invested in an immovable property

a) Annuity b) Sinking fund


c) Capital value d) Reversionary value

51.12. Mr. X get a rate of return of 6% from his investment on commercial shop of value
1 crore. What is the annuity?

a) Rs. 6,00,000 b) Rs. 50,000


c) Rs. 10,00,000 d) Rs. 60,000

51.13. Mr. X invests money in a nationalised bank as fixed deposit. The bank gives 8% as
annual interest on fixed deposit. The monthly interest amount Rs. 80,000. What is
the fixed deposit amount (Capital value)?

a) Rs. 1,20,00,000 b) Rs. 1,00,00,000


c) Rs. 1,50,00,000 d) Rs. 1,10,00,000

51.14. Mr. X invests money Rs. 15,00,000 in a bank and he gets ever month Rs. 10,000
as monthly interest. What is the rate of interest (Rate of capitalisation)?

a) 8% b) 7% c) 6% d) 5%

* * *
Answers :
51.1 - a 51.5 - a 51.9 - c 51.13 - a
51.2 - b 51.6 - a 51.10 - c 51.14 - a
51.3 - d 51.7 - d 51.11 - a
51.4 - c 51.8 - d 51.12 - a
50

Q51 A. ANNUITIES, CAPITALIZATION, RATE OF CAPITALIZATION, YEARS PURCHASE,


SINKING FUND, REDEMPTION OF CAPITAL, REVERSIONARY VALUE

51.1. What is annuity?

a) Investment value b) Net annual income


c) Reversionary value d) Deferment value

51.2. What is valuation concept of an annuity?

a) Investment value b) Time value, interest rate, future


value
c) Reversionary value d) Deferment value

51.3. What is Capitalization?

a) Present Amount invested b) Invested Amount + interest


received
c) Interest received d) Profit amount received

51.4. What is yield rate?

a) Profit interest rate of income b) Interest versus capital value


c) Compound interest d) Simple interest

51.5. What is Discount rate?

a) Terminable rate b) Inverse interest rate


c) Reversionary yield d) Compounded interest

51.6. What is the Remunerative rate of interest?

a) Rate of capitalization b) Comparable rates with other


investments
c) Interest Rate of short term d) Interest Rate of profit
investments
51

51.7. What is accumulative rate of interest?

a) Interest portion received b) Total interest portion on


accumulation
c) Terminable rate d) Interest Rate of investments

51.8. What is the Year’s Purchase?

a) Present capital value b) Method of valuing future income


c) Average annual cash-flow d) Future Capital value

51.9. What is Rate of return?

a) Interest rate on capital value b) % net income of capital value


invested investment
c) Capital appreciation or capital d) Inverse interest rate
erosion
.
51.10. What is sinking fund?

a) Present Amount invested b) Reserve fund for future capital


expense
c) Net annual income d) Interest Rate of profit

51.11. What will be the range of Rate of sinking fund?

a) 1% b) 3 % to 8%
c) 2% d) More than 8%

51.12. What is redemption of capital?

a) Invested capital amount b) Capital+ accumulative interest


recoupment
c) Reserve fund d) Capital appreciation

51.13. What is the rate of redemption of capital when compared to market rate?

a) Equal b) Lower
c) Higher d) Cannot judge
52

51.14. For short term period leased properties or building having a shorter life span, the
rate of redemption of capital has to be dealt with

a) Net annual income b) Terminable income


c) High rate of recoupment d) Sinking fund

51.15. What is reversionary value?

a) Investment value b) Difference of capitalization value


and market rent value
c) Future Capital value d) Deferment value

51.16. What is reversionary yield?

a) Annualized rents percentage b) Anticipated yield on estimated


rental value
c) Higher yield for recoup investment d) Lower yield for recoup investment

* * *

Answers :
51.1 to 51.16 - b
53

Q52,53. VALUATION OF REAL ESTATE - CONSTRUCTION AND USE OF VALUATION


TABLES

1. The formula for Amount (A) is

(1 + i)n - 1 r
a) b)
i (1 + r)n - 1
1
1-
(1 + i)n
c) A = P (1 + i)n d)
i

2. The formula for Amount of Re. 1 per annum (APA) is

r
a) A = P (1 + i)n b)
(1 + r)n - 1
1
n 1-
(1 + i) - 1 (1 + i)n
c) d)
i i

3. The formula for Annual Sinking Fund (ASF) is

(1 + i)n - 1
a) A = P (1 + i)n b)
i
1
1-
r (1 + i)n
c) d)
(1 + r)n - 1 i

4. The formula for Present value of Re. 1 per annum at Years Purchase is

(1 + i)n - 1
a) A = P (1 + i)n b)
i
1
1-
(1 + i)n r
c) d)
i (1 + r)n - 1
54

5. The formula for Present value (PV)

(1 + i)n - 1
a) A = P (1 + i)n b)
i

1 r
c) (1 + i)n d) (1 + r)n - 1

6. A leasehold property is normally valued by

a) Cost approach b) Market approach


c) None of the above d) Income approach

7. If years purchase (Y.P.) is 12.5, then the rate of return is

a) 10% b) 6% c) 10% d) 8%

8. If the unexpired period in lease is short, then the lessor’s share will be

a) less b) equal c) depends d) more

9. If the unexpired period is very long, the value of lessee’s share will be

a) less b) depends c) stable d) more

10. The value in the hands of lessor and lessee mainly depends on

a) Rental income b) Rate of return


c) Advance received d) Terms & conditions of the deed

* * *

Answers :
1 to 5 - c 6 to 10 - d
55

Q52,53. CONSTRUCTION AND USE OF VALUATION TABLES

1. What is the gross amount which will fetch for Rs 1,000 at 10% on simple Interest for
2 years term?

a) Rs. 1,100 b) Rs. 1,200 c) Rs. 1,000 d) Rs. 1,150

2. What is the gross amount which will fetch for Rs 1,000 at 10% on Compound interest
amount for 2 years term?

a) Rs. 1,000 b) Rs. 1,210 c) Rs. 1,100 d) Rs. 1,200

3. Present value of rupee Calculation - What is the Present value of rupee for Capital
amount receivable at a future date of Rs 10,00,000 at 6 % compound interest rate for
10 year term?

a) Rs. 5,85,390 b) Rs. 5,58,390 c) Rs. 8,55,390 d) Rs. 5,26,850

4. Amount of Re. 1 / year (annum) Calculation - Rs 500 deducted every month and
invested annually towards PF account from salary for a period of 20 year terms and at
a 7% of compound interest?

a) Rs. 2,56,000 b) Rs. 2,46,000 c) Rs. 2,26,000 d) Rs. 1,50,000

5. Annual sinking fund Calculation - To find out the depreciated worth of the building to
set aside annually for 10 lakhs as Capital recoupment amount expected at a 4% interest
rate for the period unexpired period of lease of 60 years

a) Rs. 2,400 b) Rs. 4,200 c) Rs. 3,200 d) Rs. 4,000

6. Present value of future income of Re. 1 / year (Single rate basis) - Annual Rental
income from property is Rs 48,000 /-. If the building is demolished after 40 years, what
will be the present value of the property @ 7% interest rate?

a) Rs. 6,00,000 b) Rs. 6,39,936 c) Rs. 6,28,745 d) Rs. 6,93,935

7. Present value of future income of Re. 1 / year (Duel rate basis) - The annual rent
received from the property is rs 48000 /-. Expected rate of return is 10% future life of
the building is 50 years. Recoupment rate is 4% on capital. Find the purchase price.

a) Rs. 4,85,672 b) Rs. 4,50,672 c) Rs. 4,60,672 d) Rs. 4,75,672

* * *
Answers :
1 to 7 - b
56

Q54. URBAN INFRASTRUCTURE AND ITS INFLUENCE ON VALUE OF REAL


ESTATE

54.1. What affects the real estate value?

a) Increased Rapid urbanization b) Availability of urban infrastructure


c) Quick Implementation of services d) Poor Govt. budgetary allocations

54.2. When the real estate value comes down?

a) Availability of water supply b) Availability of sewage & sanitation


c) Availability of drainage d) No urban infrastructure
development

54.3. Which is not a finance type used for urban infrastructure development?

a) Market Based Finances b) Pooled Financing


c) Project Finance d) JNNURM linking

54.4. Who is the authority for Rating of bonds?

a) MOUD b) SEBI
c) CRISIL d) IT Act

54.5. Which is not a Public - Private Finance options?

a) Project Finance b) O & M


c) Joint Sector &BOT d) Bonds

54.6. Which is not a function of JNNURM?

a) Improved service delivery systems b) Improved economic level


c) Enhanced quality of life d) Centralization of Governance

* * *

Answers :
54.1 to 54.3 - d 54.5 to 54.6 - d
54.4 - c
57

Q55,56. REAL ESTATE MARKET AND ITS CHARACTERISTICS, INVESTMENT IN REAL


ESTATE, FACTORS INFLUENCING DEMAND AND SUPPLY SCHEDULE IN
REAL ESTATE

1. Investment in immovable properties is considered as

a) Sound investment b) Unstable investment


c) Not a wise investment d) Wrong investment

2. When compared to the yield rate available for long term investment in Government
security, the return of return on investment in real estate is considered as

a) Higher b) Lower
c) Not preferable d) Same

3. In case of any sale or transfer, getting back the capital invested is

a) Not certain b) Certain


c) Doubtful d) Not possible at all

4. For owner occupied properties in a developed area, capital appreciation is

a) Fairly bad chance b) Fairly good chance


c) Worst chance d) No chance

* * *

Answers :
1 to 2 - a 3 to 4 - b
58

Q57 to 61 - INCOME APPROACH TO VALUE

57.1. If the Land alone is fully developed and rented out, the method to be used

a) Any method b) Cost Approach


c) Market Approach d) Income approach

57.2. Land fully developed, partly owner occupation partly rented out

a) Only by income approach b) Only by market approach


c) Owner by market approach & d) Composite rate approach
rented portion by income approach

57.3. Land partly developed and rented, additional FSI available in the property

a) Income approach b) Market Approach


c) Cost Approach d) Rental method & market approach

58.1. Rental method is applicable for properties like

a) Public building b) Agricultural


c) Owner occupied d) Tenanted property

58.2. As per which Act, the tenancy right is only occupancy right

a) Succession Act b) SARFAESI Act


c) Easement Act d) Rent control Act

58.3. As per which Act, the eviction right is to the landlord

a) Succession Act b) Easement Act


c) SARFAESI Act d) Rent control Act

58.4. As per which Act, Reversion of the property is to the landlord

a) Succession Act b) Easement Act


c) SARFEASI Act c) Rent Control Act

58.5. The value of the Rent Controlled properties will have a

a) High market value b) Both low and high


c) No difference d) Low market value
59

58.6. What is the Government Act for lease

a) Succession Act b) SARFEASI Act


c) Rent Control Act d) Transfer of property Act

58.7. Deduction allowable under Indian Act on rental income of house property

a) 15% b) 20% c) 25% d) 30%

58.8. License is defined in Section 52 of

a) Succession Act b) Rent Control Act


c) SARFEASI Act d) Indian Easement Act

59.1. When a parcel of land is given on lease, the rent reserved in lease is known as

a) Standard rent b) Rack rent


c) Virtual rent d) Ground rent

59.2. The rent mutually fixed by the owner and the tenant is called as

a) Fair rent b) Standard rent


c) Head rent d) Contractual rent

59.3. The rent which can be legally charged by a landlord from a tenant or the rent derived
by a court of law is called as

a) Head rent b) Rack rent


c) Virtual rent d) Standard rent

59.4. The rent which includes the premium is called

a) Head rent b) Rack rent


c) Standard rent d) Virtual rent

59.5. When the lessor leases the property on lease he will get

a) Virtual rent b) Rack rent


c) Standard rent d) Head rent

59.6. When the head lessee sub leases the property he will get

a) Virtual rent b) Head rent


c) Standard rent d) Rack rent
60

59.7. The difference amount between the amount received by the head lessee by sub leasing
the leasehold property and amount paid by the head lessee to the lessor is called

a) Virtual rent b) Head rent


c) Rack rent d) Profit rent

59.8. What is the difference in rate of return of Secured ground rent in comparison with
unsecured ground rent?

a) 2% to 3% more b) 2% to 3% less
c) 1% to 2% more d) 1% to 2% less

59.9. What is the rent assumed if the property is Owner occupied or vacant premises

a) Profit rent b) Head rent


c) Standard rent d) Notional rent

60.1. Lease requires registration as per the

a) Rent Control Act b) Easement Act


c) SARFEASI Act d) Transfer of Property Act

60.2. 99 years with due provision for renewal is specifically called as

a) Long lease b) Short lease


c) Building lease d) Perpetual lease

60.3. Lessor gives an Open plot of land on lease to the lessee for the construction of
building for a lease period. This lease is

a) Occupational lease b) Sub - lease


c) Life lease d) Building lease

60.4. Lessor gives a building (i.e.) both land and building on lease to the lessee for a lease
period. This lease is

a) Building lease b) Full repair lease


c) Life lease d) Occupational lease

60.5. A lease agreement in which the lessee pays all outgoings apart from his head rent.
Such lease is called

a) Building lease b) Occupational lease


c) Life lease d) Full repair lease
61

60.6. What is the lease name when the lease period is fixed till the death of lessee and the
lease period expires on lessee’s death

a) Full repair lease b) Occupational lease


c) Building lease d) Life lease

60.7. A perpetual lease can be valued in the hands of

a) Lessor b) Tenant c) Head lessor d) Lessee

60.8. A leasehold property is normally valued by

a) All methods b) Cost approach


c) Market approach d) Income approach

60.9. If years purchase (Y.P.) is 12.5, then the rate of return is

a) 10.00% b) 12.50% c) 6.00% d) 8.00%

61.1. The value in the hands of lessor and lessee mainly depends on

a) Rental income b) Rate of return


c) Advance received d) Deed Terms

61.2. If the unexpired period in lease is short, then the lessor’s share normally will be

a) Less b) Same as lessee c) Equal d) More

61.3. If the renewal clause is not stipulated in the lease agreement then the lessor’s share
will be

a) Less b) Same as lessee c) Equal d) More

61.4. If the lessee surrenders the development in the property free of cost after expiry of
lease period then the lessor’s share will be

a) Less b) Same as lessee c) Equal d) More

61.5. If the periodical rent revision in terms of market rent during lease period, then the
lessor’s share

a) Less b) Same as lessee c) Equal d) More


62

61.6. If the unexpired period is very long, the value of lessee’s share will be

a) Same as lessor b) Equal c) Less d) More

61.7. If there is no restrictive conditions in the lease deed, then the value of lessee’s share
will be

a) Same as lessor b) Equal c) Less d) More

61.8. If the profit rent is more and balance lease period is more, then the value of lessee’s
share will be

a) Same as lessor b) Equal c) Less c) more

61.9. Premium means

a) None b) Refundable deposit amount


c) Annual lease rent d) Non-refundable Price

61.10. Premium is defined in transfer of Property Act

a) Section 108 b) Section 107


c) Section 106 d) Section 105

61.11. Remunerative rate / Accumulated rate of investment of lease period for 50 years
and above will be

a) cannot be ascertained b) More


c) Less d) Equal

* * *

Answers :

57.1 - d 58.6 - d 59.6 - d 60.5 - d 61.4 - d


57.2 - c 58.7 - d 59.7 - d 60.6 - d 61.5 - d
57.3 - d 58.8 - d 59.8 - d 60.7 - d 61.6 - d
58.1 - d 59.1 - d 59.9 - d 60.8 - d 61.7 - d

58.2 - d 59.2 - d 60.1 - d 60.9 - d 61.8 - d


58.3 - d 59.3 - d 60.2 - d 61.1 - d 61.9 - d
58.4 - d 59.4 - d 60.3 - d 61.2 - d 61.10 - d
58.5 - d 59.5 - d 60.4 - d 61.3 - d 61.11 - d
63

Q 62, 63.

1. Real estate economics is the application used for

a) Predict patterns of prices b) Predict patterns of supply


c) Predict patterns of demand d) All the above

2. Who are not the main participants in real estate markets?

a) Owner using as an investment b) Tenant


or for renting out or lease
c) Financial institutions d) Developers

3. Who are all forming demand side of the market?

a) Owner using for his own b) Owner for investment purpose


c) Renter or tenant d) all the above

4. Which people are all not forming supply side of the market?

a) Property developers b) Building renovators


c) Financial institutions d) Insurance companies

5. What are the characteristics of real estate economics?

a) Durability on construction b) Heterogeneity in terms of its


location, the building, and its
financing.
c) Pricing of real estate d) All the above

6. What are the market adjustment process subject to?

a) Time delays b) Due to delayed finance


c) Design and construction d) All the above

7. When the real estate market considered in terms of both an investment and
consumption good?

a) The expectation of attaining b) the intention of using it


a return
64

c) Both the above

8. What are the variables affecting demand schedule in real estate?

a) Population size and population b) More people in the economy


growth
c) Family size, family age d) All the above
composition, net migration,
death rates

9. How the market demand is calculated?

a) Household demand and b) Income or permanent income or


measure of wealth annual income
c) The price and quality factor d) All the above

10. What are the variables affecting supply schedule in real estate?

a) Availability of land b) Availability of labour


c) Electrical Power, water and d) All the above
building materials

11. How supply restrictions can affect substitutability?

a) Lack of supply of skilled labour b) Land availability


c) Land-use controls such as d) All the above
zoning bylaws

12. How supply factor reacts to increase in costs?

a) Developers will find their b) Developers may leave the industry


business less profitable
c) Quantity of housing starts will d) All the above
decrease and prices will tend
to rise

13. What are called as property concepts?

a) Exclusive right of acquiring b) Right of possessing


c) Enjoying and disposing d) All the above
65

14. Define Ownership?

a) An unlimited right in point b) Unrestricted in point of disposition


of duration

c) Subject to control of reduction d) All the above


by contractual obligation

15. How land ownership affected by government?

a) Imposes taxation and the right b) Right to revert back in the


for compulsory acquisition legitimate inheritance absence

c) Imposes restrictions on land use d) All the above


and rights of interest by
development through zoning
regulations, Environmental
requirements

16. The owner is entitle to?

a) Use the property in any manner b) Enjoy by exclusive possession,


or abuse the property derive benefits, income/ profits
from it

c) Disposing it during his lifetime d) All the above


by sale, gift or will

* * *

Answers :
1 - d 5 - d 9 - d 13 - d
2 - c 6 - d 10 - d 14 - d
3 - d 7 - c 11 - d 15 - d
4 - d 8 - d 12 - d 16 - d
66

Q64. MARKET APPROACH - TYPES OF MARKET, DEMAND AND SUPPLY, CURVE,


BELL CURVE FOR OVERALL SALES PERFORMANCE

64.1. When the supply is in excess of demand, it is called as

a) Buyers market b) Sellers market


c) Stable market d) No market

64.2. When the supply is less than demand, it is called as

a) Buyers market b) Sellers market


c) Stable market d) No market

64.3. In buyers market, the king is

a) Public b) Seller c) Buyer d) None

64.4. In sellers market, the king is

a) Broker b) Buyer c) Public d) Seller

64.5. Expectations of people about likely fall in price of real estate in near future results in

a) Stable market b) Buyers market


c) Sellers market d) No market

64.6. Under market approach, we can normally value only

a) Non - marketable properties b) Marketable properties


c) Religious properties d) Leasehold properties

64.7. Non - marketable properties can be valued by

a) Income approach b) Market approach


c) Cost approach d) Composite rate method

64.8. ‘Highest and Best use’ aspect is to be considered in


67

a) Composite rate method b) Cost approach


c) Income approach d) Market approach

64.9. ‘Market is Supreme’ - This aspect holds good in

a) Market approach b) Cost approach


c) Income approach d) Composite rate method

64.10. In the demand and supply curve, price is plotted as

a) X axis b) Y axis c) Z axis d) None

64.11. In the demand and supply curve, quantity of goods available is plotted as

a) Z axis b) Y axis c) X axis d) None

64.12. Transaction takes place after higgle haggle (bargaining) for the price. This
process is explained by

a) Sellers curve b) Supply and demand curve


c) Buyers curve d) Bell curve

64.13. The four factors - economic, physical, social and legal factors mainly affect

a) Value b) Cost c) Price d) None

64.14. Sale recorded at Registrar’s office is one of the sources where

a) Market value can be collected b) Sale instances can be collected


c) Cost can be collected d) Price can be collected

* * *

Answers :
64.1 - a 64.5 - a 64.9 - a 64.13 - a
64.2 - b 64.6 - b 64.10 - b 64.14 - b
64.3 - c 64.7 - c 64.11 - c
64.4 - d 64.8 - d 64.12 - d
68

Q65. MARKET APPROACH - COMPARISON OF SALE INSTANCES - FACTORS OF


COMPARISON AND WEIGHTAGES OF ADJUSTMENT IN VALUE

65.1. Formula for success in any valuation assignment is

a) Inspect before you value b) Allot the work to your staff

c) Receive the fees before d) Valuation report


accepting the assignment

65.2. For collecting data for sale comparisons, a good source of information is

a) Court b) Registrar of documents


c) Corporation d) Town planning department

65.3. Before undertaking comparison with sale transactions, the valuer’s prime duty is

a) Study the tax receipt b) Study the drawing


c) Study the title deed d) Study the encumbrance certificate

65.4. Sale transaction records cannot be fully relied upon because of

a) Wrong representation b) Under valuation


c) Over valuation d) Black money transaction
(parallel economy)

65.5. The weightages adopted in sale transaction may

a) Change from locality to locality b) Be constant everywhere


c) Change from purpose to purpose d) Change from time to time

65.6. The weightage to be applied to sale instance rate in the case of landlocked land
is

a) + 15% b) - 75%
c) + 25% d) - 15%
69

65.7. The weightage to be applied for forced sale by authorities is say,

a) + 5% b) + 10%
c) - 15% d) + 15%

65.8. The weightage to be applied for low lying plots

a) - 50% b) - 25%
c) - 30% d) Deduct for cost of earth filling

65.9. The weightage to be applied for large size plot

a) - 15% to - 40% b) + 15% to + 40%


c) + 60% d) - 100%

65.10. The weightage to be applied for properties pending litigation (stay in the court
against sale)

a) + 10% b) - 30% or more


c) + 20% d) Zero

* * *

Answers :
65.1 - a 65.5 - a 64.9 - a
65.2 - b 65.6 - b 64.10 - b
65.3 - c 65.7 - c
65.4 - d 65.8 - d
70

Q66. MARKET APPROACH TO VALUE - HEDONIC MODEL AND ADJUSTMENT GRID


MODEL UNDER SALES COMPARISON METHOD

66.1. The adjustment grid model can be adopted mainly for

a) Flats b) Restaurant
c) Cinema theatre d) Agricultural land

66.2. For adjustment grid model, the (minimum) main attributes of the properties are

a) 3 (Location, size, age) b) 4 (Location, size, age, specification)


c) 2 (Location, size) d) 1 (Location)

66.3. In the adjustment grid model, negative weightages are given for

a) Normal specification b) Superior specification


c) Inferior specification d) Posh specification

66.4. In the adjustment grid model, positive weightages are given for

a) Substandard specification b) Inferior specification


c) Normal specification d) Superior specification

66.5. Under the adjustment grid model, the rate adjustment is mainly to be given first
for

a) Time b) Location c) Size d) Age

66.6. The factors to be considered under Hedonic Pricing Model (Adhoc Comparison
Technique) are

a) Size, Time, Location b) Size, Time, Location, Age


c) Size, Time d) Size

* * *
Answers :
66.1 - a 66.5 - a
66.2 - b 66.6 - b
66.3 - c
66.4 - d
71

Q67. MARKET APPROACH TO VALUE - LAND CHARACTERISTICS AND ITS


EFFECT ON LAND VALUES

67.1. The value of land mainly depends on

a) Economic, technical, social and legal aspects


b) Size, encumbrance, climate, building bye-laws
c) Topography, prestige aspect, view aspect, Tenure
d) Community aspect, vaastu aspect, landlocked concept, environment aspect

67.2. If the land is situated in a business area of the town, the rental value is

a) Less b) More
c) Double d) Stable

67.3. The important criteria for the selection of the best land in the town is

a) More width of road b) Commercial zone


c) Location, location & location d) Residential zone

67.4. If the land is situated where there are no infrastructure facilities like water, power,
drainage, the value is

a) Half the value of other plot b) More


c) Stable d) Less

67.5. When the land is large, the land rate is comparatively

a) Lower b) Higher
c) Stable d) Unstable

67.6. Wider frontage plot in a commercial zone is valued

a) Less b) More
c) Constant d) No value
72

67.7. Return frontage is known as

a) Tandem plot b) Intermittent plot


c) Corner plot d) Middle plot

67.8. The plots having road in the front and rear side is called as

a) Single frontage b) Return frontage


c) Multi frontage d) Double frontage

67.9. When the width of road is more, normally the value is said to be

a) More b) Less
c) Double the normal value d) Half the normal value

67.10. The concept of floating FSI or transferable right for development of the land is

a) Building bye-laws b) TDR aspect


c) Tenure aspect d) Environment aspect

* * *

Answers :
67.1 - a 67.5 - a 67.9 - a
67.2 - b 67.6 - b 67.10 - b
67.3 - c 67.7 - c
67.4 - d 67.8 - d
73

Q68. MARKET APPROACH TO VALUE - HYPOTHETICAL PLOTTING SCHEME FOR


VALUE OF LARGE SIZE LAND - RESIDUAL TECHNIQUE AND OTHER
DEVELOPMENT METHODS - VALUATION FOR JOINT VENTURE DEVELOPMENT
OF PROPERTY

68.1. Hypothetical plotting scheme is mainly used to value

a) Large open land b) Flats


c) Industries d) Residential building

68.2. Hypothetical building scheme is normally used for

a) Large utilised property b) Under utilised property


with more FSI with less FSI
c) Industries d) Flats

68.3. Sale comparison method and development method (Residual technique) are the
two main methods under

a) Income approach b) Cost approach


c) Market approach d) Composite rate method

68.4. When total unavailability of sale instances are there, the method useful to find
the land rate is called as

a) Profit method b) Sale comparison method


c) Land & building method d) Development method
(Residual technique)

68.5. By using this method, land value can be arrived at. The method is called as

a) Development method b) Profit method


c) Composite rate method d) Income capitalisation method

68.6. Joint venture is mostly applicable for

a) Individual residential building b) Apartment building


c) Cinema theatre d) Hotel
74

68.7. In a joint venture development, the ratio normally represents

a) Landlord : Contractor b) Landlord : Promoter


c) Promoter : Landlord d) Engineer : Landlord

68.8. In a joint venture, the land rate is to be considered based on

a) Average of market & guideline rate b) Guideline rate


c) Whichever is more d) Prevailing market rate

68.9. In a joint venture agreement, the building is rate is based on

a) The proposed specifications b) State PWD rates


of building
c) CPWD rates d) Rates adopted by the
registrar’s office

68.10. In a joint venture arrangement, the share of the landlord will be more if the land rate is

a) Less b) High
c) Average d) Adopted based on Guideline rate

68.11. In a joint venture agreement, the share of the builder is less when

a) Building rate is equal to the land component


b) Building rate is more than the land component
c) Building rate is less than the land component
d) The FSI is less

68.12. In a joint venture agreement, the two main factors which decide the ratio normally is

a) Demand b) Plot coverage


c) Approved plan d) Land rate & FSI

* * *
Answers :
68.1 - a 68.5 - a 68.9 - a
68.2 - b 68.6 - b 68.10 - b
68.3 - c 68.7 - c 68.11 - c
68.4 - d 68.8 - d 68.12 - d
75

Q69. COST APPROACH TO VALUE - METHODS OF COST ESTIMATES FOR


BUILDINGS

69.1. The actual cost of construction of a building in terms of money as actually spent
by the owner is called as

a) Historic cost b) Original cost


c) Replacement cost d) Reproduction cost

69.2. The cost of building in the year 1990 is 5 lakhs to Mr. A; If it is sold to Mr. B for
Rs. 10 lakhs in the year 2000, the original cost to Mr. B is

a) Rs. 5 lakhs b) Rs. 10 lakhs


c) Rs. 7.5 lakhs d) Rs. 15 lakhs

69.3. The depreciated replacement cost of the building to the new owner is called

a) Price b) Historic cost


c) Original cost d) Value

69.4. The term - loss in money value due to age, usage, wear and tear is called as

a) Scrap b) Appreciation
c) Salvage d) Depreciation

69.5. To arrive at the book value, the essential data is

a) Historic cost b) Replacement cost


c) Original cost d) Depreciated value

69.6. The area of the building multiplied by unit rate of building is called as

a) Book value method b) Flat rate method


c) Cost index method d) Detailed quantity method
76

69.7. The method devised by CPWD to work out the cost estimate for the building
proposed to be constructed is called as

a) Detailed quantity method b) Flat rate method


c) Cost index method d) Book value method

69.8. Latest CPWD plinth area rates are as on

a) 01.01.1955 b) 01.01.1992
c) 01.10.1976 d) 01.10.2012

69.9. The most reliable and accurate method to estimate the cost is

a) Detailed estimate method b) Cost index method


c) Flat rate method d) Reinstatement method

69.10. This method is videly used to file claim under land acquisition act and for
insurance companies

a) Detailed estimate method b) Reinstatement method


c) Flat rate method d) Cost index method

* * *

Answers :
69.1 - a 69.5 - a 69.9 - a
69.2 - b 69.6 - b 69.10 - b
69.3 - c 69.7 - c
69.4 - d 69.8 - d
77

Q70. COST APPROACH TO VALUE - LIFE OF BUILDING - ECONOMIC / PHYSICAL


/ LEGAL

70.1. The actual service life (planned life) of the building is called

a) Economic life b) Physical life


c) Life due to legal constrains d) Life due to obsolescence

70.2. Economic life of a RCC roofed load bearing building is normally considered as

a) 100 Years b) 50 - 60 Years


c) 40 Years d) 25 Years

70.3. The actual survival life of the building before collapse is called as

a) Life due to legal constrains b) Economic life


c) Physical life d) Life due to obsolescence

70.4. The life of building becomes obsolete due to changes in life style of society. This
is called

a) Life due to legal constrains b) Economy life


c) Physical life d) Obsolescence life

70.5. A building is erected in a lease land which has 30 year lease period. The valuer
has to adopt the life of the building as 30 years only. This is called

a) Life due to legal constrains b) Obsolescence life


c) Physical life d) Economic life

70.6. Excess wear and tear

a) Increases the life b) Reduces the life


c) Maintain the life d) None
78

70.7. The economic life of RCC framed structure is normally

a) 30 - 40 Years b) 40 - 60 Years
c) 60 - 80 Years d) 20 - 30 Years

70.8. For the purpose of calculating depreciation, a valuer normally adopts

a) Physical life b) Obsolescence life


c) Life due to legal constrains d) Economic life

70.9. High rise RCC framed buildings which are not designed for earthquake
resistance features like shear walls may have

a) Reduced life b) Increase the life


c) Constant life d) None

70.10. Materials used in the construction, workmanship, soil conditions, weather


conditions, etc., determines

a) Depreciation of the building b) Total life of building


c) Present value of the building d) Historic cost

* * *

Answers :
70.1 - a 70.5 - a 70.9 - a
70.2 - b 70.6 - b 70.10 - b
70.3 - c 70.7 - c
70.4 - d 70.8 - d
79

Q71. MARKET APPROACH TO VALUE - VARIOUS METHODS OF COMPUTATION OF


DEPRECIATION, FUNCTIONAL, TECHNOLOGICAL AND ECONOMIC
OBSOLESCENCE.

71.1. Decrease in value of the property through wear, deterioration and obsolescence is
called as

a) Depreciation b) Appreciation
c) Functional obsolescence d) Technical obsolescence

71.2. The normal wear and tear which occurs due to the usage of the asset is called as

a) Economic depreciation b) Physical depreciation


c) Functional obsolescence d) Technological obsolescence

71.3. A residential building existing on the plot which is placed in commercial zone is a
glaring example of

a) Technological obsolescence b) Functional obsolescence


c) Economic obsolescence d) Physical obsolescence

71.4. Dilapidation of building or heavy structural repair is also

a) Physical obsolescence b) Functional obsolescence


c) Technological obsolescence d) Economic obsolescence

71.5. Many palaces are converted into Hotels. This is due to

a) Functional obsolescence b) Economic obsolescence


c) Physical obsolescence d) Technological obsolescence

71.6. A computer may become obsolete within two or three years due to

a) Physical obsolescence b) Functional obsolescence


c) Technological obsolescence d) Economic obsolescence

71.7. Old load bearing structure with more thick walls is an example for

a) Physical obsolescence b) Economic obsolescence


c) Technological obsolescence d) Functional obsolescence
80

71.8. The method of depreciation widely used by Chartered Accountants for Taxation
purpose is called as

a) Sinking fund method b) Straight line method


c) Constant percentage method d) Written down value method

71.9. Statutory depreciation method is also called as

a) Written down method b) Constant percentage method


c) Straight line method d) Sinking fund method

71.10. The method where equal percentage of depreciation is allowed on its original cost
(Replacement cost) for each year of life is called as

a) Written down method b) Straight line method


c) Constant percentage method d) Sinking fund method

71.11. In this method, constant rate of depreciation is first assumed and depreciation is
calculated by applying formula.

a) Written down method b) Straight line method


c) Constant percentage method d) Sinking fund method

71.12. Optimum economic benefit of the land and building is not achieved. This is called as

a) Depreciation due to functional b) Depreciation due to physical


obsolescence obsolescence
c) Depreciation due to technological d) Economic obsolescence
obsolescence

71.13. An asset suffering from severe economic or functional obsolescence may have

a) High depreciation b) High appreciation


c) Normal depreciation d) Normal appreciation

* * *

Answers :
71.1 - a 71.5 - a 71.9 - a 71.13 - a
71.2 - b 71.6 - b 71.10 - b
71.3 - c 71.7 - c 71.11 - c
71.4 - d 71.8 - d 71.12 - d
81

Q72. COST APPROACH TO VALUE - REPRODUCTION COST / REPLACEMENT


COST, DEPRECIATED REPLACEMENT COST (DRC) WORKING, ADOPTING
DRC AS VALUE SUBJECT TO DEMAND AND SUPPLY ASPECT.

72.1. Cost of creating a new building having identical utility and performing similar
functions as being performed by the existing old asset is called as

a) Replacement cost b) Reproduction cost


c) Original cost d) Historic cost

72.2. It is cost to produce exactly similar asset (i.e. exact replica - mirror image) of the
old existing asset to be valued. It has same utility, functions, similar specification.
It is called as

a) Replacement cost b) Reproduction cost


c) Original cost d) Historic cost

72.3. Current replacement cost minus depreciation is called as

a) Scrap value b) Depreciation value


c) Depreciated cost (Net present value) d) Salvage cost

72.4. To calculate depreciated replacement cost, the value to be considered is only

a) Historic cost b) Book value


c) Written down value d) Replacement cost

72.5. Replacement cost and reproduction cost is one and the same.

a) Not same b) Same


c) May be same d) May not be same

72.6. For building insurance, only this cost is to be used

a) Replacement cost b) Reproduction cost


c) Historic cost d) Book cost
82

72.7. In land acquisition matters, if any ornamental building is acquired, this cost to be
considered is only

a) Book cost b) Net present value


c) Depreciated reproduction cost d) Written down cost

72.8. Net present value is used to indicate

a) Historic cost of the asset b) Written down value of the asset


c) Book value of the asset d) Current value of the asset

72.9. Net present value is also called as

a) Depreciated cost b) Depreciation cost


c) Book cost d) Historic value

72.10. Market value of the building is estimated by using the formula Net Present Value
= Replacement cost - Depreciation. This approach is called as

a) Income approach b) Cost approach


c) Market approach d) Composite rate method

* * *

Answers :
72.1 - a 72.5 - a 72.9 - a
72.2 - b 72.6 - b 72.10 - b
72.3 - c 72.7 - c
72.4 - d 72.8 - d
83

Q73. LAND VALUE BY MARKET APPROACH AND BUILDING VALUE BY COST


ESTIMATION METHOD FOR OWNER OCCUPIED BUNGALOWS, FACTORIES,
PUBLIC BUILDINGS.

73.1. This method is not helpful in estimating values of premises which are rented

a) Cost approach b) Market approach


c) Income approach d) Composite rate method

73.2. In this method, land is separately valued and building is separately valued

a) Market approach b) Cost approach


c) Income approach d) Composite rate method

73.3. Land married with structure does not fetch same price in market as a

a) Recess land b) Tandem plot


c) Virgin land d) Belt shaped plot

73.4. In the areas where much sale transactions have not taken place, land value
estimation is

a) Not at all difficult b) Very easy


c) Not difficult d) Very difficult

* * *

Answers :
73.1 - a 73.3 - c
73.2 - b 73.4 - d
84

h. VALUATION OF REAL ESTATE


Q74

74.1. Forced sale value and orderly liquidation value are the two classifications under

a) Liquidation b) Market value


c) Fair market value d) Open market value

74.2. When the property is to be partitioned according to predetermined shares, the


basis of valuation should always be

a) Guideline value b) Market value


c) As per rent control Act d) As per PWD rates

74.3. Forced sale value and realisable value are one and the same?

a) Same b) Not related


c) Not same d) Depends on purpose

74.4. Value of brick field is usually calculated on the basis of .......... years profit for the
lessee’s interest

a) 20 years b) 10 years
c) 30 years d) 40 years

74.5. When doing valuation of fisheries, the year’s purchase for such fisheries normally
can be

a) 20 to 30 on net profit b) 10 to 12 on gross income


c) 10 to 12 on net profit d) 20 to 30 on gross income

74.6. Collecting stamp duty for the deeds registered in the registrar’s office is

a) State subject b) Central subject


c) Quasi government subject d) CPWD subject
85

74.7. The procedure adopted by many states for fixing guideline rate is

a) Same b) Not uniform


c) Can be same d) Need not be the same

74.8. Market value by adopting guideline rate fixed by local registrar’s office and by
adopting prevailing market rate - are they one and same?

a) Can be same b) Same


c) Not same d) No comparison

74.9. An applicant has proposed to construct a residential building and he applies loan
from a bank. The bank releases loan amount in installments. After the completion
of the building, the bank asks the valuer to certify the

a) Cost b) Price
c) Value d) Market value

74.10. A borrower is pledging his property as a collateral security to a bank. The bank
directs the valuer to certify the

a) Market value b) Distressed rate value


c) Auction value d) Salvage value

74.11. The borrower’s account becomes NPA. The bank has taken symbolic possession.
Under the SARFAESI act, the bank directs the value to certify the

a) Reserve value b) Upset value


c) Auction value d) Salvage value

74.12. When the pledged property is to be auctioned, the upset price is to be fixed by the

a) Advocate b) Valuer
c) Bank d) Owner of the property

74.13. Debt Recovery Tribunal and SARFAESI are one and the same?

a) Not related b) They are same

c) They may be the same on d) They are not the same


few occasions
86

74.14. The reserve price (upset price) will be fixed based on the

a) Auction department b) Advice from the advocate


c) Request from the borrower d) Value certified by the valuer

74.15. Reserve price will be

a) As suggested by the b) More than the value certified by the


advocate valuer

c) Same value as certified d) Less than the value certified by the


by the valuer valuer

74.16. In the probate petition of the legal heirs, a valuation report is required to

a) Decide the court fees b) To know the cost


c) To know the liquidation value d) To know the depreciated
value

74.17. When a person dies after leaving a WILL, it is necessary for the executor of the
WILL to obtain a ............. from the high court.

a) Clearance b) Probate
c) No objection d) Endorsement

74.18. Even if the WILL is signed by the deceased, it will be declared null and void if it is
not countersigned by

a) Wife b) Sons
c) Daughters d) Two witnesses

74.19. The value of the property for probate purpose is to be determined as on

a) The date of filing probate b) The date of death of the deceased


petition to the court

c) The date of signing the d) The date of sign by the witnesses


will by the deceased
87

74.20. Courts have upheld that valuation of property for probate purposes and court fees
for probate cases should be estimated as per the provisions of

a) Fair rent act b) Income tax rules


c) Wealth tax rules d) Principles of valuation

74.21. The process of officially proving a WILL to be valid is called

a) Probate b) Notarised
c) Testimony d) Endorsement

74.22. Certificate issued by a court that the WILL of a deceased is legally valid and that
the executors appointed under the WILL are authorised to administer. This process
is called

a) Endorsement b) Notarised
c) Testimony d) Probate

74.23. Probate for WILL is required to be carried out

a) Only in Calcutta, Madras, b) Only in Delhi, Calcutta, Madras,


Mumbai Mumbai
c) All the places in the country d) It is not required now

* * *

Answers :

74.1 - a 74.5 - c 74.9 - a 74.13 - d 74.17 - b


74.2 - b 74.6 - a 74.10 - a 74.14 - d 74.18 - d
74.3 - c 74.7 - b 74.11 - c 74.15 - d 74.19 - a
74.4 - b 74.8 - c 74.12 - c 74.16 - a 74.20 - c

74.21 - a
74.22 - d
74.23 - a
88

Q75. MORTGAGE

75.1. A person who mortgages his property in lieu of security payment or loan is called as

a) Mortgagor b) Mortgagee
c) Borrower d) Applicant

75.2. The person who advances funds or loan amount against the security offered by
the property owner is called as

a) Mortgagee b) Mortgagor
c) Applicant d) Borrower

75.3. Mortgagor does not give possession of the property to the mortgagee but he gives
personal undertaking to repay loan amount with interest. This mortgages is called
as

a) Usufructuary mortgage b) Mortgage by conditional sale


c) Equitable mortgage d) Simple mortgage

75.4. The mortgage deed provides for conditional sale of the property by the mortgagor
to the mortgagee. This mortgage is called as

a) Usufructuary mortgage b) Simple mortgage


c) Equitable mortgage d) Mortgage by conditional sale

75.5. The Mortgagor delivers possession of the property to the mortgagee. The
mortgagee receives rents and profits from the property and retains possession till
the full loan money is paid. This mortgage is called as

a) Equitable mortgage b) Mortgage by conditional sale


c) Usufructuary mortgage d) Simple mortgage

75.6. The Mortgagor delivers to the mortgagee title deeds with intent to create a security
thereon. This mortgage is called as

a) Mortgage by conditional sale b) Simple mortgage


c) Equitable mortgage d) Usufructuary mortgage
89

75.7. If an asset is created through finance obtained from the bank, such security is
known as

a) Collateral security b) Primary security


c) Preliminary security d) Secondary security

75.8. The additional security (apart from the primary security) pledged to the bank is
called as

a) Primary security b) Collateral security


c) Preliminary security d) Secondary security

75.9. For the construction of a residential building, an applicant obtains loan from a bank.
On completion of the construction, the bank directs the panel valuer to certify

a) The cost on completion b) Market value


c) Forced sale value d) Auction value

75.10. In the case of collateral securities, the bank directs the valuer to certify

a) Market value b) Cost of the property


c) Written down value d) Book value

75.11. If there is a default of repayment of loan by the borrower, the account becomes
N.P.A. The bank decides to auction the property to recover the loan. In this case,
the bank directs the valuer to certify

a) Cost of the property b) Auction value


c) Written down value d) Book value

75.12. Valuers prefer to estimate forced sale value a certain percentage less than the fair
market value. This percentage normally is

a) 30% b) 15%
c) 40% d) 50%

75.13. The term in common parlance would mean net money likely to be realised after the
sale of the property is

a) Auction value b) Market value


c) Forced sale value d) Realisable value
90

75.14. Valuers prefer to estimate auction value a certain percentage less than the fair
market value. This percentage normally is

a) 5% b) 15%
c) 10% d) 30%

75.15. An estimate of minimum price likely to be offered by the bidders in the public
auction of the mortgaged property is called as

a) Realisable value b) Market value


c) Reserve price d) Forced sale value

75.16. Before proceeding with valuation, is it the duty of a valuer to ask for documents like
title deeds, plan, etc.

a) It is the bank’s duty to process b) It is not correct


the document

c) Yes, it is his duty d) It is the advocate’s job to look


into it

75.17. Valuer is not a fortune teller and he can at the most foresee market conditions only
for

a) 1 year b) 5 years
c) 7 years d) 10 years

75.18. Is there any hard and fast rule that auction value should be always lower than the
fair market value?

a) No, there is no rule b) Yes, it is always lower


c) Not related d) It is always higher

75.19. Authorised officer under this act has more powers than a court receiver. He can
take not only symbolic possession of the property but can also take physical
possession of the property from the tenants also. This act is known as

a) Transfer of property Act b) DRT


c) LARAR Act d) SARFAESI Act 2002
91

75.20. A temple is a

a) Property that be sold easily b) Marketable property


c) Marketable but not saleabled) Non marketable property

75.21. To ascertain the rent for the premises proposed to be occupied by Central
government department, who is ascertaining the rent for them?

a) CPWD b) State Public Works Department


c) Panel engineer d) Local authority

75.22. Which rate is being adopted by CPWD at the time of fixing rent for the buildings
proposed to be occupied by the Central government departments?

a) Plinth area rates prescribed by b) Plinth area rates prescribed by


CPWD State PWD

c) Plinth area rates prescribed by d) Prevailing plinth area rates


Supt. Engineer of the district

75.23. While fixing rent for the premises, CPWD follows the method purely based on
recognised principles of valuation?

a) Yes b) Also on the basis of comparative


study of prevailing rent in that
locality
c) They compare the rent as per d) They fix arbitrarily
the local government rent

75.24. In the case of metropolitan cities, the percentage rate of return to fix rent for
non-residential purposes adopted by CPWD is

a) 8% b) 10% c) 6% d) 4%

75.25. In the case of mofussil towns the percentage rate of return to fix rent for
non-residential purposes adopted by CPWD is

a) 5% b) 7% c) 9% d) 3%

75.26. In the case of metropolitan cities, the percentage rate of return to fix rent for
residential purposes adopted by CPWD is
92

a) 5.5% b) 7.5% c) 9.5% d) 3.5%

75.27. In the case of mofussil towns, the percentage of rate of return to fix rent for
residential purposes adopted by CPWD is

a) 2.5% b) 6.5% c) 4.5% d) 8.5%

75.28. To ascertain the rent based on the recognised principles, which method of
depreciation is adopted by CPWD?

a) Sinking fund method b) Constant percentage method


c) Linear method d) Straight line method

75.29. The cost inflation index is prescribed by

a) CBDT b) CPWD
c) Valuation cell d) Public works department

75.30. When a property is sold after 01.04.2017, the fair market value of the property (for
the purpose of ascertaining capital gain) is to be ascertained as on

a) 01.04.2001 b) 01.04.1981 c) 01.04.1991 d) 01.04.1971

75.31. When a property is sold on 31.03.2017, the fair market value of the property (for
the purpose of computing the capital gains) is to be ascertained as on

a) 01.04.2001 b) 01.04.1981 c) 01.04.1991 d) 01.04.1971

75.32. If the FMV is to be ascertained as on 01.04.1981, then cost inflation index for
1981 - 82 is

a) 110 b) 100 c) 120 d) 130

75.33. If the FMV is to be ascertained as on 01.04.2001, then the cost inflation index for
2001 - 02 is

a) 520 b) 426 c) 100 d) 626

75.34. Any profit or gains arising from the transfer of a capital asset effected from the
previous year is called as
93

a) Net gain b) Capital loss c) Capital gain d) Gross gain

75.35. The capital gain tax percentage for an individual is

a) 50% b) 30% c) 40% d) 20%

75.36. The capital gain tax percentage for association of persons (AOP) is

a) 50% b) 20% c) 40% d) 30%

75.37. The capital gain tax percentage for companies is

a) 50% b) 20% c) 30% d) 40%

75.38. Cost inflation index and cost index are one and the same?

a) They are not same b) They are same


c) May be same d) May not be same

75.39. A capital asset held by an assessee for not more than 24 months immediately
proceeding the date of its transfer is called as

a) Medium term capital asset b) Long term capital asset


c) Short term capital asset d) Narrow term capital asset

75.40. Sale consideration minus indexed cost of acquisition minus indexed cost of
improvement is

a) Capital loss for seller b) Capital gain for purchaser


c) Taxable capital gain for seller d) Income in the hands of the seller

* * *

Answers :

75.1 - a 75.9 - a 75.17 - a 75.25 - c 75.33 - c


75.2 - a 75.10 - a 75.18 - a 75.26 - c 75.34 - c
75.3 - d 75.11 - b 75.19 - d 75.27 - d 75.35 - d
75.4 - d 75.12 - b 75.20 - d 75.28 - d 75.36 - d

75.5 - c 75.13 - d 75.21 - a 75.29 - a 75.37 - d


75.6 - c 75.14 - d 75.22 - a 75.30 - a 75.38 - a
75.7 - b 75.15 - c 75.23 - b 75.31 - b 75.39 - c
75.8 - b 75.16 - c 75.24 - b 75.32 - b 75.40 - c
94

Q 76. ASSET VALUATION UNDER SARFAESI ACT 2002

76.1. SARFAESI ACT was enacted based on the recommendation from

a) Narasimhan committee b) Rajendran committee


c) Rajeswaran committee d) Thillainayagam committee

76.2. In which year SARFAESI ACT was enacted?

a) 2002 b) 1993 c) 2012 d) 1992

76.3. Debt Recovery Tribunal and SARFAESI are one and the same

a) They are not same b) They are same


c) They may be same d) Not relevant to each other

76.4. NPA means

a) Non - productive account b) Non - performing asset


c) Non - productive asset d) No performance asset

76.5. Under the SARFAESI Act, the powers of taking possession and sell them are vested
with

a) Lawyer b) Banks c) Court d) Police

76.6. SARFAESI act consists of chapters

a) 5 b) 6 c) 4 d) 3

76.7. Regulations of securitisation and Reconstruction of Financial Assets of banks and


financial institution is covered in chapter

a) Chapter V b) Chapter III c) Chapter IV d) Chapter II


95

76.8. Enforcement of security interest is covered in

a) Chapter VI b) Chapter IV c) Chapter V d) Chapter III

76.9. Offences and penalties are covered in chapter

a) Chapter II b) Chapter III c) Chapter I d) Chapter V

76.10. Under which rule, ‘approved value’ is dealt with?

a) Rule 8 (5) b) Rule 5 c) Rule 2 d d) Rule 12

76.11. Under which rule, valuation of immovable properties are dealt with

a) Rule 8 (5) b) Rule 2 d c) Rule 5 d) Rule 12

76.12. Under which rule, sale of immovable secured assets are dealt with

a) Rule 5 b) Rule 2 d c) Rule 8 (5) d) Rule 12

76.13. Which are the properties that cannot be considered as security under SARFAESI
act

a) Aircraft, ship, agricultural lands b) Plot, building, petrol bunk


c) Commercial ship, flat d) Cinema theatre

76.14. Who is the authority to empanel the approved valuer under SAFAESI act (as on
2017)

a) Board of directors of the bank b) IBA


c) IBBI d) MCA

76.15. LARAR Act deals with

a) Land acquisition b) Bank loan recovery


c) Licence and Rent control d) Debt recovery

76.16. LARAR Act was implemented on

a) 2013 b) 1993 c) 2003 d) 2013


96

76.17. LARAR Act is under the purviews of

a) Ministry of law and justice b) Ministry of finance


c) Ministry of social affairs d) Ministry for company act

76.18. The provisions of this Act relating to land acquisition, compensation, rehabilitation
and settlement shall apply

a) When the government acquires b) When the government takes the


land for its own use land on lease basis

c) When the government takes the d) When any private party purchases
land on monthly rental basis the land

76.19. The number of chapters involved in LARAR Act is

a) 12 b) 13 c) 11 d) 10

76.20. Chapter I of LARAR Act deals with

a) Determination of social impacts b) Preliminary


and public purpose

c) Social provision to safeguard d) Notification and acquisition


food security

76.21. Chapter 5 of LARAR Act deals with

a) Procedure and manner of b) Rehabilitation and Resettlement


Rehabilitation and resettlement award

c) National monitoring committee d) Establishment of land acquisition


of rehabilitation

76.22. Chapter 9 of LARAR Act deals with

a) Payment b) Apportionment of compensation


c) Temporary occupation of land d) Offences and penalties
97

76.23. The collector shall determine the market value of land

a) As per the oral enquiry b) As per the prevailing market rate


c) As per the stamp duty value d) As per the income capitalisation

76.24. The date of determination of market value shall be the

a) Date of processing b) Date of award


c) Date of notification d) Date of appeal

76.25. The collector having determined the total compensation to be paid, shall, to arrive
at the final award, impose a ‘solatium’ amount equivalent to ................% of the
compensation amount

a) 50% b) 30% c) 100% d) 10%

76.26. The collector in determining the market value of building and other immovable
property used the services of a

a) Surveyor b) Architect
c) Competent engineer d) Draftsman

76.27. The collector for the purpose of determining the value of trees and plants attached
to the land acquired uses the services of

a) Land surveyor b) Plant & Machinery valuer

c) Civil engineer d) Experienced persons in the


field of agriculture

* * *

Answers :

76.1 - a 76.9 - d 76.17 - a 76.25 - c


76.2 - a 76.10 - c 76.18 - a 76.26 - c
76.3 - a 76.11 - c 76.19 - b 76.27 - d
76.4 - b 76.12 - c 76.20 - b

76.5 - b 76.13 - a 76.21 - b


76.6 - b 76.14 - a 76.22 - b
76.7 - d 76.15 - a 76.23 - c
76.8 - d 76.16 - a 76.24 - c
98

Q77. CONCEPT OF TRANSFERABLE DEVELOPMENT RIGHTS (TDR), CONCEPT OF


TIME SHARE INTEREST IN REAL PROPERTY, VALUATION OF TDR, TIME SHARE
INTEREST AND EASEMENT RIGHTS

77.1. The right granted by a landowner to an owner of another property for the non-exclusive
use of a portion of the land of a specific purpose or enjoyment of certain rights, is
called as

a) Easement rights b) Transferable development rights


c) Time share interests d) Leasehold rights

77.2. When a family is partitioned and its members agree amongst themselves that in case
any of the parties wish to sell even at a future date, they must do so to one of the other
members. This is called as

a) Self-imposed easement b) Legal easement


c) Time share interest d) Transferable development rights

77.3. When a co-operative society admits only vegetarians as its members, it is called as

a) Legal easement b) Self imposed easement


c) Time share interest d) Transferable development rights

77.4. To fix electric lines over certain lands or the right to construct underground drains over
certain lands or right to dispose water through adjoining property, right to use water
from a reservoir constructed over another property is called as

a) Self imposed easement b) Legal easement


c) Time share interest d) Transferable development rights

77.5. Easements are not attached to owners, but they are attached only to

a) Local authorities b) Tenants


c) Properties d) Owner

77.6. Easements can be created or extinguished by owners and cannot be created by

a) Advocate b) Neighbours
c) Lessees / Tenants d) Municipal authorities
99

77.7. When the benefit of an easement is not exercised over a long period of time, then the
easement may be considered as

a) Automatic renewal b) Full right


c) Continue to enjoy right d) Extinguished or released

77.8. Ownership of a particular property (say a flat) is held for a specified period of time
during a year - is called as

a) Transferable development right b) Temporary ownership


c) limited ownership d) Time share property

77.9. The concept of time sharing was initially introduced in the mid 1960s at a resort in the

a) French Alps b) Suiss Alps c) Australia d) New Zealand

77.10. In India, the concept of time sharing started in the year

a) 1980 b) 1990 c) 1970 d) 1960

77.11. The promoter runs the business and lets to the owner to use the premises, as per the
agreement, for the specific time of the year. The agreement may be for 20 years.
Once the period is over, the entire property reverts to the original developers. This
type of arrangement is called as

a) Form of rent on a long term basis b) Form of lease on a long term basis
c) Form of rent on a short term basis d) Form of lease on perpetuity

77.12. Time share concept is advantageous to

a) Developer b) Developer and purchaser


c) Purchaser d) None

77.13. In a hill station, this period of time share will be more valuable comparatively to other
period

a) December b) January c) May d) October

77.14. Transfer of Development Rights was initially developed in

a) Australia b) U.K. c) USA d) Canada


100

77.15. If a person is denied the right to use his land in a particular way, he should be offered
alternative facilities to ulitise these rights in some other way so that a public purpose
is served and at the same time the interest of the land owner is preserved. It is called
as

a) Leasehold Rights b) Easement Rights


c) Time Share Rights d) Transfer of Development Rights

77.16. TDR is a concept in

a) Forest management b) Air management


c) Water management d) Land management

77.17. TDR scheme was first started in the city of

a) Bombay b) Kolkatta c) Chennai d) Delhi

77.18. Transferring the building potential from one plot (originating plot) to some other site
(receiving plot) under certain terms and conditions is called

a) Transfer of Development Rights b) Time Share Rights


c) Easement Rights d) Leasehold Rights

77.19. One of the essential conditions of the TDR is both the originating and receiving plot
must fall within the territorial jurisdiction of the

a) Different locality b) Same local authority


c) Anywhere in the country d) Neighbouring local authority

77.20. The Development Right certificate can be sold to

a) Neighbour b) Any other person


c) None d) Government only

77.21. The DRC is issued by

a) Magistrate b) Collector
c) Commissioner of the local authority d) Chief Engineer, PWD

77.22. For the use of TDR, areas near railway track, highways and coastal areas are
101

a) Can be included / excluded depending b) Included


upon the decision of local authority
c) Excluded d) None

77.23. The rights under “Development Right Certificate” can

a) Be tradable and not transferable b) Not be transferred


c) Be transferred and not tradable d) Be transferable and tradable

77.24. TDR originated out of free surrender of plot reserved for public purposes like garden,
school, play ground, etc. is called as

a) Heritage TDR b) Road TDR


c) Slum TDR d) General TDR

77.25. TDR originated out of free surrender of plot or portion of plot affected by new road or
road widening of existing municipal road, is called as

a) Road TDR b) General TDR


c) Slum TDR d) Heritage TDR

77.26. TDR granted to developers, in lieu of carrying out redevelopment work of ‘slum area’
plot as per government policy & norms is called

a) Slum TDR b) General TDR


c) Road TDR d) Heritage TDR

77.27. Owner of plot on which ‘Heritage’ building exists and when FSI of such plot is
underutilised and when such owner is prevented to use such unutilised FSI on his
land, this TDR is granted to owner which corresponds to unulitised FSI area in the
plot. Such TDR is called as

a) General TDR b) Heritage TDR


c) Slum TDR d) Road TDR

77.28. If originating TDR is from residential zone, it cannot be used on plot in industrial or
commercial zone and vice versa. This is

a) False b) True
c) Not applicable d) No such rule
102

77.29. The definition for easement is given in

a) Section 4 of SARFAESI b) Section 4 of Transfer of Property


Act Act
c) Section 4 of Indian d) Section 4 of Debt Recovery
Easement Act Tribunal Act

77.30. A contract for leave and licence is a right of ‘Licence’ as per section 52 of

a) Sarfaesi Act b) Transfer of property Act


c) Indian Easement Act d) DRT Act

* * *

Q78. STUDY OF INDIAN ACCOUNTING STANDARDS (IND AS) AS APPLICABLE TO


VALUATION OF REAL ESTATE. STUDY OF INTERNATIONAL VALUATION
STANDARDS (IVS) AS APPLICABLE TO VALUATION OF REAL ESTATE

78.1. “Market value is the estimated amount for which a property should exchange on the
date of valuation between a willing buyer and a willing seller in an arm’s length
transaction after proper marketing where in parties had each acted knowledgeably
predently and without compulsion”. This is as per which standard?

a) Indain Standard b) American Standard


c) British Standard d) International Valuation Standard

* * *

Answers :

77.1 - a 77.9 - a 77.17 - a 77.25 - a


77.2 - a 77.10 - a 77.18 - a 77.26 - a
77.3 - b 77.11 - b 77.19 - b 77.27 - b
77.4 - b 77.12 - b 77.20 - b 77.28 - b

77.5 - c 77.13 - c 77.21 - c 77.29 - c


77.6 - c 77.14 - c 77.22 - c 77.30 - c
77.7 - d 77.15 - d 77.23 - d
77.8 - d 77.16 - d 77.24 - d 78.1 - d
103

Q79,80. IMPORTANT CASE LAWS ON PRINCIPLES OF VALUATION OF REAL ESTATE

Sl.no. CASE NAME JUDGEMENT

79.1. K.P. VARHESE vs The estimate of FMV vary form individual to


INCOME TAX OFFICER individual. The difference between one honest
valuation and another may range upto 15%.

79.2. GOLD COAST TRUST vs Valuation is not an exact science. Mathematical


HUMPHRAY certainty is not demanded, nor indeed is it
possible.

79.3. RUSTAM C. COOPER vs Property means the highest right a man can have
UNION OF INDIA to anything, it includes ownership, estates and
interest in coporeal things and also rights such as
trademarks, copy rights, patents, etc. Court has
included all types of property viz tangible.

79.4. HAYS WILL TRUST vs The most likely price, in the absence of
HAYS AND OTHERS consultation between the valuers representing
conflicting interests, would presumably be the
mean price.

79.5. V.C. RAMACHANDRAN vs If there are more than one valuation for the same
COMMISSIONER OF property, the one which is reasonable and nearer
WEALTH TAX, to the correct market value, having due regard to
KARNATAKA HC all the relevant facts and circumstances of the case
alone should be accepted.

79.6. SUBH KARAN CHOUDHURY Valuation of fully tenanted property should be made
vs INSPECTING ASSISTANT on the basis of capitalization of rental method.
COMMISSIONER (IAC)

80.1. WENGER & CO vs DISTRICT Combination of methods (Owner occupied - sale


VALUATION OFFICER - comparison & tenanted portion - capitalization
DELHI HC method) - adopted by DVO and his approach is
not only acceptable but also in accordance with
the principles of evaluation.
104

80.2. SORAB TALATI vs The court approved of investment theory to fix


JOSHEPH MICHEM standard rent of the rent controlled premises.
Return or yield from Gilt Edged Security is the
basis for determining the fair return to the landlord
for his investments in land & buildings.

80.3. COMMISSIONER OF In determining the values of the leasehold interest


WEALTH TAX vs of the assessee in the land for the purpose of
P.N. SIKAND wealth tax assessment, the price would have to
be reduced by 50% of the unearned increase in
the value of the land (which is diverted to the
lessor).

80.4. SPECIAL LAND Value fetched by small extent of land cannot be


ACQUISITION OFFICER adopted for large extent of land. Loss
vs JASTI ROHINI of land for road & park, expenses for development
should be deducted.

80.5. SHUBH RAM & OTHERS vs The value of one sq.yd of undeveloped land is
STATE OF HARYANA not the same as one sq.yd of developed
residential plot. Various expenses are required to
develop such large extent of undeveloped plot. All
these expenditure and factors are standardized
into 33% deduction towards expenses of
development.

80.6. JAWAJI NAGNATHAN vs Basic valuation register prepared and maintained


REVENUE DIVISIONAL for the purpose of collecting stamp duty cannot
OFFICER form foundation to determine the market value of
the property under registration.

80.7. CHIMANLAL The court cannot take into account the award
HARIGOVINDDAS vs passed by the SLAO unless it is produced and
SPECIAL LAND proved before the court. The market value is to be
ACQUISITION OFFICER determined as on date of publication of the
notification under sec 4 of LAA. Plus factors and
minus factors are to be considered while
determining the market value.

* * *
105

Q79, 80 - IMPORTANT CASE LAWS ON PRINCIPLES OF VALUATION

79.1. The difference between one honest valuation and another may range upto 15%

a) Gold Coast Trust Ltd. vs Humphray (1949) 17 ITR 19


b) K.P. Varghese vs ITO (1981) 131 ITR 597 (SC)
c) Rustam C Cooper vs Union of India AIR 1970 SC 564
d) Hays Will Trust vs Hays and Others (1971) 1 WLR 758

79.2. Valuation is not an exact science. Mathematical certainty is not demanded, not
indeed is it possible.

a) K.P. Varghese vs ITO (1981) 131 ITR 597 (SC)


b) Gold Coast Trust Ltd. vs Humphray (1949) 17 ITR 19
c) Rustam C Cooper vs Union of India AIR 1970 SC 564
d) Hays Will Trust vs Hays and Others (1971) 1 WLR 758

79.3. Property includes ownership, estates and interests in corporeal things and also
rights such as trademarks, copyrights, patents.

a) K.P. Varghese vs ITO (1981) 131 ITR 597 (SC)


b) Rustam C Cooper vs Union of India AIR 1970 SC 564
c) Gold Coast Trust Ltd. vs Humphray (1949) 17 ITR 19
d) Hays Will Trust vs Hays and Others (1971) 1 WLR 758

79.4. The most likely price, in the absence of consultation between the valuers representing
conflicting interests, would presumably be the mean price.

a) K.P. Varghese vs ITO (1981) 131 ITR 597 (SC)


b) Hays Will Trust vs Hays and Others (1971) 1 WLR 758
c) Gold Coast Trust Ltd. vs Humphray (1949) 17 ITR 19
d) Rustam C Cooper vs Union of India AIR 1970 SC 564

79.5. If there are more than one valuation of the same property, the one which is
reasonable and nearer to the correct market value, having due regard to all the
relevant facts and circumstances of the case alone should be accepted.
106

a) Subh Karan Choudhury vs IAC (1979) 118 ITR 777 Kolkatta HC (Special Value
/ FMV)
b) V.C. Ramchandran vs CWT (1979) 126 ITR 157 Karnataka HC
c) Wenger & Co. vs DVO (1978) 115 ITR 648 Delhi HC (Combination of
Methods)
d) Sorab Talati vs Josheph Michem Appeal 101 of 1949 - Vol.- 2 of SOC - page
162 (Bombay) (Invest Theory of Rent)

79.6. Valuation of fully tenanted property should be made on the basis of capitalisation of
rental method.

a) V.C. Ramchandran vs CWT (1979) 126 ITR 157 Karnataka HC


b) Subh Karan Choudhury vs IAC (1979) 118 ITR 777 Kolkatta HC (Special Value
/ FMV)
c) Wenger & Co. vs DVO (1978) 115 ITR 648 Delhi HC (Combination of
Methods)
d) Sorab Talati vs Josheph Michem Appeal 101 of 1949 - Vol.- 2 of SOC - page
162 (Bombay) (Invest Theory of Rent)

80.1. For owner occupied portion, the District valuation officer calculated the value on
the basis of what were the rates prevalent for sale of commercial flats in cannaught
place. For the tenanted portion, he capitalised the rental value. The method adopted
by him is acceptable.

a) V.C. Ramchandran vs CWT (1979) 126 ITR 157 Karnataka HC


b) Wenger & Co. vs DVO (1978) 115 ITR 648 Delhi HC (Combination of
Methods)
c) Subh Karan Choudhury vs IAC (1979) 118 ITR 777 Kolkatta HC (Special Value
/ FMV)
d) Sorab Talati vs Josheph Michem Appeal 101 of 1949 - Vol.- 2 of SOC - page
162 (Bombay) (Invest Theory of Rent)

80.2. In this case, the court approved of investment theory in preference to comparable
rent theory to fix standard rent of the rent controlled premises. The court
considered return or yield from Gilt Edged security as the basis.

a) V.C. Ramchandran vs CWT (1979) 126 ITR 157 Karnataka HC


b) Sorab Talati vs Josheph Michem Appeal 101 of 1949 - Vol.- 2 of SOC - page
162 (Bombay) (Invest Theory of Rent)
c) Wenger & Co. vs DVO (1978) 115 ITR 648 Delhi HC (Combination of
Methods)
107

d) Subh Karan Choudhury vs IAC (1979) 118 ITR 777 Kolkatta HC (Special Value
/ FMV)

80.3. Value fetched by sale of small extent land cannot be adopted for large extent land.
Loss of land for road, and park, expenses for development should be deducted.

a) CWT vs P.N. Sikand (1977) 107 ITR 922 SC


b) SLAO (Eluru) vs Jasti Rohini (1995) 1 SCC 717 SC
c) Shubh Ram and Others vs State of Haryana (2010) 1 SCC 444
d) Jawaji Nagnathan vs REV. DIV. Officer (1994) SCC - 4 Page 595 SC

80.4. Basic valuation register for the purpose of collecting stamp duty cannot form a
foundation to determine the market value

a) CWT vs P.N. Sikand (1977) 107 ITR 922 SC


b) Jawaji Nagnathan vs REV. DIV. Officer (1994) SCC - 4 Page 595 SC
c) Shubh Ram and Others vs State of Haryana (2010) 1 SCC 444
d) SLAO (Eluru) vs Jasti Rohini (1995) 1 SCC 717 SC

80.5. The developer who undertakes the development (of a layout) and invests money
for development would also expect a reasonable profit when the plots are sold. All
these expenditure and factors are standardized into 33% deduction towards
expenses of development

a) CWT vs P.N. Sikand (1977) 107 ITR 922 SC


b) Shubh Ram and Others vs State of Haryana (2010) 1 SCC 444
c) SLAO (Eluru) vs Jasti Rohini (1995) 1 SCC 717 SC
d) Jawaji Nagnathan vs REV. DIV. Officer (1994) SCC - 4 Page 595 SC

80.6. Basic valuation register prepared and maintained for the purpose of collecting stamp
duty cannot form foundation to determine the market value of the property under
registration.

a) CWT vs P.N. Sikand (1977) 107 ITR 922 SC


b) Jawaji Nagnathan vs REV. DIV. Officer (1994) SCC - 4 Page 595 SC
c) Shubh Ram and Others vs State of Haryana (2010) 1 SCC 444
d) SLAO (Eluru) vs Jasti Rohini (1995) 1 SCC 717 SC

80.7. The court cannot take into account the award passed by the SLAO unless it is
produced and proved before the court. The market value is to be determined as on
108

date of publication of the notification under sec 4 of LAA. Plus factors and minus
factors are to be considered while determining the market value.

a) Jawaji Nagnathan vs REV. DIV. Officer (1994) SCC - 4 Page 595 SC


b) Chimanlal Harigovinddas vs SLAO Special Land Acquisition Officer
c) Shubh Ram and Others vs State of Haryana (2010) 1 SCC 444
d) SLAO (Eluru) vs Jasti Rohini (1995) 1 SCC 717 SC

* * *

FAQ

1. The Doctrine of Unearned Increase was enunciated because of a famous court


judgement. Select the correct judgement.

a) CIT vs. Smt. Ashima Sinha (1979) 116 ITR 26 (Calcutta), 1980 Tax 56(1) 19
(Calcutta).
b) Commissioner of Wealth Tax, New Delhi vs. Sri P.N. Sikand (1977) 107 ITR
922 (SC).
c) Controller of Estate Duty vs. Radha Devi Jalan (1968) 67 ITR 761, Calcutta
High Court.
d) C.W.T. vs. Venugopal Konar&Ors. (1977) 109 ITR 52, Madras High Court.

2. The landmark judgement, Commissioner of Wealth Tax, New Delhi Vs. Sri P.N.
Sikand (1979) 107 ITR 922 (SC) states that:

a) only the lessor has the right to the increase in value


b) the increase in value of the leasehold interest of the property leased is to be
equally shared by both the lessor and lessee.
c) only the lessee has the right to the increase in value
d) the ratio of the lessor’s and lessee’s share is to be determined by negotiation.

* * *

Answer for all questions : b


109

Q81. VALUER AS AN EXPERT WITNESS IN COURT

1. A valuer is not witness of facts but he is witness of opinion of facts, i.e., on fair value
of the property. Is this correct?

a) May be correct b) Not correct


c) Correct d) May not be correct

2. If A has sold a property to B for 20 lakhs through a broker, then the broker is a

a) Witness of opinion b) Witness of fact


c) Not a witness of fact d) Not a witness of opinion

3. A has sold a property to B for Rs. 20 lakhs through a broker,. The valuer as expert
witness deposes that the value of said property is Rs. 25 lakhs. Then, the valuer is

a) Witness of fact b) Witness of opinion


c) Not a witness of opinion d) Not a witness of fact

4. What must be the first requirement of a valuer if he is directed to give an expert


witness?

a) Give me the copy of my valuation report (documents)


b) Give me the copy of drawing
c) Give me the copy of legal opinion
d) Give me the copy of approval letter

5. The first important quality of an expert honest valuer while being cross examined is

a) Never tell a lie b) Do not tell the truth


c) Never admit the mistake d) Never say yes

6. A valuer should always remember that

a) He is not witness of fact but witness of opinion


b) He is a witness of fact but not a witness of opinion
c) His words are final in the court’s verdict
d) His words will be taken as perfect by the court

7. Any opinion without reasoning has no value

a) This is not true b) This is true


110

c) May be true d) May not be true

8. A valuer who has never faced cross examination by a senior counsel in the court,
will never reach perfection in field of valuation, in spite of several years of practice
as valuer

a) Not correct b) This is wrong notion


c) Need not be correct d) This is correct

9. It is science of morals, that branch of philosophy which is concerned with human


character and conduct ..... It is called as

a) Ethics b) Conduct
c) Discipline d) Character

10. Inadequate physical inspection or doing valuation without inspecting the property
is considered as

a) Indiscipline of a valuer b) Negligence of a valuer


c) Incompetence of a valuer d) Irresponsibility of a valuer

11. Failure to ascertain restrictive covenants is considered as

a) Indiscipline of a valuer b) Negligence of a valuer


c) Incompetence of a valuer d) Irresponsibility of a valuer

12. Which of the following is expressed by a valuer while giving expert evidence in the
examination - in - chief in the Court?

a) Opinions regarding values b) Evidence of facts


with reasoning
c) Answers without reasoning d) Answers only in ‘yes’ or ‘no’

13. A flat was valued by ‘A’ at Rs. 21 Lakhs and then purchaser ‘B’ purchased from seller
‘C’ said flat for Rs. 20 Lakhs with the help of broker ‘D’. In a court case about correct
sale value, which of the following is not called a ‘witness of fact’?

a) Purchaser ‘B’ b) Valuer ‘A’


c) Seller ‘C’ d) Broker ‘D’

* * *

Ans
1 - c, 2 - b, 3 - b, 4 - a, 5 - a, 6 - a, 7 - b, 8 - d, 9 - a, 10 - b, 11 - b, 12 - a, 13 - b
111

i. PRINCIPLES OF INSURANCE AND LOSS ASSESSMENT (Q82 to 86)

1. Valuation of property for insurance purpose does not involve

a) Land valuation b) Building valuation


c) Machinery valuation d) Equipments valuation

2. Land does not need any insurance because it

a) Is not depreciable b) Is not destructible


c) Can be destroyed d) Can be deteriorated

3. The insurance company do not insure

a) Machinery b) Building c) Land d) Equipments

4. A building is a ............... and hence requires insurance

a) Moveable asset b) Immovable asset


c) Solid asset d) Destructible asset

5. In the language of insurance, the risks like fire, earth quake, riot, terrorism,
explosion, flooding. tsunami, storm, cyclone, lightening are called

a) Perils b) Non - perils c) Damage d) Destroy

6. Insurance policy is basically a ....................... under which insurance company


promises to indemnify the insured person for the loss or damage caused by the
insured peril (fire, flood) to the insured property.

a) Registered deed b) Contract document


c) Oral agreement d) Lease deed

7. Insurance was first established in the country

a) France b) USA c) Britain d) Russia


112

8. The number of subsidiaries to the head company general insurance (India) is

a) 10 b) 3 c) 2 d) 4

9. To regulate the insurance business in India, Government of India introduced a


body named as

a) IRDA b) SARFAESI c) DRT d) IBBI

10. The insurance company who undertakes risk of the person taking insurance policy
is called as

a) Insured b) Insurer c) Company d) Institution

11. The person who seeks indemnity and insurance cover for his asset from an
insurance company is called as

a) Owner b) Insurer c) Insured d) Applicant

12. The sum for which the asset is insured by the insurer is called as

a) Value b) Premium c) Deposit d) Insurable amount

13. The annual sum payable by the insured to the insurance company for risk
protection of assets is called as

a) Premium b) Insurable amount


c) Caution deposit d) Pakadi

14. The (fair) reasonable value (True value) of the asset on the day of taking out of
insurance policy is called as

a) Over value b) Value at risk


c) Under value d) No value

15. If the insured amount is higher than the true worth of the asset (value at risk), than
it is called as

a) Fair value b) Under insured


c) Over insured d) Unfair value
113

16. If the insured amount is lower than the true worth of the asset / value at risk, then
it is called as

a) Unfair value b) Over insured


c) Fair value d) Under insured

17. Concept of insurance is based on principle of

a) Utmost good faith b) Natural justice


c) Fairness d) Sympathy

18. Fundamental principle of insurance is

a) Insured person should make b) Insured person should not make


profit out of insurance money profit out of insurance money

c) The insurance company should d) The insured person should be at


make profit loss

19. Immediately on payment of premium, the insurance company issues a ‘cover note’
and later releases the insurance policy. A risk is covered even from the

a) Date of cheque payment b) Only after the date of issuing policy


c) Date of issue of cover note d) Date of accident

20. The period of an insurance policy is normally

a) One month b) 6 months c) 3 months d) One year

21. The types of building insurance policy which are available with the insurance
company are

a) Two b) Four c) One d) Eight

22. Value at risk and insurable amount are determined on the basis of depreciated
value of building. This type policy is called

a) Reinstatement value policy b) Market value policy


c) Replacement policy d) Agreed value policy
114

23. In this type of policy, a clause is added that full reinstatement of the asset will be
done by the insured company, in case of total destruction of the building. It is called
as

a) Replacement policy b) Market value policy


c) Reinstatement policy d) Salvage value policy

24. Reinstatement policy is available for the buildings of age upto

a) 30 years b) 20 years
c) 25 years d) 15 years

25. This policy is available only for marine insurance and goods in transit through sea
way and not for building insurance

a) Agreed value basis policy b) Market value policy


c) Reinstatement policy d) Replacement policy

26. In building insurance, the normal fire policies are

a) 4 b) 3 c) 2 d) 6

27. It is an all peril policy. All types of risks are covered. Damage not only due to fire but
by flood and earthquake are also covered. In this type of policy, 15% variation in
insured amount estimate and actual value at risk are admissible. If difference is
more, average clause is made applicable. This policy is offered for non industrial
buildings like residential building, office building, hotel building, hospital, etc.

a) Fire policy C b) Fire policy B


c) Fire policy A d) Fire policy A1

28. This policy covers restricted peril viz. damage by fire. However it also covers risks
due to gas explosion and damage by lightening. In this type of policy 15% variation
in estimation is not permitted. This type of policy is also offeref for non industrial i.e.
for residential and commercial structures.

a) Fire policy A1 b) Fire policy C


c) Fire policy A d) Fire policy B
115

29. This policy is offered mainly for industrial buildings and godown type structures.
Risk covered under the policy is also mainly fire, lightening and explosion only. In
this policy also 15% variation between sum insured and value at risk is not allowed.

a) Fire policy C b) Fire policy A1


c) Fire policy A d) Fire policy B

30. In India reinstatement clause concept was first introduced from

a) 1945 b) 1935 c) 1955 d) 1965

31. The unauthorised and illegal structures are

a) Insured under special b) Insured


conditions

c) Not insured d) Insured with heavy premium

32. For the high valued properties under insurance the premium amount will be

a) More or less b) Less c) Constant d) More

33. If the insured sum is 30% less than the value at risk, then the insurance company
will pay ................. amount of the insured sum

a) 30% less b) 30% more c) 60% less d) 0%

34. The premium rates depend on the perils and

a) It is constant b) It changes from time to time

c) It does not change d) It will be compulsorily less


every year

35. Several property owners insure the flats, shops, office for full purchase price. This
is not proper, because

a) The purchase price includes b) The purchase price includes


registration charges drawing approval expenses
116

c) The purchase price mainly d) The purchase price includes


includes the land component deposits paid electricity, drainage,
and land cannot be insured etc.

36. All RCC framed and load bearing structures with brick or concrete walls, RCC slab
belong to the category of

a) Class ‘D’ building b) Class ‘B’ building


c) Class ‘C’ building d) Class ‘A’ building

37. The temporary type semi-permanent structures with AC / GI shade roof, belong to
the category of

a) Class ‘B’ building b) Class ‘A’ building


c) Class ‘C’ building d) Class ‘D’ building

38. In claiming the compensation for peril,

Sum insured
Amount payable = Assessed loss x
Risk insurable

This is called as

a) Assessing the loss b) Average clause


c) Fire policy A c) Fire policy B

39. Fire insurance is an agreement between two parties,

a) The insured and contractor b) The insurer and builder


c) The insurer and insured d) Contractor and insurance company

* * *

Answers :
1 - a 9 - a 17 - a 25 - a 33 - a
2 - b 10 - b 18 - b 26 - b 34 - b
3 - c 11 - c 19 - c 27 - c 35 - c
4 - d 12 - d 20 - d 28 - d 36 - d

5 - a 13 - a 21 - a 29 - a 37 - a
6 - b 14 - b 22 - b 30 - b 38 - b
7 - c 15 - c 23 - c 31 - c 39 - c
8 - d 16 - d 24 - d 32 - d
117

j. REPORT WRITING

Q87. REPORTS - QUALITY, STRUCTURE AND STYLE

87.1. What should be the first column of valuation report?

a) What is the purpose of valuation? b) Documents perused


c) Valuation is undertaken based d) Name of the owner
on the request from

87.2. Writing a valuation report requires

a) Technical and communication skill b) High imagination


c) Letter from the client d) Good typing

87.3. A valuation report is to be written

a) Only after the valuation is b) Before taking up the


completed valuation assignment
c) During the process of valuation d) At the time of physical
inspection

87.4. Formatted valuation reports tend to be arbitrary and are not

a) Exhaustive b) Detailed
c) Brief d) Sufficient sometimes

87.5. Writing a report on valuation is the result of instruction from the

a) Client b) Advocate c) Auditor d) Engineer

87.6. While preparing a valuation report, the valuer must satisfy himself that whatever
he writes is

a) True b) False c) Notional d) Approximate

87.7. The valuer himself must verify and get satisfied that the evidence and facts stated
in the report are

a) True and correct b) Notional c) False d) Approximate

* * *
Answers :

1 to 7 - a
118

Q88. REPORT WRITING - QUALITY, STRUCTURE AND STYLE, REPORT WRITING


FOR VARIOUS PURPOSES OF VALUATION, CONTENTS OF THE REPORT.

88.1. Non technical reports are

a) News paper or media reports b) Valuation reports


c) Structural design reports d) Structural stability reports

88.2. Reports for income tax valuation should be furnished in

a) FORM 0-2 b) FORM 0-1


c) FORM 0-3 d) FORM 0-4

88.3. IVS 103 deals with

a) General concepts and principles b) Scope of work


c) Reporting d) Investigation & compliance

88.4. Valuation date is defined in IVS as

a) Date of inspection b) Date of issuing report


c) Date of valuation report d) Date on which the estimate of
value applies

88.5. Valuer may not be required to give evidence as expert in the court for the following:

a) Drawing b) Acquisition of land


c) Fair rent d) Insurance

88.6. Supporting materials to valuation report are not the following:

a) Building plans b) Photographs


c) Pay slip of owner d) Statement of floor areas

* * *
Answers :

1 - a 5 - a
2 - b 6 - c
3 - c
4 - d
119

PART - III

TWELVE MARKS CASE STUDIES (Q89 TO 94)


120

This page is kept vacant intentionally.


121

Part - III

CASE STUDIES ON VALUATION (Q 89 to 94)

CONTENTS

1. Valuation of Building - 123 - 137

2. Valuation by Land & Building method - 138 - 156

3. Written down value & Book value - 157 - 160

4. Insurance - 161 - 172

5. Valuation by Cost index method - 173 - 176

6. Valuation by Belting method - 177 - 181

7. Valuation of Petrol bunk - 182 - 185

8. Valuation of Leasehold properties - 186 - 196

9. Valuation by Profit method - 197 - 199

10. Bank valuation - 200 - 201

11. Ground rent - 202 - 203

12. Valuation of Tenanted properties - 204 - 209

13. Residual value method - 210 - 211

14. Capital gain - 212 - 225

15. Apartments & J.V. ratio - 226 - 237

16. Miscellaneous topics - 238 - 238


122

DISCLAIMER

While every effort is taken to avoid errors or omissions in this publication, any
mistake or omission that might have crept in is not intentional. It may be taken note
of that neither the publisher nor the author will be responsible for any damage or
loss of any kind arising to any one in any manner of account of such errors and
omissions. A few case studies have been taken from the book of Mr. R.K. Gandhi.
123

1. VALUATION OF BUILDING

Exercise 1 :

It is a load bearing structure. Age is 8 years. Life is 60 years.

i) What is the percentage depreciation by straight line method assuming a salvage


value of 10%.

ii) What is the depreciation by constant percentage method if the depreciation rate is
1.5%.

i) Age = 8 years
Life = 60 years
Salvage = 10%
8
Depreciation = x (100 - 10) = 12%
60

r n
ii) Depreciation = 1 - (1 - )
r 100
Formula A = P (1 - 100 )n
1.5 8
= 1 - (1 - )
A = depreciated value 100
P = replacement value
n = age = 1 - (0.985)8
r = rate of depreciation
Depreciation factor = 1 - (0.886) = 0.1138 or
Depreciation percentage = 0.1138 x 100 = 11.38%

Exercise 2 :

It is a load bearing structure of 20 years old. Plinth area : 1275 sq.ft.. Replacement rate =
Rs. 1,650/sq.ft. What is the depreciated value of the building (Life : 60 years, salvage
value = 10%) by adopting straight line method (SLM)?

Plinth area = 1,275 sq.ft.


Replacement rate = Rs. 1,650/sq.ft.
Replacement value = Rs. 21,03,750/-
Age of the building = 20 Years
Life of the building = 60 years
Salvage value = 10%
124

20
Depreciation percentage = x (100 - 10) = 30%
60
Depreciated value or = 0.7 x 21,03,720
Net Present Value or = Rs. 14,72,625/-
Depreciated Replacement cost
or NCRC

Exercise 3 :

The built up area of a GF building is 5,000 sq.ft. and the carpet area is 4,000 sq.ft. Plot area
is 10,000 sq.ft. What is the FSI? What is plot coverage?

Builtup area
FSI =
Plot area
5,000
= = 0.5
10,000

GF area
Plot coverage = x 100
Plot area
5,000
= x 100 = 50%
10,000

Exercise 4 :

A building of 8,000 sq.ft (GF & FF - 4,000 sq.ft each) is existing in a plot of 8,000 sq.ft.
What is the plot coverage?

Plinth area of GF
Plot coverage = x 100
Plot area

4,000
= x 100 = 50%
8,000

Exercise 5 :

20 years factory building of 5,000 sq.ft. is situated in 1 acre of industrial land. The unit
replacement rate of building is Rs. 1,000/-. Assuming the life as 40 years and a salvage
value of 30%, find the depreciated value and salvage value of the building.

Plinth area = 5,000 sq.ft.


Replacement rate = Rs. 1,000/sq.ft.
125

Replacement value = Rs. 50,00,000


Age of the building = 20 years
Life assumed = 40 years
Salvage value assumed = 30%
20
Depreciation percentage = x (100 - 30) = 35%
40
Depreciated value = 0.65 x 50,00,000 = Rs. 32,50,000
Salvage value = 0.30 x 50,00,000 = Rs. 15,00,000

Exercise 6 :

Building area = 1,200 m2 ; Age = 25 years ; Life = 50 years ; Salvage value = Nil ; Plot area
= 2,000 m2 ; Land rate = Rs. 8,000/m2 ; Replacement cost of building = Rs. 25,000/m2.
What is the value?

Land value = 2,000 x 8,000 = Rs. 1,60,00,000


25
Depreciation percentage = x 100 = 50% (salvage value is nil)
50
Depreciated value of the building = 1,200x25,000x0.5 = Rs. 1,50,00,000

Total value (Land + building) = Rs. 3,10,00,000

Exercise 7 :

The plinth area of a RCC roofed load bearing residential building (16 years old) is
1,000 sq.ft. The life of the building as 60 years and a salvage value of 10%,

Questions :

1) Calculate the depreciated value if the unit replacement cost is Rs. 1,800/-.

2) For the above building, if the age of the first floor is 10 years, what will be the
depreciated value of first floor of built up area 1,200 sq.ft. assuming the unit rate of
construction as Rs. 1,400/-.

Data :

Type of structure = Load bearing


Plinth area = 1,000 sq.ft.
Life = 60 years
126

Age of the building = 16 years


Salvage value = 10%

Calculations :

GF
Plinth area = 1,000 sq.ft.
Replacement rate = Rs. 1,800/sq.ft.
Replacement value = Rs. 18,00,000
Age = 16 years
Life = 60 years
Salvage value = 10%
16
Depreciation percentage = x 90 = 24%
60
Depreciated value = 0.76 x 18,00,000
= Rs. 13,68,000/- (1)

FF
Plinth area = 1,200 sq.ft.
Age = 10 years
Life = 60 Years
Depreciation (as of GF) = 24%
Replacement rate = Rs. 1,400/sq.ft.
Replacement value = 1,200 x 1,400 = 16,80,000
Depreciated value = 0.76 x 16,80,000
= Rs. 12,76,800/- (2)

Answers :

1) Rs. 13,68,000/- 2) Rs. 12,76,800/-

Exercise 8 :

A RCC framed structure building consists of front portion (1,500 sq.ft. - 24 years age) and
rear portion (1,200 sq.ft. - 16 years). The replacement unit rate of construction is
Rs. 1,600 per sq.ft. Life - 80 years. Salvage value - 10%.

Questions :

1) What is the depreciated value of rear portion?


127

2) What is the depreciated value of the front portion?

Data :

Number of portions = 2
Type of structure = RCC framed
Area of front portion = 1,500 sq.ft.
Age of front portion = 24 years
Area of rear portion = 1,200 sq.ft.
Age of rear portion = 16 years
Replacement rate of construction = Rs. 1,600/sq.ft. (average)
Life = 80 years
Salvage value = 10%

Calculations :

Rear portion :
Plinth area of rear portion = 1,200 sq.ft.
Replacement rate = Rs. 1,600/sq.ft.
Replacement value = 1,200 x 1,600
= Rs. 19,20,000
Age of the building = 16 years
Life of the building = 80 years
Salvage value = 10%
16
Depreciation percentage = x 90 = 18%
80
Depreciation value = 0.18 x 19,20,000
= Rs. 3,45,600
Depreciated value = 19,20,000 - 3,45,600
= Rs. 15,74,400/- (1)

Front portion :
Plinth area of front portion = 1,500 sq.ft.
Replacement rate = Rs. 1,600/sq.ft.
Replacement value = 1,500 x 1,600
= Rs. 24,00,000
Age = 24 years
Life = 80 years
Salvage value = 10%
128

24
Depreciation percentage = x 90 = 27%
80
Depreciation value = 0.27 x 24,00,000
= Rs. 6,48,000
Depreciated value = 24,00,000 - 6,48,000
= Rs. 17,52,000/- (2)

Answers :

1) Rs. 15,74,400/- 2) Rs. 17,52,000/-

Exercise 9 :

It is a residential building of GF & FF. The age of GF is 16 years and FF is 8 years. Plinth
area of each floor is 1,200 sq.ft. Replacement unit rate of GF & FF is Rs. 1,600 & 1,200
respectively. Assume life as 60 years and salvage value as 10%.

Questions :

1. What is the depreciated value of GF?


2. What is the depreciated value of FF?

Data :

Number of floors = GF & FF


Plinth area of ground floor = 1,200 sq.ft.
Age of ground floor = 16 years
Replacement rate of ground floor = Rs. 1,600/-
Plinth area of first floor = 1,200 sq.ft.
Age of first floor = 8 years
Replacement rate of first floor = Rs. 1,200/-
Life = 60 years
Salvage value = 10%

Calculations :

GF FF
Plinth area = 1,200 sq.ft. 1,200 sq.ft.
Replacement rate = Rs.1,600/sq.ft. Rs.1,200/sq.ft.
Replacement value = Rs.19,20,000 Rs.14,40,000
129

Age = 16 years 8 years


Life = 60 years 60 years
Salvage vlaue = 10% 10%
16 8
Depreciation = x 90 = 24% x 90 = 12%
60 60
But, 24% is adopted
(as of GF)
Depreciation value = 19,20,000 x 0.24 14,40,000 x 0.24
Rs. 4,60,800 Rs. 3,45,600
Depreciated value = Rs. 14,59,200/- Rs. 10,94,400/-
(1) (2)

Answers :

1) Rs. 14,59,200/- 2) Rs. 10,94,400/-

Exercise 10 :

A load bearing building (1,500 sq.ft.) of 20 years old is existing in a plot of 2,400 sq.ft. The
unit land rate of plot is Rs. 2,000 and replacement unit rate of construction is
Rs. 1,700 sq.ft. It is a collateral security. Salvage value = 10%.

Questions :

1) Determine the market value assuming it is a marketable property?


2) Determine the forced value (assuming a reduction factor as 15%)?

Data :

Type of structure = Load bearing


Plinth area = 1,500 sq.ft.
Age = 20 years
Plot area = 2,400 sq.ft.
Land rate = Rs. 2,000/-
Replacement rate of construction = Rs. 1,700/sq.ft.
Salvage value = 10%
Purpose = Collateral security to bank
130

Calculations :

Land value = 2,400 x 2,000 = Rs. 48,00,000

Building area = 1,500 sq.ft.


Replacement rate = Rs. 1,700/sq.ft.
Age of the building = 20 years
Life of the building = 60 years
Salvage value = 10%
20
Depreciation percentage = x 90 = 30%
60

Depreciated value of building = 0.7 x 1,500 x 1,700


= Rs. 17,85,000
Total value = Rs. 65,85,000/- (1)
48,00,000 + 17,85,000

Forced sale value 0.85x65,85,000= Rs. 55,97,250/- (2)

Answers :

1) Rs. 65,85,000/- 2) Rs. 55,97,250/-

Exercise 11 :

Plinth area is 1,000 sq.ft. Replacement rate of construction is Rs. 2,000/sq.ft. Age is
20 years. Life is 60 years. Salvage value is 10%.

Questions :

1) What is replacement value?


2) What is depreciation percentage by adopting straight line method?
3) What is the net present value?
4) What is the depreciation percentage by constant percentage method assuming a
rate of depreciation as 1.5%.
5) What is the balance economic life?

Data :

Plinth area = 1,000 sq.ft.


Replacement rate of construction = Rs. 2,000/sq.ft.
131

Age of the building = 20 years


Life of the building = 60 years
Salvage value = 10%

Calculations :

Plinth area = 1,000 sq.ft.


Replacement rate = Rs. 2,000/sq.ft.
Replacement value = Rs. 20,00,000/- (1)

Age of the building = 20 Years


Life of the building = 60 Years
Salvage value = 10%
Depreciation = (20/60) x 90 = 30% (2)

Depreciation value = 0.3 x 20,00,000 = Rs. 6,00,000


Net present value = 20,00,000 - 6,00,000
= Rs. 14,00,000/- (3)

r n
Depreciation percentage = 1-(1 - )
100
by constant %age method
r
Formula A = P (1 - 100 )n 1.5 20
= 1-(1 - )
A = depreciated value 100
P = replacement value
= 1 - (0.985)20
Depreciation factor = 0.26087
Depreciation percentage = 0.26087 x 100 = 26.09% (4)

Balance economic life = 60 - 20 = 40 Years (5)

Answers :

1) Rs. 20,00,000/- 4) 26.09%


2) 30% 5) 40 Years
3) Rs. 14,00,000/-

Exercise 12 :

Ground floor of a residential bungalow was constructed in 1985 at a (historic) cost of


132

Rs. 3,50,000. First floor was constructed in 1990 at a cost of Rs. 6,00,000/-. Work out
replacement cost of bungalow for the year 2003 by Book value method. The building cost
multiplier factor with 1960 as base year for year 1985, 1990 and 2003 were 14.16, 27.08
and 87.50 respectively. (Courtesy : Mr. R.K. Gandhi).

Questions :

1. What is the replacement cost of ground floor by book value method?


2. What is the replacement cost of first floor by book value method?
3. What is the total replacement cost of the building by book value method?

Data :

Year of construction of GF = 1985


Cost invested for GF = Rs. 3,50,000/-
Year of construction of FF = 1990
Cost invested for FF = Rs. 6,00,000/-
Cost multiplier factor (1960 as base = 14.16
year) for the year 1985
Cost multiplier factor (1960 as base = 27.08
year) for the year 1990
Cost multiplier factor (1960 as base = 87.50
year) for the year 2003

Calculations :

1. Ground floor cost in 1985 = Rs. 3,50,000/-


Cost factor for 1985 = 14.16
Cost factor for 2003 = 87.50
Replacement cost of GFin 2003 3,50,000
= x 87.50
by Book value method 14.16
= Rs. 21,62,782/- (1)

2. First floor cost in 1990 = Rs. 6,00,000/-


Cost factor for 1990 = 27.08
Cost factor for 2003 = 87.50
Replacement cost of FF by 6,00,000
= x 87.50
Book value method 27.08
= Rs. 19,38,700/- (2)
133

3. Total replacement cost = 21,62,782 + 19,38,700


= Rs. 41,01,482/- (3)

Answers :

1. Rs. 21,62,782/- 3. Rs. 41,01,482/-


2. Rs. 19,38,700/-

Exercise 13 : (IBBI)

The ground floor of an RCC framed residential building was constructed in 1978. The first
floor of the building was constructed in 1992 and second floor was in 2010. A major structural
renovation took place in 2015. The areas of ground floor, first floor and second floor are
1200 sq ft., 1200 sq. ft and 800 sq ft respectively. The cost of construction of similar type
of building in 2015 as per CPWD Plinth Area Rate method is INR 1600 per sq ft. The Cost
Index in 2018 is 114. The remaining economic life of the building in 2018 is another
65 years.

Questions :
2010 800
1) What is the physical age of first floor as on date?
2) What is the effective age of the building? 1992 1200
3) What is the replacement value of the building in 2018?
4) What is the depreciation percentage of the first floor in 2018? 1978 1200
5) What is the depreciated value of the building in 2018?

Data :

Year of construction of GF = 1978


Year of construction of FF = 1992
Year of construction of SF = 2010
Year of major renovation = 2015
Plinth area of GF = 1,200 sq.ft.
Plinth area of FF = 1,200 sq.ft.
Plinth area of SF = 800 sq.ft.
Replacement rate of construction = Rs. 1,600/sq.ft.
in 2015
Cost index in 2018 = 114
Remaining economic life as on = 65 years
2018
134

Calculations :

1) Year of construction of FF = 1992


Age of FF = 2018 - 1992 = 26 years
Year of construction of GF = 1978
Age of the GF = 2018 - 1978 = 40 years

For the purpose of calculating the


depreciation of upper floors, the
age of GF is mainly considered.
.
. . the physical age of FF = 2018 - 1978 = 40 years (1)

2) Age of GF = 40 years
Remaining economic life = 65 years
Effective life of the building = 65 + 40 = 105 years

3) Replacement rate of construction = Rs. 1,600/sq.ft. (2)


in 2015
Cost index in 2018 = 114
1,600
Replacement rate in 2018 = x 114 = Rs. 1,824/-
100
Plinth area of GF = 1,200 sq.ft.
Plinth area of FF = 1,200 sq.ft.
Plinth area of SF = 800 sq.ft.
Total plinth area of the building = 3,200 sq.ft.
Replacement value of the building= 1,824 x 3,200
in 2018
= Rs. 58,36,800/- (3)

4) Age of GF =
40 years
Life of GF =
105 years
Salvage value assumed as =
10%
40
Depreciation percentage = x 90 = 34.29% (4)
105
This %age is assumed as %age of depreciation for FF also.

5) Replacement value of building = Rs. 58,36,800/-

Depreciation value = 0.3429 x 58,36,800


= Rs. 20,01,439/-
135

Depreciated value = 58,36,800 - 20,01,439


= Rs. 38,35,361/- (5)

Answers :

1) 40 years 4) 34.29%
2) 105 years 5) Rs. 38,35,361/-
3) Rs. 58,36,800/-

Exercise 14 :

There is a commercial building of GF + 2 in a busy commercial locality. GF (2,000 sq.ft.) is


a load bearing structure of age 40 years. The economic life can be assumed as 60 years
with a salvage value of 10%. The FF (2,200 sq.ft.) & SF (2,200 sq.ft.) is a framed structure
of age 20 years which rest on independent separate foundation. The economic life of this
new structure can be assumed as 80 years. The replacement cost of load bearing
structure is Rs. 1,600/sq.ft. and the average replacement cost of FF & SF is Rs. 1,800/
sq.ft. The external services is 10% for all the floors. The plot area is 4,000 sq.ft. and the
prevalent rate of land is Rs. 5,000/sq.ft. Salvage value for FF is 10%.
11 12
12
11 SF 12
12
11 12
12 20Y
11 2,200 12
12
Questions : 11 12
12
11 FF 12
12 20Y
11 2,200 12
12
1) What is the value of the plot? 11 12
12
11 12
12
2) What is the depreciated value of GF? GF 11 12
12 40Y
11
2,000 12
12
3) What is the depreciated value of FF & SF? 11 12
12
4) What is the value of the entire building?
5) What is the value of the building for the purpose of fire insurance assuming 20%
as the value of foundation?
6) What is the value of property?

Data :

• Plot area = 4,000 sq.ft.


Rate for land = Rs. 5,000/sq.ft.

• Plinth area of GF = 2,000 sq.ft.


Age of GF = 40 years
Type of structure = Load bearing
Economic life = 60 years
136

Replacement cost = Rs. 1,600/-


External service = 10%

• Plinth area of FF & SF = 2,200 sq.ft. & 2,200 sq.ft.


Age of FF & SF = 20 years
Economic life of FF & SF = 80 years
Type of structure = RCC framed with independent
foundation
Replacement rate = Rs. 1,800/-
External services = 10%

Calculations :

Plot area = 4,000 sq.ft.


Rate of plot = Rs. 5,000/sq.ft.
Value of plot = 4,000 x 5,000
= Rs. 2,00,00,000/- (1)

GF
Plinth area = 2,000 sq.ft.
Replacement rate = Rs. 1,600/-
Replacement value = 2,000 x 1,600
= Rs. 32,00,000
Add 10% for external services = Rs. 3,20,000
Total = Rs. 35,20,000/-
Age of ground floor = 40 years
Life of ground floor = 60 years
Salvage value = 10%
40
Depreciation percentage = x 90 = 60%
60
Depreciation value = 0.6 x 35,20,000
= Rs. 21,12,000/-
Depreciated value of GF = 35,20,000 - 21,12,000
= Rs. 14,08,000/- (2)

FF & SF
Built up area of first floor = 2,200 sq.ft.
Built up area of second floor = 2,200 sq.ft.
Total built up area = 4,400 sq.ft.
Replacement rate = Rs. 1,800
137

Add 10% for external services = Rs. 180


Rate + Service = Rs. 1,980/-
Replacement value = 4,400 x 1,980
= Rs. 87,12,000/-
Age of FF & SF = 20 years
Life of FF & SF = 80 years
Salvage value = 10%
20
Depreciation percentage = x 90 = 22.5%
80
Depreciation value = 0.225 x 87,12,000
= Rs. 19,60,200/-
Depreciated value of FF & SF = 87,12,000 - 19,60,200
= Rs. 67,51,800/- (3)

Total value of building


GF - load bearing - 2,000 sq.ft. = Rs. 14,08,000
FF & SF - framed structure - 4,400 sq.ft.= Rs. 67,51,800
Value of building = Rs. 81,59,800/- (4)

Depreciated value of building = Rs. 81,59,800


Less value of foundation (-20%) = (-) Rs. 16,31,960
Value for the purpose of insurance= Rs. 65,27,840 (5)

Value of plot = Rs. 2,00,00,000


Value of building = Rs. 81,59,800
Total value of the property = Rs. 2,81,59,800/- (6)

Answers :

1) Rs. 2,00,00,000/- 4) Rs. 81,59,800/-


2) Rs. 14,08,000/- 5) Rs. 65,27,840/-
3) Rs. 67,51,800/- 6) Rs. 2,81,59,800/-

* * *
138

2. VALUATION BY LAND & BUILDING METHOD

Exercise 1 :

In 2008, Mr. X purchased a residential plot of 3,000 sq.ft. for Rs. 15,00,000/-. In the year
2010, he constructed a residential building of GF for 1,500 sq.ft. and in the year 2012, he
constructed FF for 1,200 sq.ft. In 2018, a valuation report is required. Replacement cost of
GF is Rs. 2,000/sq.ft. and FF is 1,600/sq.ft. Prevailing market rate of plot is Rs. 2,000/sq.ft.
and the guide line rate is Rs. 2,500/sq.ft. Assume the life as 60 years and salvage value is
10%.

Questions :

1. What is the total replacement value of the building?


2. What is the total depreciation value of the entire building?
3. What is the total depreciated value of the entire building?
4. What is the prevailing market value of the plot?
5. What is the total value of the property as on date that can be certified?
6. What is the book value of the plot as on 2018?

Data :

Plot area = 3,000 sq.ft.


Purchased cost of plot (2008) = Rs. 15,00,000/-
Area of building GF (2010) = 1,500 sq.ft.
Area of building FF (2012) = 1,200 sq.ft.
Replacement cost of building GF (2018) = Rs. 2,000/sq.ft.
Replacement cost of building FF (2018) = Rs. 1,600/sq.ft.
Prevailing market rate of plot = Rs. 2,000/sq.ft.
Guideline rate = Rs. 2,500/sq.ft.
Life of the building = 60 years
Salvage value = 10%
Date of valuation = 2018

Calculations :

Value of GF

Plinth up area of Ground floor = 1,500 sq.ft.


139

Replacement rate of GF = Rs. 2,000/sq.ft.


Replacement value 1,500 x 2,000 = Rs. 30,00,000
Age 2018 - 2010 = 8 years
Life = 60 years
Salvage value = 10%
Depreciation percentage (8/60) x 90 = 12%
Depreciation value of GF = 0.12 x 30,00,000
= Rs. 3,60,000
Depreciated value of GF = 30,00,000 - 3,60,000
= Rs. 26,40,000/-

Value of FF

Built up area of First floor = 1,200 sq.ft.


Replacement rate of FF = Rs. 1,600/sq.ft.
Replacement value 1,200 x 1,600 = Rs. 19,20,000
Age 2018 - 2012 = 6 years
6
Depreciation percentage = x 90 = 9%
60
Depreciation of GF is adopted (i.e. 12%)
Depreciation value 0.12 x 19,20,000 = Rs. 2,30,400
Depreciated value 19,20,000 - 2,30,400 = Rs. 16,89,600/-

Value of GF + FF

Total replacement value 30,00,000 + 19,20,000= Rs. 49,20,000/- (1)


Total depreciation value 3,60,000 + 2,30,400= Rs. 5,90,400/- (2)
Total depreciated value 26,40,000 + 16,89,600= Rs. 43,29,600/- (3)

Value of Plot

Extent of plot = 3,000 sq.ft.


Prevailing market rate = Rs. 2,000/sq.ft.
Value - 3,000 x 2,000 = Rs. 60,00,000/- (4)

Total value of property

Value of plot = Rs. 60,00,000


Value of building = Rs. 43,29,600
Total value = Rs. 1,03,29,600/- (5)
140

Book value

Book value of plot = Rs. 15,00,000/- (6)

Answers :

1) Rs. 49,20,000/- 4) Rs. 60,00,000/-


2) Rs. 5,90,400/- 5) Rs. 1,03,29,600/-
3) Rs. 43,29,600/- 6) Rs. 15,00,000/-

Exercise 2 : (IBBI)

A doctor purchased a plot of 2,000 Sq.m. in a posh locality in a city in the year 1997 for a
price of Rs. 50,00,000/-. In the year 1998, he constructed a hospital having 500 Sq.m.
built up floor area at ground level and 200 Sq.m. built up area at first floor level at the cost
of Rs. 20,00,000/-. Prevalent replacement cost of similar hospital as on 2018 is
Rs. 35,000 per Sq.m. Prevalent land price in the locality at present is Rs.80,000 per Sq.m.
Age of building is 20 years and the total life of the building is 60 years.

Questions :

1. What will be the depreciation amount of the hospital building by adopting straight
line method of depreciation and considering scrap value at 10% ?

2. What will be the depreciation amount of the hospital building by adopting constant
percentage method of depreciation?

3. What will be the total market value of the plot at present?

4. What will be the total market value of the hospital property for bank loan purpose?

5. What is the balance economic life of the building?

6 Which of the following will not be considered for the estimation of present value of
building?

a) Age b) Area of the building


c) Replacement cost d) Land rate
141

Data :

Extent of plot = 2,000 sq.m.


Year of purchase of plot = 1997
Purchased amount = Rs. 50,00,000/-
Year of construction = 1998
Plinth area of the building GF = 500 sq.m.
Built up area of the building FF = 200 sq.m.
Cost of building GF + FF (500 + 200) = Rs. 20,00,000/-
Replacement rate of the building = Rs. 35,000/sq.m.
Prevalent land rate = Rs. 80,000/sq.m.
Age of the building = 20 years
Life of the building = 60 years
Salvage value assumed = 10%

Calculations :

1. Total built up area = 700 sq.m.


Replacement rate / sq.m. = Rs. 35,000
Replacement value - 700 x 35,000 = Rs. 2,45,00,000
Age = 20 years
Life = 60 years
Salvage value = 10%
20
Depreciation percentage = x 90 = 30%
60
Depreciation amount : 0.3x2,45,00,000= Rs. 73,50,000/- (1)

2. Life = 60 years
100
Rate of depreciation = = 1.66 %
60
1.66 20
Depreciation amount = [
P 1-(1-
100
) ]
r n = 2,45,00,000 [1 - 0.7155]
[
P 1-(1-
100
) ] = 2,45,00,000 x 0.2845
= Rs. 69,70,250/- (2)

3. Extent of plot = 2,000 m2


Prevalent market rate = Rs. 80,000/m2
Market value of land - 2,000 x 80,000= Rs. 16,00,00,000/- (3)
142

4. Land value - 2,000 x 80,000 = Rs. 16,00,00,000


Depreciated value of the building = 0.7 x 2,45,00,000
= Rs. 1,71,50,000
Total value - Land + building = Rs.17,71,50,000/- (4)

5. Total economic life of building = 60 years


Age of the building = 20 years
Balance economic life : 60 - 20 = 40 years (5)

6. While estimating the present market value of the building,


1. Age is to be considered.
2. Area is to be considered.
3. Replacement cost is to be considered.

Land rate need not be considered. (6)

Answers :

1) Rs. 73,50,000/- 4) Rs. 17,71,50,000/-


2) Rs. 69,70,250/- 5) 40 years
3) Rs. 16,00,00,000/- 6) Land rate need not be
considered.

Exercise 3 :

In the year 2000, a plot of 4,800 sq.ft. was purchased by Mr. X for Rs. 4,80,000/-. In 2008,
he constructed GF for an area of 1,400 sq.ft. In 2015, he constructed FF for an area of
1,200 sq.ft. It is a load bearing structure. The replacement rate of construction of GF & FF
is Rs. 1,800 & Rs. 1,500 respectively. The guideline (circle) rate of plot is Rs. 1,540/sq.ft.
and the prevailing market rate is Rs. 1,000/sq.ft. Assume a salvage value 10%, Date of
valuation is 2018.

The questions are :

1. What is the land value in 2018?


2. What is the depreciated value of GF?
3. What is the depreciated value of FF?
4. What is the market value of the property assuming it is a marketable property?
5. What is the forced sale value of the property assuming a reduction factor of 15%?
6. What is the book value of the plot in 2018?
143

Data :

Plot area = 4,800 sq.ft.


Purchased cost (2000) = Rs. 4,80,000/-
Area of GF (2008) = 1,400 sq.ft.
Area of FF (2015) = 1,200 sq.ft.
Type of structure = Load bearing
Replacement rate of GF = Rs. 1,800/sq.ft.
Replacement rate of FF = Rs. 1,500/sq.ft.
Circle rate of plot = Rs. 1,540/sq.ft.
Market rate of plot = Rs. 1,000/-
Salvage value assumed = 10%
Date of valuation = 2018

Calculations :

Value of land in 2018 - (4,800 x 1,000) = Rs. 48,00,000/- (1)

Plinth area of GF = 1,400 sq.ft.


Age of GF - (2018 - 2008) = 10 years
Economic life of load bearing structure = 60 years
Salvage value = 10%
Depreciation percentage - (10/60) x 90 = 15%
Replacement rate of GF = Rs. 1,800/-
Replacement value - 1,400 x 1,800 = Rs. 25,20,000/-
Depreciated value - 25,20,000 x 0.85 = Rs. 21,42,000/- (2)

Age of FF - (2018 - 2015) = 3 years


3
Depreciation percentage = x 90 = 4.5%
60
Depreciation percentage adopted = 15% (as of GF)
Replacement rate of FF = Rs. 1,500/-
Replacement value - 1,200 x 1,500 = Rs. 18,00,000/-
Depreciated value - 18,00,000 x 0.85 = Rs. 15,30,000/- (3)

Market value of the property (assuming it is marketable) :

Plot value = Rs. 48,00,000


Building - GF = Rs. 21,42,000
Building - FF = Rs. 15,30,000
Total value = Rs. 84,72,000/- (4)
144

Forced sale value 0.85 x 84,72,000 = Rs. 72,01,200/- (5)

Book value of the plot in 2018 = Rs. 4,80,000/- (6)

Answers :

1. Rs. 48,00,000/- 4. Rs. 84,72,000/-


2. Rs. 21,42,000/- 5. Rs. 72,01,200/-
3. Rs. 15,30,000/- 6. Rs. 4,80,000/-

Exercise 4 : (IBBI)

A business man purchased a plot of 1000 sq.mt. in a posh locality of a city in the year 1987
for a price of Rs. 30,00,000. In the year 1988, he constructed a residential bungalow
having 300 sq.mt. built up floor area at ground level and 100 sq.mt. built up area at first
floor level at the cost of Rs. 14,00,000. Prevalent replacement cost of similar bungalow as
on today is Rs. 30,000 per sq.mt. Prevalent land price in the locality at present is
Rs. 60,000 per sq.mt. Age of building is 30 years and the total life of the building is
60 years.

Questions :

1. What will be the depreciation amount of the bungalow by adopting straight line
method of depreciation and considering scrap value at 10 % ?

2. What will be the depreciation amount of the bungalow by adopting constant


percentage method of depreciation?

3. What will be the market value of the land at present?

4. What will be the total market value of the bungalow property for the bank loan
purpose?

5. What is the balance economic life of the building?

6. Which of the following will not be considered for the estimation of present market
value of above property?
a) Depreciation b) Replacement cost
c) Current land rate d) Economic obsolescence

Data :

Extent of plot = 1,000 sq.m.


145

Year of purchase of plot = 1987


Purchased amount = Rs. 30,00,000/-
Year of new construction = 1988
Plinth area of the building GF = 300 sq.m.
Built up area of the building FF = 100 sq.m.
Cost of building GF + FF (300 + 100) = Rs. 14,00,000/-
Replacement cost = Rs. 30,000/sq.m.
Prevalent land rate = Rs. 60,000/sq.m.
Age of the building = 30 years
Life of the building = 60 years
Salvage value assumed = 10%

Calculations :

1. Total built up area = 400 sq.m.


Replacement rate / sq.m. = Rs. 30,000
Replacement value - 400 x 30,000 = Rs. 1,20,00,000
Age of the building = 30 years
Life of the building = 60 years
Salvage value = 10%
30
Depreciation percentage = x 90 = 45%
60
Depreciation amount : 0.45x1,20,00,000= Rs. 54,00,000/- (1)

2. Life = 60 years
100
Rate of depreciation = = 1.66 %
60
1.66 30
Depreciation amount = [
P 1-(1-
100
) ]
r n
[
P 1-(1-
100
) ] = 1,20,00,000 x 0.3948
= Rs. 47,37,600/- (2)

3. Extent of plot = 1,000 m2


Prevalent market rate = Rs. 60,000/m2
Market value of land - 1,000 x 60,000= Rs. 6,00,00,000/- (3)

4. Land value - 1,000 x 60,000 = 6,00,00,000


Depreciated value of the building = 0.55 x 1,20,00,000
(SLM) = Rs. 66,00,000
146

Total value - Land + building= Rs. 6,66,00,000/- (4)

5. Total economic life of building = 60 years


Age of the building = 30 years
Balance economic life 60 - 30 = 30 years (5)

6. While estimating the present market value of the property,


1. Depreciation is to be considered.
2. Replacement cost is to be considered.
3. Current land rate is to be considered.

Economic obsolescence need not be considered. (6)

Answers :

1) Rs. 54,00,000/- 4) Rs. 6,66,00,000/-


2) Rs. 47,37,600/- 5) 30 years
3) Rs, 6,00,00,000/- 6) Economic obsolescence
need not be considered.

Exercise 5 :

Twenty years back, Mr. X purchased a plot of 3,000 sq.ft. for 4 lakhs. In this plot, he
constructed a residential building of 1,000 sq.ft. 16 years back. The replacement rate of
construction including services today is 1,800/sq.ft. Assume the life as 80 years and
salvage value as 10%. The prevalent rate of plot as Rs. 1,500/sq.ft.

1) What is the value of the property (Land + building) as on date?


2) What is the depreciation amount for the building as on date? (by adopting straight
line method)
3) What is the forced sale value of the property assuming 15% as the reduction
factor?
4) What is the auction value of the property assuming 30% as the reduction factor?
5) What will be the upset price if the bank fixes 10% as the reduction factor?
6) What is the cost of the plot for balance sheet purpose?

Data :

Plot (3,000) purchased cost = Rs. 4,00,000


Plinth area of building = 1,000 sq.ft.
147

Age of the building = 16 years


Replacement rate of building = Rs. 1,800/sq.ft.
Life = 80 years
Salvage value = 10%
Prevalent land rate = Rs. 1,500/sq.ft.

Calculations :

a. Extent of plot = 3,000 sq.ft.


Prevalent market rate = Rs. 1,500/sq.ft.
Value of plot = Rs. 45,00,000/-

b. Plinth area of building = 1,000 sq.ft.


Replacement rate = Rs. 1,800/sq.ft.
Replacement value = Rs. 18,00,000
Age of the building = 16 years
Life of the building = 80 years
Salvage value = 10%
Depreciation percentage = (16/80) x 90 = 18%
Depreciation value = Rs. 3,24,000
0.18 x 18,00,000
Depreciated value = Rs. 14,76,000/-
(18,00,000 - 3,24,000)

c. Value of the property = Rs. 59,76,000/- (1)


45,00,000 + 14,76,000

The depreciation amount of the building = Rs. 3,24,000/- (2)

Value of the property = Rs. 59,76,000


Forced sale value = 0.85 x 59,76,000
= Rs. 50,79,600/- (3)

Value of the property = Rs. 59,76,000


Auction value = 0.7 x 59,76,000
= Rs. 41,83,200/- (4)

Auction value certified by the valuer = Rs. 41,83,200


Less 10% = (-) 4,18,320
Upset price fixed by the bank = Rs. 37,64,880/- (5)
148

The purchased amount of plot will be the cost for balance sheet purpose.
Cost is Rs. 4,00,000/-.

Answers :

1) Rs. 59,76,000/- 4) Rs. 41,83,200/-


2) Rs. 3,24,000/- 5) Rs. 37,64,880/-
3) Rs. 50,79,600/- 6) Rs. 4,00,000/-

Exercise 6 :

A load bearing building having 1,000 sq.m. built-up floor area is constructed in the year
1992. Total area of the plot is 5,000 sq.m. Replacement cost of building in March 2012 is
Rs. 7,500/sq.m. Prevalent Land rate is Rs. 1,200/sq.m. in the locality.

Questions :

1. What is the value of the plot?


2. What is the replacement value of building?
3. What is the depreciation percentage by adopting straight line method assuming
the life as 60 years and salvage value as 10%?
4. What is the depreciation value?
5. What is the depreciated value?
6. What is the total value?

Data :

Building type = RCC roofed load bearing


Builtup area of the building = 1,000 sq.m.
Year of construction = 1992
Replacement cost of building 2012= Rs. 7,500/sq.m.
Plot area = 5,000 sq.m.
Prevalent land rate = Rs. 1,200/sq.m.
Value to be calculated as on = 2012

Calculations :

Plot area = 5,000 sq.m.


Prevalent land rate = Rs. 1,200/sq.m.
Land value = 5,000 x 1,200
= Rs. 60,00,000/- (1)
149

Building area = 1,000 sq.m.


Replacement rate = Rs. 7,500
Replacement value = 1,000 x 7,500
= Rs. 75,00,000/- (2)

Age of the building = 2012 - 1992 = 20 years


Life assumed = 60 years
Depreciation percentage = (20 / 60) x 90 = 30% (3)

Depreciation value = 0.3 x 75,00,000


= Rs. 22,50,000/- (4)

Net present value (NPV) or = 75,00,000 - 22,50,000


Depreciated value
= Rs. 52,50,000/- (5)

Total value of the property = 60,00,000 + 52,50,000


= Rs. 1,12,50,000/- (6)

Answers :

1. Rs. 60,00,000/- 4. Rs. 22,50,000/-


2. Rs. 75,00,000/- 5. Rs. 52,50,000/-
3. 30% 6. Rs. 1,12,50,000/-

Exercise 7 :

A residential load bearing structure having 280 sq.m. built-up floor area is constructed in
1961 at Delhi. Area of plot is 650 sq.m. Calculate value of property as on 01.04.1981, if
prevalent land rate in 1981 in that locality was Rs. 800 per sq.m. Cost index for Delhi in
1981 was 176 with base year 01.10.1976 as 100. Rate for bungalow in 1976 was Rs. 325/
sq.m. Plumbing cost/unit was Rs. 6,000 and electrification cost was Rs. 5,700/unit as per
C.P.W.D. memorandum of 01.10.1976. Life is 60 years & salvage value is 10%.
(Courtesy : Mr. R.K. Gandhi)

Questions :

1. What is the value of the plot as on 1981?


2. What is the replacement value of the building as on 1981?
3. What is the depreciation percentage by adopting straight line method assuming life
as 60 years & salvage value as 10%?
150

4. What is the depreciation value as on 1981?


5. What is the Net present value of the building?
6. What is the total value of the property as on 1981?

Data :

Structure = Load bearing


Builtup area = 280 sq.m.
Year of construction = 1961
Place = Delhi
Area of plot = 650 sq.m.
Land rate prevailing (1981) = Rs. 800/sq.m.
Cost index for Delhi with base = 100
year 01.10.1976
Cost index for Delhi (1981) = 176
Rate of bungalow 1976 = Rs. 325/sq.m.
Plumbing cost/unit = Rs. 6,000/unit
Electrification = Rs. 5,700/unit

Calculations :

Area of plot = 650 sq.m.


Rate in 1981 = Rs. 800/sq.m.
Value of plot in 1981 = 650 x 800
= Rs. 5,20,000/- (1)

Building replacement cost in 1976


Basic rate = Rs. 325/sq.m.
Builtup area = 280 sq.m.
Building cost = 280 x 325
= Rs. 91,000
Add for plumbing = Rs. 6,000
Add for electrification = Rs. 5,700
Total replacement cost in 1976 = Rs. 1,02,700
This is for cost index of = 100
Cost index in Delhi in 1981 = 176
... Replacement cost of building 1,02,700
= x 176
in 1981 100
= Rs. 1,80,752/- (2)
151

Age of the building = 1981 - 1961 = 20 years


Life assumed = 60 years
Salvage value = 10%
20
Depreciation percentage = x (100 - 10) = 30% (3)
60
Depreciation value = 0.3 x 1,80,752
= Rs. 54,225/- (4)

Net present value or = 1,80,752 - 54,225


Depreciated value
= Rs. 1,26,527/- (5)

Total value of the property = 5,20,000 + 1,26,527


as on 1981
= Rs. 6,46,527/- (6)

Answers :

1. Rs. 5,20,000/- 4. Rs. 54,225/-


2. Rs. 1,80,752/- 5. Rs. 1,26,527/-
3. 30% 6. Rs. 6,46,527/-

Exercise 8 :

A bungalow having G + 2 upper floor is for sale. Area of plot is 500 sq.m. Ground floor
having 200 sq.m. built-up area was built in 1975. 1st and 2nd floor having total 300 sq.m.
built-up area were raised in 1995. Prevalent land rate in locality, in 2012, is Rs. 46,000/
sq.m. and replacement cost is Rs. 18,000/- per sq.m. Date of valuation is 2012.

Questions :

1. What is the value of the plot as on 2012?


2. What is the replacement value of the building?
3. What is the depreciation percentage that can be adopted for entire building by
adopting straight line method assuming life as 60 years and salvage value as 10%?
4. What is the depreciation value of the building?
5. What is the Net present value of the building?
6. What is the total value that can be certified for the entire property?
152

Data :

Area of plot = 500 sq.m.


Area of GF = 200 sq.m.
Year of construction of GF = 1975
Area of FF & SF = 300 sq.m.
Year of construction of FF & SF = 1995
Land rate in 2012 = Rs. 46,000/sq.m.
Replacement cost of building = Rs. 18,00,000/sq.m.

Calculations :

Area of plot = 500 sq.m.


Land rate = Rs. 46,000/sq.m.
Value of plot = 500 x 46,000
= Rs. 2,30,00,000/- (1)

Area of GF = 200 sq.m.


Area of FF & SF = 300 sq.m.
Total area = 500 sq.m.
Replacement rate = Rs. 18,000/sq.m.
Replacement value = 500 x 18,000
= Rs. 90,00,000/- (2)

Year of construction of GF = 1975


Valuation as on = 2012
Age as on 2012 = 2012 - 1975 = 37 years
Life assumed = 60 years
Salvage value assumed = 10%
37
Depreciation of GF = x (100 - 10) = 55.5% (3)
60

Depreciation of FF & SF = Adopted same as GF


Depreciation value = 0.555 x 90,00,000
= Rs. 49,95,000/- (4)

Depreciated value or = 90,00,000 - 49,95,000


Net present value of the building
= Rs. 40,05,000/- (5)
153

Present value of the property = Land value + Building value


= 2,30,00,000 + 40,05,000
= Rs. 2,70,05,000/- (6)

Answers :

1. Rs. 2,30,00,000/- 4. Rs. 49,95,000/-


2. Rs. 90,00,000/- 5. Rs. 40,05,000/-
3. 55.5% 6. Rs. 2,70,05,000/-

Exercise 9 :

An existing two storeyed framed structure stands on land measuring 2 grounds (1 ground
= 2,400 sq.ft.). The ground floor and first floor each has an area of 1,000 sq.ft. The ground
floor was constructed 20 years ago and the first floor 12 years ago. The prevailing land
market value of a similar adjacent vacant plot was Rs. 90,000 per ground. The
replacement cost of new similar construction (including foundation) is Rs. 300 per sq.ft.
for ground floor and Rs. 250 per sq.ft. for the first floor. External services, amenities,
boundary wall, etc. provided can be taken at 15% of the depreciated cost of the structure.
Value the property. Assume life as 80 years & salvage value as 10%.
(Courtesy : Mr. R.K. Gandhi)

Questions :

1. What is the value of plot?


2. What is the net present value of ground floor?
3. What is the depreciation percentage of first floor?
4. What is the net present value of first floor?
5. What is the value of services?
6. What is the total value of property?

Data :

Structure = Framed structure


Plot area = 2 grounds
Plinth area of GF = 1,000 sq.ft.
Age of GF = 20 years
Plinth area of FF = 1,000 sq.ft.
Age of FF = 12 years
Replacement cost of GF = Rs. 300/sq.ft.
Replacement cost of FF = Rs. 250/sq.ft.
154

Prevailing market rate of plot = Rs. 90,000/sq.ft.


Services = 15%

Calculations :

Area of plot = 2 grounds


Prevailing market rate of plot = Rs. 90,000/ground
Value of plot = 2 x 90,000
= Rs. 1,80,000/- (1)

GF : Plinth area of GF = 1,000 sq.ft.


Replacement rate = Rs. 300/sq.ft.
Replacement value = 1,000 x 300
= Rs. 3,00,000
Age of the building = 20 years
Life assumed = 80 years
Salvage value = 10%
20
Depreciation percentage = x 90 = 22.5%
80
Depreciation value = 0.225 x 3,00,000 = 67,500
Depreciated value or = 3,00,000 - 67,500
Net Present Value
= Rs. 2,32,500/- (2)

FF : Plinth area of FF = 1,000 sq.ft.


Replacement rate = Rs. 250/sq.ft.
Replacement value = 1,000 x 250
= Rs. 2,50,000
Depreciation percentage of FF = Same as GF (i.e. 22.5%) (3)

Depreciation value = 0.225 x 2,50,000 = 56,250


Depreciated value or = 2,50,000 - 56,250
Net Present Value = Rs. 1,93,750/- (4)

Total value of building (GF + FF) = 2,32,500 + 1,93,750


= Rs. 4,26,250/-
External services 15% = 0.15 x 4,26,250
= Rs. 63,938/- (5)
155

Total value of property = 1,80,000 + 4,26,250 + 63,938


(Plot + Building + Services)
= Rs. 6,70,188/- (6)

Answers :

1) Rs. 1,80,000/- 4) Rs. 1,93,750/-


2) Rs. 2,32,500/- 5) Rs. 63,938/-
3) 22.5% 6) Rs. 6,70,188/-

Exercise 10 :

Land extent is 500 sq.m. in which a building of 300 sq.m. is existing. Year of construction
is 2002. Present replacement cost is Rs. 20,000/sq.m. Prevailing market rate of land is
Rs. 22,000/sq.m. What is the selling price as on today (omit salvage value)?

Data :

Extent of land = 500 sq.m.


Market rate = Rs. 22,000/sq.m.
Building area = 300 sq.m.
Replacement rate of building = Rs. 20,000/sq.m.
Year of construction = 2002
Salvage value = Nil

Calculation :

Plot

Extent of land = 500 sq.m.


Market rate = Rs. 22,000/sq.m.
Value : 500 x 22,000 = Rs. 1,10,00,000/-

Building

Area of building = 300 sq.m.


Replacement rate = Rs. 20,000/sq.m.
Replacement value = 300 x 20,000
= Rs. 60,00,000/-
156

Age : 2018 - 2002 = 16 years


Life assumed = 60 years
Salvage value = Nil
16
Percentage of depreciation = x 100
60
= 26.67%
Depreciation value = 0.2667 x 60,00,000
= Rs. 16,00,200/-
Depreciated value = Rs. 43,99,800/-

3.0. Total value

Value of plot = Rs. 1,10,00,000


Value of building = Rs. 43,99,800
= Rs. 1,53,99,800/-

* * *
157

3. WRITTEN DOWN VALUE & BOOK VALUE

Exercise 1 :

An assessee has spent Rs. 1,20,00,000 in his new building in the year March 2014. What will
be the written down value (WDV) of the above building as on 31.03.2018 assuming a rate of
depreciation as 10%. This is required for preparing balance sheet for Income Tax
purpose.

Book value as on 31.03 2014 = Rs. 1,20,00,000


Less 10% depreciation = (-) Rs. 12,00,000
WDV as on 31.03.2015 = Rs. 1,08,00,000
Less 10% depreciation = (-) Rs. 10,80,000
WDV as on 31.03.2016 = Rs. 97,20,000
Less 10% depreciation = (-) Rs. 9,72,000
WDV as on 31.03.2017 = Rs. 87,48,000
Less 10% depreciation = (-) Rs. 8,74,800
WDV as on 31.03.2018 = Rs. 78,73,200/-

r n
Formula A = P(1- )
100
10 4
= 1,20,00,000 ( 1 - )
100

= 0.661 x 1,20,00,000
= Rs. 78,73,200/-

Exercise 2 :

In the year 2015, Mr. ‘X’ has spent Rs. 87,00,000/- in purchasing a vacant site of
10,000 sq.ft. which includes registration charges, stamp duty, brokerage, etc. What will be
the book value of the plot as on 2017?

Book value is the amount spent originally in procuring the site.

... Book value = Rs. 87,00,000/-


158

Exercise 3 :

A factory building with 30’ roof height was constructed in 1974 at the cost of Rs. 6,40,000.
Calculate the replacement cost (by book value method) in the year 1995 if prevalent
building construction cost in 1974 and 1995 were Rs. 530/sq.m. and Rs. 5,800/sq.m.
respectively.

Historical cost 1974 = Rs. 6,40,000

Historical cost in 1974


Replacement cost = x cost factor in 1995
Cost factor in 1974

5,800
= 6,40,000 x
530

= Rs. 70,03,774/-

Exercise 4 :

A machine was purchased in year 1993 at the cost of Rs. 2,20,000/-. Cost Index factor for
year 1993 was 37.50 with base year 1960 as 1.00. Calculate replacement cost of machine
in year 2003 if Cost Index factor for year 2003 is 87.50 with same base year.

Book value as on 1993


Replacement cost (2003) = x Cost Index factor for 2003
Cost index factor for 1993

87.50
= 2,20,000 x
37.50

= Rs. 5,13,333/-

Exercise 5 :

In April 2012, Mr. ‘X’ has purchased a residential plot of 3,000 sq.ft. for an amount of
Rs. 9,00,000/- and has paid Rs. 1,22,000 for the registration charges, stamp paper,
brokerage expenses, etc. In this plot, he constructed a commercial building of 2,200 sq.ft.
for an amount of Rs. 25,25,000. The construction was completed in February 2013.
Calculation of book value is required for the purpose of income tax. Assume a
depreciation of, say, 10%.
159

Questions :

1. What is the book value of the property as on 31.03.2013?


2. What is the book value of the property as on 31.03.2014?
3. What is the book value of the property as on 31.03.2015?
4. What is the book value of the property as on 31.03.2016?
5. What is the book value of the property as on 31.03.2017?
6. What is the book value of the property as on 31.03.2018?

Data :

Purchased cost of plot 3,000 sq.ft. = Rs. 9,00,000


in April 2012
Registration expenses stamp duty = Rs. 1,22,000
and brokerage
Commercial building - 2,200 sq.ft. = Rs. 25,25,000
Depreciation percentage = 10%
Purpose of valuation = Income tax

Calculations :

Cost of the plot in April 2012 (i.e. 2012 - 13) = Rs. 10,22,000/-
9,00,000 + 1,22,000

Historic cost of the building in February 2013 = Rs. 25,25,000/-


(i.e. 2012 - 13)

S.no. As on Land Rs. Building Rs. Book value Rs.

1. 31.03.2013 10,22,000 25,25,000 35,47,000 (1)

2. 31.03.2014 10,22,000 25,25,000 x 0.9 32,94,500 (2)


= 22,72,500

3. 31.03.2015 10,22,000 22,72,500 x 0.9 30,67,250 (3)


= 20,45,250
160

4. 31.03.2016 10,22,000 20,45,250 x 0.9 28,62,725 (4)


= 18,40,725

5. 31.03.2017 10,22,000 18,40,725 x 0.9 26,78,653 (5)


= 16,56,653

6. 31.03.2018 10,22,000 16,56,653 x 0.9 25,12,988 (6)


= 14,90,988

Answers :

1. Rs. 35,47,000/- 4. Rs. 28,62,725/-


2. Rs. 32,94,500/- 5. Rs. 26,78,653/-
3. Rs. 30,67,250/- 6. Rs. 25,12,988/-

* * *
161

4. INSURANCE

Exercise 1 :

The value of a building on completion in 2015 is 25 lakhs excluding foundation and the
owner has insured for Rs. 25 lakhs. The value of the building in 2018 is 30 lakhs exclusive
of the value of foundation. In 2018, there is a damage to the building to the extent of
Rs. 3,00,000. How much the owner will get compensation from the insurance company?

Sum insured in 2015


Compensation = x Damage
Value as on 2018

25,00,000
Compensation = x 3,00,000 = Rs. 2,50,000/-
30,00,000

Exercise 2 :

There is a property with the following specification.

a) Plinth area = 1,100 m2


b) Age of the building = 15 years
c) Life of the building = 60 years
d) Replacement rate = Rs 10,500/m2
e) Assume cost of foundation = 15%
f) Salvage value = Nil

Question :

Calculate the insurable value of this property.

Calculations :

Value of building as if new = 1,100 x 10,500


= Rs. 1,15,50,000/-

15
Depreciation = = 25%
60
162

Deduct value of foundation = 15%


= 1,15,50,000 - 17,32,500
= Rs. 98,17,500/-
Deduct depreciation 25% = Rs. 24,54,375/-
Insurable value = Rs. 73,63,125/-
say Rs. 73.63 lakhs.

Answers :

Rs. 73,63,000/-

Exercise 3 :

A standard fire policy is there for 50 lakhs for a factory building 700 Sq.m. of 20 years old.
Replacement rate is Rs 20,000 / sq.m. Fire loss is Rs 10 lakhs.

Questions :

1. Claim payable is how much?


2. If policy excess of Rs.10,000/- is to be considered, then, what is the claim payable?
3. What is the present market worth less foundation before fire damage?
4. What is the replacement cost of new building today deducting 10% for foundation?
5. What is the depreciation of bldg excl. foundation on straight line method?
Life : 60 years; Salvage = Nil

Data :

Sum insured = Rs. 50,00,000/-


Area of the building = 700 sq.m.
Age of the building = 20 years
Replacement rate = Rs. 20,000/sq.m.
Fire loss = Rs. 10,00,000/-

Calculation :

Area of the building = 700 sq.m.


Replacement rate = Rs. 20,000/sq.m.
Replacement value = 700 x 20,000 = Rs. 1,40,00,000/-
Age of the building = 20 years
Life of the building assumed = 60 years
163

Salvage value assumed = Nil


20
Depreciation percentage = x 100 = 33.33%
60
Depreciation value = 0.3333 x 1,40,00,000
= Rs. 46,66,200/-

Depreciated value = 1,40,00,000 - 46,66,200


= Rs. 93,33,800/-
Less foundation 10% = 0.1 x 93,33,800
= Rs. 9,33,380

Depreciated value of the = Rs. 84,00,420/-


building less foundation
93,33,800 - 9,33,380

Answers :

1) Fire loss = Rs. 10,00,000


Sum insured = Rs. 50,00,000
Depreciated value of the = Rs. 84,00,420
building
50,00,000
Under insurance factor = = 59.52%
84,00,420
.
. . Claim payable = 0.5952 x 10,00,000
= Rs. 5,95,200/- (1)

2) Claim = Rs. 5,95,200


Less policy excess = Rs. 10,000
Net claim payable = Rs. 5,85,200/- (2)

3) Present worth less = Rs. 84,00,420/- (3)


foundation

4) Replacement cost of new


building today deducting = 0.9 x 1,40,00,000
10% for foundation
= Rs. 1,26,00,000/- (4)

5) Depreciation of building excluding foundation on straight line method :


164

Replacement value = Rs. 1,40,00,000


Value excluding foundation= 0.9 x 1,40,00,000
= Rs. 1,26,00,000/-
Depreciation percentage
20
assuming the life as = x 100 = 33.33%
60
60 years & salvage value
as NIL

Depreciation of building = 0.3333 x 1,26,00,000


excluding foundation
= Rs. 41,99,500/- (5)

Exercise 4 :

A standard fire policy was taken for Rs. 161 lakhs for a factory building (RCC roof) 1,400
sq.m. of 15 years old. Replacement cost is Rs. 18,000 / sq.m. Fire loss is Rs. 30 lakhs.
Assume life as 60 years. Salvage value : NIL.

Questions :

1. Claim payable is how much?


2. If policy excess of Rs. 10,000/- is to be considered, then what is the claim payable?
3. What is the present worth less foundation before fire damage?
4. What is the replacement cost of new building today deducting 15% for foundation?
5. What is the depreciation of building excluding foundation on straight line method?
6. What will be the depreciation percentage if the salvage value is 20%?

Data :

Sum insured = Rs. 1,61,00,000/-


Area of the building = 1,400 sq.m.
Age of the building = 15 years
Replacement cost = Rs. 18,000/sq.m.
Fire loss = Rs. 30,00,000/-
Life = 60 years

Calculation :

Area of the building = 1,400 sq.m.


165

Replacement rate = Rs. 18,000/sq.m.


Replacement value = 1,400 x 18,000 = Rs. 2,52,00,000
Age of the building = 15 years
Life of the building = 60 years
Salvage value = Nil
15
Depreciation percentage = x 100 = 25%
60
Depreciation value = 0.25 x 2,52,00,000
= Rs. 63,00,000/-
Depreciated value = 2,52,00,000 - 63,00,000
= Rs. 1,89,00,000/-
Less foundation 15% = 0.15 x 1,89,00,000
= Rs. 28,35,000
Depreciated value of the = Rs. 1,60,65,000/-
building less foundation
1,89,00,000 - 28,35,000

Answers :

1) Fire loss = Rs. 30,00,000


Sum insured = Rs. 1,61,00,000
Depreciated value of the = Rs. 1,60,65,000
building
.
. . Sum insured = Adequate (There is no under
insurance)
... Claim payable = 100% of Rs. 30,00,000/-
= Rs. 30,00,000/- (1)

2) Claim = Rs. 30,00,000


Less policy excess = Rs. 10,000
Net claim payable = Rs. 29,90,000/- (2)

3) Present worth less = Rs. 1,60,65,000/- (3)


foundation

4) Replacement cost of new


building today deducting = 0.85 x 2,52,00,000
15% for foundation
= Rs. 2,14,20,000/- (4)
166

5) Depreciation of building excluding foundation on straight line method :

Replacement value = Rs. 2,52,00,000


Value excluding foundation= 0.85 x 2,52,00,000
= Rs. 2,14,20,000/-
Depreciation percentage
assuming the life as 15
= x 100 = 25%
60 years & salvage value 60
as NIL

Depreciation of building = 0.25 x 2,14,20,000


excluding foundation
= Rs. 53,55,000/- (5)

6) Age of the building = 15 years


Life of the building = 60 years
Salvage value = 20%
15
Depreciation percentage = (100 - 20) = 20% (6)
60

Exercise 5 :

A standard fire policy with reinstatement clause was taken for Rs. 161 lakhs for a factory
building (RCC roof) 1,400 sq.m of 15 years old. Replacement cost is Rs. 18,000/sq.m. Fire
loss is Rs. 30 lakhs. Assume life as 60 years. Salvage value : Nil. Plinth & foundation : 15%.

Questions :

1) Claim payable is how much?


2) If policy excess of Rs. 10,000/- is to be considered, then, what is the claim payable?

Data :

Sum insured = Rs. 1,61,00,000


Area of the building = 1,400 sq.m.
Age of the building = 15 Years
Replacement rate = Rs. 18,000 /sq.m.
Fire loss = Rs. 30,00,000
167

Life = 60 Years
Salvage value = Nil
Plinth & foundation = 15%
Special clause = Reinstatement value clause
included
Calculation :

Area of the building = 1,400 sq.m.


Replacement rate = Rs. 18,000/sq.m.
Replacement value = 1,400 x 1,800
= Rs. 2,52,00,000/-
Age of the building = 15 Years
Life of the building = 60 Years
Salvage value = Nil

Reinstated Value of the building


Less foundation 15% = Less 15% of 2,52,00,000
= 2,52,00,000 x (100-15)%
= Rs. 2,14,20,000/-

Answers :

1) Fire loss = Rs. 30,00,000


Sum insured = Rs. 1,61,00,000
Reinstated value of the
building less plinth & = Rs. 2,14,20,000/-
foundation
Sum insured = Inadequate (There is under
insurance)
Under Insurance Quotient = Sum insured/Value of the asset
1,61,00,000
= x 100 = 75%
2,14,20,000
Claim payable = Value assessed as loss x
Under Insurance Quotient
= Rs. 30,00,000 x 0.75
= Rs. 22,50,000/- (1)

2) It is a fire policy and the peril for the loss is fire. Hence there is a policy
excess of Rs. 10,000/-
168

Policy excess = Rs. 10,000


Claim payable = 22,50,000 - 10,000
= Rs. 22,40,000/- (2)

Exercise 6 :

One factory got damaged. The sum insured is Rs. 50,00,000/-. Claim made by the owner
- Rs. 10,00,000/-. The property is 20 years old. Present replacement rate of a similar new
building is Rs. 7,000/- per sq.m. Builtup area - 2,000 sq.ft.

1) What is replacement value of the building?


2) What would be the claim approved by insurance company?.

Opinion :

Data :

Sum insured = Rs. 50,00,000/-


Claim made by the owner = Rs. 10,00,000/-
Age of the building = 20 years
Built up area = 2,000 sq.ft.
Replacement cost = Rs. 7,000/sq.m.

Solution :

Builtup area = 2,000 sq.ft. or 185.87 sq.m.


Replacement rate = Rs. 7,000/sq.m.
Replacement value = 185.87 x 7,000
= Rs. 13,01,090/- (1)

Age = 20 years
Life assumed = 40 years (since it is a factory)
Salvage value = Nil (assumed)
20
Depreciation percentage = x 100 = 50%
40
Depreciation value = 0.5 x 13,01,090
= Rs. 6,50,545/-
Depreciated value = 13,01,090 - 6,50,545
= Rs. 6,50,545/- (2)
169

Answers : 1) The replacement value is Rs. 13,01,090/-.

2) • The depreciated value (including the foundation is Rs. 6,50,545/-.


The owner has insured the building for Rs. 50,00,000/-. The sum
insured in adequate. These is no under insurance.

• Though the owner has claimed Rs. 10,00,000/-, the actual


depreciated value is only Rs. 6,50,545/-. Therefore the sum payable
is Rs. 6,50,545/- (minus the policy excess).

• (Note : If policy is made only for the super structure only, value of the
superstructure (assuming 15% for foundation) = 0.85 x 6,50,545 =
Rs. 5,52,963/-. Sum payable is Rs. 5,52,963/- (less policy excess)).

Exercise 7 : (IBBI)

Factory building of built-up area 700 sq.m. 20 years old, total life of the building 40 years with
a specification equivalent to the current replacement cost of Rs. 20,000/sq.m. is insured for
Rs. 50,00,000/- in a standard fire policy. There is a partial damage to the building to a total
loss of Rs. 10,00,000/- due to peril. 10% cost of foundation. (Courtesy : Mr. S. Pichaiya)

Questions :

1. What is the amount payable by the insurer to the insure for the loss due to fire?

a) Rs. 2,00,000/- b) Rs. 5,00,000/-


c) Rs. 7,70,000/- d) Rs. 10,00,000/-

2. What is the present market worth of the building before fire damage (excluding
foundation)?

3. Reinstatement value of building excluding foundation?

4. What is the depreciation of the building excluding foundation? (Neglecting scrap


value).

5. Which peril is not covered under standard fire policy?

a) Impact damage b) STFI


c) Earthquake d) Fire
170

Data :
Sum insured = Rs. 50,00,000/-
Area of the building = 700 sq.m.
Age of the building = 20 years
Replacement rate = Rs. 20,000/sq.m.
Fire loss = Rs. 10,00,000/-
Life = 40 years

Calculation :

Area of the building = 700 sq.m.


Replacement rate = Rs. 20,000/sq.m.
Replacement value = 700 x 20,000 = Rs. 1,40,00,000/-
Age of the building = 20 years
Life of the building assumed = 40 years
Salvage value assumed = Nil
20
Depreciation percentage = x 100 = 50%
40
Depreciation value = 0.5 x 1,40,00,000
= Rs. 70,00,000/-

Depreciated value = 1,40,00,000 - 70,00,000


= Rs. 70,00,000/-
Less foundation 10% = 0.1 x 70,00,000
= Rs. 7,00,000/-

Depreciated value of the = Rs. 63,00,000/-


building less foundation
70,00,000 - 7,00,000

Answers :

1) Fire loss = Rs. 10,00,000


Sum insured = Rs. 50,00,000
Depreciated value of the = Rs. 63,00,000
building
50,00,000
Under insurance factor = x 100 = 79.37%
63,00,000
... Claim payable = 0.7937 x 10,00,000
= Rs. 7,93,700/-
= say Rs. 7,70,000/- (1)
171

2) Present worth less = Rs. 63,00,000/- (2)


foundation

3) Replacement cost of new


building today deducting = 0.9 x 1,40,00,000
10% for foundation
= Rs. 1,26,00,000/- (3)

4) Depreciation of building excluding foundation on straight line method :

Replacement value = Rs. 1,40,00,000


Value excluding foundation= 0.9 x 1,40,00,000
= Rs. 1,26,00,000/-
Depreciation percentage
20
assuming the life as = x 100 = 50%
40
40 years & salvage value
as NIL

Depreciation of building = 0.5 x 1,26,00,000


excluding foundation
= Rs. 63,00,000/- (4)

5. Earthquake (5)

Exercise 8 :

RCC roofed building of a 30 years is required to be insured under standard fire policy.
Advise on fair ‘Insurable value’ of the factory building on depreciated cost basis from the
following data. Calculate depreciation by SLM. (Courtesy Mr. R.K. Gandhi)

Plinth area of factory : Ground floor = 500 sq.m.


First floor = 300 sq.m.
Replacement cost today = Rs. 7,500/sq.m.
Age of building = 30 years
Total life of building = 60 years
Foundation = 10%

Solution :

Total builtup area : 500 + 300 = 800 sq.m.


172

Total replacement value = Rs. 60,00,000/- (a)


800 sq.m. x Rs. 7,500/sq.m.
Less 10% towards cost of = Rs. 6,00,000/- (b)
foundation & plinth
Net value of the super structure = Rs. 54,00,000/- (c)

Age = 30 years
Life = 60 years
Salvage = Nil
30
Depreciation percentage = x 100 = 50%
60
Depreciated value = 0.5 x 54,00,000
= Rs. 27,00,000/-
Insurable value = Rs. 27,00,000/-

Advise factory owner to insure the building for Rs. 27,00,000/-

* * *
173

5. VALUATION BY COST INDEX METHOD


(Courtesy : Mr. R.K. Gandhi)

Exercise 1 :

A factory building was constructed in the year 1985 at the total cost of Rs. 25,50,000/-.
Work out replacement cost of said factory building in year 2011 if Building Cost index in
year 1985 and 2011 were 14.16 and 142 respectively with base year 1960 at 1.00.

Replacement cost of Book value cost


= x Cost Index factor for 2011
factory in year 2011 Cost index for 1985

25,50,000
= x 142
14.16

= Rs. 2,55,72,033/-

Exercise 2 :

A boeing repair shop hanger (Area 10,648 sq.m.) was constructed at Mumbai in year 1999
at the total cost of Rs. 68.00 crores. Find out its replacement cost in year 2011, if cost of
construction of normal residential building was Rs. 8,600/sq.m. in 1999 and Rs.18,300/
sq.m. in year 2011.

Historic cost = Rs. 68,00,00,000

68,00,00,000
Present day replacement cost = x 18,300
8,600

= Rs. 1,44,69,76,744

Exercise 3 :

Building cost for the residential building in Delhi, as per 01.01.1992 cost index as 100, was
Rs. 2,810/sq.m. Now if Cost Index of Mumbai in 2005 is 250 as compared to 1992 base
index 100, work out replacement cost for a residential building at Mumbai for the year
2005.

Flat rate for building cost for residential house in Mumbai for the year 2005 as per
CPWD memorandum of 1992 will be :
174

2,810
= x 250
100
= Rs. 7,025/sq.m.

Exercise 4 :

A residential building was built in the year 1978 at an actual cost of Rs. 5,00,000/-. If
Building Cost Index for year 1978 and 1998 were 125 and 1442 respectively, with 01.10.1976
as base index 100, work out replacement cost of the building for the year 1998.

5,00,000
Replacement cost in 1998 = x 1,442
125

= Rs. 57,68,000/-
Exercise 5 :

An R.C.C. framed building at Delhi, in 01.01.1992 would cost Rs. 2,810/sq.m. If Cost
Index of V.V. Nagar is 139 in 1997, calculate rate of cost of construction for similar R.C.C.
building at V.V. Nagar for the year 1997.

Rate of cost of construction in 2,810


= x 139
1997 at V.V. Nagar : 100

= say Rs. 3,906/- per sq.m.

Exercise 6 :

A load bearing residential family house was built in year 1969 at Nagpur. Built-up floor
area is 200 sq.m. on ground floor and 100 sq.m./floor on each of 1st and 2nd floor. Total
plot area is 1,200 sq.m. Calculate sale value of property as in March 1989 if Building Cost
Index of Nagpur was 394 in 1989 with Delhi base year 01.10.1976 as 100. Building cost
for base year was Rs. 385/sq.m. and plumbing and electrification costs were Rs. 6,000/
unit and Rs. 5,700/unit respectively. Prevalent land rate in 1989 was Rs. 800/sq.m.
Building is wholly provided with marble floor. Marble cost was Rs. 250/sq.m. and mosaic
tile cost was Rs. 60/sq.m. in 1989.

Questions :

1. What is the value of land as on 1989?


2. What is the replacement cost of the building?
175

3. What is the depreciation percentage by adopting straight line method assuming


economic life as 60 years and salvage value as 10%?
4. What is the depreciation value of the building?
5. What is the depreciated value of the building?
6. What is the total value of the property?

Data :

Place = Nagpur
Year of construction = 1969
GF area = 200 sq.m.
FF area = 100 sq.m.
SF area = 100 sq.m.
Plot area = 1,200 sq.m.
Cost index in Delhi for base year = 100
01.10.1976
Building cost index for Nagpur = 394
in 1989
Building cost for base year (1976) = Rs. 385/sq.m.
Plumbing cost for base year (1976)= Rs. 6,000/unit
Electrification cost for base year = Rs. 5,700/unit
Flooring cost in 1989 = While marble Rs. 250/sq.m.
Mosaic tile cost in 1989 = Rs. 60/sq.m.
Prevalent land rate in 1989 = Rs. 800/sq.m.

Calculations :

Area of plot = 1,200 sq.m.


Unit rate of plot (1989) = Rs. 800/sq.m.
Value of plot in 1989 = 1,200 x 800
= Rs. 9,60,000/- (1)

Area of ground floor = 200 sq.m.


Area of first floor = 100 sq.m.
Area of second floor = 100 sq.m.
Total area of all floors = 400 sq.m.
Unit rate of building (1976) = Rs. 385/sq.m.
Civil work cost (1976) = 400 x 385
= Rs. 1,54,000
Plumbing 3 floors x 6,000 = Rs. 18,000
Electrification 3 floors x 5,700 = Rs. 17,100
176

Replacement cost at Delhi (1976) = Rs. 1,89,100


This is for cost index for = 100
Cost index at Nagpur (1989) = 394
1,89,100
Replacement cost in Nagpur = x 394
100
= Rs. 7,45,054/-

Add for difference of marble & Mosaic


Rs. 250 - 60 = 190/sq.m.

Carpet area = 85% of built up area


= 0.85 x 400 =340 sq.m.

Extra cost of marble = 340 x 190


= Rs. 64,600

Total building cost = 7,45,054 + 64,600


= Rs. 8,09,654/- (2)

Age = 1989 - 1969 = 20 years


Life assumed = 60 Years
Salvage assumed = 10%
20
Depreciation percentage = x 90 = 30% (3)
60

Depreciation value = 0.3 x 8,09,654


= Rs. 2,42,896/- (4)

Net present value or = 8,09,654 - 2,42,896


depreciated value = Rs. 5,66,758/- (5)

Total value of the property = 9,60,000 + 5,66,758


= Rs. 15,26,758/- (6)

Answers :

1. Rs. 9,60,000/- 4. Rs. 2,42,896/-


2. Rs. 7,45,054/- 5. Rs. 5,66,758/-
3. 30% 6. Rs. 15,26,758/-

* * *
177

6. VALUATION BY BELTING METHOD &


Hypothetical plotting scheme method

Exercise 1 :

Estimate the value of plot 40’ x 150’ by belting method. The prevailing market rate for one
ground plot in the nearby locality is Rs. 600/sq.ft. Standard depth is 60’.
40’
II
90’
400/-
150’
I
60’
600/-
Road

Extent Basic rate Unit rate Estimated


Belts Size sq.ft. for standard adopted value
depth. (Rs.) Rs. Rs.

I Belt 40’ x 60’ 2,400 600 600 14,40,000


II Belt 40’ x 90’ 3,600 600 400 14,40,000
40’ x 150’ 6,000 --- --- 28,80,000

Exercise 2 :

Find out the value of the plot 50’ x 200’ by belting method. The prevailing market rate of
the neighbouring plot 40’ x 60’ located on the main road is Rs. 600/-.
50’
Questions : III 300/- 50’

1. What is the area of I belt? II 400/- 90’


Rs. 600
2. What is the area of II belt?
3. What is the area of III belt? 60’ I 600/- 60’
40’
4. What is the value of I belt?
50’
5. What is the total value of 50’ x 200’?
6. While certifying market value for collateral security to bank, which factor is most
important?
a) Unit rate of land b) Age of the building
c) Location d) Marketability
178

Data :

Size of plot = 50’ x 200’


Market rate of 40’ x 60’ plot = Rs. 600/-
Standard depth assumed as = 60’

Calculations :

Standard depth is assumed as 60’

Size of I belt = 50’ x 60’


Area of I belt = 3,000 sq.ft. (1)
Unit rate 100%of 600 = Rs. 600/sq.ft.
Value of I belt - 3,000 x 600 = Rs. 18,00,000/- (4)
Size of II belt = 50’ x 90’
Area of II belt = 4,500 sq.ft. (2)
Unit rate (2/3) x 600 = Rs. 400/sq.ft.
Value of II belt- 4,500 x 400 = Rs. 18,00,000/-

Size of III belt = 50’ x 50’


Area of III belt = 2,500 sq.ft. (3)
Unit rate 50% of 600 = Rs. 300/sq.ft.
Value of III belt- 2,500 x 300 = Rs. 7,50,000/-

Total value of plot 50’ x 200’ = Rs. 43,50,000/- (5)


18,00,000 + 18,00,000 + 7,50,000

Marketability is the most important factor while certifying the (6)


market value for collateral security to bank purposes.

Answers :

1. 3,000 sq.ft 4. Rs.18,00,000/-


2. 4,500 sq.ft. 5. Rs. 43,50,000/-
3. 2,500 sq.ft. 6. Marketability

Exercise 3 : (IBBI)

Value the plot of 150 m x 350 m by belting method. The depth of first belt X is 50 m. The
depth of second belt is 2X. The depth of third belt is 4X.
179

Land rate for I belt = Rs. 300/m2 150 m

200 m
2 x (2X)
Rate for II belt = 40% less from I belt

4X
III belt
Rate for III belt = 40% less from II belt

350 m

100 m
2X

2X
II belt
Questions :

50 m
I belt

X
X
150 m
1. What is the value of I belt?
Road
2. What is the value of II belt?
3. What is the value of III belt?
4. What is the total value of the plot 150 m x 350 m?
5. What is the name of the method?

Answers :

1. I belt : 150 x 50 x Rs. 300/sq.m. = Rs. 22,50,000/-

2. II belt : 150 x 100 x (300 x 0.6) = Rs. 27,00,000/-

3. III belt : 150 x 200 x (180 x 0.6) = Rs. 32,40,000/-

4. Value of the entire plot


22,50,000 + 27,00,000 + 32,40,000 = Rs. 81,90,000/-

5. Hypothetical Plotting Scheme.

Exercise 4 : (IBBI)

In a situation, subject land is located in such a place where, instances of sale of large size
plots in the locality are not available. Small sized road side developed plots are available at
the rate of Rs. 300 per sq.m. Plot is located in developing area of town where demand for
housing site exists. The subject land is not surrounded by agricultural lands. The subject plot
is of sufficiently large size which can be divided into several small size plots. The depth of the
plot is 450 meters considerably more as compared to the road frontage of 150 meters.

Questions :

1. What is value of 1st portion from road side if the plot is considered as 50 metres in
depth in Rs.?

a) Rs. 22,50,000/- b) Rs. 20,50,000/-


c) Rs. 15,00,000/- d) Rs. 18,75,000/-
180

2. What is value of 2nd portion from road side if the plot is considered as 100 metres
in depth and rate considered for 40 per cent lesser than the 1st one in Rs.?

a) Rs. 27,00,000/- b) Rs. 8,00,000/-


c) Rs. 30,00,000/- d) Rs. 45,00,000/-

3. What is value of 3rd portion from road side if the plot is considered as rest of the
plot and rate considered for 40 per cent lesser than the 2nd one in Rs.?

a) Rs. 32,40,000/- b) Rs. 14,40,000/-


c) Rs. 21,60,000/- d) Rs. 54,00,000/-

4. What is the value of entire land?

a) Rs. 98,10,000/- b) Rs. 88,10,000/-


c) Rs. 78,10,000/- d) Rs. 68,10,000/-

5. As Gujarat HC said this method of valuation is arbitrary & artificial, instead of that
which method of valuation is accepted in case of huge plot area to be valued?

a) Plotting scheme method b) Sales comparison method


c) Net present value method d) Transfer of development
right method

Solution :
300 III Portion

Hypothetical plotting scheme :


450 100 II Portion
Unit rate = Rs. 300/sq.m.

50 I Portion
150’
Road

1. Area of I portion = 50 x 150 = 7,500 sq.m.


Unit rate = Rs. 300/m2
Value of I portion = 7,500 x 300
= Rs. 22,50,000/-

Ans : “a”
181

2. Area of II portion = 100 x 150 = 15,000 sq.m.


Unit rate = 40% less then I portion
(0.6 x 300 = 180/sq.m.)
Value of II portion = 15,000 x 180
= Rs. 27,00,000/-

Ans : “a”

3. Area of III portion = 300 x 150 = 45,000 sq.m.


Unit rate = 40% less than II portion
(0.6 x 180 = Rs. 108/sq.m.)
Value of III portion = 45,000 x 108
= Rs. 48,60,000/-

Answer not tallying.

4. Value of I portion = Rs. 22,50,000


Value of II portion = Rs. 27,00,000
Value of III portion = Rs. 48,60,000
Value of all portions = Rs. 98,10,000/-

Ans : “a”

5. Ans “a” - Plotting scheme method.

* * *
182

7. VALUATION OF PETROL BUNK

Exercise 1 :

1,500 m2 of plot abutting a Highways is proposed to be taken on lease by a firm. Fix the
lease rent of the plot, if yield rate is 6% & land rate is Rs. 4,000/m2.

Solution :
Value of land = 1,500 x 4,000
= Rs. 60,00,000/-

6 1
Lease rent = 60,00,000 x x
100 12

= Rs. 30,000 / month

Exercise 2 : (IBBI)

A client wants to purchase a petrol bunk outlet situated on the main road in the center of
town. The main road has traffic of 300 PCU. For the land, the company pays the rent
Rs. 4,00,000/ per annum. Total annual income from sale of petrol and diesel and other
items is Rs. 2,00,00,000/-. Property tax Rs. 50,000/6 months. Staff salary and other out
goings are Rs. 60,000/ per month. Other expenses for running the business is Rs.
1,70,00,000/-. Rate of capitalisation is 12%.

Question :

1. What is the total income for the owner?


2. What is the total expense for the owner?
3. What is the net profit?
4. What is the method to be used?
5. What is Y.P.?
6. What is the amount the client can pay to purchase the bunk?

Answers :

Given data :

Ground rent = Rs. 4,00,000 / annum


183

Income from sale of petrol = Rs. 2,00,00,000 / annum


Property tax = Rs. 50,000 / 6 months
Staff salary & other out goings = Rs. 60,000 / month
Other expenses = Rs. 1,70,00,000 / yearly
Rate of capitalisation = 12%

Solution :

1.0. Income to the owner of the bunk

Ground rent = Rs. 4,00,000


Income from sale of petrol&diesel= Rs. 2,00,00,000
Total income for the owner = Rs. 2,04,00,000 (1)

2.0. Expenses for the owner

Property tax (50,000 x 2) = Rs. 1,00,000


Staff salary & other out goings = Rs. 7,20,000
(60,000 x 12)
Other expenses for running the = Rs. 1,70,00,000
business
Total expenses = Rs. 1,78,20,000 (2)

3.0. Net profit

Net income / Net profit = 2,04,00,000 - 1,78,20,000


Net profit = Rs. 25,80,000 (3)

100
4.0. Years purchase = = 8.33 (4)
12

5.0. Method = Profit method (5)

6.0. Value

Value of the bunk by capitalising = 25,80,000 x (100 / 12)


@ 12%

Capitalised value = Rs. 2,15,00,000/-


184

... The amount that can be paid for the = Rs. 2.15 crores (6)
purchase of the bunk

Exercise 3 :

A petrol bunk is situated on the main road. Mr. ‘X’ is the dealer of the bunk and he gets a lease
rent (ground rent) from the petroleum company. Other details are :
(Courtesy : Mr. R.K. Gandhi)

Sale of Petrol = 2,42,000 litres @ Rs. 76/lit


Sale of Diesel = 3,75,000 litres @ Rs. 47/lit
Lease rent paid to the dealer = Rs. 4,00,000 /year
by company for the land

Sale of car parts, oil and lubricants= Rs. 6,00,000/year


Profit of sale of goods = 15%
Cars to be serviced/year = 1,600 nos.
Charge for servicing the car = Rs. 700/car

Cost of petrol = Rs. 71/lit


Cost of diesel = Rs. 42/lit
Staff salary = Rs. 50,000/month
Property taxes = Rs. 35,000/half yearly
Electricity charges = Rs. 41,000/year
Telephone, postage & stationery = Rs. 26,000/year
Travelling & maintenance = Rs. 21,000/year
Insurance premium = Rs. 8,000/year
Grease & Oil for car service = Rs. 40 /car
Miscellaneous expenses = Rs. 36,000/year
Rate of return = 12%

Questions :

1. What is the net profit for the dealer?


2. What is the years’ purchase, If the rate of capitalisation is 12%?
3. What is the value of the property for the purpose of purchasing assuming a rate of
return as 12%?
185

1.0. Income

Petrol sale amount : 2,42,000 lit x 76/lit = Rs. 1,83,92,000


Diesel sale amount : 3,75,000 lit x 47/lit = Rs. 1,76,25,000
Ground rent for land received from the petrol = Rs. 4,00,000
company
Sale of oil & lubricants of car parts = Rs. 6,00,000
Income from car services : 1,600 x Rs. 700 = Rs. 11,20,000
Gross income = Rs. 3,81,37,000

2.0. Expenses

Cost of petrol : 2,42,000 x 71 = Rs. 1,71,82,000


Cost of diesel : 3,75,000 x 42 = Rs. 1,57,50,000
Staff salary : 50,000 x 12 = Rs. 6,00,000
Property tax : 35,000 x 2 = Rs. 70,000
Electricity & water charges = Rs. 41,000
Telephone, postage & stationery = Rs. 26,000
Travelling & Maintenance expenses = Rs. 21,000
Insurance premium = Rs. 8,000
Car service - grease / oil (1,600 x 40) = Rs. 64,000
Misecellaneous expenses = Rs. 36,000
Expenses for sale of oil & lubricants = Rs. 5,10,000
85% of 6,00,000
Gross expenses total = Rs. 3,43,08,000

3.0. Net profit = 3,81,37,000 - 3,43,08,000


= Rs. 38,29,000/year (1)

4.0. Years’ purchase

Rate of capitalisation = 12
Years purchase (100 / 12) = 8.33 (2)

5.0. Capitalising at 12% yield, value of business = Rs. 38,29,000 x (100/12)


= Rs. 3,19,08,333
say Rs. 3,19,00,000/- (3)

... Purchase price for the outlet is Rs. 3,19,00,000/-.

* * *
186

8. VALUATION OF LEASEHOLD PROPERTIES

Exercise 1 :

A freehold site is rented out for 99 years to a developer at a ground rent of Rs. 1,00,000
per annum, net of outgoings. It is renewable. The lessee developer has constructed a
building fetching an annual rent of Rs. 5,00,000/-. Value the freeholder’s interest
assuming an yield of 6%.

Value in the hands of lessor :


Net income from ground rent = Rs. 1,00,000
Yield = 6%
100
Years purchase = = 16.67
6
Value in the hands of lessor = 1,00,000 x 16.67
= Rs. 16,67,000/-

Exercise 2 :

Value the freehold interest of a shop which has been let out for a rent of Rs. 1,00,000 (Net)
per month. The rent is renewable. Yield is 5%.

Yearly rent = 1,00,000 x 12 = Rs. 12,00,000


Net income = Rs. 12,00,000
100
Y.P. for a yield of 5% = = 20
5
Capitalised value = 12,00,000 x 20
= Rs. 2,40,00,000/-

Exercise 3 :

An industrial corporation has decided to lease 40,000 sq.ft. plot for an user for 60 years
period. The land rate is 2,000 per sq.ft. Assuming an yield of 6%, what will be the monthly
lease?

Extent of land = 40,000 sq.ft.


Market rate = Rs. 2,000/sq.ft.
Value of land = Rs. 8,00,00,0000
187

Lease rent yield = 6%


6
Annual rent = 8,00,00,000 x
100
= Rs. 48,00,000
48,00,000
Monthly rent = = Rs. 4,00,000/-
12

Exercise 4 :

A private trust had leased 10,000 sq.ft. plot for 99 years lease which can be renewed for
further period. Fix lease rent if the land rate is Rs. 1,500/sq.ft. Assume lease rent as 8%.

Extent of land = 10,000 sq.ft.


Land rate = Rs. 1,500/sq.ft.
Land value = Rs. 1,50,00,000
Lease rent yield assumed = 8%
8
Annual lease rent = 1,50,00,000 x
100
= 12,00,000
Monthly lease rent = Rs. 1,00,000/-

Exercise 5 :

A lessor leased his 3,000 sq.ft. of land to a lessee for 99 years on a monthly rent of
Rs. 1,000 per month. Lease is renewable.

In this land, the lessee has constructed a residential building and rented out on a total rent
of Rs. 5,500 / month. All outgoings are 40% of rental income excluding ground rent.

Questions :

1. What is the value of lessor’s interest? Rate of return (yield) is 7%.


2. What is the lessee’s interest assuming a rate of return as 8%.

Data :

Period of lease = 99 years - renewable


Monthly rent = Rs. 1,000
Rate of return = 7%
Lessor’s interest = ?
188

Calculations :

Lessor :
Monthly rent = Rs. 1,000
Yearly rent = 1,000 x 12 = Rs. 12,000
Type of lease = Perpetual. can be treated as free
hold
Rate of return / yield = 7%
100
Value of lessor’s right = 12,000 x
7
= Rs. 1,71,428/- (1)

Lessee :
Monthly rent = Rs. 5,500
Yearly rent = 5,500 x 12 = Rs. 66,000
Less outgoings 40% = (-) Rs. 26,400
Less ground rent 1,000 x 12 = (-) Rs. 12,000
Net annual income = Rs. 27,600
Rate of return = 8%
100
Value of lessee’s interest = 27,600 x
8
= Rs. 3,45,000/- (2)

Answers :

1) 1,71,428/- 2) Rs. 3,45,000/-

Exercise 6 : (IBBI)

State government industrial development corporation leased industrial plot to the


industrialist in the year 1998 for a period of 99 years by charging one time premium of
Rs. 450/sq.mt. for a total land area of 4,000 sq.mt. Lease rent was fixed at Rs.1 per year.
Lessee built a factory (total built up area 2,000 sq.mts) on the plot in 1998 at the cost of
Rs. 60,000/-. Land rate as on 2018 is Rs. 1,250/sq.mt and replacement cost of building for
2018 is Rs. 25,000/sq.mt.. Total life of the factory building is 40 years. Lease provides that
the lessor is entitled to charge 50 percent unearned increase in land value as transfer/as-
signment charges in case of sale/tranfer of the property.

Questions :

1. What is the lessor’s interest in the property in 2018?


189

a) Rs. 32,00,000/- b) Rs. 16,00,000/-


c) Rs. 50,00,000/- d) Nil

2. What is the market value of the property in 2018 if land was not of leasehold tenure
and it was a free hold land? Salvage value - 10%.

a) Rs. 7,75,00,000/- b) Rs. 10,00,00,000/-


c) Rs. 7,50,00,000/- d) Rs. 5,00,00,000/-

3. What is the lessee’s interest in property in 2018?

a) Rs. 5,00,00,000/- b) Rs. 7,75,00,000/-


c) Rs. 2,75,00,000/- d) Rs. 7,59,00,000/-

4. Depreciation amount of the factory value in 2018 on striaght line method of


depreciation and assuming 10% scrap value?

a) Rs. 2,50,00,000/- b) Rs. 2,25,00,000/-


c) Rs. 2,75,00,000/- d) Rs. 50,00,000/-

5. Which of the following statement is correct?

a) Lessor’s interest in the property is right to receive 50% unearned increase in


land value only.

b) Lessor’s interest in the property is value of right to receive lease rent in


property plus right to receive 50% unearned unearned increase in land value.

c) Lessee’s interest is estimated by estimating capitalised value of profit rent


receivable for 20 years.

d) Lessee’s interest in the property is to be increased by amount of 50%


unearned increase in land value payable to lessor as per P. N. Sikand’s
case.

6. Which of the following statement is true?

a) Lessor is entilted to take income tax of the depreciation of the building.


b) Lessor' s interest in the property is nil.
c) Lessee is virtually having a right to use property for life time only.
d) Balance economical life is 20 years.
190

Data :

Year of lease = 1998


Period of lease = 99 years
One time premium = Rs.450/sq.m. for land extent
Land area = 4,000 sq.m.
Lease rent = Re. 1/year
Lessee built a factory of builtup area = 2,000 sq.m.
Year of construction of factory by the = 1998
lessee
Cost of factory (Lessee) = Rs. 60,000/-
Land rate as on 2018 (date of valuation) = Rs. 1,250/sq.m.
Replacement cost of building in 2018 = Rs. 25,000/sq.m.
Total life of the building = 40 years

Condition : Lessor is entitled to charge 50% unearned increase in land


value in case of sale / transfer.

Opinion :

1. Lessor’s interest :

This case of lease of land is by state government. It is assumed as a perpetual


lease and reversionary value of land is negligible. The lease rent is only Re. 1/year
and hence its capitalised value will be negligible. Lessor’s interest in land value
would be therefore is restricted to claim 50% of unearned increase in land value in
case of sale.

Land area = 4,000 sq.m.


Prevailing land rate 2018 = Rs. 1,250/sq.m.
One time premium charged in 1998 = Rs. 450/sq.m.
Unearned increase 1,250 - 450 = Rs. 800/sq.m.
The percentage the lessor is entitled to = 50%
charge in case of transfer
Unearned increase the lessor can enjoy = 0.5 x 800 = Rs. 400/sq.m.
The lessor’s value 4,000 x 400 = Rs. 16,00,000/- (1)

.
. . The answer is ‘b’.
191

2. Value of property assuming it is a freehold :

(i) Land :

Land area = 4,000 sq.m.


Unit rate of land = Rs. 1,250/sq.m.
Land value - 4,000 x 1,250 = Rs. 50,00,000/-

(ii) Building :

Building area = 2,000 sq.m.


Replacement cost = Rs. 25,000/sq.m.
Replacement value = 2,000 x 25,000
= Rs. 5,00,00,000/-
Age of the building : 2018 - 1998 = 20 years
Life of the factory = 40 years
Salvage value assumed = 10%
20
Depreciation percentage = x 90 = 45%
40
Depreciation value = 0.45 x 5,00,00,000
= Rs. 2,25,00,000/-
Depreciated value = Rs. 2,75,00,000/-
5,00,00,000 - 2,25,00,000

(iii) Total value :

Value of land = Rs. 50,00,000/-


Depreciated value of building = Rs. 2,75,00,000/-
Total value = Rs. 3,25,00,000/- (2)

(Note : The options given in the question is not tallying with this answer).

3. Value of lessee’s interest :

Total value of land : 4,000 x 1,250= Rs. 50,00,000/-


Value of lessee’s interest = Total value of land - Value of
lessor’s interest
= 50,00,000 - 16,00,000
= Rs. 34,00,000/-
192

Lessee also holds in the building value.

Depreciated value of building = Rs. 2,75,00,000/-

Total value :
Land = Rs. 34,00,000
Depreciated value of building = Rs. 2,75,00,000/-
= Rs. 3,09,00,000/-

(Note : The options given in the question is not tallying with this answer).

4. Depreciation amount by straight line method

20
= 2,000 x 25,000 x ( x 90)
40

= 0.45 x 5,00,00,000
= Rs. 2,25,00,000/-

.
. . The answer is ‘b’.

5. Lessor’s interest in the property is right to receive 50% unearned increase in land
value only.

.
. . The answer is ‘a’.

6. Total life = 40 years


Age 2018 - 1998 = 20 years
Balance economic life = 40 - 20 = 20 years

.
. . The answer is ‘d’.

Exercise 7 :

A government M.I.D.C. gives 8,000 sq.m. of land on 99 years lease @ 1/- P.A. lease rent
and charged one time premium of Rs. 450 / sq.m. in the year 1998. The lessee in the year
1998 constructed an industrial shed 4,000 sq.m. of BU area with his own expenditure. The
age of the shed is 20 years as on year 2018 and total life of the shed is 40 years. The land
rate is Rs. 2,000 / sq.m. and replacement cost is Rs. 25,000 / sq.m. Lease provides that
the lessor is entitled to charge 50% unearned increase in land value as transfer / assignment
charges in case of sale / transfer of the property. Calculate the following :
193

1. What is the lessors interest?


2. What is the total value of property considering a freehold property?
3. What is the lessee interest?
4. What is the reversionary value of the leasehold land?
5. What is the depreciated value of shed?

Data :

Year of lease = 1998


Period of lease = 99 years
One time premium = Rs.450/sq.m. for land extent
Land area = 8,000 sq.m.
Lease rent = Re. 1/year
Lessee built a factory of builtup area = 4,000 sq.m.
Year of construction of factory by the = 1998
lessee
Land rate as on 2018 (date of valuation) = Rs. 2,000/sq.m.
Replacement cost of building in 2018 = Rs. 25,000/sq.m.
Total life of the building = 40 years
Date of valuation = 2018
Age of the shed 2018 - 1998 = 20 years

Opinion :

1. Lessor’s interest :

This case of lease of land is by state government. It is assumed as a perpetual


lease and reversionary value of land is negligible. The lease rent is only Re. 1/year
and hence its capitalised value will be negligible. Lessor’s interest in land value
would be therefore is restricted to claim 50% of unearned increase in land value in
case of sale.

Land area = 8,000 sq.m.


Prevailing land rate 2018 = Rs. 2,000/sq.m.
One time premium charged in 1998 = Rs. 450/sq.m.
Unearned increase 2,000 - 450 = Rs. 1,550/sq.m.
The percentage the lessor is entitled to = 50%
charge in case of transfer
Unearned increase the lessor can enjoy = 0.5 x 1,550 = Rs. 775/sq.m.
The lessor’s value - 8,000 x 775 = Rs. 62,00,000/- (1)
194

2. Value of property assuming it is a freehold :

(i) Land :

Land area = 8,000 sq.m.


Unit rate of land = Rs. 2,000/sq.m.
Land value - 4,000 x 1,250 = Rs. 1,60,00,000/-

(ii) Building :

Building area = 4,000 sq.m.


Replacement cost = Rs. 25,000/sq.m.
Replacement value = 4,000 x 25,000
= Rs. 10,00,00,000/-

Age of the building : 2018 - 1998 = 20 years


Life of the factory = 40 years
Salvage value assumed = 10%
20
Depreciation percentage = x 90 = 45%
40
Depreciation value = 0.45 x 10,00,00,000
= Rs. 4,50,00,000/-
Depreciated value = Rs. 5,50,00,000/-
10,00,00,000 - 4,50,00,000

(iii) Total value :

Value of land = Rs. 1,60,00,000/-


Depreciated value of building = Rs. 5,50,00,000/-
Total value = Rs. 7,10,00,000/- (2)

3. Value of lessee’s interest :

Total value of land : = Rs. 1,60,00,000/-


8,000 x 2,000

Value of lessee’s interest = 1,60,00,000 - 62,00,000


= Rs. 98,00,000/-

Lessee also holds in the building value.


195

Depreciated value of building = Rs. 5,50,00,000/-

Total value :

Land = Rs. 98,00,000


Building = Rs. 5,50,00,000
= Rs. 6,48,00,000/-

4. Reversionary value is negligible and hence not considered.

5. Depreciated value of shed = Rs. 5,50,00,000/-

Exercise 8 : (IBBI)

A warehouse property is situated close to a port facility in a major port twn. It is let out on a
50 years lease. The lessee is paying to the lessor an exclusive ground rent @ INR 2,000
per annum, after payment of an one time premium of INR 25,00,000. The rack rental value
on full repairing terms amounts to INR 1,20,000 per annum. The yield from freehold ware
houses in similar locations is considered to be 10% and for long term lease is 15%.
(Valuation of Real property : Page no. 69 - Mr. Symales Datta)

Questions :

1. What is the outgoing for lessor?


2. What is the net income for the lessor during the term period?
3. What is the YP during the term period?
4. What is the YP during the reversionary value calculations?
5. What is the value of freeholder’s interest?
6. What is the market rent?

Calculation :

Data :

Lease = 50 years
Ground rent to lessor = Rs. 2000/- per annum
Premium paid to lessor = Rs. 25,00,000/-
Rack Rent on full repairing terms = Rs. 1,20,000/- per annum
Yield for freehold ware houses = 10%
Yield for long term lease = 15%
196

Answers :

1. What is the outgoing for lessor?

The outgoing for lessor is nil on the assumption that the lease is on full repairing
terms, however this is not specifically mentioned in the question. But could be
inferred as such, as the rack rent mentioned is on full repairing terms.

2. What is the net income for the lessor during the term period?

Rs. 2000/- per annum

3. What is the YP during the term period?

Y.P. = 100 / 15 = 6.66

4. What is the YP during the reversionary value calculations?

The YP during reversionary value calculations is 100 / 10 = 10.

5. What is the value of freeholder’s interest?

Value of freeholder’s interest = value of term + value of reversion

i) Value of term (lessor’s interest) = 2000 x 6.66 = 13,320

ii) Value of reversion


Market value = Rs. 1,20,000/-
Y.P. @ 10% = 100 / 10 = 10
Capitalised value = 1,20,000 x 10 = 12,00,000
Y.P. in perpetuity deferred 1
= = 0.008518
50 years @ 10% (1 + 0.1)50
Value of reversion = 12,00,000 x 0.008518
= 10,222

iii) Value of freeholder’s interest = 13,320 + 10,222


= Rs. 23,542/-

6. What is the market rent?

Rs. 1,20,000/- per annum.

(Courtesy : Mr. Mahendra Kakule)

* * *
197

9. VALUATION BY PROFIT METHOD

Exercise 1 :

A hotel has 100 rooms. Room rent is Rs. 1,500/day. Occupancy ratio is 65%. Income from
restaurant is Rs. 200 lakhs/year. Conference hall rental income is Rs. 150 lakhs/year.
Corpoartion tax, Electricity, insurance and other expenses are Rs. 200 lakhs. Staff salary
Rs. 125 lakhs. Food & beverage expenses are Rs. 150 lakhs. Miscellaneous expenses
Rs. 50 lakhs. Ascertain the value of the hotel by profit method assuming an yield as 10%.

1. Gross income :

a. From Rooms

Number of rooms = 100


Daily rent = Rs. 1,500/day
Income from 100 rooms/day = 1,500 x 100 = Rs. 1,50,000
Income from 100 rooms/year = 1,500 x 100 x 365
Occupancy ratio = 65%
Yearly income for 65% = 1,500 x 100 x 365 x 0.65
= Rs. 3,55,87,500/-

b. Income from restaurant = Rs. 2,00,00,000/-

c. Income from conference hall = Rs. 1,50,00,000/-

Gross income = Rs. 7,05,87,500/-

2. Expenses :

Corporation tax, electricity, etc. = Rs. 2,00,00,000


Staff salary = Rs. 1,25,00,000
Food & beverage expenses = Rs. 1,50,00,000
Miscellaneous expenses = Rs. 50,00,000
Total expenses = Rs. 5,25,00,000/-
198

3. Net income :

Gross income = Rs. 7,05,87,500


Expenses = Rs. 5,25,00,000
Net income = Rs. 1,80,87,500/-

4. Value :

Net income = Rs. 1,80,87,500


Yield = 10%
100
Value = 1,80,87,500 x
10

= Rs. 18,08,75,000/-

Exercise 2 :

It is a marriage hall in a town. The daily rental charge is Rs. 25,000/-. The number of booking
per year is 50 percent. Expenses are : Property tax - Rs. 25,000/half year, Staff salary - Rs.
40,000/month, Yearly Insurance - Rs. 35,000/-, Repairs & Maintenance - Rs. 15,000/month,
Electricity - Rs. 50,000/month, Miscellaneous expenses - Rs. 25,000/month, Management
expenses : Rs. 1,00,000/month.

Determine the value by profit method assuming an yield of 12%.

1. Gross income :

Daily rental charges = Rs. 25,000


Yearly rental charge = 25,000 x 365
Occupation ratio = 50%
Gross income = 25,000 x 365 x 0.5
= Rs. 45,62,500/-

2. Expenses :

Property tax 25,000 x 2 = Rs. 50,000


Staff salary 40,000 x 12 = Rs. 4,80,000
Insurance = Rs. 35,000
Repairs & maintenance = Rs. 1,80,000
15,000 x 12
199

Electricity - 50,000 x 12 = Rs. 6,00,000


Miscellaneous expense = Rs. 3,00,000
25,000 x 12
Management expense = Rs. 12,00,000
1,00,000 x 12
Total expenses = Rs. 28,45,000/-

3. Net income :

Gross income = Rs. 45,62,500


Expenses = Rs. 28,45,000
Net income = Rs. 17,17,500

4. Value :

Net income = Rs. 17,17,500


Yield = 12%
17,17,500
Value = x 100
12

= Rs. 1,43,12,500/-

* * *
200

10. BANK VALUATION

Exercise 1 :

In a plot of 2,400 sq.ft., Mr. X has proposed to construct a building of 1,200 sq.ft. He has
obtained loan. Basement completed (25%). Land rate is Rs. 1,000/sq.ft. The unit
construction cost is Rs. 1,800/-. Determine the stage value of the property for primary
security purpose to bank.

Land value = 2,400 x 1,000 = Rs. 24,00,000


Building value = 0.25 x 1,200 x 1,800 = Rs. 5,40,000
Total value = Rs. 29,40,000/-

Exercise 2 :

The plot area is 3,000 sq.ft. The land rate is Rs. 1,500/sq.ft. The owner wishes to
construct a building of 3 floors of 1,200 sq.ft. each. The average unit rate of construction
is Rs. 1,600/-. The total estimated amount is Rs. 57.60 lakhs and the bank has sanctioned
a loan of 43.20 lakhs. The owner has completed 40% of the civil works. In order to pay the
first installment of loan, the bank directs the valuer to certify the stage cost of the building
alone.

Number of floors = 3
Built up area of each floor = 1,200 sq.ft.
Total built up area 3 x 1,200 = 3,600 sq.ft.
Unit rate of construction = Rs. 1,600/-
Total value of completion = 3,600 x 1,600
= Rs. 57,60,000/-
Stage precentage completed = 40%
Stage value = 0.4 x 57,60,000
= Rs. 23,04,000/-

Exercise 3 :

In the year April 2018, Mr. X has purchased plot of 2,400 sq.ft. for Rs. 24,00,000. In the
same year (April to December) he has constructed a residential building for Rs. 18,00,000.
He wants to sell. He quoted (Jan 2019) Rs. 48,00,000/-. The borrower approached the
bank and the bank directed its panel valuer to inspect the site and give a report. The
valuer certified as Rs. 45,00,000/-as on February 2019.
201

Now,
1) What is the cost of the property for 2018 - 19?
2) What is the price?
3) What is the value?

Answers :

1) Cost = 24,00,000 + 18,00,000 = Rs. 42,00,000/-


2) Price is Rs. 48,00,000/-
3) Value is Rs. 45,00,000/-

* * *
202

11. GROUND RENT

Exercise 1 :

Mr. ‘X’ is owning a vacant site of 8,000 sq.ft. near the bus stand. He wants to let out. The
prevailing unit market rate is Rs. 1,000 and the guideline rate is Rs. 1,500/sq.ft. Mr. Y
wants this site for parking vehicles. Mr. Z also wants this site and wishes to construct a
shed. Assume rate of return of 4% for secured ground rent and 5% for unsecured ground rent.

Questions :

1. What is the market value to determine the rent for Mr. Y?


2. What is the market value to determine the rent for Mr. Z?
3. What is the yearly ground rent that can be fixed for Mr. Y?
4. What is the monthly ground rent that can be fixed for Mr. Y?
5. What is the yearly ground rent that can be fixed for Mr. Z?
6. What is the monthly ground rent that can be fixed for Mr. Z?

Data :

Extent of site = 8,000 sq.ft.


Market rate of site = Rs. 1,000/sq.ft.
Guideline rate = Rs. 1,500/sq.ft.
Rate of return for secured = 4%
ground rent
Rate of return for unsecured = 5%
ground rent

Calculations :

For Y & Z :

Extent of site = 8,000 sq.ft.


Prevailing unit rate = Rs. 1,000/sq.ft.
Market value (for Y & Z) = Rs. 80,00,000/- (1&2)
8,000 x 1,000
203

For Y :

Market value = Rs. 80,00,000


Type of rent = Unsecured
Rate of return assumed = 5%
Yearly ground rent = 80,00,000 x (5/100)
= Rs. 4,00,000/- (3)

Monthly ground rent = 4,00,000 / 12


= Rs. 33,333/- (4)

For Z :

Market value = Rs. 80,00,000


Type of rent = Secured
Rate of return assumed = 4%
Yearly ground rent = 80,00,000 x (4/100)
= Rs. 3,20,000/- (5)

Monthly ground rent = 3,20,000 / 12


= Rs. 26,667/- (6)

Answers :

1) Rs. 80,00,000/- 4) Rs. 33,333/-


2) Rs. 80,00,000/- 5) Rs. 3,20,000/-
3) Rs. 4,00,000 /- 6) Rs. 26,667/-

* * *
204

12. VALUATION OF TENANTED PROPERTIES

Exercise 1 :

The monthly rent (Net) of a shop of 540 sq.ft. is Rs. 12,000/-. Calculate the approximate
value by rent capitalisation method by adopting a rate of return as 5%.

Monthly rent = Rs. 12,000


Yearly rent = 12,000 x 12
= Rs. 1,44,000/-
Rate of return adopted = 5%
100
Capitalised value = 1,44,000 x
5
= Rs. 28,80,000/-

Exercise 2 :

The net monthly rent of a residential building of 1,250 sq.ft. is Rs. 16,500/-. Find the
approximate value of the property by rent capitalisation method by adopting a rate of
return as 3%.

Monthly rent = Rs. 16,500


Yearly rent = 16,500 x 12
= Rs. 1,98,000/-
Rate of return = 3%
100
Capitalised value = 1,98,000 x
3
= Rs. 66,00,000/-

Exercise 3 :

A new shop was purchased for Rs. 10,00,000 which was rented out for Rs. 5,000 per
month. What is the yield?

Capital value = Rs. 10,00,000


Yearly rent = Rs. 5,000 x 12 = Rs. 60,000
60,000
Yield = x 100
10,00,000
= 6%
205

Exercise 4 : (IBBI)

A fully developed building in a plot has a total of 4 floors. Total plot area is 1,000 sq.m. and
total builtup for area of the building is 250 sq.m / per floor. Permissible FSI is 1.00. There
are 4 tenants per floor and tenants of lower 2 floors pay a rent of
Rs. 750 / month / tenement. which includes property tax. Top 2 floors are occupied by the
owners of the property itself. Total property taxes are Rs. 25,000 / 6 months for 4 floors.

Tenant’s rent includes 50% of total tax, Non - agricultural (N.A.) tax of the plot is Rs. 800 /
year and building insurance premium is Rs. 1,000 / year.

Assume repair cost at 6% of the gross rent and collection & management charges at 3%
of the gross rent. Stamp duty paid at the time of purchase is Rs. 9,000/-. The land is of
freehold tenure. Prevalent land rate of freehold land in the locality at present is Rs. 8,000/
sq.m. The rate of ownership flats in the locality for similar construction as on today is
Rs. 30,000/sq.m.

Questions :

1. What will be the total annual rent receivable by the landlord from all the tenants?

a) Rs. 6,000/- b) Rs. 72,000/-


c) Rs. 1,44,000/- d) Rs. 12,000/-

2. What will be the total outgoings including repairs allowance & collection charges
for the tenanted portion of the building?

a) Rs. 32,380/- b) Rs. 57,380/-


c) Rs. 33,280/- d)

3. What will be the present market value of the tenanted portion of the building if
rental income is assumed to be in perpetuity & rate of capitalisation is adopted
@ 8%

a) Rs. 9,90,500/- b) Rs. 1,50,00,000/-


c) Rs. 77,50,000/- d) Rs. 4,95,250/-

4. What will be the present market value of the owner occupied portion of the
building?
206

a) Rs. 75,00,000/- b) Rs. 1,50,00,000/-


c) Rs. 10,00,000/- d) Rs. 78,00,000/-

5. Which of the following is not considered as outgoing for computing net rent
received by the landlord?

a) Property tax b) Repair cost


c) Stamp duty paid d) Management charges

6. What is the market value of the balance potential in the property?

a) Rs. 1,50,000/- b) Rs. 15,00,000/-


c) Zero d) Reversionary value of the land

Data :

Property tax for 4 floors = Rs. 25,000 / 6 months


Non Agricultural tax for Mumbai = Rs. 800 / year
Building insurance = Rs. 1,000 / year
Repair cost & maintenance = 6% Gross rent
Rent collection charge = 3% Gross rent
Market rate of land = Rs. 8,000 / sq.m.
Prevalent unit rate of flat = Rs. 30,000 / sq.m.

Opinion :

1. Rent received by the owner :

Tenants occupied portions = GF & FF


Number of tenants in each flat = 4
Total number of flats in all flats = 2x4=8
Monthly rent for each flat = Rs. 750/-
Monthly rent for all flats = 750 x 8 = Rs. 6,000
Yearly rent for all flats = 6,000 x 121 = Rs. 72,000/-

The answer is “b”.

2. Outgoings :

Property tax = Rs. 50,000


N.A. (Non-Agricultural tax) = Rs. 800
Insurance premium = Rs. 1,000
= Rs. 51,800
207

Since the tenants are bearing 50% of the


above expenses, the actual outgoings
of the owner = Rs. 25,900
Maintenance charges 6% of gross rent = Rs. 4,320
0.06 x 72,000
Rent collection charge 3% of gross rent = Rs. 2,160
0.03 x 72,000
Total outgoings = Rs. 32,380/-

The answer is “a”.

3. Capitalisation amount :

Gross income = Rs. 72,000


Outgoes = Rs. 32,380
Net income = Rs. 39,620
Yield = 8%
Capitalised amount = 39,620 x (100 / 8)
= Rs. 4,95,250/-

The answer is “d”.

4. Value of the building - free holder (land owner) :

FSI = 1
Area of the flat 2 x 250 = 500 sq.m.
Unit rate of flat = Rs. 30,000/sq.m.
Value 500 x 30,000 = Rs. 1,50,00,000/-

The answer is “b”.

5. While computing net rent received by the landlord, Stamp duty is not to be
considered.

.
. . The answer is “c”.

6. The market value of the balance potential in the property is zero.

.
. . The answer is “c”.
208

Exercise 5 : (IBBI)

An apartment carries 4 floors built on a plot of area 1,000 sq.m. Each floor area is
250 sq.m. The GF & FF have been rented and SF & TF is in possession of the owner. Each
floor carries 4 tenements, and tenants pay @ Rs. 750 / tenement as rent. The property tax
being paid is @ Rs. 25,000 / six month. Rs. 900 / year is non agri - tax. 6% per annum
towards management cost. Rs. 9,000/- stamp duty cost. 3% towards rent collection
charge. Cost of land is Rs. 2,000 / sqm and cost of construction is Rs. 25,000 / sqm, FSI
is 1. Calculate the following :

1. What is the total rent?


2. What is the total outgoes?
3. What is the valuation of owner occupied portion?
4. What is the balance potential in building?
5. What is the depreciated cost of building?
6. What is the tenanted portion value @ rate of return of 8% PA?
Data :

Property tax = Rs. 25,000 / 6 months


Non Agricultural tax (for Mumbai) = Rs. 900 / year
Management cost = 6%
Stamp duty = Rs. 9,000/-
Cost of land = Rs. 2,000/sq.m.
Cost of construction = Rs. 25,000/sq.m.
Rent collection charge = 3%

Opinion :

1. Rent received by the owner :

Tenants occupied portions = GF & FF


Number of tenants in each flat = 4
Total number of flats in all flats = 2x4=8
Monthly rent for each flat = Rs. 750/-
Monthly rent for all flats = 750 x 8 = Rs. 6,000
Yearly rent for all flats = 6,000 x 121 = Rs. 72,000/-

2. Outgoings :

Note : (It is assumed that the tenents are bearing 50% of the property tax, N.A.
tax). It is the practice in Maharashtra.
209

Property tax = Rs. 50,000


N.A. (Non-Agricultural tax) = Rs. 900
= Rs. 50,900

Since the tenants are bearing 50% of the


above expenses, the actual outgoings
for the owner = Rs. 25,450

Management charges 6% of gross rent = Rs. 4,320


0.06 x 72,000

Rent collection charge 3% of gross rent = Rs. 2,160


0.03 x 72,000

Total outgoings = Rs. 31,930/-

3. Value of the building - free holder (land owner) :

FSI = 1
Area of the flat 2 x 250 (SF & TF) = 500 sq.m.
Unit rate of flat = Rs. 25,000/sq.m.
Value - 500 x 25,000 = Rs. 1,25,00,000/-

4. The market value of the balance potential in the property is zero.

5. Data is not adequate to calculate the depreciated cost of the building.

6. Capitalisation amount :

Gross income = Rs. 72,000


Outgoes = Rs. 31,930
Net income = Rs. 40,070
Yield = 8%
Capitalised amount = 40,070 x (100 / 8)
= Rs. 5,00,875/-

* * *
210

13. RESIDUAL VALUE METHOD

Exercise 1 :

Plot area = 3,000 sq.ft. Building area = 2,400 sq.ft. The age of the building = 20 years (Life
can be assumed as 60 years & salvage value as 10%). Replacement cost including
services is Rs. 1,800/sq.ft. This property was sold for Rs. 60,24,000. Calculate the land rate
by residual technique.

Building area = 2,400 sq.ft.

Replacement rate = 1,800/sq.ft.

Replacement value 2,400 x 1,800 = Rs. 43,20,000/-

Age of the building = 20 years

Life assumed = 60 years

Salvage value = 10%

Depreciation percentage = (20 / 60) x 90 = 30%

Depreciation value 0.3x43,20,000 = Rs. 12,96,000/-

Depreciated value of building = Rs. 30,24,000

Sale value (land + building) = Rs. 60,24,000

Value of land alone = Rs. 30,00,000


60,24,000 - 30,24,000

Extent of land = 3,000 sq.ft.

Rate of land 30,00,000 / 3,000 = Rs. 1,000/sq.ft.


211

Exercise 2 :

In a plot of 4,000 sq.ft., a flat promoter constructed 8 flats of 1,000 sq.ft. each. Building rate
including all services is Rs. 2,500/sq.ft. He sold one flat for Rs. 66,00,000/-. Assuming his
profit margin as 20%, calculate the land rate by residual technique.

Area of 8 flats 8 x 1,000 = 8,000 sq.ft.

Area of plot = 4,000 sq.ft.

FSI : 8,000 / 4,000 = 2

Selling price [(Land + Building) + = Rs. 66,00,000


+ Profit]

Unit rate of flat 66,00,000/1,000 = Rs. 6,600 (This is composite rate)

Promoter’s profit = 20%

Land & building excluding profit = 6,600 / 1.2


= Rs. 5,500/-

Deduct building unit rate = (-) Rs. 2,500

Land component alone = Rs. 3,000

FSI = 2

Land rate - 3,000 x 2 = Rs. 6,000/-

* * *
212

14. CAPITAL GAIN

Exercise 1 :

On 07.12.1989, a property was acquired by Mr. X for 8.08 lakhs. In June 1992,
improvements were made for 12.06 lakhs. On 10.12.2014, the property was sold to
1.93 crores. (172, 223, 1024 are the cost inflation index for 1989 - 90, 1992 - 93, 2014 - 15
respectively).

Questions :

1. What is the Indexed cost of acquisition?


2. What is the indexed cost of improvement?
3. What is the total cost of acquisition & improvement?
4. What is the taxable capital gain?
5. What is the tax to be paid by Mr. ‘X’?
6. If the property is owned by a company, what is the capital gain tax?

Calculations :

Date of acquisition = 07.12.1989 (1989 - 90)


Cost of acquisition (12/1989) = Rs. 8,08,000
Cost of improvements (6/1992) = Rs. 12,06,000 (1992 - 93)
Date of transfer = 10.12.2014 (2014 - 15)
Sale consideration = Rs. 1,93,00,000
Cost inflation index 1989 - 90 = 172
Cost inflation index 1992 - 93 = 223
Cost inflation index 2014 - 15 = 1,024

1. Indexed cost of acquisition = 8,08,000 x (1,024/172)


= Rs. 48,10,419/- (1)

2. Indexed cost of improvement = 12,06,000 x (1,024/223)


= Rs. 55,37,865/- (2)

3. Total indexed cost of acquisition = 48,10,419 + 55,37,865


& indexed cost improvement
= Rs. 1,03,48,284/- (3)
213

4. Taxable capital gain = 1,93,00,000 - 1,03,48,284


= Rs. 89,51,716/- (4)

5. Tax in the hand of Mr. ‘X’ - 20% = 0.2 x 89,51,716


= Rs. 17,90,343/- (5)

6. If it is owned by a company, = 0.4 x 89,51,716


tax - 40%
= Rs. 35,80,686/- (6)

Answers :

1) Rs. 48,10,419/- 4) Rs. 89,51,716/-


2) Rs. 55,37,865/- 5) Rs. 17,90,343/-
3) Rs. 1,03,48,284/- 6) Rs. 35,80,686/-

Exercise 2 :

On 09.01.1990, a property was acquired by Mr. X for 9.49 lakhs. In August 1992,
improvements were made for 14.76 lakhs. On 17.12.2014, the property was sold to
1.97 crores. 172, 223, 1024 are the cost inflation index for 1989 - 90, 1992 - 93, 2014 - 15
respectively.

Questions :

1. What is the indexed cost of acquisition?


2. What is the indexed cost of improvement?
3. What is the total indexed cost of acquistions & improvement?
4. What is the taxable capital gain?

Calculations :

Date of acquisition = 09.01.1990 (1989 - 90)


Cost of acquisition (12/1989) = Rs. 9,49,000/-
Cost of improvements (6/1992) = Rs. 14,76,000 (1992 - 93)
Date of transfer = 17.12.2014 (2014 - 15)
Sale consideration = Rs. 1,97,00,000
Cost inflation index 1989 - 90 = 172
Cost inflation index 1992 - 93 = 223
Cost inflation index 2014 - 15 = 1,024
214

1,024
Indexed cost of acquisition = 9,49,000 x
172
= Rs. 56,49,860/- (1)

Indexed cost of improvement 1,024


= 14,76,000 x
223
= Rs. 67,77,686/- (2)

Total indexed cost of acquisition


= 56,49,860 + 67,77,686
& improvement
= Rs. 1,24,27,546/- (3)

Taxable capital gain = 1,97,00,000 - 1,24,27,546


= Rs. 72,72,454/- (4)

Answers :

1) Rs. 56,49,860/- 3) Rs. 1,24,27,546/-


2) Rs. 67,77,686/- 4) Rs. 72,72,454/-

Exercise 3 :

On 10.10.1982, Mr. X acquired a property consisting of 3,000 sq.ft. of plot and 4,500 sq.ft.
of building in Chennai for a cost of Rs. 10,00,000/-. On 06.02.2017, he sold his property
for a sale consideration of Rs. 2,00,00,000/-. 109 & 1125 are the cost inflation index for
1982 - 83 & 2016 - 17 respectively.

Questions :

1. What will be the indexed cost of acquisition?


2. What is the capital gain?

Calculations:

Date of acquisition = 10.10.1982 (1982 - 83)


Cost of acquisition = Rs. 10,00,000
C.I.I. for 1982 - 83 = 109
Date of transfer = 06.02.2017 (2016 - 17)
C.I.I. for 2016 - 17 = 1,125
215

10,00,000
1) Indexed cost of acquisition = x 1,125
109
= Rs. 1,03,21,100/- (1)

2) Capital gain = 2,00,00,000 - 1,03,21,100


= Rs. 96,78,900/- (2)

Answers :

1) Rs. 1,03,21,100/- 2) Rs. 96,78,900/-

Exercise 4 :

Mr. ‘X’ acquired a property in June 1990 for 12.05 lakhs. On 10.12.2014, this property was
sold for a sale consideration of 85.14 lakhs. 182, 1024 are the cost inflation index for
1990 - 91 & 2014 - 15.

Questions :

1. What is the cost of acquistion?


2. What is the taxable capital gain?

Calculations :

Date of acquisition = June 1990 (1990 - 91)


Cost of acquisition = Rs. 12,05,000
Fair market value as on 1.4.81 = Not applicable here
Date of transfer = 10.12.2014 (2014 -15)
Sale consideration = Rs. 85,14,000
Cost inflation index 1990 - 91 = 182
Cost inflation index 2014 - 15 = 1,024
1,024
Indexed cost of acquisition = 12,05,000 x
182
= Rs. 67,79,780/- (1)

Taxable capital gain = 85,14,000 - 67,79,780


= Rs. 17,34,220/- (2)
216

Answers :

1) Rs. 67,79,780/- 2) Rs. 17,34,220/-

Exercise 5 :

An individual owned property was originally acquired in 01.10.1972 for 1.02 lakhs. The fair
market value of the property as on 01.04.1981 is 5.25 lakhs. On 10.12.2014, this property
was sold for a sale consideration of 75.05 lakhs. 100, 1024 are the cost inflation index for
1981 - 82 & 2014 - 15.

Questions :

1. What is the indexed cost of acquisition?


2. What is the taxable capital gain?

Calculations :

Date of acquisition = 01.10.1972


Cost of acquisition = Rs. 1,02,000
Fair market value as on 1.4.81
= Rs. 5,25,000 (1981 - 82)
as worked out
Date of transfer = 10.12.2014 (2014 - 15)
Sale consideration = Rs. 75,05,000
Cost inflation index 1981 - 82 = 100
Cost inflation index 2014 - 15 = 1,024
1,024
Indexed cost of acquisition = 5,25,000 x
100
= Rs. 53,76,000/- (1)

Taxable capital gain = 75,05,000 - 53,76,000


= Rs. 21,29,000/- (2)

Answers :

1) Rs. 53,76,000/- 2) Rs. 21,29,000/-

Exercise 6 :

On 12.12.2010, a property was acquired by Mr. Y for 75.28 lakhs. On 10.12.2014, the
217

same was sold for 1.03 crores. 711, 1024 are the cost inflation index for 2010 - 11
& 2014 - 15.

Questions :

1. What is the indexed cost of acquisition?


2. What is the taxable capital gain?

Calculations :

Date of acquisition = 12.12.2010 (2010 - 11)


Cost of acquisition = Rs. 75,28,000
Date of transfer = 10.12.2014 (2014 -15)
Sale consideration = Rs. 1,03,00,000
Cost inflation index 2010 - 11 = 711
Cost inflation index 2014 - 15 = 1,024
1,024
Indexed cost of acquisition = 75,28,000 x
711
= Rs. 1,08,42,014/- (1)

Taxable capital gain = 1,03,00,000 - 1,08,42,014


= (-) Rs.5,42,014 (2)
It is a loss, there is no taxable gain
(Capital loss)

Answers :

1) Rs. 1,08,42,014/- 2) (-) Rs. 5,42,014


It is a loss, there is no taxable gain.

Exercise 7 : (IBBI)

A flat was purchased in 1981 for Rs. 2,40,000/-. As a gift from his uncle, the assessee
received this flat having an area of 80 sq.m. in June 2001.The assessee has made
improvements in the flat in August 2005 at a cost of Rs. 15,00,000/-. He sold this flat in
2018 for Rs. 2,40,00,000/-. Society transfer charges was Rs. 50,000/- and the brokerage
charges were Rs. 1,00,000/-. Prevailing rate of flat as on 2001 is Rs. 40,000/m2.

Cost inflation index as on 2001 is 100. Cost inflation index on 2005 is 117 and cost
inflation index on 2018 is 272.
218

Questions :

1) What is the indexed cost of improvement?


2) What is the indexed cost of acquisition?
3) Compute capital gain at 20%?
4) What are the deductions as per Section 48(i)?
5) Whether the assessee is liable for paying capital gains?

Solution :

1. Indexed cost of improvements :

Cost of improvement made in 2005 = Rs. 15,00,000


Cost inflation index in 2005 = 117
Cost inflation index in 2018 = 272

. 15,00,000
. . Indexed cost of improvements in 2018= x 272
117

= Rs. 34,87,179/- (1)

2. Indexed cost of acquisition :

Flat was purchased in = 1981 - Rs. 2,40,000/-


Flat was gifted in = 2001 - 80 sq.m.
Flat was sold in = 2018 - Rs. 2,40,00,000/-

For the purpose of computing capital gain, the FMV as on 01.04.2001 is to be


determined.

Area of the flat = 80 sq.m.


Rate prevailing in 2001 as given = Rs. 40,000/sq.m.
.
. . Value of the flat as on 2001 = 80 x 40,000
= Rs. 32,00,000/-
Cost inflation index in 2001 = 100
Cost inflation index in 2018 = 272

. 32,00,000
. . Indexed cost of Acquisition = x 272
100

= Rs. 87,04,000/- (2)


219

3. Computation of capital gain :

Indexed cost of acquisition = Rs. 87,04,000


Indexed cost of improvements = Rs. 34,87,179
Total indexed cost of acquisition = Rs. 1,21,91,179/-
& improvements
Sale consideration = Rs. 2,40,00,000
Less expenses : = (-) Rs. 1,50,000
Society transfer charges= Rs. 50,000
Brokerage = Rs. 1,00,000
Total expenses = Rs. 1,50,000

Net income from sale = Rs. 2,38,50,000/-


(2,40,00,000 - 1,50,000)

Less indexed cost of acquistion & = Rs. 1,21,91,179


improvements
Capital gains = Rs. 1,16,58,821
Capital gain tax percentage = 20%
Capital gain tax 0.2 x 1,16,58,821 = Rs. 23,31,764/- (3)

4. Deductions :

Brokerage = Rs. 1,00,000


Society transfer for charges = Rs. 50,000
Total = Rs. 1,50,000/- (4)

5. The assessee is liable for paying capital gains. (5)

Exercise 8 : (IBBI)

Gift from the year 2000 a flat of carpet area of 80 sq.m. purchased by his uncle in 1981 for
a price of Rs. 2,40,000/-. Flat was transferred in the name of A in the year June 2001.

Assessee started using the flat in 2000 only.

In 2005, he carried out substantial improvement works inside the flat by spending a total sum
of Rs. 15,00,000/-. (Courtesy : Mr. S. Pichaiya)
220

Flat was sold by A in the month of February 2018 for a total price of Rs. 2,40,00,000/-. Prevailing
rate of similar ownership flats in the locality in April 2001 was Rs. 40,000/sq.m.

Questions :

1. Which of the following statement is false?

a) Cost of acquisition to the assessee is Rs. 2,40,000/-


b) Capital gain tax would be levied at the rate of 20%
c) The assessee is entitled to deduct index cost of improvement
d) The assessee is liable to pay capital gain tax in the matter

Ans : (a)

2. Which of the following statement is true?

a) Cost of acquisition to the assessee is Rs. 32,00,000/-


b) Cost of acquisition to the assessee is Rs. 2,40,000/-
c) The assessee is entitled to deduct indexed cost of improvement only
d) The assessee is not liable to pay in the matter since it was received by his
as gift

80 sq.m. x Rs. 40,000 / sq.m. = Rs. 32,00,000/-

Ans : (a)

3. What will be the indexed cost of flat sold in 2018 for the purpose of calculating gain
tax by the assessee A if cost of inflation index for the financial year 2017 / 2018 is
272 for the years 2001 / 2002 it was 100

a) Rs. 32,00,000/- b) Rs. 87,04,000/-


c) Rs. 6,52,800/- d) Rs. 62,00,000/-

Indexed cost of flat = 32,00,000 x (272 / 100)


= Rs. 87,04,000/-

Ans : (b)

4. What will be the indexed cost of improvement works carried out in the flat for cost
inflation index for the year 2005 / 2006 was 117 and 2018 is 272
221

a) Rs. 34,87,179/- b) Rs. 40,87,179/-


c) Rs. 15,00,000/- d) Rs. 30,00,000/-

Indexed cost of improvement = 15,00,000 x (272 / 117)


carried out in 2005
= Rs. 34,87,179/-

Ans : (a)

5. What will be the deduction permissible to the assessee while computing capital
gain from the sale price of flat under capital gain tax provision if assessee has
spent Rs. 1,20,000/- for the brokerage charges and Rs. 25,000/- paid to the society
for transfer charges

a) Rs. 87,04,000/- b) Rs. 1,23,36,179/-


c) Rs. 1,21,91,179/- d) Rs. 1,45,000/-

34,87,179 + 87,04,000 + 1,20,000 + 25,000 = Rs. 1,23,36,179/-

Ans : (b)

6. What was the total capital gain tax 20% rate if assessee has not invested sales
proceeds anywhere

a) Rs. 48,00,000/- b) Rs. 30,60,000/-


c) Rs. 23,32,764/- d) Rs. 40,00,000/-

2,40,00,000 - 1,23,36,179 = Rs. 1,16,63,821/-


Capital gain tax @ 20% is = Rs. 23,32,764/-

Ans : (c)

Exercise 9 :

On 04.01.2005, a property was acquired by Mr. X for 8.08 lakhs. In June 2010,
improvements were made for 12.06 lakhs. On 28.08.2018, the property was sold to
83 lakhs. (113, 167, 280 are the cost inflation index for 2004 - 05, 2010 - 11, 2018 - 19
respectively).
222

Questions :

1. What is the Indexed cost of acquisition?


2. What is the indexed cost of improvement?
3. What is the total indexed cost of acquisition & improvement?
4. What is the taxable capital gain?
5. What is the tax to be paid by Mr. ‘X’?
6. If the property is owned by a company, what is the capital gain tax?

Calculations :

Date of acquisition = 04.01.2005 (2004 - 05)


Cost of acquisition (01/2005) = Rs. 8,08,000
Cost of improvements (6/2010) = Rs. 12,06,000 (2010 - 11)
Date of transfer = 28.08.2018 (2018 - 19)
Sale consideration = Rs. 83,00,000
Cost inflation index 2004 - 05 = 113
Cost inflation index 2010 - 11 = 167
Cost inflation index 2018 - 19 = 280

1. Indexed cost of acquisition = 8,08,000 x (280/113)


= Rs. 20,02,124/- (1)

2. Indexed cost of improvement = 12,06,000 x (280/167)


= Rs. 20,22,036/- (2)

3. Total indexed cost of acquisition = 20,02,124 + 20,22,036


& indexed cost improvement
= Rs. 40,24,160/- (3)

4. Taxable capital gain = 83,00,000 - 40,24,160


= Rs. 42,75,840/- (4)

5. Tax in the hand of Mr. ‘X’ - 20% = 0.2 x 42,75,840


= Rs. 8,55,168/- (5)

6. If it is owned by a company, = 0.4 x 42,75,840


tax - 40%
= Rs. 17,10,336/- (6)
223

Answers :

1) Rs. 20,02,124/- 4) Rs. 42,75,840/-


2) Rs. 20,22,036/- 5) Rs. 8,55,168/-
3) Rs. 40,24,160/- 6) Rs.17,10,336/-

Exercise 10 :

On 11.06.2004, a property was acquired by Mr. X for 9.49 lakhs. In August 2012,
improvements were made for 14.76 lakhs. On 01.04.2017, the property was sold to
67 lakhs. 113, 200, 272 are the cost inflation index for 2004 - 05, 2012 - 13, 2017 - 18
respectively.

Questions :

1. What is the indexed cost of acquisition?


2. What is the indexed cost of improvement?
3. What is the total indexed cost of acquistions & improvement?
4. What is the taxable capital gain?

Calculations :

Date of acquisition = 11.06.2004 (2004 - 05)


Cost of acquisition = Rs. 9,49,000/-
Cost of improvements (08/2012) = Rs. 14,76,000 (2012 - 13)
Date of transfer = 01.04.2017 (2017 - 18)
Sale consideration = Rs. 67,00,000
Cost inflation index 2004 - 05 = 113
Cost inflation index 2012 - 13 = 200
Cost inflation index 2017 - 18 = 272
272
Indexed cost of acquisition = 9,49,000 x
113
= Rs. 22,84,319/- (1)

Indexed cost of improvement 272


= 14,76,000 x
200
= Rs. 20,07,360/- (2)

Total indexed cost of acquisition


= 22,84,319 + 20,07,360
& improvement
= Rs. 42,91,679/- (3)
224

Taxable capital gain = 67,00,000 - 42,91,679


= Rs. 24,08,321/- (4)

Answers :

1) Rs. 22,84,319/- 3) Rs. 42,91,679/-


2) Rs. 20,07,360/- 4) Rs. 24,08,321/-

Exercise 11 :

On 10.10.1982, Mr. X acquired a property consisting of 3,000 sq.ft. of plot and 4,500 sq.ft.
of building in Chennai for a cost of Rs. 10,00,000/-. On 31.03.2017, he sold his property
for a sale consideration of Rs. 2,00,00,000/-. 109 & 1125 are the cost inflation index for
1982 - 83 & 2016 - 17 respectively.

Questions :

1. What will be the indexed cost of acquisition?


2. What is the capital gain?

Calculations:

Date of acquisition = 10.10.1982 (1982 - 83)


Cost of acquisition = Rs. 10,00,000
C.I.I. for 1982 - 83 = 109
Date of transfer = 31.03.2017 (2016 - 17)
C.I.I. for 2016 - 17 = 1,125

10,00,000
1) Indexed cost of acquisition = x 1,125
109
= Rs. 1,03,21,100/- (1)

2) Capital gain = 2,00,00,000 - 1,03,21,100


= Rs. 96,78,900/- (2)

Answers :

1) Rs. 1,03,21,100/- 2) Rs. 96,78,900/-


225

Exercise 12 :

An individual owned property was originally acquired in 01.10.1972 for Rs. 45,000/-. The
fair market value of the property as on 01.04.2001 is 5.25 lakhs. On 01.04.2017, this
property was sold for a sale consideration of Rs. 25,05,000/-. 100, 272 are the cost
inflation index for 2001 - 02 & 2017 - 18.

Questions :

1. What is the indexed cost of acquisition?


2. What is the taxable capital gain?

Calculations :

Date of acquisition = 01.10.1972


Cost of acquisition = Rs. 45,000
Fair market value as on 1.4.2001
= Rs. 5,25,000 (2001 - 02)
as worked out
Date of transfer = 01.04.2017 (2017 - 18)
Sale consideration = Rs. 25,05,000
Cost inflation index 2001 - 02 = 100
Cost inflation index 2017 - 18 = 272
272
Indexed cost of acquisition = 5,25,000 x
100
= Rs. 14,28,000/- (1)

Taxable capital gain = 25,05,000 - 14,28,000


= Rs. 10,77,000/- (2)

Answers :

1) Rs. 14,28,000/- 2) Rs. 10,77,000/-

* * *
226

15. APARTMENTS & J V RATIO

Exercise 1 :

i) In a plot of 3,000 sq.ft., 3 flats of same built up area 1,500 sq.ft each are
constructed. What is the Undivided share (UDS) of land for each flat?

ii) If 3 flats of 1,500, 800, 700 are constructed in the plot of 3,000 sq.ft., what is the
UDS of land for 1,500 sq.ft. of flat?

i) Built up area = 3 x 1,500 = 4,500 sq.ft.


Plot area = 3,000 sq.ft.
FSI = 4,500/3,000 = 1.5
UDS = 1,500 / 1.5 = 1,000 sq.ft.

ii) Built up area = 1,500 + 800 + 700= 3,000 sq.ft.


Plot area = 3,000 sq.ft.
FSI = 3,000/ 3,000 = 1
UDS for = 1,500 / 1 = 1,500 sq.ft.
1,500 sq.ft. flat

Exercise 2 :

Land rate = Rs. 5,500 / sq.ft. FSI is 2. Building unit rate is Rs. 2,000/sq.ft. Assuming the
promoter’s profit as 20%, what is the composite rate?

Prevailing land rate = Rs. 5,500 / sq.ft.


FSI = 2
Land component = 5,500 / 2 = Rs. 2,750
Building rate = Rs. 2,000
Land + building component = Rs. 4,750
Add promoter’s profit, 20% = Rs. 950
Composite rate = Rs. 5,700/-

Exercise 3 :

In an apartment building, the sum of the plinth area of all the flats is 5,000 sq.ft. Common
227

area is 500 sq.ft. The super plinth area is 5,500 sq.ft. What is percentage of common area in
the apartment building?

Sum of plinth area of all flats = 5,000 sq.ft.


Common area = 500 sq.ft.

500
Percentage of common area = x 100 = 10%
5,000

Exercise 4 :

An apartment building consists of 12 flats of super built up area 1,050 sq.ft. The net
monthly rent of a flat is Rs. 9,000. The prevailing rate of return is 2.5%. Find the
approximate value of one flat by rent capitalisation method.

Net monthly rent = Rs. 9,000


Yearly rent = Rs. 1,08,000
Rate of return = 2.5%
100
Value = 1,08,000 x
2.5
= Rs. 43,20,000/-

Exercise 5 :

In a plot of 3,600 sq.ft., an apartment building of GF + 2 is existing. 3 flats of 600, 800,


1000 are existing in one floor. What is the UDS of land for i) flat 600 sq.ft & ii) flat
1,000 sq.ft.?

Built up area = (600 + 800 + 1000) x 3 = 7,200 sq.ft.


Plot area = 3,600 sq.ft.
FSI = 7,200 / 3,600 = 2
UDS = 600 / 2 = 300 sq.ft.
= 1,000 / 2 = 500 sq.ft.

Exercise 6 :

In a plot of 8,608 sq.ft., the landlord Mr. ‘X’ intends to construct an apartment through joint
venture for a total built up area of 17,216 sq.ft. There will be 16 flats of super built up area
of 1,076 sq.ft. The prevailing market rate for plot is Rs. 10,000 per sq.ft. and the guideline
228

rate is Rs. 20,000 per sq.ft. The building construction rate is Rs. 2,500/-. Assume the
promoter’s profit as 20%.

Questions :

1. What is FSI?
2. What is the undivided share (UDS) for each flat?
3. What is the composite rate?
4. What is the selling price of each flat?
5. What is Joint venture Ratio? (Promoter : Landlord)
6. Whether there is any impact of Guideline rate while fixing the composite rate and
joint venture ratio?

Data :

Extent of plot = 8,608 sq.ft.


Proposed builtup area = 17,216 sq.ft.
No. of flats proposed = 16
Built up area of each flat = 1,076 sq.ft.
Market rate of plot = Rs. 10,000/sq.ft.
Guideline rate = Rs. 20,000/sq.ft.
Building construction rate = Rs. 2,500/sq.ft.
Promoter’s profit = 20%

Calcluations :

1. Total built up area = 17,216 sq.ft.


Extent of plot = 8,608 sq.ft.
FSI 17,216 / 8,608 = 2 (1)

2. Super built up area of one flat = 1,076 sq.ft.


FSI = 2
UDS of a flat 1,076 / 2 = 538 sq.ft. (2)

3. Land component 10,000 / 2 = Rs. 5,000


Building rate = Rs. 2,500
Land rate + Building rate = Rs. 7,500
Add 20% for promoter’s profit = Rs. 1,500
Composite rate = Rs. 9,000 / sq.ft. (3)
229

4. Super built up area of one flat = 1,076 sq.ft.


Composite rate = Rs. 9,000/sq.ft.
Selling price 1,076 x 9,000 = Rs. 96,84,000/- (4)

5. Landlord’s share 10,000/2 = Rs. 5,000


Promoter’s share = Rs. 2,500
Total - Land lord + developer = Rs. 7,500
Landlord’s percentage share
5,000/7,500 = 67%
Promoter’s percentage share
2,500/7,500 = 33%
.
. . Joint venture Ratio is = 33 : 67 (5)

6. Guideline rate is meant for fixing stamp duty only and hence plays no role
while fixing the composite rate and joint venture ratio. (6)

Answers :

1) 2 4) Rs. 96,84,000/-
2) 538 sq.ft. 5) 33 : 67
3) Rs. 9,000/sq.ft. 6) Guideline rate plays no role while
fixing the composite rate and joint
venture ratio

Exercise 7 :

In a plot of 8,000 sq.ft., the promoter has constructed an apartment building of super built
up area 20,000 sq.ft. It consists of 16 flats of super plinth area 1,000 sq.ft. and 8 flats of
super plinth area of 500 sq.ft. The market rate of plot is Rs. 6,000/sq.ft. and the guideline
rate is Rs. 7,500/sq.ft. The building rate is Rs. 2,500/sq.ft. The promoter’s profit is 15%.

Questions :

1. What is FSI?
2. What is UDS for 1,000 sq.ft. of flat?
3. What is UDS for 500 sq.ft. of flat?
4. What is the composite rate for the flat?
5. Assuming a common area of 4,000 sq.ft., what is the common area percentage?
6. What is the joint venture ratio?
230

Data :

Extent of plot = 8,000 sq.ft.


Total built up area = 20,000 sq.ft.
Number of flats = 16 + 8
Builtup area of each flat = 1,000 sq.ft + 500 sq.ft.
Market rate of plot = Rs. 6,000/sq.ft.

Guideline rate = Rs. 7,500/-


Building rate = Rs. 2,500/-
Promoter’s profit = 15%

Calculations :

1. Total built up area = 20,000 sq.ft.


Plot area = 8,000 sq.ft.
FSI - 20,000 / 8,000 = 2.5 (1)

1,000
2. UDS for 1,000 sq.ft. of flat = = 400 sq.ft. (2)
2.5

500
3. UDS for 500 sq.ft. of flat = = 200 sq.ft. (3)
2.5

4. Land component = 6,000/2.5 = Rs. 2,400


Building component = Rs. 2,500
Land & Building component = Rs. 4,900
Promoter’s profit 15% = Rs. 735
Composite rate = Rs. 5,635/- (4)

5. Total super built up area of all flats= 20,000 sq.ft.


Common area = 4,000 sq.ft.
Plinth area of all flats = 16,000 sq.ft.
4,000
Common area percentage =
16,000
= 25% (5)

6. Joint venture ratio


Land rate = Rs. 6,000
FSI = 2.5
231

Land component = 6,000/2.5 = 2,400


Building component = 2,500
Land & Building = 4,900
Promoter’s ratio = 2,500/4,900 = 0.51 (51%)
Landlord’s share = 2,400/4,900 = 0.49 (49%)

Joint venture Ratio - (Promoter : Lordlord) = 51 : 49 (6)

Answers :

1. 2.5 4. Rs. 5,635/-


2. 400 sq.ft. 5. 25%
3. 200 sq.ft. 6. 51 : 49

Exercise 8 :

It is an apartment building with GF + 2 floors. Mr. ‘X’ has booked a flat (1,320 sq.ft.). UDS
(Undivided share) of land is 660 sq.ft. The composite rate is Rs. 6,000/sq.ft. The land rate
is Rs. 5,000/sq.ft. Sale deed for UDS of land has been executed (Rs. 33,00,000/-) and the
builder’s agreement has been signed. Total value of the flat on completion is
1,320 x 6,000 = Rs.79,20,000/-. Mr. X has applied loan from a bank. The bank directs the
valuer to certify the value in stages (break up for Rs. 77,20,000 : Land UDS (660) =
Rs. 33,00,000/- and Building (1,320) = Rs. 46,20,000/-).

Questions :

1) Before commencement of the building construction, what is the stage value?

2) Mr. X has booked a flat (1,320 sq.ft.) in first floor. Basement completed (18%). UDS
sale deed executed. What is the stage value?

3) Mr. X has booked a flat in first floor (1,320 sq.ft.). Frame works of all floors
completed. RCC roof for all the floors has been cast. For the concerned flat in FF,
brick work has been completed, doors & windows frames have been fixed, inside
plastering of walls and ceiling finish have been completed. Percentage of works
completed is 75%. What is the stage value?

4) Construction is fully completed in all respects. Flat is fit for use. What is the value to
be certified?
232

5) Mr. Y has booked a flat in 3rd floor. RCC columns have been raised upto second
floor. What is the stage value?

6) What is the cost to be certified on completion for the purpose of income tax?

Data :

Number of floors = 3
UDS = 660 sq.ft.
Composite rate = Rs. 6,000/sq.ft.
Land rate = Rs. 5,000/sq.ft.
Sale deed for 660 sq.ft. of UDS = Rs. 33,00,000/-
Value of flat on completion = Rs. 79,20,000/-

Calculations :

1) UDS of land has been executed.


.
. . the value = 660 x 5,000 = Rs. 33,00,000/- (1)

2) UDS 660 x 5,000 = Rs. 33,00,000


Building - 0.18 x 46,20,000 = Rs. 8,31,600
Total stage value = Rs. 41,31,600/- (2)

3) UDS of land = 660 x 5,000 = Rs. 33,00,000


Building = 0.75 x 46,20,000 = Rs. 34,65,000
Total stage value = Rs. 67,65,000/- (3)

4) Composite rate = Rs. 6,000 / sq.ft.


Built up area = 1,320 sq.ft.
Value on completion = 1,320 x 6,000
Value to be certified = Rs. 79,20,000/- (4)

5) UDS of land = 660 x 5,000 = Rs. 33,00,000


Value upto basement (18%) = Rs. 8,31,600
Total stage value = Rs. 41,31,600/- (5)

6) The cost to be certified for the = Rs. 79,20,000/- (6)


purpose of income tax
233

Answers :

1) Rs. 33,00,000/- 4) Rs. 79,20,000/-


2) Rs. 41,31,600/- 5) Rs. 41,31,600/-
3) Rs,. 67,65,000/- 6) Rs. 79,20,000/-

Exercise 9 :

It is a joint venture proposal. The landlord is having a plot of 8,250 sq.ft. and he wishes to
construct an apartment for an FSI of 2. The land rate is Rs. 5,000/sq.ft. A promoter has
approached the landlord for developing an apartment for which the unit rate of
construction is Rs. 2,500/-.

Questions:

1. What will be the promoter’s share?


2. What will be the landlord’s share?

Data :

Proposal for = Joint venture


Plot area owned by landlord = 8,250 sq.ft.
FSI proposed = 2
Land rate = Rs. 5,000/sq.ft.
Unit rate of construction = Rs. 2,500/-

Calculations :

Land rate = Rs. 5,000 / sq.ft.


FSI = 2
Land component = 5,000 / 2 = Rs. 2,500
Building rate = Rs. 2,500
Land + building = Rs. 5,000

2,500
Promoter’s share = x 100 = 50% (1)
5,000

2,500
Landlord’s share = x 100 = 50% (2)
5,000
234

Answers :

1) 50% 2) 50%

Exercise 10 :

An apartment building consisting of 70 flats of equal super built up area of 1,000 sq.ft.
each is proposed to be constructed on a land of 35,000 sq.ft. 10% of the area of land has
to be left as OSR (Open Space Reservation) and separate deed has to be executed in
favour of the corporation.

Question :

1) What is the UDS for each flat?

Data :

Number of flats proposed = 70 Nos.


Built up area of each flat = 1,000 sq.ft.
Land area = 35,000 sq.ft.
OSR = 10%

Calculations :

Super built up area of one flat = 1,000 sq.ft.


Super built up area of seventy flats = 70,000 sq.ft.
Extent of land = 35,000 sq.ft.
FSI : 70,000 / 35,000 = 2
Percentage to be left for OSR = 10%
Area of land to be left for OSR = 3,500 sq.ft.
Net extent of land left with the
= 31,500 sq.ft.
Promoter 35,000 - 3,500
FSI now : 70,000 / 31,500 = 2.22
UDS for 1,000 sq.ft. of flat (31,500/70) = 1,000 / 2.22
= 450 sq.ft.

Answer :

1. 450 sq.ft.
235

Exercise 11 :

Mr. ‘X’ is having a commercial building of 20,000 sq.ft. situated in a plot of 10,000 sq.ft. He
wants to sell one shop of plinth area 1,000 sq.ft. to Mr. ‘Y’. He approaches a valuer to
suggest him the UDS of land of the shop for the purpose of executing a sale deed in
favour of ‘Y’. The common area percentage is 10%.

Question :

1. What is the UDS of land?

Data :

Plot area = 10,000 sq.ft.


Built up area of building = 20,000 sq.ft.
Plinth area of 1 shop = 1,000 sq.ft.
Common area of 1 shop = 10%

Calculations :

Building area = 20,000 sq.ft.


Plot area = 10,000 sq.ft.
FSI = 20,000 / 10,000 = 2
Plinth area of shop = 1,000
Common area percentage = 10%
Super builtup area = 1,000 x 1.1 = 1,100 sq.ft.
Super builtup area
UDS =
FSI
1,100
= = 550 sq.ft.
2

Answer :

1. 550 sq.ft.

Exercise 12 :

An apartment of built up area of 25,000 sq.ft. is proposed to be constructed in a land of


12,500 sq.ft. Prevailing market rate of land is Rs. 10,000/sq.ft. Unit rate of construction is
Rs. 3,000/sq.ft. Assume the profit of the promoter as 25%.
236

Question :

1. What is the composite rate of the flat?

Data :

Total built up area = 25,000 sq.ft.


Land area = 12,500 sq.ft.
Land rate = Rs. 10,000/-
Rate of construction = Rs. 3,000/sq.ft.
Promoter’s profit = 25%

Calculations :

Built up area = 25,000 sq.ft.


Plot area = 12,500 sq.ft.
25,000
FSI = = 2
12,500
Prevailing market rate of plot = Rs. 10,000
Land rate 10,000
Land component = =
FSI 2

= Rs. 5,000

Building rate = Rs. 3,000 / sq.ft.


Land & Building (5,000 + 3,000) = Rs. 8,000
Add promoter’s profit 25% = Rs. 2,000
Composite rate = Rs. 10,000/-

Answer :

1. Rs. 10,000/-

Exercise 13 :

A landlord has a plot of 15,000 sq.ft. A promoter has approached the landlord for a joint
venture stating that he wishes to construct an apartment building for 30,000 sq.ft. The
prevailing market rate of land is Rs. 14,000/sq.ft. and the guideline rate is
Rs. 24,000/sq.ft. The construction cost is Rs. 3,000/sq.ft.
237

Question :

1. What is the Joint Venture ratio (Promoter : Landlord)?

Data :

Plot area = 15,000 sq.ft.


Proposed building area = 30,000 sq.ft.
Land rate = Rs. 14,000/-
Guideline rate = Rs. 24,000/-
Construction cost = Rs. 3,000/-

Calculations :

Plot area = 15,000


Building area = 30,000
30,000
FSI = = 2
15,000
Land rate = Rs. 14,000
FSI = 2
Proportionate land rate for the
purpose of joint venture 14,000
= = Rs. 7,000
(landlord) 2

Building rate (Promoter) = Rs. 3,000


Landlord + Promoter = Rs.10,000
3,000
Promoter’s share = x 100 = 30%
10,000
7,000
Landlord’s share = x 100 = 70%
10,000

... Ratio - Promoter : Landlord = 30 : 70

Answer :

1. 30 : 70

* * *
238

16. MISCELLANEOUS TOPICS & SOME MORE EXERCISES

Exercise 1 :

What is the amount of Rs. 5,000 at the end of 5 years @ 5% compound interest per annum?

r n
Amount A = P (1+ )
100
5 5
= 5,000 ( 1 + )
100

= 5,000 x (1.05)5
= 5,000 x 1.276
= Rs. 6,380/-

Exercise 2 :

In 2013, a valuer valued a residential property in a mofusil town for Rs. 68.56 lakhs. Assuming
an annual escalation of 10% per year, what will be the value of the property as on 2018 by
applying the formula?

P = Rs. 68.56 lakhs


r = 10%
n = 2018 - 2013 = 5 years

r n
Amount A = P (1+ )
100
10 5
= 68.56 ( 1 + )
100

= 68.56 x (1.1)5
= 68.56 x 1.6105
= Rs. 110.42 lakhs

Exercise 3 :

Mr. X is selling 2,400 sq.ft. of plot to Mr. Y for a mutually agreed amount of Rs. 24,00,000. But
in sale deed, they mention as Rs. 12,00,000/-. Guideline rate is Rs. 510/-. What is the
intrinsic value?, What is the agreement value? & What is stamp duty value?

a. Intrinsic value = Rs. 24,00,000/-


b. Agreement value = Rs. 12,00,000/-
c. Stamp duty value = 2,400 x 510
= Rs. 12,24,000/-

* * *
239

PART - IV

ONE MARK CASE STUDIES - 52 Nos.


240

This page is kept vacant intentionally.


241

Part - IV

ONE MARK CASE STUDIES

Exercise 1 : (IBBI)

A machine was purchased for Rs. 1,00,000/- @ 15% depreciation of SLM. What is the writ-
ten down value after 2 years?

a) Rs. 70,000/- b) Rs. 80,000/-


c) Rs. 60,000/- d) Rs. 90,000/-

Depreciation for 1 year by = 1,00,000 x 0.15


Straight line method
Depreciation amount = Rs. 15,000/year

Depreciated value after 1 year = 1,00,000 - 15,000


= Rs. 85,000/-

Depreciated value after the = 85,000 - 15,000


second year
= Rs. 70,000/-

Written down value after 2 years = Rs. 70,000/-

Ans ‘a’

Exercise 2 : (IBBI)

A property has a net income of Rs. 30,000/-. One appraiser decides to use a 12 percent
capitalisation rate, while a second appraiser uses a 10 percent rate. What is the
difference in appraisal value of the two valuers?

a) Rs. 50,000/- b) Rs. 60,000/-


c) Rs. 40,000/- d) Rs. 70,000/-
242

First appraiser :

100
Capitalised value = 30,000 x = Rs. 2,50,000/-
12

Second appraiser :

100
Capitalised value = 30,000 x = Rs. 3,00,000/-
10

By using a higher rate of return, the value is decreased by Rs. 50,000/-.

Ans ‘a’

Exercise 3 : (IBBI)

The net income was reported at Rs. 21,000/- and the property was sold for Rs. 3,00,000.
What capitalisation rate is applied to this sale?

a) 5% b) 8%
c) 6% d) 7%

Capitalised value = Rs. 3,00,000/-


Net income = Rs. 21,000/-
Net income
Capitalised value = x 100
X
21,000
3,00,000 = x 100
X
21,000 x 100
X =
3,00,000
= 7%

Ans ‘d’

Exercise 4 : (IBBI)

A mobile phone was purchased for Rs.50,000/-. Its salvage value is Rs. 10,000. Total life
time used 60,000 hours. Used time 20,000 hours. What is the depreciation of the cell
phone?
243

a) Rs. 20,000/- b) Rs. 3,333/-


c) Rs. 13,333/- d) Rs. 23,333/-

Phone purchased for = Rs. 50,000/-


Salvage value = Rs. 10,000/-
Net value = Rs. 40,000/-
Used time = 20,000 hours
Total life = 60,000 hours
20,000 1
Depreciation = =
60,000 3
40,000
Depreciation value =
3
= Rs. 13,333/-.

Ans ‘c’

Exercise 5 :

A Contractor took a loan of Rs. 36,00,000/- from a bank for construction of modern building
2 years back. He has to repay the loan at the Interest of 10%. If the sale of the property is
yet to take one year, calculate the amount to be paid by the contractor?

a) Rs. 46,00,000 b) Rs. 48,91,600


c) Rs. 49,00,000 d) Rs. 47,91,600

Amount borrowed = Rs. 36,00,000


Amount to which Re. 1/- will = (1 + i)n
accumulate @ 10% in 3 years
= (1 + 0.1)3
= 1.331

Amount to be repaid by the = (1.331 x 36,00,000)


contractor
= Rs. 47,91,600/-

Ans ‘d’

Exercise 6 :

A promoter purchased a residential property for Rs. 60,00,000/- and immediately carried
out certain interior decorations works for Rs. 20,00,000/-.
244

He intends to dispose of the property at the end of 4 years. Calculate the cost for Purchaser
if he expects a return of 12% on his investment.

a) Rs. 1,25,92,000 b) Rs. 1,35,92,000


c) Rs. 1,20,00,000 d) Rs. 1,55,63,000

Initial investment = 60,00,000 + 20,00,000


= 80,00,000

Amount to which Re. 1/- will = (1 + 0.12)n


accumulate @ 12% in 4 years
= (1 + 0.12)4
= 1.574

Cost for the purchaser = 1.574 x 80,00,000


= Rs. 1,25,92,000/-

Ans ‘a’

Exercise 7 :

An Investor has the right to receive Rs. 25,00,000/- from a property after a period of
9 years. Assuming the rate of interest of 8% Find out the amount for which the investor will
be ready to relieve his future right over the property.

a) Rs. 13,50,000 b) Rs. 12,50,000


c) Rs. 14,00,000 d) Rs. 15,00,000

Amount receivable in 9 years time= Rs. 25,00,000/-


1
P.V. of Re. 1/- in 9 years @ 8% =
(1 + i)n
1
=
(1 + 0.08)9
= 0.5
P.V. (Present value) = 25,00,000 x 0.5
= Rs. 12,50,000/-

Ans ‘b’
245

Exercise 8 :

A property owner is able to save Rs 50,000/-per year from the net income of his property
and he invest this amount each year to earn interest at 7%. Find out the amount which will
be available at the end of 18 years.

a) Rs. 18,00,000 b) Rs. 16,00,000


c) Rs. 17,00,000 d) Rs. 20,00,000

Annual saving = Rs. 50,000/-


Amount Re. 1/- P.A. at 7% for 18 years
Formula (1 + i)n - 1 (1 + 0.07 )18 - 1
APA = =
i 0.07
2.3799
=
0.07
= 34

Total amount available after 18 years :


= 34 x 50,000
= Rs. 17,00,000/-

Ans ‘c’

Exercise 9 :

A Promoter at Chennai constructed 4 flats of 1700, 1400, 1300, 1600 Sq.ft. in a plot area
of 4000 Sq.ft.

(i) What is the FSI?

a) 1.2 b) 1.3 c) 1.6 d) 1.5

(ii) What is UDS of land for the flat of 1600 Sq.ft.?

a) 1066.7 Sq.ft. b) 1055 Sq.ft.


c) 1000 Sq.ft. d) 1006 Sq.ft.

Area of the site = 4,000 sq.ft.


Built up area = 6,000 sq.ft.
(1,700 + 1,400 + 1,300 + 1.600)
246

Total built up area


FSI =
Area of the site
6,000
= = 1.5
4,000
Flat area 1,600
UDS = =
FSI 1.5
= 1066.7 sq.ft.
i) Ans ‘d’
ii) Ans ‘a’

Exercise 10 :

A Real estate promoter has approached the landlord for joint venture development of an
apartment for which the unit rate of construction is Rs. 3000. The land rate is
Rs. 6000/Sq.ft. FSI = 2, What will be the joint venture ratio of promoter & Landlord?

a) 35 : 65 b) 40 : 60 c) 50 : 50 d) 37 : 67

FSI = 2
Land rate = Rs. 6,000/sq.ft.
Land component (6,000/2) = Rs. 3,000
Building rate = Rs. 3,000
Land + Building = Rs. 6,000/sq.ft.
3,000
Landlord’s share = x 100
6,000
= 50%

Ans ‘c’

Exercise 11 :

Mr. ‘X’ acquired a property at Coimbatore on 06.09.1972 for 2.25 lakhs. Fair market value
of the property as on 1.4.81 is 9.50 lakhs. The property was sold on 25.11.2014 for a sale
consideration of 105 lakhs. What is the Taxable capital gain? C.I.I. for 1981 & 2014 are
100 & 1,024 respectively.

a) Rs. 6,62,000/- b) Rs. 5,00,000/-


c) Rs. 7,72,000/- d) Rs. 8,00,000/-

1,024
Indexed cost of acquisition= 9,50,000 x
100
247

= Rs. 97,28,000/-

Taxable capital gain = 1,05,00,000 - 97,28,000


= Rs. 7,72,000/-

Ans ‘c’

Exercise 12 :

A Businessman acquires a property at Tiruchirappalli on 24.12.2010 for Rs. 95 lakhs. The


same was sold for Rs. 130 lakhs in 2014. What is the capital gain? C.I.I. for 2010 & 2014
are 711 & 1,024 respectively.

a) 5.82 L b) 7.82 L
c) 8.50 L d) Capital Loss

1,024
Indexed cost of acquisition = 95,00,000 x
711
= Rs. 1,36,82,000/-
Taxable capital gain = 1,30,00,000 - 1,36,82,000
= Negative
= Capital loss

Ans ‘d’

Exercise 13 :

Built-up area of an apartment building at Chennai 9000 Sq.ft. is proposed to be constructed


in a site of 4500 Sq.ft. Prevailing market rate of land is Rs 6000/Sq.ft. Unit rate of construction
is Rs 2800/Sq.ft. Promoter’s profit = 20% What is the composite rate of the flat?

a) 7960 Sq.ft. b) 6500 Sq.ft.


c) 6960 Sq.ft. d) 7600 Sq.ft.

Built up area = 9,000 sq.ft.


FSI = 9,000 / 4,500 = 2
Prevailing market rate of land = Rs. 6,000/sq.ft.
Land rate 6,000
Land component = =
FSI 2
= 3,000/sq.ft.
248

Building rate = Rs. 2,800/sq.ft.


Land + Building = 3,000 + 2,800
= Rs. 5,800
Add 20% profit = Rs. 1,160
Composite rate = Rs. 6,960/sq.ft.

Ans ‘c’

Exercise 14 :

A 600 sq.ft. of shop building at T. Nagar Chennai is occupied by a tenant. The net monthly
rent is Rs. 45,000/-. Find out the value of the property by R.C.Method by adopting a rate of
return 6%

a) 85,00,000/- b) 95,00,000/-
c) 90,00,000/- d) 80,00,000/-

Monthly rent = Rs. 45,000/-


Yearly rent = 12 x 45,000
= Rs. 5,40,000/-
Rate of return = 6%
100
Value of the property = 5,40,000 x
6
= Rs. 90,00,000/-

Ans ‘c’

Exercise 15 :

Total extent of a site is 0.162 Hectare. Total built-up area of the building is 26,136 sq.ft.
What is the FSI?

a) 1.2 b) 1.5 c) 2.0 d) 1.0

Total built up area


FSI =
Area of the site
26,136 x 0.0929 26,136
= or
0.162 x 10,000 0.162 x 2.47 x 43,560
= 1.5

Ans ‘b’
249

Exercise 16 :

An apartment building at Tiruchirappalli consists of 4-floor, eight flat of 1125 Sq.ft. at each
floor. Area of the site = 2230 Sq.m. What is the UDS of land for each flat?

a) 1500 Sq.ft. b) 1200 Sq.ft.


c) 600 Sq.ft. d) 750 Sq.ft.

4 x 8 x 1,125
FSI =
2,230 x 10.76
= 1.50

1,125
UDS =
1.5
= 750 sq.ft.

Ans ‘d’

Exercise 17 :

A Contractor took a loan Rs 40,00,000/-from the bank, at the rate of Interest 9%. What is
the amount to be paid by the contractor at the end of 4th year?

a) 40.8 L b) 56.48 L c) 60.8 L d) 56.8 L

Amount to which Re. 1/- will accumulate at 9% in 4 years


= (1 + r)n
= (1 + 0.9)4
= 1.412

Amount to be repaid by contractor = (1.412 x 40,00,000)


= Rs. 56,48,000/-

Ans ‘b’

Exercise 18 :

An immovable property yields a net annual income of Rs 60,000. The income is expected
to continue for next 99 years. What is the present value of the property if the rate of
interest is 6% p.a?
250

a) Rs 8,00,000 b) Rs 10,00,000
c) Rs 12,00,000 d) Rs 15,00,000

Year’s purchase (Y.P.) = 100 / R


= 100 / 6
= 16.67
Present value = Net annual income x Y.P.
= 60,000 x 16.67
= Rs. 10,00,200/-
say Rs. 10,00,000/-

Ans ‘b’

Exercise 19 :

An Investor has invested an amount of Rs 10,00,000 in purchasing an urban site on taking


a loan of 9% compound rate of interest from a bank. What amount he will have to repay to
the bank after 3 years?

a) 11,95,000 b) 13,95,000
c) 12,50,000 d) 12,95,000

i = 0.09
n = 3
So, the amount to which his loan is accumulated
= 10,00,000 x (1 + i)n
= 10,00,000 x (1 + 0.09)3
= 10,00,000 x 1.295
= Rs. 12,95,000/-

Ans ‘d’

Exercise 20 :

5 Years back Mr.Sanjay had constructed a house at an intrinsic cost of Rs 80,00,000/- and
he has incurred an expenditure 2 years back of Rs 15,00,000/- for construction of a boundary
wall around that house. What is the total accumulated cost of his investment today at a
compound interest of 15% per annum?

a) 160.75 L b) 180.75 L
c) 190.75 L d) 165.90 L
251

i = 0.15
n = 5
Accumulated cost of his house = 80,00,000 x (1.15)5
= Rs. 1,60,91,000 (A)

Accumulated cost of his = 15,00,000 x (1.15)2


boundary wall
= Rs. 19,84,000 (B)

Total cost of his investment A + B = Rs. 1,80,75,000/-

Ans ‘b’

Exercise 21 :

A framed structure building is at Coimbatore. Age is 16 years. Assume the life is 70 years.
What is the depreciation by constant percentage method, if the depreciation is 1.5%

a) 29.5% b) 31.5%
c) 24.5% d) 21.5%

r = 1.5%
n = 16
r
Depreciation = 1 - (1 - )n
100
1.5 16
= 1 - (1 - )
100
= 1 - (0.785)
= 0.215 or
= 21.5%

Ans ‘d’

Exercise 22 :

A promoter at Chennai expects 15% profit. Building unit rate = Rs 2,500/Sq.ft. FSI is 2.
Land rate is Rs 6,000/Sq.ft. What will be the composite rate?

a) Rs. 6,325 b) Rs 5,325


c) Rs 7,325 d) Rs 5,900
252

FSI = 2
Prevailing land rate = Rs. 6,000/sq.ft.
Land component = 6,000 / 2 = Rs. 3,000/-
Building rate = Rs. 2,500/sq.ft.
Land + Building = Rs. 5,500/sq.ft.
Add, profit 15% = Rs. 825/sq.ft.
Composite unit rate = Rs. 6,325/sq.ft.

Ans ‘a’

Exercise 23 :

It is a load bearing residential building at Chennai. Ground floor : 20 years old, First Floor
: 10 years old, Plinth area of Ground floor : 1,000 sq.ft. First floor = 800 sq.ft. Assume
salvage value is 10% Life of the building is 60 years. What is the depreciated value of the
first floor building? Replacement cost of ground floor : 2,000/sq.ft. & first floor : 1,700/sq.ft.

a) 8.52 L b) 9.52 L
c) 4.08 L d) 13.60 L

20
Ground floor depreciation = x (100 - 10)
60
= 30%
Replacement value of first floor = 800 x 1,700
building
= Rs. 13,60,000/-
Depreciation value of FF = 13,60,000 x 0.30
= Rs. 4,08,000/-
Depreciated value of the first floor= 13,60,000 - 4,08,000
building
= Rs. 9,52,000/-

Ans ‘b‘

Exercise 24 :

A newly constructed apartment building at Chennai having block “A” & block “B”, consists
of 30 flats in each block of 1200Sq .ft. equal super Built-up area of each flat. Area of the
land : 48,000 sq.ft. Assume 10% land of OSR (Open space reserve). What is the UDS of
the each flat?
253

a) 820 Sq.ft. b) 620 Sq.ft.


c) 720 Sq.ft. d) 1100 Sq.ft.

Super built up area of block A & B= 2 x 30 x 1,200


= 72,000 sq.ft.
Extent of land = 48,000 sq.ft.
FSI = 72,000 / 48,000 = 1.5
OSR % = 10%
Area of the land to be left for OSR= 4,800 sq.ft.
Net extent of land = 48,000 - 4,800
= 43,200 sq.ft.
FSI now = 72,000 / 43,200 = 1.6667
UDS = 1,200 / 1.667 = 719.85,
say 720 sq.ft.

Ans ‘c’

Exercise 25 :

A businessman is having a commercial building at Coimbatore. It consists of 5 shops of


equal plinth area 1200 Sq.ft./each shop. He proposed to sell the 2 shops. What will be the
UDS of land for which he has to mention in the sale deed? Plot area is 3,000 sq.ft.

a) 1200 Sq.ft. b) 1280 Sq.ft.


c) 1480 Sq.ft. d) 1300 Sq.ft.

Total built up area = 5 x 1,200


= 6,000 sq.ft.
Plot area = 3,000 sq.ft.
FSI = 6,000 / 3,000 = 2
Plinth area of 2 shops = 2 x 1,200
= 2,400 sq.ft.
UDS = 2,400 / 2
= 1,200 sq.ft.

Ans ‘a’

Exercise 26 :

An apartment building at Chennai, consists of 4 floors, each floor built-up area is 2,400
sq.ft. Area of the plot is 4,800 sq.ft. What is the plot coverage?
254

a) 60% b) 70%
c) 50% d) 40%

Built up area of GF
Plot coverage = x 100
Area of the site
2,400
= x 100
4,800
= 50%

Ans ‘c’

Exercise 27 :

The net monthly rent of a Ground floor residential building of 1300 Sq.ft. is Rs. 18,000,
and Rs 15,000 for First floor building of same area. Find the approximate value of the
property by rent capitalization method by adopting a rate of return as 4%?

a) 89 L b) 79 L
c) 69 L d) 99 L

Monthly rent = Rs. 18,000 + 15,000


= Rs. 33,000/-
Yearly rent = 12 x 33,000
= Rs. 3,96,000
Rate of return = 4%
Value = 3,96,000 x (100 / 4)
= Rs. 99,00,000/-

Ans ‘d’

Exercise 28 :

An apartment building consists of 8 flats of super built-up area : 1,200 sq.ft. The gross
monthly rent of each flat is Rs. 9,000. Outgoing are 15% of the gross rent. The prevailing
rate of return is 3.5%. Find the approximate value of a flat by rent capitalization method?

a) 219.8 L b) 229.8 L
c) 209.8 L d) 240 L

Gross monthly rent = 8 x 9,000


= Rs. 72,000
255

Outgoings 15% (-) = Rs. 10,800


Net monthly rent = Rs. 61,200
Yearly rent = 12 x 61,200
= Rs. 7,34,400
Rate of return = 3.5%
Value = 7,34,400 x (100 / 3.5)
= Rs. 2,09,82,857/-

Ans ‘c’

Exercise 29 :

A 9,000 sq.ft. of factory building at Tiruchirappalli is situated in the 2 Acres of land. Age of
the building is 15 years. Salvage value 25%. Replacement rate of building is Rs. 900/sq.ft.
Find the salvage value & depreciated value of the building? Life - 30 years.

(i) a) 18.25 L b) 20.25 L


c) 25 L d) 23.25 L

(ii) a) 40.63 L b) 20.25 L


c) 50.63 L d) 60.63 L

Plinth area = 9,000 sq.ft.


Replacement rate = Rs. 900/sq.ft.
Replacement value = Rs. 81,00,000
Age = 15 years
Life = 30 years
Salvage value = 25%
Depreciation = (15/30) x 75 = 37.5%
Salvage value = 0.25 x 81,00,000
= Rs. 20,25,000/-
Depreciated value of the building = 0.625 x 81,00,000
= Rs. 50,63,000/-

i) Ans ‘b’ ii) Ans ‘c’

Exercise 30 :

A Commercial property at Chennai was valued by a valuer for Rs 95L during the year
2009. What will be the value of the property as on 2017 by using the formula? Assume
12% escalation per year.
256

a) 235 L b) 245 L
c) 225 L d) 200 L

r
Amount = P(1+ )n
100
12 8
= 95,00,000 ( 1 + )
100

= 95,00,000 (2.4759)
= Rs. 2,35,00,000/-

Ans ‘a’

Exercise 31 :

Mr. “Y” constructed a load bearing building of 232 sq.mt during the year 1999. Area of the
plot is 5,000 sq.ft. What is the depreciation value & value of the property in the year 2018
for bank loan purpose? Assume life : 60 years, Salvage value : 10%. Land rate -
Rs. 2,300/-. Replacement rate of building is Rs. 2,000/sq.ft.

(i) Depreciation value


a) 14.23 L b) 24.23 L
c) 18.23 L d) 10.23 L

(ii) Value of the property


a) 14.71 L b) 150.71 L
c) 165 L d) 160.71 L

Extent of land = 5,000 sq.ft.


Land rate = Rs. 2,300/sq.ft.
Land value = 5,000 x 2,300
= Rs. 1,15,00,000/-
Builtup area = 232 sq.m. = 2,497 sq.ft.
Unit rate of construction = Rs. 2,000/sq.ft.
Replacement value = 2,497 x 2,000
= Rs. 49,94,000/-
Age 2018 - 1999 = 19 years
Life = 60 years
Depreciation percentage = (19/60) x 90
= 28.5%
257

Depreciation value = 0.285 x 49,94,000


= Rs. 14,23,000

Present value of the building = 49,94,000 - 14,23,000


= Rs. 35,71,000/-
Total value of the property = 1,15,00,000 + 35,71,000
= Rs. 1,50,71,000/-

i) Ans ‘a’
ii) Ans ‘b’

Exercise 32 :

An investor purchased a plot of land for Rs 6L, and spent Rs 75,000/- towards stamp duty
and brokerage charges. He started construction of house on plot after 3 years. Calculate
the amount that is blocked up in land investment after 3 years on the basis of purchase
price of land and other expenses by considering 7% compound rate of interest?

a) 9.27 L b) 7.27 L
c) 8.27 L d) 9.10 L

Principal sum = 6,00,000 + 75,000


= Rs. 6,75,000/-
Value of land (A) after 3 years = 6,75,000
7 3
= (1 + )
100
= 6,75,000 x 1.225
= Rs. 8,27,000/-

Ans ‘c’

Exercise 33 :

A residential building at Chennai yields a net rental income (Annuity) of 2.4L/year. What is
the capitalised value of the property at 7% rate of interest?

a) 39.29 L b) 34.29 L
c) 43.29 L d) 31.29 L

Y.P. = 100 / R
258

= 100 / 7 = 14.286
Present value of the building = 2,40,000 x 14.286
= Rs. 34,28,640/-
say Rs. 34,29,000/-

Ans ‘b’

Exercise 34 :

Ground floor is a load bearing structure of age 30 years. Life - 60 years. First floor is a
framed structure with independent foundation. Age is 10 years. Life is 80 years. What is
the depreciation for FF, assuming a salvage value of 10%.

a) 11.25% b) 45%
c) 33.75% d) 30%

Ground floor :

30
Depreciation = x 90 = 45%
60

First floor :

10
Depreciation = x 90 = 11.25%
80

Ans ‘a’

Exercise 35 :

Lessee receive an income of Rs. 30,000 per annum. He pays Rs. 16,000/- rent to
landlord. If the lessee receives a rent of 8% return, how much the landlord will expect his
return.

a) 0.09 b) 0.07
c) 0.01 d) 0.10

The return of lessee is (atleast) 1% more than the rate of return of lessor.

Hence the rate of return for lessor is 7%.

Ans ‘b’
259

Exercise 36 :

The lessor receives a ground rent of Rs. 50,000/- from the lessee. The lessee is going to
construct a building and let it out. If the lessor receives a rate of return of 6% from his
lessee, what will be the rate of return that can be expected by the lessee.

a) 6% b) 4%
c) 5% d) 7%

The lessee expects a rate of return of atleast 1% more than the rate of return of
lessor. Hence the rate of return is 7%.

Ans ‘d’

Exercise 37 :

Plot area is 4,800 sq.ft., building 2,400 sq.ft., age is 20 years, life is 60 years, salvage
value is 15%, land rate is Rs. 1,200/sq.ft., replacement rate of building is Rs. 2,100/sq.ft.
Valuation is for security to bank. What is the forced sale value assuming the reduction
factor is 15%.

a) Rs. 78,94,800/- b) Rs. 92,88,000/-


c) Rs. 1,08,00,000/- d) Rs. 91,80,000/-

Land value = 4,800 x 1,200


= Rs. 57,60,000/-
Building depreciation = (20/60) x 90
= 30%
Depreciated value of building = 0.7 x 2,400 x 2,100
= Rs. 35,28,000/-
Value of land & building = Rs. 92,88,000/-
57,60,000 + 35,28,000
Forced sale value = 0.85 x 92,88,000
= Rs. 78,94,800/-

Ans ‘a’

Exercise 38 : (IBBI)

Total age of this building is 4 years. After four years, the depreciated value is equal to 24%
of the cost. Find out the percentage of depreciation (near to answer) by WDV method.
260

a) 24 b) 25
c) 30 d) 35

Method 1 :

r
Formula A = P (1 - )n
100

Depreciated value = Replacement cost x Depreciation


= Replacement cost x 24%

r 4 24
a) For 24% (1 - ) = (1 - )4
100 100

76 76 76 76
= x x x
100 100 100 100
= 0.3336 = 33%

25 4
b) For 25% = (1 - )
100

75 75 75 75
= x x x
100 100 100 100
= 0.3164 = 32%

30 4
c) For 30% = (1 - )
100

70 70 70 70
= x x x
100 100 100 100
= 0.2401 = 24%

35 4
d) For 35% = (1 - )
100

65 65 65 65
= x x x
100 100 100 100
= 0.1785 = 18%

The answer is 30% - ‘c’


261

Method 2 :

24% 25% 30% 35%

Value 100 100 100 100


Less depreciation - 24 - 25 - 30 - 35

After 1 year 76 75 70 65
Less depreciation - 18.24 - 18.75 - 21 - 22.75

After 2 years 57.76 56.25 49.0 42.25


Less depreciation - 13.86 - 14.06 -14.7 -14.79

After 3 years 43.90 42.19 34.3 27.46


Less depreciation - 10.54 - 10.55 - 10.3 - 9.61

After 4 years 33.36 31.64 24 17.85

The answer is 30% - ‘c’.

Exercise 39 :

Total age of this building is 3 years. After 3 years, the depreciated value is equal to 34.30%
of the cost. Find out the percentage of depreciation by WDV method.

a) 15 b) 20 c) 25 d) 30

15% 20% 25% 30%

Value 100 100 100 100


Less depreciation - 15 - 20 - 25 - 30

After 1 year 85 80 75 70
Less depreciation - 12.75 - 16 - 18.75 - 21

After 2 years 72.25 64 56.25 49


Less depreciation - 10.84 - 12.80 - 14.06 -14.70

After 3 years 61.41 51.20 42.19 34.30

The answer is 30% - ‘d’.


262

Exercise 40 :

A building is 40 years old. It has a total life span of 80 years. Current replacement cost of
the building is INR 40,00,000. The salvage value of the materials of the building at the end
of the life is 10% of CRC. What is the depreciation in percentage today?

a) 55% b) 45%
c) 35% d) 65%

Age = 40
Life = 80
Salvage value = 10%
40
Depreciation = x 90 = 45%
80

Answer is ‘b’

Exercise 41 : (IBBI)

A machine was purchased of Rs. 18,000/- before 2 years. It is sold for Rs. 16,000/-
considering 10% depreciation (of Rs. 18,000/-) per annum. The machinery was sold for

a) 2,000 less b) 1,600 less


c) 1,600 profit d) No loss and no gain

Purchased cost two years back = 18,000


Less 10% depreciation = 1,800
Value after one year = 16,200
Less 10% depreciation = 1,800
Value after two years = 14,400
Sold for = Rs. 16,000

... Profit = 16,000 - 14,400 = Rs. 1,600

Answer is ‘c’

Exercise 42 : (IBBI)

The property value is Rs. 1,00,000, expected salvage is Rs. 2,000 after 5 years, what is rate
of depreciation?
263

a) 20 b) 19.60 1,00,000 - 2,000


c) 30 d) 15 5
98%
5 = 19.6%

Ans : b

Exercise 43 : (IBBI)

A mobile phone was purchased for Rs. 60,000/-. It is salvage is Rs. 10,000/-. Total life time
use 40,000 hours. Used time 20,000/-. What is the depreciation of the cell phone?

a) Rs. 12,000/- b) Rs. 15,000/-


c) Rs. 18,000/- d) None of above

60,000 - 10,000 20,000


Ans = 1 x 40,000
= Rs. 25,000/-

Ans : d

Exercise 44 : (IBBI)

Cost of acquisition is Rs. 8,000/-. Salvage value is Rs. 1,000/-. Life of the machine is 3 years.
For WDV, what is the depreciation rate?
8,000
-50% 4,000
a) 50% b) 25% 4,000
-50% 2,000
c) 66% d) 100% 2,000
-50% 1,000
Salvage value 1,000

Ans : a

Exercise 45 : (IBBI)

The net income was reported at Rs. 24,000/- and the property sold for Rs. 3,00,000. What
capitalisation rate is applied to this sale?
24,000 x 100
Ans = 3,00,000
a) 7% b) 8%
c) 9% d) 10%

Ans : b
264

Exercise 46 : (IBBI)

The age of the building is 20 years. The life of the building is 40 years. The replacement cost
of the building as on 2018 is Rs. 5,000/-. The salvage value is Rs. 500/-. Using straight line
method, what is depreciated rate?
5,000 - 500 20
1 x 40
a) Rs. 2,250/- b) Rs. 2,500/-
c) Rs. 2,750/- d) Rs. 3,000/-

Ans : c

Exercise 47 : (IBBI)

A person seeks an income of Rs. 1,000 per annum from an investment. He wishes this is to
be an 8% return on his investment. What is the amount he has to invest?

a) Rs. 1,000/- b) Rs. 80/-


c) Rs. 12,500/- d) Rs. 10,000/-

Annual income = Rs. 1,000


Rate of return = 8%
1,000
Capital value = x 100
8
= Rs. 12,500/-

Ans : c

Exercise 48 : (IBBI)

What would be the written down value of a machine purchased at the cost of Rs. 30,000/-
after 3 years of service life at 5% rate of depreciation?

a) Rs. 26,720/- b) Rs. 25,720/-


c) Rs. 27,720/- d) Rs. 28,720/-

Cost = Rs. 30,000


Less 5% = 1,500
WDV - 1st year = 28,500
Less 5% = 1,425
WDV - 2nd year = 27,075
265

Less 5% = 1,355
WDV - 3rd year = 25,720

Ans : b

Exercise 49 : (IBBI)

A machine was purchased 2 years back at cost of Rs. 4,00,000/-. Total life is 20 years.
Salvage value = 10%. What is the depreciated present value after 2 years

a) Rs. 3,74,000/- b) Rs. 3,54,000/-


c) Rs. 3,64,000/- d) Rs. 3,44,000/-

Purchased cost = Rs. 4,00,000/-


Age = 2 years
Life = 20 years
Salvage value = 10%
2
Depreciation percentage = x 90 = 9%
20
Depreciated value = 0.91 x 4,00,000
= Rs. 3,64,000/-

Ans : c

Exercise 50 : (IBBI)

Workout N.P.V. of a building having 20 years of age and 60 years of total life. Its replacement
cost as on today is Rs. 4,30,000/-. Salvage value 10%. Adopt SLM

a) Rs. 3,11,000/- b) Rs. 4,11,000/-


c) Rs. 3,01,000/- d) Rs. 4,01,000/-

Replacement value = Rs. 4,30,000/-


Age = 20 years
Life = 60 years
Salvage value = 10%
20
Depreciation percentage = x 90 = 30%
60
Net present value = 0.7 x 4,30,000
= Rs. 3,01,000/-

Ans : c
266

Exercise 51 : (IBBI)

What is the N.P.V. by constant percentage method (linear method). Replacement cost is
Rs. 3,50,000/-. Life : 75 years. Age : 15 years.

a) Rs. 3,86,300/- b) Rs. 4,86,300/-


c) Rs. 1,86,300/- d) Rs. 2,86,300/-

100
Depreciation percentage = = 1.33
75

r n
A = P(1- )
100
1.33 15
= 3,50,000 ( 1 - )
100
= 3,50,000 ( 0.9867)15
= 3,50,000 x 0.818
= Rs. 2,86,300/-

Ans : d

Exercise 52 : (IBBI)

A single storeyed house was constructed in 1993, cost Rs. 10,00,000/-, What is the value in
the year 2000 by cost index method of CPWD? Index in 1993 - 244, Index in 2000 - 447,
Base index is 100 for 1981 .

a) Rs. 10,00,000 x (447 - 244) b) Rs. 10,00,000 x (244 / 447)


c) Rs. 10,00,000 x (244 + 447) d) Rs. 10,00,000 x (447 / 244)

Cost of the building in 1993 = Rs. 10,00,000/-


Cost index in 1993 = 244
Cost index in 2000 = 447
Value of the building in 2000 447
= 10,00,000 x
by applying CPWD cost index 244

Ans : d

* * *
267

PART - V

A FEW MORE ONE MARK


THEORY QUESTIONS - 156 Nos.
268

This page is kept vacant intentionally.


269

Part - V

A FEW MORE ONE MARK THEORY QUESTIONS

1. A property was acquired in 01.04.1972. The property was sold on 31.03.2017. On


which date, the FMV is to be ascertained

a) 01.04.1981 b) 01.04.2001
c) 01.04.1972 d) 31.03.2017

Ans : a

2. A property was acquired on 01.04.1972. The property was sold on 01.04.2017. On


which date, the FMV is to be ascertained?

a) 01.04.1981 b) 01.04.2001
c) 01.04.1972 d) 01.04.2017

Ans : b

3. According to the recent amendment in capital gains computations, what is the base
year for which cost inflation index is 100?

a) 2001 - 02 b) 1981 - 82
c) 2016 - 17 d) 2002 - 03

Ans : a

4. According to the recent amendment in capital gains, what is the cost inflation index
for 2001 - 02?

a) 100 b) 110
c) 120 d) 130

Ans : a

5. Cost is a
270

a) Fact b) Policy
c) Opinion d) None of the above

Ans : a

6. Price is

a) Fact b) Policy
c) Opinion d) None of the above

Ans : b

7. Value is

a) Fact b) Policy
c) Opinion d) None of the above

Ans : c

8. 99 years lease with renewal clause is called as

a) Long lease b) Short lease


c) Perpetual lease d) None of the above

Ans : c

9. When valuation is required for a lease property, the main requirements are

a) Location of the property b) Lessor & lessee details

c) Encumbrance details d) Terms & conditions stipulated


in the lease deed

Ans : d

10. The types of lease are

a) Building lease b) Occupational lease


c) Life lease d) All the above

Ans : d
271

11. Annuity is defined as

a) Capitalisation b) Sinking fund


c) Deferment d) Net annual payment

Ans : d

12. A person seeks an income of Rs. 1,000 per annum from an investment. He wishes
this is to be an 8% return on his investment. What is the amount he has to invest?

a) Rs. 1,000/- b) Rs. 80/-


c) Rs. 12,500/- d) Rs. 10,000/-

Ans : c

13. If ‘A’ leases property to ‘B’ for Rs. 2,000/- and ‘B’ in turn subleases to ‘C’ for a rent
of Rs. 3,000/- monthly. Rs. 2,000/- is head rent and Rs. 3,000/- is improved rent.
What is the difference value of profit rent?

a) Rs. 2,000/- b) Rs. 1,000/-


c) Rs. 3,000/- d) Rs. 5,000/-

Ans : b

14. A flat is normally valued by the method namely,

a) Market approach b) Cost approach


c) Income approach d) Composite rate method

Ans : d

15. If any property is to be valued for mortgage purpose to bank as collateral security,
the important factor to be considered is

a) Loan amount b) Value of the property


c) Location of the property d) Marketability & enforceability

Ans : d

16. Sinking fund can be defined as an


272

a) Rate of capitalisation b) Rate of reversion


c) Net annual payment d) Annual recurring fund

Ans : d

17. While calculating depreciation value by constant percentage method, which is not
considered?

a) Age b) Depreciation percentage


c) Salvage value d) Replacement value

Ans : c

18. Which are same?

a) Straight line method b) Linear method and


and linear method constant percentage method

c) Linear method and d) Sinking fund and


sinking fund method straight line method

Ans : b

19. Which type of property enjoys absolute ownership?

a) Leasehold b) Freehold
c) Licensed holder d) Easement holder

Ans : b

20. All the land locked lands enjoy only 25% of the value?

a) Yes, only 25% b) They enjoy less than 25%


c) They enjoy no value d) That depends upon ‘Demand’

Ans : d

21. For which property, profit method of valuation is not appropriate?

a) Petrol bunk b) Hotel


273

c) Lodge d) Residential building

Ans : d

22. For which property, composite rate method is most suitable?

a) Petrol bunk b) Hotel


c) Restaurant d) Flat

Ans : d

r n
23. A=P(1- ) - this formula is to find the
100

a) Depreciated value by constant percentage method


b) Depreciated value by straight line method
c) Depreciated value by sinking fund method
d) Replacement value by straight line method

Ans : a

24. In an apartment, common area percentage is

a) Common area / sum of plinth area of all flats


b) Common area / sum of super plinth area of an flats
c) Common area / FSI
d) None of the above

Ans : a

25. Mere offer for sale cannot be construed as

a) Guideline value b) Market value


c) Distressed value d) Auction value

Ans : b

26. The meaning of cost of construction with reference to income tax valuation is
274

a) The value of the property purchased


b) The market value of land and depreciated value of building
c) The amount spent by the assessee in the construction of his new
building
d) The replacement value of the building

Ans : c

27. In book value method, .................. of the asset in the year of acquisition is taken as
the basis

a) Historic cost b) Intrinsic cost


c) Replacement cost d) Reinstatement cost

Ans : a

28. Temple, church, museum, school, factories can be normally valued by

a) Profit method b) Composite rate method


c) Land & building method d) Comparable sale instance method

Ans : c

29. Land and building method is also known as

a) Depreciated cost method b) Detailed estimate method


c) Book value method d) Composite rate method

Ans : a

30. If the land is situated in central business district area of town, it fetches

a) Low rental value b) High rental value


c) Normal rental value d) None of the above

Ans : b

31. The three important criteria for selection of best land in a town are

a) Availability of power, water and electricity


275

b) Width, depth and size of the plot


c) Soil condition, topography and width of the road
d) Location, location and location

Ans : d

32. The unit rate of large sized lands (say 30,000 sq.m.) is comparatively ...................
when compared to a small size plot of 500 sq.m.

a) More b) Less
c) Same d) None of the above

Ans : b

33. A virgin plot is said to be normally more by a ........ percentage when compared to a
plot with a building

a) 10 to 15% b) 15 to 20%
c) 20 to 25% d) 25 to 30%

Ans : a

34. Ideal proportion of width to depth of a plot could be

a) 1 : 1.5 or 1 : 2 b) 1:3
c) 1:4 d) 1:5

Ans : a

35. Front foot value is the norm used

a) In valuing the land b) In valuing the building


c) In valuing the machinery d) In valuing any property

Ans : a

36. A corner plot is normally enjoys a higher rate by ............ percentage when
compared to the rate of single frontage plot

a) 10 to 15% b) 15 to 20%
276

c) 20 to 25% d) 25 to 30%

Ans : a

37. The plot not having any legal access is known as

a) Recess land b) Tandem plot


c) Land locked land d) None of the above

Ans : c

38. The open unobstructed view in front of the building is known as

a) Vista b) Elevation
c) Clear view d) Excellent

Ans : a

39. Fee simple is called as

a) Freeholder land b) Leasehold land


c) Tenanted property d) None of the above

Ans : a

40. Tenure means condition of holding land. It is a

a) Greek word b) English word


c) German word d) Trench word

Ans : a

41. Value of a leasehold land gets divided into ............... interests

a) 4 b) 3
c) 2 d) 5

Ans : c
277

42. Land may have been used as burial ground or some crime or murder story is linked
with such land. This effect is called as

a) Stigma effect b) Vaastu effect


c) Fang shui effect d) Nuisance effect

Ans : a

43. Belting theory method and Hypothetical plotting scheme (method) of valuing land
are corollaries of

a) Cost approach b) Market approach


c) Income approach d) Composite rate method

Ans : b

44. Belting method cannot be applied for

a) Agricultural lands b) Moffusil lands


c) Urban lands d) Corporation lands

Ans : a

45. Actual cost in terms of money as actually spent by the owner in erecting a building
on the plot is known as

a) Historic cost b) Original cost


c) Intrinsic cost d) None of the above

Ans : a

46. In reinstatement method, depreciation

a) Is to be allowed b) Is not to be allowed


c) Can be / cannot be d) Not applicable
allowed

Ans : b

47. To pay compensation for shifting any religious building during land acquisition, this
method is used
278

a) Reinstatement method b) Replacement method


c) Depreciation method d) Book value method

Ans : a

48. Loss in service value due to usage of an asset and passage of time - this is called
as

a) Appreciation b) Depreciation
c) Escalation d) Depletion

Ans : b

49. A building along sea shore may deteriorate faster than the building away from the
sea shore - this is called as

a) Physical depreciation b) Economic obsolescence


c) Functional obsolescence d) Technological obsolescence

Ans : a

50. A factory building in chenmical zone would wear out faster than the factory in
engineering zone - this is called as

a) Physical depreciation b) Economic obsolescence


c) Functional obsolescence d) Technological obsolescence

Ans : a

51. The buildings which are located in areas which are frequently subjected to earth
quake, cyclone, tsunami, flooding observe sudden damage - this is called as

a) Physical depreciation b) Economic obsolescence


c) Functional obsolescence d) Technological obsolescence

Ans : a

52. Due to structural design defects, serious damage is caused to the building - this is
called as
279

a) Physical depreciation b) Economic obsolescence


c) Functional obsolescence d) Technological obsolescence

Ans : a

53. The assets are under utilised. Optimum economic benefit of the land & building is
not achieved - this is called as

a) Physical depreciation b) Economic obsolescence


c) Functional obsolescence d) Technological obsolescence

Ans : b

54. An asset is put into inferior usage of residence instead of commercial use - this is
called as

a) Physical depreciation b) Economic obsolescence


c) Functional obsolescence d) Technological obsolescence

Ans : b

55. The policy of government to protect slums and not to permit removal of unauthorised
hutments is an example of

a) Physical depreciation b) Economic obsolescence


c) Functional obsolescence d) Technological obsolescence

Ans : b

56. Dilapidation of building or heavy structural cost for the building is

a) Physical depreciation b) Economic obsolescence


c) Functional obsolescence d) Technological obsolescence

Ans : b

57. Rental value of a premises in a particular locality falls severely due to bad
neighbourhood - this is called as

a) Physical depreciation b) Economic obsolescence


280

c) Functional obsolescence d) Technological obsolescence

Ans : b

58. The assets are outdated and their planning and designing are contrary to the present
day requirements - this is called as

a) Physical depreciation b) Economic obsolescence


c) Functional obsolescence d) Technological obsolescence

Ans : c

59. An old palace is a glaring example for

a) Physical depreciation b) Economic obsolescence


c) Functional obsolescence d) Technological obsolescence

Ans : c

60. A single screen theatre is obsolete - this is called as

a) Physical depreciation b) Economic obsolescence


c) Functional obsolescence d) Technological obsolescence

Ans : c

61. A computer may suffer from ..................... hardly within 2 or 3 years if more
advanced computers are there in the market

a) Physical depreciation b) Economic obsolescence


c) Functional obsolescence d) Technological obsolescence

Ans : c

62. A new machine may become obsolete, if product manufactured by said machine
do not have any demand whatsoever in the market - this obsolescence is called as

a) Physical depreciation b) Economic obsolescence


c) Functional obsolescence d) Technological obsolescence

Ans : c
281

63. Load bearing structures are replaced by framed structure. Timber structures are
replaced by steel framed structure. Wooden windows are replaced by aluminium
windows. These are due to the modern technologies - this obsolescence is called
as

a) Physical depreciation b) Economic obsolescence


c) Functional obsolescence d) Technological obsolescence

Ans : d

64. In this method of estimating depreciation, the valuer decides lumpsum


depreciation for the asset by physical inspection. This is called

a) Direct appraisal method b) Written down value method


c) Straight line method d) Constant percentage method

Ans : a

65. The method generally adopted by chartered accountants for preparation of


balance sheet of a company. This is called as

a) Direct appraisal method b) Written down value method


c) Straight line method d) Constant percentage method

Ans : b

66. What would be the written down value of a machine purchased at the cost of
Rs. 30,000/- after 3 years of service life at 5% rate of depreciation?

a) Rs. 26,720/- b) Rs. 25,720/-


c) Rs. 27,720/- d) Rs. 28,720/-

Ans : b

67. In this method, final depreciated value of the asset, after several years of its useful
life, is not shown at zero but it is shown at token Re. 1 till it is sold or transferred.

a) Direct appraisal method b) Written down value method


c) Straight line method d) Constant percentage method

Ans : b
282

68. In this method, equal percentage of depreciation is allowed on its original capital
cost for each year of life. Depreciation amount for each year is exactly same till full
cost is written off.

a) Direct appraisal method b) Written down value method


c) Straight line method d) Constant percentage method

Ans : c

69. A machine was purchased 2 years back at cost of Rs. 4,00,000/-. Total life is 20
years. Salvage value = 10%. What is the depreciated present value after 2 years

a) Rs. 3,74,000/- b) Rs. 3,54,000/-


c) Rs. 3,64,000/- d) Rs. 3,44,000/-

Ans : c

70. Workout N.P.V. of a building having 20 years of age and 60 years of total life. Its
replacement cost as on today is Rs. 4,30,000/-. Salvage value 10%. Adopt SLM

a) Rs. 3,11,000/- b) Rs. 4,11,000/-


c) Rs. 3,01,000/- d) Rs. 4,01,000/-

Ans : c

71. It gives higher depreciation in initial years of the asset when actual wear and tear is
minimum. It gives exactly same depreciation amount for each year, even for a later
period of asset when actual deterioration is very high. This method is

a) Direct appraisal method b) Written down value method


c) Straight line method d) Constant percentage method

Ans : c

72. In this method, rate of depreciation is first assumed. The formula straight away
gives the N.P.V. This method is called

a) Direct appraisal method b) Written down value method


c) Straight line method d) Constant percentage method

Ans : d
283

73. What is the N.P.V. by constant percentage method (linear method). Replacement
cost is Rs. 3,50,000/-. Life : 75 years. Age : 15 years.

a) Rs. 3,86,300/- b) Rs. 4,86,300/-


c) Rs. 1,86,300/- d) Rs. 2,86,300/-

Ans : d

74. In this method, rate of depreciation is adopted as prescribed in Income tax Act -
This is called as

a) Statuatory depreciation method b) Linear method


c) Straight line method d) Sinking fund method

Ans : a

75. There are ......... types of life of the building.

a) 4 b) 3
c) 2 d) 5

Ans : a

76. It is the actual service life of a building. It is also called as planned life or income
yielding life. This life is called

a) Economic life b) Physical life


c) Life due to obsolescence d) Life due to legal constrains

Ans : a

77. It is the actual survival life of the building before collapse. It may be more or less
than the planned life. This life is called

a) Economic life b) Physical life


c) Life due to obsolescence d) Life due to legal constrains

Ans : b

78. It is life of the building by which it becomes obsolete due to changes in life style in
the society. The life is called
284

a) Economic life b) Physical life


c) Life due to obsolescence d) Life due to legal constrains

Ans : c

79. Life of a residential building in an industial zone and life of a factory building in a
residential zone are examples are

a) Economic life b) Physical life


c) Life due to obsolescence d) Life due to legal constrains

Ans : c

80. A building erected on leasehold land which has only 30 years lease period. Income
from building would lease after 30 years. This type of life is called as

a) Economic life b) Physical life


c) Life due to obsolescence d) Life due to legal constrains

Ans : d

81. This approach mainly consists of estimating value of land and building separately
and adding values to arrive at total cost of property.

a) Cost approach b) Market approach


c) Income approach d) None of the above

Ans : a

82. Vyagramukhi plot has .............. front width along the road and ............. width in the
rear side.

a) Wider, narrow b) Equal, equal


c) Narrow, wider d) None of the above

Ans : a

83. As per belting theory, rate of land under 3rd belt is adopted as .....% of the front belt
land rate.
285

a) 50% b) 67%
c) 33% d) 40%

Ans : a

84. Return frontage plots are commonly known as

a) Corner plot c) Tandem plot


c) Recess plot d) None of the above

Ans : a

85. A gaumukhi shaped plot has ............. front width along the road and .............. width
on the rear side

a) Narrow, wider b) Wider, narrow


c) Equal, equal d) None of the above

Ans : a

86. In this type of mortgage, mortgager does not give possession of the property to the
bank, but he gives personal undertaking that he will repay loan. This is

a) Simple mortgage b) Mortgage by conditional sale


c) Usufructuary mortgage d) English mortgage

Ans : a

87. In this mortgage, the mortgage deed provides for conditional sale of the property
by the mortgager to the mortgagee

a) Simple mortgage b) Mortgage by conditional sale


c) Usufructuary mortgage d) English mortgage

Ans : b

88. The mortgager delivers possession of the property to the mortgagee. The
mortgagee receives rent and profits from the property and retains the possession
till the full mortgage money is paid. This mortgage is called as
286

a) Simple mortgage b) Mortgage by conditional sale


c) Usufructuary mortgage d) English mortgage

Ans : c

89. The mortgager binds himself to repay mortgage money on a certain date and he
also transfers the mortgaged property absolutely to the mortgagee, but subject to a
proviso that mortgagee will transfer the property to the mortgager upon full
repayment of the mortgage money. This mortgage is called as

a) Simple mortgage b) Mortgage by conditional sale


c) Usufructuary mortgage d) English mortgage

Ans : d

90. Where a person delivers to the creditor documents of title of immovable property
with intent to create a security thereon, the transaction is called

a) Mortgage by deposit of title deeds or Equitable mortgage


b) Mortgage by conditional sale
c) Simple mortgage
d) English mortgage

Ans : a

91. Land, land with building, flat, office, shop, etc. which are proposed to be purchased
from the finance obtained from the bank are called as

a) Primary security b) Collateral security


c) Current assets d) Stocks

Ans : a

92. Additional securities pledged to the bank in addition to the primary security are
called as

a) Primary security b) Collateral security


c) Current assets d) Stocks

Ans : b
287

93. Bank gives loan to a borrower. He completes the building. Bank requires a report
which is called as

a) Cost report b) Market value report


c) Forced sale value report d) Auction value report

Ans : a

94. The minimum realisable value of property under public auction is called as

a) Reserve price b) Market value


c) Forced sale value d) Auction value

Ans : a

95. It is an estimate of the price of the property would fetch in open market on ‘as is
where is basis’ in a short possible time is called as

a) Reserve price b) Market value


c) Forced sale value d) Auction value

Ans : c

96. The net money likely to be realised by owner by sale of his property is commonly
called as

a) Realisable value b) Market value


c) Forced sale value d) Auction value

Ans : a

97. The estimated amount that one would expect to achieve at properly promoted,
conducted and attended auction sale held at site is called as

a) Realisable value b) Market value


c) Forced sale value d) Auction value

Ans : d

98. It is an estimate of minimum price likely to be offered by the bidders in the public
auction
288

a) Reserve price b) Market value


(upset price)
c) Forced sale value d) Auction value

Ans : a

99. Collapse of American Economy in 2008 - 2009 was due to

a) Lehman Brothers scam b) Joseph Edwin scam


c) Charles Brothers scam d) Winston (p) Ltd scam

Ans : a

100. A valuer is required to consider what will be the position of the property if it is sold
within 12 months of his valuation. This case in English court is called as

a) Corisand case b) Edwin Joseph case


c) John Miltn case d) Kennady brothers case

Ans : a

101. There is a range of the prices of the property. Only competent valuer can fix the
market value of the property. This case is

a) Hays Will Trust case b) Corisand case


c) John Milton case d) Kennady brothers case

Ans : a

102. Authorised officer under this Act has more powers than a court receiver. He can
take not only symbolic possession of the property but can also take physical
possession. What is the Act?

a) SARFAESI Act b) LARAR Act


c) Transfer of Property Act d) DRT Act

Ans : a

103. .......................... possession is taken by the bank when the borrower is in


possession and occupation of the property or when the property is occupation of
third party like tenant, lessee. This is called as
289

a) Symbolic possession b) Physical possession


c) Actual possession d) Legal possession

Ans : a

104. ..................... possession of the property is taken by the bank when borrower
surrenders the possession pursuant to the notice or such possession is taken after
due process of law. It is called as

a) Symbolic possession b) Physical possession


c) Actual possession d) Legal possession

Ans : b

105. A temple property is a

a) Non-marketable property b) Marketable property


c) Primary security d) Collateral property

Ans : a

106. Reverse mortgage scheme (RMS) was introduced in 2007 for the benefit of

a) Senior citizen over 60 years b) Young engineers


c) New entrepreneurs d) Honest business people

Ans : a

107. It is a process by means of which dissolution of an organisation is brought about,


the assets disposed of and in realisation payment towards debts made. Such
process is called as

a) Auction b) Liquidation
c) Sale d) None of the above

Ans : b

108. The value of a property derived from the transactions in which sellers are unwilling
sellers and marketing time is not sufficient - is called as
290

a) Realisable value b) Liquidation value


c) Sale value d) Market value

Ans : b

109. In order to equalise the assets value according to the predetermined exact shares,
there has to be what is known as ................ money exchanged between the
concerned parties

a) Royalty money b) Compensation money


c) Owelty money d) Extra money

Ans : c

110. A right granted by a land owner to an owner of another property for non-exclusive
use of a portion of the land of a specific purpose or enjoyment of certain rights - this
is called as

a) Leasehold right b) Easement right


c) Freehold right d) None of the above

Ans : b

111. In easement right, the land over which the right is imposed is termed as servient
tenement and the owner has a

a) Servient ownership b) Dominant ownership


c) Legal ownership d) None of the above

Ans : a

112. When a co-operative society admits its members only people who are vegetarians
such type of easement is called as

a) Legal easement b) Self imposed easement


c) Conditional easement d) None of the above

Ans : b

113. When the benefit of an easement is not exercised over a long period of time, the
easement
291

a) May be considered b) Is automatically renewed


extinguished

c) Pending decision d) None of the above

Ans : a

114. Any servient easement to a property makes it automatically less attractive and
hence tends to diminish its ................

a) Value b) Right
c) Cost d) None of the above

Ans : a

115. According to this concept, the ownership of a particular property or a flat is held for
a specified period of time during a year. This is called as

a) Time share b) Short lease


c) Temporary accommodation d) Service apartment

Ans : a

116. The most commonly used method for valuing mineral lands is

a) Cost approach b) Market approach


c) Income approach d) Composite method

Ans : c

117. The purpose of imposing stamp duty when a transaction on an immovable property
takes place is

a) To legalise the transaction b) To augment the revenue of


State government

c) The government must know any d) To augment the revenue of


transaction between any parties Central government

Ans : b
292

118. When a land is under sale, the value of land (for the purpose of stamp duty) is fixed
by the Registrar’s office based on the

a) Market rate b) Guideline rate


c) Recent sale instances d) As per intrinsic value

Ans : b

119. Money, cash in bank, gold, silver, jewellery and personal belongings are called as

a) Personal property b) Real property


c) Unreal property d) None of the above

Ans : a

120. Land, land with building are called as

a) Personal property b) Real property


c) Mofussil property d) None of the above

Ans : b

121. Rented properties, hotels, cinemas, malls are termed as

a) Income fetching marketable properties


b) Non-income fetching marketable properties
c) Non-income fetching - non-marketable properties
d) None of the above

Ans : a

122. Owner occupied bungalows, offices, factories are termed as

a) Income fetching marketable properties


b) Non-income fetching marketable properties
c) Non-income fetching - non-marketable properties
d) None of the above

Ans : b
293

123. Temple, church, school, college, public buildings, museum, fire station,
government buildings are termed as

a) Income fetching marketable properties


b) Non-income fetching marketable properties
c) Non-income fetching - non-marketable properties
d) None of the above

Ans : c

124. This approach is generally useful to value income fetching marketable properties

a) Income approach b) Market approach


c) Cost approach d) Composite rate method

Ans : a

125. This approach is generally recommended for the non-income fetching marketable
properties

a) Income approach b) Market approach


c) Cost approach d) Composite rate method

Ans : b

126. This approach is generally adopted for non-income fetching - non-marketable


property

a) Income approach b) Market approach


c) Cost approach d) Composite rate method

Ans : c

127. ................. normally includes profit of seller over and above cost of labour and cost
of materials that has been incurred by the seller in creation of the said asset

a) Cost b) Price
c) Value d) None of the above

Ans : b
294

128. “Value is an estimate of the price as it ought to be” - who said this?

a) Justice Hadley b) Viscount Simon


c) Lee Coca d) Prof. Jean Canonne

Ans : a

129. .............................. value is an estimated amount of the interest or legal right of a


person in a property, to derive existing and future benefit by putting land and
building to best possible legal and potential use.

a) Market value b) Legal value


c) Sale value d) Cost value

Ans : a

130. Sometimes property owners expect some likely changes in government and
expect the value of their property to rise in the near future. Such value is called as

a) Hope value b) Special value


c) Potential value d) Desired value

Ans : a

131. When a property is to be sold as quickly as possible with minimum time for market
exposure, it is termed as

a) Forced liquidation value b) Realisable value


c) Fair market value d) Open market value

Ans : a

132. A property is situated in a highly developed are. Due to heavy demand and less
supply, this property can be sold at a fancy price. This is called

a) Monopoly value b) Hope value


c) Mortgage value d) Intrinsic value

Ans : a
295

133. It is an imaginary value of the property worked out for special purposes of taxation
like wealth tax. This value is called as

a) Notional value b) Potential value


(Hypothetical value)

c) Market value d) Cost value

Ans : a

134. It is a value of the property to the speculator who invests in the property with sole
motive of selling at profit after short period of time. It is called as

a) Speculative value b) Special value


c) Potential value d) Notional value

Ans : a

135. Valuation is not an exact science. Mathematical certainly is not demanded, not
indeed is it possible. Who said this?

a) Justice Viscount Simon b) Justice Hadley


c) Prof. Jean Canonne d) Lee Coca

Ans : a

136. Temple property is not marketable, yet it has got value, we may call it as

a) Notional value b) Market value


c) True value d) Actual value

Ans : a

137. A property which can be physically touched and seen like cash, car, land, etc. are
called as

a) Intangible assets b) Tangible assets


c) Fixed assets d) None of the above

Ans : b
296

138. The assets which cannot be seen or touched but its effect can be notionally seen
and felt - They are called as

a) Intangible assets b) Tangible assets


c) Fixed assets d) None of the above

Ans : a

139. What is the number of elements of value?

a) 5 b) 2
c) 3 d) 4

Ans : d

140. Economic and taxation policies of the government, money market situation,
expected rental yields & returns on investment in real estate are the ......................
factors affecting the value

a) Economic factors b) Physical (technical) factors


c) Social factors d) Legal factors

Ans : a

141. Income fetching capacity of the property, cyclical boom & recession periods in real
estate market, employment opportunities are the .................. factors affecting the
value.

a) Economic factors b) Physical (technical) factors


c) Social factors d) Legal factors

Ans : a

142. Swimming pool, garden, lift, security system, health club, good network of roads,
water supply, drainage system, power supply are some of the ................. factors
affecting the value.

a) Economic factors b) Physical (technical) factors


c) Social factors d) Legal factors

Ans : b
297

143. Proximity of shops, market, school, cinema, hospital, railway station, bus stand,
temple, place of workship are some of the ................ factors affecting the value.

a) Economic factors b) Physical (technical) factors


c) Social factors d) Legal factors

Ans : c

144. Ecological restriction, Easement Act, Covenants in a lease deed, Income tax Act
are some of the ............... factors affecting the value.

a) Economic factors b) Physical (technical) factors


c) Social factors d) Legal factors

Ans : d

145. Reservation under different Acts, Height restriction rules near the airport area, safety
distance rules from High tension lines, railway tracks, highways, water courses are
some of the ............. factors affecting the value.

a) Economic factors b) Physical (technical) factors


c) Social factors d) Legal factors

Ans : d

146. A valuation report originates with the instruction given by the ............. in response
to which the task of writing a report is undertaken by a valuer.

a) Lawyer b) Client
c) Concerned department d) None of the above

Ans : b

147. The main requirement to write a valuation report

a) Technical and communication skill


b) Skill in the English knowledge
c) To write a detailed report like a project report
d) Good presentation

Ans : a
298

148. Valuation reports are to be prepared

a) Aft the time of receiving the relevant documents


b) After valuation is completed
c) Before undertaking valuation
d) During the inspection of the said property

Ans : b

149. Many a times, use of ‘cut’, ‘copy’ and ‘paste’ functions in computer make valuers
lazy about report writing which may result in enquiry by ................

a) Clients b) User

c) Investigation agency like d) Lawyer


CBI, Policy, etc.

Ans : c

150. News paper or media reports giving only information which may be without
expressing any opinion or advice which is left to the reader to make out. This is

a) Non-technical report b) Technical reports


c) Legal reports d) Project reports

Ans : a

151. It is the exercise of technical expertise and therefore gives all the necessary
information and data with their analysis and expresses definite conclusion or
opinion. The user takes appropriate decision based on this report. This is

a) Non-technical reports b) Technical report


c) Legal reports d) Project reports

Ans : b

152. A written report casts high degree of ........ on the valuer as it acts as a formal record
and becomes a document/

a) Accountability b) Duty
299

c) Integrity d) Honesty

Ans : a

153. At the time of valuation, the valuer should have high degree of ................ with the
client and property to be valued.

a) Attachment b) Detachment
c) Intimacy d) None of the above

Ans : b

154. While estimating the market value for mortgage, a valuer should consider himself
to be ..............

a) a lawyer b) a mortgage
c) a mortgagor d) both mortgagor and mortgagee

Ans : d

155. The first column of any valuation report should be

a) Name of the client b) Documents perused


c) Location d) Purpose of valuation

Ans : d

156. It means reduce in numbers of exhaust. It is the word applied to the consumption of
natural resources. Typical examples are natural gas, oil, coal, water, mineral

a) Depreciation b) Depletion
c) Exhaust d) None of the above

Ans : b

* * *
300

This page is kept vacant intentionally.


301

PART - VI

FREQUENTLY ASKED QUESTIONS


IN THE EXAMINATION - 450 Nos.
302

This page is kept vacant intentionally.


303

Part - VI

FREQUENTLY ASKED QUESTIONS (FAQ) IN THE EXAMINATION

1. Which of the following is not a natural attribute of a land, but artificially created by
man?

a) Benefit b) Tenure
b) Location d) Frontage

Ans : b

2. Onus of proof is a ................ in the evaluation of evidence.

a) Fact b) Principle
c) Continuous process d) One time process

Ans : c

3. Which of the following terms is not relevant to valuation of contaminated properties?

a) Environmental stigma b) Impaired value


c) Reinstatement value for d) Remediation cost
insurance

Ans : c

4. In perfect competition, an individual firm .................

a) has a price elastically of supply equal to one.


b) faces unitary elasticity of demand.
c) has a price elasticity of supply equal to infinity.
d) faces infinitely elastic demand.

Ans : c

5. Which of the following distinguishes investment in real estate vis-a-vis other forms
of investment?
304

a) Capital erosion b) Redemption of capital


c) Income generation d) Solvency

Ans : b

6. Which out of the following is not a factor that affects demand & supply of real estate?

a) Economic growth b) Interest rate


c) Increased population d) Number of cars in household

Ans : d

7. Under sec 61 of Indian succession Act, a will or any part of a will, which has been
caused by fraud and which takes away the free agency of the testator, is ..................

a) Illegal b) Valid
c) Voidable d) Void

Ans : a

8. Utility means existing and anticipated ............... due to ownership & use of property.

a) Marketability b) Scarcity
c) Transferability d) Benefits

Ans : d

9. The value of the rent controlled properties will have ............. value than vacant property.

a) 100 percent higher b) 50 percent higher


c) Exactly same d) lower

Ans : d

10. In case of gift, if donee dies before acceptance, then .................

a) Gift is valid. b) Gift is void.


c) Gift is reduced. d) Court has to decide.

Ans : b
305

11. Which valuation method should be used to calculate the market value of vacant
building excluding land?

a) Sales comparable b) Depreciated replacement cost


c) Income capitalization d) Residue technique

Ans : b

12. Which of the following is consistent with the law of demand?

a) A decrease in the price of a gallon of milk causes a decrease in the


quantity of milk demanded.
b) An increase in the price of a soda causes a decrease in the quantity of
soda demanded.
c) An increase in the price of a tape causes an increase the quantity of
tapes demanded.
d) A decrease in the price of juice causese no change in the quantity of
juice demanded.

Ans : b

13. H.A. Sturges formula for determining the number of classes is :

a) 1 - 3.322 * log N b) 1 + 33.22 * log N


c) 1 - 33.22 * log N d) 1 + 3.322 * log N

Ans : d

14. Sec 112 under Transfer of Property Act for waiver of forfeiture but where rent is
accepted after the institution of a suit to eject the lessee on the ground of forfeiture,
such acceptance .........................

a) is a forfeiture b) is not forfeiture


c) is determination of lease d) is transfer of lease

Ans : b

15. Oridinal utility analysis is otherwise known as :

a) Gossens second law b) Cardinallty approach


306

c) Indifference curve analysis d) Rationality approach

Ans : a

16. Interest is paid because ;

a) Capital is scarce b) Capital is productive


c) Capital is attractive d) Capital is surplus

Ans : b

17. Severance of joint family status takes place from the date when the communication
.................

a) is received by the first coparcenary


b) is received by the last coparcenary
c) is received by the karta
d) is sent

Ans : d

18. Any profit or loss on the sale of sinking deprecation fund investments is transferred to
...............

a) Profit and loss account b) Asset account


c) Sinking fund a/c d) Depreciation account

Ans : a

19. A owes B Rs. 1,000 payable on 1st December 2017 with interest. On 1st June 2017,
A offers to pay the amount with interest up to 1st of June 2017. It is

a) a valid tender b) not a valid tender of performance


c) a legal tender d) an Illegal tender

Ans : a

20. Premature termination of lease is a major .......... factor affecting the value of leasehold
property.

a) Physical b) Legal
307

c) Social d) economic

Ans : d

21. Data obtained by conducting a survey is called ……………

a) Primary data b) Secondary data


c) Measured data d) Regular data

Ans : a

22. Which of the following is not a criterion in determining the highest and best use of
land?

a) Physically possible b) Legally permissible


c) Financially feasible d) Socially acceptable

Ans : d

23. Which of the following is not a method used to calculate depreciation?

a) Straight line method b) Declining Balance method


c) Average of the year digits d) Constant percentage method

Ans : c

24. Where there is frequent fluctuations in stocks/ stock values, which of the following
provides suitable cover?

a) Fire floating policy b) Fire policy with Escalation


clause
c) Fire Reinstatement value policy d) Fire Declaration policy

Ans : b

25. Where rent reserved in an occupational lease is less than the full rental value, it is
called ..................

a) Acknowledgement rent b) Head rent


c) Standard rent d) Virtual rent

Ans : a
308

26. Rehabilitations and resettlement award for affected as per land acquisition,
rehabilitation and resettlement act 2013 is made by

a) Central government b) Local panchayat, municipal (or)


village authority
c) State Government d) Tribunal authority

Ans : c

27. Before accepting the gift, if the donee expired, this is

a) Void b) Voidable
c) Valid d) The court will finalise

Ans : a

28. In case of lease hold property with perpetuity of 99 years lease and If the lease deed
is having renewable class of for further 99 years, the lessor’s interest is

a) More b) Not more


c) less d) revocable

Ans : c

29. The reversionary value of property is one

a) residual value of property


b) lease value of the property
c) value of the property in the hands of lessee
d) value of the property at the end of lease period in the hands of lessor

Ans : d

30. If the promiser promises to perform the promise of third person or to discharge the
liability (or) obligation of a third person in the case of the latter’s default if fullfilled by
person is

a) Indemnity b) Promiser
c) Guarantee d) none of above

Ans : c
309

31. ........... is appointed to inform agriculturalist about the value, type of pesticides and its
level of poison.

a) ICAR b) CIBRAC
c) Rural bank d) RBI

Ans : b

32. As per RERA Act, the promoter has to collect from customer, maintain and spend the
money through escrow account. The percentage of money deposited is ........

a) 50% b) 60%
c) 70% d) 80%

Ans : c

33. Out of the following, the one which is not belonging to mobile pollution is

a) Tobacco smoking b) Gas mounted car


c) Mining d) Trucks

Ans : c

34. Deduction allowed made for rental income under income tax for the house hold rental
property is ................

a) 15% b) 25%
c) 30% d) 20%

Ans : c

35. The valuer should maintain the records for minimum of ................ years

a) 1 Year b) 3 Years
c) 5 Years d) 10 Years

Ans : b

36. The valuer should be straight forward and honest in performing professional service
because
310

a) for the owner’s interest b) for his own interest


c) for the interest of society d) none of the above

Ans : c

37. To whom can a creditor prefer an appeal in case of rejection of his claim

a) Insolvency and bankruptcy board of India


b) Jurisdictional District court
c) National Company Law Tribunal (NCLT)
d) State High court

Ans : c

38. The class of locality, neighbourhood, prestigious aspects regions factors like temple,
church, worship places, sentiments like vaasthu, etc, are ............

a) Physical factors b) Natural factors


c) Legal factors d) Social factors

Ans : d

39. For RCC roof framed building, the life of building considered for valuation purpose is
.................

a) 40 to 60 Years b) 60 to 80 Years
c) 80 to 100 Years d) 100 to 120 Years

Ans : b

40. Which method is adopted for valuation of a property located at extension under
developing area of the city?

a) Unit rate method b) Cost approach method


c) Sales comparison method d) detailed estimate method

Ans : c

41. For a free hold property owned by a person, If half portion is rented out for tenant and
half portion is owned by the land lord, the property is valued by which method?
311

a) Rent Capitalization method


b) income method
c) Cost approach method
d) The rental portion is valued by rent capitalization method and
owner’s portion is valued as free hold property.

Ans : d

42. When many number of units are available to the purchaser in excess of demand, it is

a) Sellers market b) Buyers market


c) perfect computation market d) Demand market

Ans : b
43. Saving is ............... the relationship with income is :

a) Income less consumption b) Consumption less saving


c) Saving of money d) None of above

Ans : a

44. What is extra premium for add on cover for act of god in standard peril policy?

a) 1% b) 2%
c) 3% d) None of the above

Ans : d

45. If a person having two wives dies, the property share is :

a) Him self and his sons will take equally


b) His sons will take equal share
c) His wives will take equal share
d) Wives and sons will take equal shares

Ans : d

46. As per LARAR Act, the acquisition advertisement is made in ....................

a) Any language b) English & local language


312

b) local language d) any regional language

Ans : b

47. A gift is a made to two persons. If one person dies before accepting the gift, then

a) it is void b) it is voidable
c) it is donors option to finalise, d) none of above

Ans : a

48. The forest and animal are not bounded by the following act :

a) Indian forest act b) Indian wild life act


c) Environmental protection d) Bio diversified act
act 1986

Ans : c

49. Whether a tenant can transfer the property to other tenant?

a) with the permission of land lord b) it is not transferable


c) it is transferable d) None of the above

Ans : a

50. Under fundamental right in article 14, equality is for

a) Indian citizen b) All persons in India


c) Individuals only d) Peoples of India

Ans : b

51. Contingent contract is

a) Void b) Voidable
c) valid d) Complete

Ans : c
313

52. In adjustment grid model, the first factor considered is

a) Size b) time
c) location d) age

Ans : b

53. Who has the power to acquire?

a) State government b) Central government


c) District Collector d) Corporation

Ans : c

54. If demand in unitary elastic a 25% increase in price will result in

a) 25% change in total revenue


b) No change in quantity demanded
c) 1% decrease in quantity demand
d) 25% decreases in quantity demanded

Ans : b

55. Cost of disturbance experienced during the remediation stage is known as

a) Cost of contract b) Cost of public liability


c) Cost of distribution d) Cost to operation

Ans : d

56. Price is a ................. concept

a) Exchange b) Market
c) Benefit d) Transferability

Ans : a

57. Insolvency process can be initiated by ................


314

a) Insolvency professional b) Operational creditors


c) NCLT d) DRT

Ans : b

58. Under marginal productivity theory, reward for labour is determined by

a) Owner b) Government
c) Labour d) Marginal product

Ans : d

59. A valuer should act without conflict and interest coeircen or under influence of any
party.

a) This is not requirement b) This is in public interest


of valuer
c) This is proof d) It brings higher value of asset

Ans : b

60. Fire consequential loss policy pays the insurer

a) Net profit b) Standing charges


c) Increased cost of work d) All the above

Ans : d

61. When the owner’s right is imperfect and will be made perfect on fulfilment of certain
conditions is called

a) Co-Owner ship b) Concurrent ownership


c) Contingent ownership d) Duplicate ownership

Ans : c

62. In a city, the Development Control Rules are governed by

a) Approved plan b) Location


c) FSI d) Utility

Ans : c
315

63. Two interest rates are considered when the situation of income is

a) Perpetual b) Annual
c) Terminable d) None

Ans : c

64. Accumulated sum of Rs 1/- per year for 6 years at the rate (1 + r)n - 1
APA = r
of interest of 8 %

a) 0.08/(10.08)6 -1} b) {(1+0.08)6-1}/0.08


c) {1- (10.08)6}/1 d) /(10.08)6

Ans : b

65. Key element of income approach?

a) Potential income stream b) Expenses


c) Outstanding loans d) Capitalization rates

Ans : d

66. The property value is Rs 100000, expected salvage is Rs 2000 after 5 years, what is
rate of depreciation? 1,00,000 - 2,000
5
i.e. 98%
a) 20 b) 19.60 5 = 19.6%
c) 30 d) 15

Ans : b

67. FSI is allowed 100 % for

a) Central Govt Offices b) State Govt Offices


c) Schools d) Sports complex

Ans : d

68. Where maximum rate of interest is obtained?

a) Shares & debentures b) Banks current accounts


316

c) Fixed deposits d) Investment in housing society

Ans : c

69. The difference in rent received by head lessee after giving leased property by him to
sub-lessee is called

a) Contractual rent b) Standard rent


c) Profit rent d) Rack rent

Ans : c

70. Lessor gives open land to lessee to permit him to construct building, the lease is
called

a) Building lease b) Perpetual lease


c) Occupational lease d) Life lease

Ans : a

71. If Y.P is 12.50, then the rate of return is in percentage 100 / 12.5

a) 9% b) 8% c) 10% d) 12%
Ans : b

72. When the supply is excess than demand, then the market is

a) Stable market b) Seller market


c) Buyer market d) Vegetable market

Ans : c

73. In indifferent curve, the demand curve shows

a) down-ward from left to right b) down-ward from right to left


c) upward from right to left d) upward from left to rigtht

Ans : a

74. Hypothetical grid model applies to


317

a) commercial buildings b) residential buildings


c) flats d) Govt offices

Ans : c

75. In a joint venture development, main parameters are :

a) Land rate & FSI b) Building rate & FSI


c) Land rate and building cost d) FSI & Plot coverage

Ans : c

76. The most accurate and reliable cost of construction acceptable by court is

a) cost of construction b) Quantity survey method


c) cost index method d) plinth area method

Ans : b

77. The economic life of a RCC load bearing structure is considered as ............

a) 80 years b) 60 years
c) 30 years d) 120 years

Ans : b

78. Where Transfer of Development rights was first introduced in India?

a) Bombay b) Kolkata
c) Hyderabad d) Chennai

Ans : a

79. Land can’t be ........... like other commodities.

a) transported b) bought
c) sold d) mortgaged

Ans : a
318

80. Valuation is an art or science, mathematically certainty not required, nor indeed is it
possible.

a) Rustam C Cooper Vs Union of India AIR 1970 SC 564


b) Gold Coast Trust Ltd. Vs Humphray (1949) 17 ITR 19
c) Hays Will Trust Vs Hays and others
d) K P Varghese Vs ITO (1981) 131 ITR 597 (Sc)

Ans : b

81. For owner occupied portion, the District Valuation officer calculated the value on the
basis of what were the rates prevalent for sale of commercial flats in cannaught place.
For the tenanted portion, he capitalized the rental value. The method adopted by him
is acceptable.

a) Hays Will Trust Vs Hays and others


b) K P Varghese Vs ITO (1981) 131 ITR 597 (Sc)
c) Wenger & co Vs DVO(1978) 115 ITR 648 Delhi H C (combination
methods)
d) Gold Coast Trust Ltd. Vs Humphray (1949) 17 ITR 19

Ans : c

82. Terrorism cover for industrial risks is subject to a deduction of

a) 0.5% b) 1%
c) 1.50% d) 0.025%

Ans : a

83. The sum insured under debris removal add-on cover cannot exceed —— % of the
total sum insured under the fire policy.

a) 20% b) 15%
c) 1% d) 10%

Ans : d

84. Delta saving / Delta income =


319

a) APS b) MPS
c) APC d) MPC

Ans : b

85. Theory of rent was proposed by

a) Ricardo b) Allmond Alla


c) Marshall d) F.B Hawley

Ans : a

86. Deflation means

a) Increase in price in goods b) Decrease in price in goods


c) Increase in quantity d) Decrease in quantity

Ans : b

87. Primary sector of Economics

a) Building factory that produce materials b) Building school


c) Agriculture d) Horticulture

Ans : c

88. Mr. ‘A” constructed a property, cost was Rs. 5,00,000/-, during 1990. He sold to ‘B’ in
2000 for Rs. 10,00,000/-. ..................... is the cost in the hands of Mr. ‘B’.

a) Historic cost b) Original cost


c) Replacement cost d) Reinstatement cost

Ans : b

89. When a land does not abut on any road and does not enjoy any legal approach, such
land is called

a) Land locked land b) Recess land


c) Double frontage land d) Tandom plot

Ans : a
320

90. Properties which are under developed, somebody buys such properties, develop it
with the idea to earn profit. Such properties are called

a) Development properties b) Investment properties


c) Potential properties d) Non investment property

Ans : c

91. Shopping mall, service apartments, office blocks are

a) Investment properties b) Developed properties


c) Potential properties d) Non development properties

Ans : a

92. Which of the following is not an organic water pollutant?

a) Arsenic b) Lead
c) Organic fertilizer d) Heavy metal

Ans : a

93. If land rate is high compared to cost of construction, landlord’s share in a joint venture
will be

a) High b) Equal
c) Low d) None

Ans : a

94. Which of the following is generally acceptable?

a) Demand draft b) Money


c) Land d) None

Ans : b

95. A donee dies before acceptance of the gift is

a) Valid b) Void
321

c) Voidable d) Landful

Ans : b

96. When a lessee transfers property, then the lessor, normally, claims a share in the profit.
This profit is known as ..................

a) Profit rent b) Imporoved rent


c) Unearned increase d) Head rent
in land value

Ans : c

97. Which of these is a method of calculating depreciation based on interest theories?

a) Direct appraisal method b) Straight line method


c) Declining balance method d) Sinking fund method

Ans : d

98. The Doctrine of unearned increase was enacted beacuse of a famous court
judgement. Select the correct judgement.

a) Commissioner of Wealth Tax, New Delhi vs. Sri. P.N. Sikand (1977)
107 ITR 922 (SC)
b) CIT vs. Smt. Ashima Sinha 1979) 116 ITR 26 (Calcutta), 1980 Tax 56(1)
19 Calcutta).
c) Controller of Estate Duty vs. Radha Devi Jalan (1968) 67 ITR 761,
Calcutta High Court.
d) C.W.T. vs. Venugopal Konar & Ors. ((1977) 109 ITR 52, Madras High
Court.

Ans : a

99. The landmark judgement, Commissioner of Wealth Tax, New Delhi vs. Sri. P.N. Sikand
(1977) 107 ITR 922 (SC) states that :

a) The increase in value of the leasehold interest of the property leased is


to be equally shared by both the lessor and lessee.
b) only the lessor has the right to the increase in value.
322

c) only the lessee has the right to the increase in value.


d) the ratio of the lessor’s and lessee’s share is to be determined by
negotiation.

Ans : a

100. Can the tenant carry out repairs by himself?

a) Yes b) No
c) With oral permission d) With written permission from
of landlord landlord

Ans : c

101. Under Sec 192 of Companies Act - the value of the asset is duly calculated by

a) Company Seceretary b) Chartered Accountant


c) Registered Valuer d) Accountant

Ans : c

102. Who can write a Will?

a) Minor b) Lunatic
c) Person with a sound mind, d) None of the above
not a minor

Ans : c

103. Who can write a privileged Will?

a) Mariner & Airman b) Airman


c) Mariner in Sea Expedition d) Anyone

Ans : a

104. RERA - For alteration to the plan - the developer should take the consent from atleast
.................

a) 1/3 allotees b) 2/3 allotees


323

c) All allotees d) No need of any consent

Ans : b

105. A car was bought for Rs. 2 lacs 2 years back. The market value of the car today is
1 lakh. What is the “realizable value”?

a) 1 lakh b) 2 lakhs
c) 3 lakhs d) None

Ans : a

106. S/Y =

a) APS b) APC
c) MPS d) MPC

Ans : a

107. In market approach, this is not considered :

a) Location b) Size
c) Age d) Purchase cost

Ans : d

108. In DCF technique, which is not given weightage?

a) Discount rate b) (IRR) Internal Rate of Return


c) Expenditure d) Location

Ans : d

109. The probability of occurence of all events in a random experiment is

a) Zero b) One
c) Infinity d) None

Ans : a
324

110. ................... is affected by DCR (Development Control Rules).

a) Value in use b) Value in exchange


c) HABU - Highest & Best use d) Safety (or something like this)

Ans : c

111. Commercial bank is not related in .................

a) Issuing paper money b) Issuing loans


c) Credit control d) Receiving receipts

Ans : a

112. When and where NSE (National Stock Exchange) was formed?

a) in 1993 in Mumbai b) in 1993 in Kolkatta


c) in 1993 in Chennai d) in 1992 in Delhi

Ans : a

113. Not true stigma

a) Intangible b) May not be measured


c) Off the market phenomenon d) May be measured

Ans : c

114. Algae in river how?

a) Oxygen in water b) Nutrients


c) Sunlight and Nutrients d) Sunlight

Ans : c

115. Who tells the Government on matter of forests conversation?

a) Advisory committee b) Secretary Minister of forest


c) Chief Minister of forest d) Advisor Minister of forest

Ans : a
325

116. Architect fees in insurance?

a) 1% b) 2% c) 3% d) 4%

Ans : c

117. Purchase of property lies in which concept?

a) Demand side b) Supply side


c) Exchange d) Bargain

Ans : d

118. Cost represents ..................... side of transaction.

a) Exchange b) Demand c) Supply d) Bargain

Ans : c

119. Which gives comparitively a reliable high rate of interest?

a) RD b) FD
c) Share market d) Gifted securities

Ans : b

120. The value of rent controlled properties will have

a) High market value b) Low market value


c) Lowest market value d) Same market vale

Ans : b

121. A mobile phone was purchased for Rs. 50,000/-. It’s salvage value is Rs. 10,000/-.
Total life time used is 60,000 hours. Used time is 20,000/-. What is the depreciation of
the cell phone?
50,000 - 10,000 20,000
Ans = 1 x 60,000
= Rs. 13,383/-
a) Rs. 12,000/- b) Rs. 15,000/-
c) Rs. 18,000/- d) None of above

Ans : d
326

122. A capital is .................

a) Money b) Machinery
c) Entrepreneur d) Labour

Ans : b

123. Which is not hirable?

a) Labour b) Machine
c) Capital d) Entrepreneur

Ans : d

124. Which is not a Mobile air pollunt?

a) Tobacco smoking b) Mining


c) Gas used as fuel in car d) Motor cycle

Ans : b

125. Value in use also called as ..............

a) Real value b) Subjective value


c) Market value d) None of the above

Ans : b

126. The land with high FSI or the land with low FSI - in which, the land value is high?

a) High FSI land b) Same FSI land


c) Low FSI land d) Equal FSI land

Ans : a

127. In supply and demand curves the quantity of goods in which axis

a) X - axis b) Y - axis
c) X & Y - axis d) None of above

Ans : a
327

128. Which is correct?

a) Mean > Median > Mode b) Median > Mode > Mean
c) Mode > Mean > Median d) None of the above

Ans : a

129. By benefit approach method, this property is not valued

a) Dams b) Bridge
c) Highways d) Cinema Theatre

Ans : d

130. As per building bye laws, which is permissible in Cinema Theatre complex?

a) 1 toilet for 100 person and 1 urinal for 75


b) 1 toilet for 100 person and 2 urinals for 75
c) 2 toilet for 100 person and 2 urinals for 75
d) None of the above

Ans : a

131. Under which schedule, the deprerciation is calculated?

a) Schedule - II / Companies Act b) Schedule - III / Wealth Tax


c) Schedule - IV d) None of the above

Ans : a

132. Which depreciation method gives directly the N.P. value? (Net Present Value)

a) Constant %age method b) Sinking fund method


c) Straight line method d) Lump sum method

Ans : a

133. Which of the following is not a factor affecting to the capitalisation of rate?

a) Security of capital b) Security of income


328

c) Replacement cost d) Prospectus of appreciation

Ans : d

134. A valuer may accept the following one beyond fees for his service.

a) Gift b) Hospitality
c) Sucession fee d) None of the above

Ans : b

135. Which is not the nature of real estate?

a) Benefit b) Frontage
c) Location d) Tenure

Ans : d

136. Insolvency process should be completed within ................ days.

a) 60 days b) 90 days
c) 180 days + 90 days d) 300 days

Ans : c

137. Which of the following is eligible under Transfer of Property Act to executive will?

a) Minor b) Unsound in mind


c) Major d) Intoxised person

Ans : c

138. For income generating commercial perperties, the valuation method is .............

a) Cost b) Market
c) DCF d) Comparison

Ans : c

139. For insurance purpose (fire policy) which cost is normally used?
329

a) Book cost b) Reproduction cost or


Reinstatement cost
c) Replacement cost d) Depreciated cost

Ans : d

140. Which peril is not covered under standard fire policy?

a) Impact damage b) STFI


c) Earthquake d) Fire

Ans : c

141. The assessment of yearly value of the property is known as .................

a) Rateable value b) Property Tax


c) Replacement value d) Book value

Ans : a

142. What is the common factor between distress sale value and forced sale value?

a) Market value b) Private negotiation


c) Limited time d) Sufficient time

ans : c

143. Cash, Jewellery, Car, Machines, Land and Building are called as

a) Tangible assets b) Intangible assets


c) Valuable assets d) Waste assets

Ans : a

144. Good will, brand right , copy rights, intellectual property, life interest are called as

a) Intangible assets b) Tangible assets


c) Valuable assets d) Waste assets

Ans : a
330

145. Which of the following method is to be adopted if an asset is to be used more in the
early years of its life than to that of later years.

a) SLM b) Constant percentage method


c) Sinking fund method d) Book value method

Ans : b

146. Will is executed to a widow for using the house till her life and the same can be
transferred to their sons after her death.

a) Freehold / Life estate b) Life increase


c) Leasehold d) None of the above

Ans : a

147. Following is true under the fire insurable policy?

a) Escalation clause applies to building and machinery


b) Escalation clause applies to fluctuating stock
c) Escalation clause applies to building, machinery & fluctuating stock
d) Escalation clause does not apply to building & machinery

Ans : a

148. As per IVS standard, market value consists of ------------. Which one of the following is
not the element?

a) Estimate amount b) Price


c) Willing seller d) At an arm’s length transaction

Ans : a

149. Mr. A is going to start a project, approaches B for expert opinion. Relationship
between A and B is defined as

a) Contract for services b) Consultancy services


c) Service provider d) Fiduciary relationship

Ans : d
331

150. Why should an entrepreneur do a feasibility study to start a new venture?

a) To estimate the expected sale b) Possible source of fund


c) Possible barriers to success d) Potential customer

Ans : c

151. “Carrying value”, the following asset statement best describes

a) To cost of the asset less its residual value


b) Net value of asset or value carried in book of a/c which is higher
c) The higher of due assets value in use of is recoverable amount
d) Original cost - accumulated depreciation amount

Ans : d

152. Credit card commission is charged by the bank

a) 1% to 3% b) 2% to 5%
c) 3% to 6% d) 1% to 4%

Ans : d

153. Valuer should maintain integrity because

a) It is not the interest of the values b) for the interest of the society
c) Interest of the client as well as valuer d) Shall keep public interest

Ans : b

154. Estimate of an amount for “auction sale” is known as

a) Liquidation b) Salvage value


c) Net present value d) Replacement value

Ans : a

155. Saving = Income - Consumption (S = Y - C)


Saving = Income / Consumption
Saving = Income + Consumption

Ans : a
332

156. Depreciation on building estimate ....................

a) Economic life b) Physical life


c) Obsolescence life d) Life in perperuity

Ans : a

157. Flow of income is in perpetuity, years purchase figure would ............... with decrease
in rate of interest.

a) Increase b) Decrease
c) Remain same d) Zero

Ans : a

158. SARFAESI Act 2002 - What is meant by enforcement of security interest?

a) Sale of assets of borrower by the bank


b) DRT
c) With court intervention
d) Getting bank’s charge with Central Registry

Ans : a

159. Stationary source of air pollution

a) Light duty gasoline - Powered gas b) Mining


c) Tobacco d) Truck

Ans : b

160. Development control rules of the city / town decides the ................. of the property.

a) Security b) Rise
c) Usage d) Transferability

Ans : c

161. Under market approach, we can normally value only .................

a) Religious property b) Household property


333

c) Non-marketable property d) Marketable property

Ans :. d

162. Under Transfer of property, immovable goods does not include ..............

a) Timber b) Timber, Jewellery


c) Grass only d) Timber, Jewellery, Grass or Crops

Ans : d

163. If contract specified time fails, then contracts becomes ..................

a) Invalid b) Voidable
c) Incomplete d) Disqualified

Ans : a

164. Price discrimination is not possible in .........................

a) Monopoly b) Perfect market


c) Duopoly d) Oligopoly

Ans : b

165. Which of the following details are not to be included in the valuation report?

a) Who gave instructions to prepare the report?


b) Which is the date as on which the value of the property is required to be
estimated?
c) The details of the location and the neighbourhood of the property.
d) The age of the owner of the property.

Ans : d

166. Which one is not giving rise to organic pollution in water pollution

a) Fertilizers b) Pesticides
c) Organic compounds d) Heavy metals

Ans : d
334

167. Which of the following statement is correct

a) Hotels and restaurants should be valued on the basis of their


profitability.
b) Hotels and restaurants should be valued to their tangible assets only.
c) There are no intangible assets associated with the hotel and restaurant
industry.
d) Advertisement income in the hotel and restaurants investment be treated
as scale of profit.

Ans : a

1
168. Years purchase Re. 1, 10 years, 7% Y.P. = (1 + r)n

a) 1 - (1/1 + 0.07)10 / 0.07 b) 0.07 / 1 - (1 + 0.07)10


c) 0.07 / (1 + 0.07)10 d) 1 / (1 + 0.07)10

Ans : d

169. Most appropriate method of valuation income generation commercial assets?

a) Land and building b) Any method


c) Direct comparison method d) Discounted Cash Flow method

Ans : d

170. Which of the following statements is true related to obligation of insurer on notification
of a claim.

a) To defend the insurer b) To indemnify the insured


c) To investigate and settle d) To ensure that the damage applies
potentially covered claim to the property

Ans : c

171. Under discounted cash flow method, value of property increases with ...................

a) Higher property tax b) High capitalization rate


c) Lower discount rate d) Reduced income flow

Ans : c
335

172. A single storeyed house was constructed in 1993, cost Rs. 10,00,000/-, What is the
value in the year 2000 by cost index method of CPWD? Index in 1993 - 244, Index in
2000 - 447, Base index is 100 for 1981 .

a) Rs. 10,00,000 x (447 - 244) b) Rs. 10,00,000 x (244 / 447)


c) Rs. 10,00,000 x (244 + 447) d) Rs. 10,00,000 x (447 / 244)

Ans : d

173. The average of 7 numbers 7, 9, 12, _, 5, 4, 11 is 9. The missing number is .................

a) 15 b) 16 c) 8 d) 13

Ans : a

174. Sale recorded at Registrar’s office is one of the sources from where ...............

a) Legal proof for market value of the property can be obtained.


b) Sale instances of own leasehold property can be collected.
c) Cost of property can be collected.
d) Price of only freehold property can be collected.

Ans : a

175. A valuer is honest because

a) it is not for the valuer’s interest b) it is not for the interest or client
c) it is for the interest of both d) it is for the interest of society

Ans : d

176. As per IBC 2016, who cannot initiate a fast track corporate insolvency resolution
process?

a) Financial creditor b) Operational creditor


c) Corporate debtor d) Insolvency professional

Ans : c

177. A gift to two or more donee of whom one does not accept, it is
336

a) Void as to interest which he would have taken had he accepted


b) Valid as to interest which he would have taken had he accepted
c) Voidable at option of donor
d) Valid at option of heir

Ans : a

178. Which one of the following statement is responsibility of valuer in case of value
dispute?

a) Prepare plan of property


b) Prepare machine drawings
c) Meet regulatory authorities for value discussions
d) Substantiate his valuation report by providing relevant supporting
evidence

Ans : d

179. Which of the following is not a physical characteristic of land?

a) Land is destructible b) Scarcity


c) Land has different soil strata d) Land is immobile

Ans : a

180. Which of the following legislation regards the building of hazards substance?

a) The water (cess) b) Factories Act (1978)


c) Water (P & C) d) Environment (Protection) 1986

Ans : d

181. Basic valuation principles underlying the direct comparison approach is the ...............

a) Principle of contribution b) Substitution


c) Conformity d) Standard

Ans : b

182. RERA occupancy certificate means


337

a) Preparing occupation of any b) Sanctioned plan


building plan, layout
c) Cancellation of entire project d) Completion of entire project

Ans : a

183. Omitted amendment 44th Act - 1978

a) Directives b) Right of property


c) Abolition of titles d) Abolition of enforceability

Ans : b

184. Court shall presume a fact the court .............. it is completed to take the fact as
provided, i.e., it shall have to presume the fact

a) can exercise its discretion b) cannot exercise its discretion


c) may exercise its discretion d) will exercise its discretion

Ans : c

185. Cost of acquisition is Rs. 8,000/-. Salvage value is Rs. 1,000/-. Life of the machine is
3 years. For WDV, what is the depreciation rate? 8,000
-50% 4,000
4,000
a) 50% b) 25% -50% 2,000
c) 66% d) 100% 2,000
-50% 1,000
Salvage value 1,000

Ans : a

186. An income yielding commercial property is to be valued by

a) Comparable method b) Cost approach method


c) Income capitalisation d) DCF method
method

Ans : d

187. Burden of proof lies upon a person who has to prove the fact and which
338

a) will shift b) never shift


c) may shift d) may (or) may not shift

Ans : b

188. Sec 58 Indian Easement Act the grant control render the property which is

a) Safe b) Unsafe
c) Danger d) Unsafe and danger

Ans : d

189. Section 54 of Easement Act the grant of licence may

a) Express only b) Implied only


c) Express and Implied d) Oral only

Ans : c

190. In which of the following cases, the court approved comparable sales method of
valuation for owner occupied portion of the building and rental method of valuation for
tenanted portion of the same building

a) V.C. Ramchandran v/s C.W.T. b) Wenger & co v/s D.V.O.


c) Jawajl Nagnathan v/s R.D.O. d) C.W.T. v/s P.N. Sikand

Ans : b

191. Which of the following is not nearer to money?

a) Exchange of bills b) Treeasury bill


c) Currency notes d) Bond

Ans : d

192. .................. refers to a situation where any person or a body corporate is unable to fulfil
its financial obligations (often occurring due to several factors such as a decrease in
cash flow, losses and other related issues).

a) Insolvency b) Bankruptcy
339

c) Liquidation d) Acquisition

Ans : a

193. Lessor has right to ................

a) Receive lease rent for the unexpired period of lease.


b) Receive profit rent from the property for the unexpired period of the lease.
c) Receive lease rent for the economic life of the building.
d) Receive lease rent for the physical life of the building.

Ans : a

194. Head lessee subleases the property. The rent he gets will be called as

a) Head rent b) Virtual rent


c) Standard rent d) Rack rent

Ans : d

195. .................... is not recognised model under sales comparision

a) Adjusted grid model b) Hedonic model


c) Reversible analysis model d) Weightage score system analysis

Ans : c

196. Valuer appearing in court on value is called

a) Ordinary witness b) Witness of opinion


c) Witness of fact d) Interested witness

Ans : b

197. Price elasticity of demand for any perfectly competitive fim’s output is

a) Less than 1 b) Equal to 0


c) Infinity d) 1

Ans : c
340

198. RERA is for

a) Residential building b) Commercial building


c) Residential & Commercial d) Godown

Ans : c

199. Forest (conservation) Act 1980 mainly provides for .................

a) Restriction on the dereservation of forests.


b) Restrictions on the development of wild life.
c) Conervation of any developmental activity like fencing, bridges, culverts,
dams.
d) Restriction on the entry of visitors.

Ans : a

200. Joint venture for redevelopment of property is applicable for ..................

a) Industrial b) Apartment
c) Cinema d) Hotel

Ans : b

201. Which is not a part of immovable property under transfer of property act?

a) Car b) Building embedded to earth


c) Standing timber, growing d) Jewellery
grass or crops

Ans : c

202. Value is an estimate of .................... as it ought to be

a) Price + Profit b) Price


c) Cost d) Offer amount

Ans : b

203. A lease of immovable property from year to year or exceeding one year, etc. can be
341

a) An ordinary instrument b) An unregistered instrument


c) A registered instrument d) Written instrument

Ans : c

204. Section 11 of Contract Act 1872 defines

a) An agreement b) Lawful consideration


c) Competency of parties d) Free consent

Ans : c

205. Contract of indeminity is

a) Tripartite b) Bilateral
c) Multipartite d) Multilateral

Ans : b

206. Which of the following lives of the buildings is used to determine depreciation of
building?

a) Economic life b) Physical life


c) Chronological life d) Residual life

Ans : a

207. Supply and demand is considered as ................. aspect affecting the value of the
property

a) Physical b) Social
c) Economic d) Legal

Ans : c

208. Lesee’s interest increases or decreases if the unexpired term of the lease increases?

a) Increases b) Decreases
c) No change d) None

Ans : a
342

209. Sale during time of Holi or any festival would come under which of the following?

a) Seasonal b) Cyclic
c) Regular d) None

Ans : a

210. The relationship between environmental deterioration and value

a) Inversely proportional b) Direct


c) both d) Sometimes

Ans : a

211. The net income was reported at Rs. 24,000/- and the property sold for Rs. 3,00,000.
What capitalisation rate applied to this sale?

a) 7% b) 8%
c) 9% d) 10%

Ans : b

212. In which case, the Rule of absolute liability was laid down by the Supreme Court?
24,000 x 100
Ans = 3,00,000
a) M.C. Mehta v. Union of India
b) Meneka Gandhi v. Union of India
c) Das Gupta v. Union of India
d) Sukla v. Union of India

Ans : a

213. The word ‘damages’ in law of torts means ......

a) Loss b) losses
c) Compensation d) none of the above

Ans : c

214. The exchange of letters, telex, any form of communication not denying the existence of
arbitration amounts to?
343

a) arbitration agreement b) arbitration award


c) only communication d) none of the above

Ans : a

215. Loss paid by the insurer to the insured is known as

a) The actual loss b) The accrued loss


c) Over all loss d) Indemnity

Ans : d

216. Peril based policy is

a) Automobile policy b) Marine Policy


c) Engineering policy d) Fire & Special perils Policy

Ans : d

217. Fire insurance Policy is issued to cover

a) To cover Market value of the property


b) To cover reinstated value of the property
c) To cover the escalated value of the property
d) To cover the three types said above with adequate additional premium

Ans : d

218. Taj Mahal had a great damaging impact due to pollution. Pin point the contaminant.

a) SO2 b) NO2
c) SPM d) None of the above

Ans : a

219. Acid rain is defined as

a) Rainfall which has pH less than 5.6


b) Rainfall which has pH less than 7
c) Rainfall which has pH more than 5.6
344

d) Rainfall which has pH more than 5.6

Ans : a

220. Which was the first country to impose a constitution obligation on the state and citizens
to protect and improve the environment as one of the primary duties?

a) India b) Japan
c) USA d) None of the above

Ans : a

221. The study of Supply and demand is aspect of

a) a Physical b) a Social
c) an Economic d) a legal

Ans : c

222. Value in Use is often referred to as

a) Objective value b) Subjective value


c) Exchange value d) Negotiated value

Ans : b

223. Value in Exchange is also known as

a) Cost b) Price
c) Transaction d) Interaction

Ans : b

224. Interest is defined as the possessed by an object.

a) Demand b) Price c) Value d) Legal rights

Ans : d

225. If the owner’s right which is imperfect is made perfect on fulfillment of certain conditions,
it is called as
345

a) Co-Ownership b) Concurrent Ownership


c) Duplicate Ownership d) Contingent Ownership

Ans : d

226. The exclusive right of a Flat owner over his own Flat along with rights of proportionate
share in common areas is called

a) Co-Ownership b) Concurrent Ownership


c) Duplicate Ownership d) Contingent Ownership

Ans : a

227. Years purchase .................. if number of years increases.

a) Increases b) Decreases c) remains same d) None

Ans : b

228. As the unexpired period of lease increases, the capital value after reversion goes on

a) Increasing b) Decreasing
c) Without changing d) None

Ans : b

229. Investment in Real Estate enjoy the benefits of

a) Perfect Competition b) Capital appreciation


d) Divisibility d) None

Ans : b

230. Whether gifted property attracts capital gain?

a) Yes b) No
c) Not applicable d) None of the above

Ans : b
346

231. The age of the building is 20 years. The life of the building is 40 years. The replacement
cost of the building as on 2018 is Rs. 5,000/-. The salvage value is Rs. 500/-. Using
straight line method, what is depreciated rate of construction?
5,000 - 500 20
1 x 40
a) Rs. 2,250/- b) Rs. 2,500/- 5,000 - 2,250
c) Rs. 2,750/- d) Rs. 3,000/- = 2,750

Ans : c

232. Sum insured (provisional) under the declaration policy is Rs. 1,00,00,000/-. Rate of
premium is Re. 1 per million. Average sum insured is Rs. 50,00,000/-. What is the
refund premium?

a) Rs. 5,000/- b) Rs. 6,000/-


c) Rs. 10,000/- d) Nil

Ans : a

233. Fire reinstatement value policy is not issued in respect of

a) Building c) Plant
c) Machinery d) Stocks

Ans : d

234. Turnover is Rs. 20 lakhs. Cost of production is Rs. 14 lakhs. Fixed overheads is
Rs. 4 lakhs. Net profit is Rs. 2 lakhs. Which is the correct sum insured for a
consequential loss (fire) policy?

a) Rs. 10,00,000/- b) Rs. 18,00,000/-


c) Rs. 2,00,000/- d) Rs. 6,00,000/-

Ans : d

235. Turnover is Rs. 10 lakhs. Cost of production is Rs. 7 lakhs. Fixed overheads is
Rs. 2 lakhs. What is the net profit for consequential loss (fire) policy?

a) Rs. 1,00,000/- b) Rs. 3,00,000/-


c) Rs. 5,00,000/- d) Rs. 8,00,000/-

Ans : a
347

236. Annual amount of gross profit is Rs. 1,20,000/-. What should be the sum insured
under consequential loss (fire) policy for an indemnity period of 24 months?

a) Rs. 60,000/- b) Rs. 90,000/-


c) Rs. 1,20,000/- d) Rs. 2,40,000/-

Ans : d

237. Which of the following is not a standing charge under consequential (fire) policy?

a) Salaries b) Office expenses


c) Cost of stationery d) Cost of raw materials

Ans : d

238. Annual gross profit is Rs. 1,20,000/-. What should be the sum insured under
consequential loss (fire) policy for a period of 9 months?

a) Rs. 60,000/- b) Rs. 90,000/-


c) Rs. 1,20,000/- d) Rs. 1,80,000/-

Ans : c

239. Under fire policy, the sum insured is Rs. 1 lakh. The loss is Rs. 60,000/-. Value of
the property at the time of proposal is Rs. 1,50,000/-. Value of the property at the
time of loss is Rs. 2,00,000/-. What is the amount of loss payable?

a) Rs. 40,000/- b) Rs. 30,000/-


c) Nil d) Rs. 60,000/-

Ans : b

240. In cash of Usufructuary mortgage the mortgagee is placed in possession and has a
right to enjoy the rent and profit

a) Until debt is paid b) Till contract is rescinded


c) Until debt is not paid d) Till contract is not rescinded

Ans : a

241. Which valuation methodology should be used to calculate the market value of vacant
building excluding land?

a) Sales comparable b) Depreciated replacement cost


348

c) Income capitalization d) Residue technique

Ans : b

242. After leasing of the property, the lessor’s right is sometimes called .................... right

a) diminishing b) occupational
c) residual d) possession

Ans : c

243. Return of unutilized land to the land owner after .................... years in LARAR act.

a) 2 years b) 3 years c) 4 years d) 5 Years

Ans : d

244. The property value is Rs. 1,00,000. Expected salvage value is Rs. 2,000 after 5 years.
What is the rate of depreciation?

a) 20 b) 19.60 c) 30 d) 15

1,00,000 - 2,000 98%


= = 19.6%
5 5

Ans : b

245. A mobile phone was purchased for Rs. 50,000/-. Its salvage value is Rs. 10,000/-.
Total life time use 60,000 hours. Used time 20,000. What is the depreciation of the
cell phone?

a) Rs. 12,000/- b) Rs. 15,000/- c) Rs. 18,000/- d) None of above

50,000 - 10,000 20,000


x = 13,333
1 60,000

Ans : d

246. Cost of acquisition is Rs. 8,000/-. Salvage value is Rs. 1,000/-. Life of the machine is
3 years. For WDV, what is the depreciation rate?
349

a) 50% b) 25% c) 66% d) 100%

@ 50% = 8,000 x 0.5 x 0.5 x 0.5 = 1,000 r n


A = P (1 - 100 )
@ 25% = 8,000 x 0.75 x 0.75 x 0.75 = 3,375
@ 66% = 8,000 x 0.34 x 0.34 x 0.34 = 314
@ 100% = 8,000 x 0 x 0 x 0 = 0

Ans : a

247. The net income was reported at Rs. 24,000/- and the property sold for Rs. 3,00,000.
What capitalisation rate is applied to this sale?

a) 7% b) 8% c) 9% d) 10%

24,000
x 100 = 8%
3,00,000

Ans : b

248. The age of the building is 20 years. The life of the building is 40 years. The replacement
cost of the building as on 2018 is Rs. 5,000/-. The salvage value is Rs. 500/-. Using
straight line method, what is depreciated rate?

a) Rs. 2,250/- b) Rs. 2,500/- c) Rs. 2,750/- d) Rs. 3,000/-

Depreciation rate = 5,000 - 500 20


x = Rs. 2,250 ;
1 40
Depreciated rate = 5,000 - 2,250 = Rs. 2,750

Ans : c

249. A person seeks an income of Rs. 1,000 per annum from an investment. He wishes
this to be an 8% return on his investment. What is the amount he has to invest?

a) Rs. 1,000/- b) Rs. 80/- c) Rs. 12,500/- d) Rs. 10,000/-

1,000
x 100 = Rs. 12,500
8

Ans : c
350

250. What would be the written down value of a machine purchased at the cost of
Rs. 30,000/- after 3 years of service life at 5% rate of depreciation?

a) Rs. 26,720/- b) Rs. 25,720/- c) Rs. 27,720/- d) Rs. 28,720/-

30,000 x 0.95 x 0.95 x 0.95 = Rs. 25,721 or 25,720

Ans : b

251. A machine was purchased 2 years back at a cost of Rs. 4,00,000/-. Total life is
20 years. Salvage value = 10%. What is the depreciated present value after 2 years)?

a) Rs. 3,74,000/- b) Rs. 3,54,000/- c) Rs. 3,64,000/- d) Rs. 3,44,000/-

2
Depreciation percentage = x 90 = 9%
20
Depreciated value = 0.91 x 4,00,000 = Rs. 3,64,000/-

Ans : c

252. Workout N.P.V. of a building having 20 years of age and 60 years of total life. Its
replacement cost as on today is Rs. 4,30,000/-. Salvage value 10%. Adopt SLM

a) Rs. 3,11,000/- b) Rs. 4,11,000/- c) Rs. 3,01,000/- d) Rs. 4,01,000/-

20
Depreciation percentage = x 90 = 30%
60
Net Present Value = 0.7 x 4,30,000 = Rs. 3,01,000/-

Ans : c

253. What is the N.P.V. by constant percentage method (linear method). Replacement cost
is Rs. 3,50,000/-. Life : 75 years. Age : 15 years.

a) Rs. 3,86,300/- b) Rs. 4,86,300/- c) Rs. 1,86,300/- d) Rs. 2,86,300/-

100
Depreciation rate %age = = 1.33 r n
75 A = P (1 - 100 )
15
1.33
A = 3,50,000 1 -
100
351

= 3,50,000 x 0.818 = Rs. 2,86,300

Ans : d

254. A single storeyed house was constructed in 1993, cost Rs. 10,00,000/-, What is the
value in the year 2000 by cost index method of CPWD? Index in 1993 - 244, Index in
2000 - 447, Base index is 100 for 1981 .

a) Rs. 10,00,000 x (447 - 244) b) Rs. 10,00,000 x (244 / 447)


c) Rs. 10,00,000 x (244 + 447) d) Rs. 10,00,000 x (447 / 244)

C.I.I. 2000 447


10,00,000 x = 10,00,000 x
C.I.I. 1993 244

Ans : d

255. A machine was purchased for Rs. 1,00,000/- @ 15% depreciation of SLM. What is
the written down value after 2 years?

a) Rs. 70,000/- b) Rs. 80,000/- c) Rs. 60,000/- d) Rs. 90,000/-

1,00,000 - (2 x 15,000) = Rs. 70,000

Ans : a

256. A property has a net income of Rs. 30,000/-. One appraiser decides to use a
12 percent capitalisation rate, while a second appraiser uses a 10 percent rate. What
is the difference in appraisal value of the two valuers?

a) Rs. 50,000/- b) Rs. 60,000/- c) Rs. 40,000/- d) Rs. 70,000/-

100
30,000 x = 2,50,000 3,00,000 - 2,50,000 = Rs. 50,000
12
100
30,000 x = 3,00,000
10

Ans : a

257. Lessee receives an income of Rs. 30,000 per annum. He pays Rs. 16,000/- rent to
landlord. If the lessee receives a rent of 8% return, how much the landlord will expect
his return.
352

a) 0.09 b) 0.07 c) 0.01 d) 0.10

Lessee = 8%
Lessor = 7%
(Note : Other details are not relevant)

Ans : b

258. Total age of this building is 4 years. After four years, the depreciated value is equal to
24% of the cost. Find out the percentage of depreciation (near to answer) by WDV
method.

a) 24 b) 25 c) 30 d) 35

@ 24% = 100 x 0.76 x 0.76 x 0.76 x 0.76 = 33.36


@ 25% = 100 x 0.75 x 0.75 x 0.75 x 0.75 = 31.64
@ 30% = 100 x 0.70 x 0.70 x 0.70 x 0.70 = 24.01
@ 35% = 100 x 0.65 x 0.65 x 0.65 x 0.65 = 17.85

Ans : c

259. A machine was purchased of Rs. 18,000/- before 2 years. It is sold for Rs. 16,000/-
considering 10% depreciation (of Rs. 18,000/-) per annum. The machinery was sold
for

a) 2,000 less b) 1,600 less


c) 1,600 profit d) No loss and no gain

18,000 - (1,800 x 2) = Rs. 14,400


16,000 - 14,400 = Rs. 1,600 Profit

Ans : c

260. Factory building has 1200 S.M. built-up area. Plot area is 2000 S.M. Building is
25 years old and total life is 50 years. Replacement cost today is Rs. 25000 per S.M.
Industrial plot is available for Rs. 8000 per S.M. Which of the following is the fair sale
price (ignoring savage) for the property?

a) Rs. 4,60,00,000 b) Rs. 3,10,00,000


c) Rs. 1,60,00,000 d) Rs. 1,84,00,000
353

2,000 x 8,000 = Rs. 1,60,00,000

25
1,200 x 25,000 x = Rs. 1,50,00,000
50

= Rs. 3,10,00,000

Ans : b

261. The W.D.V of an asset after three years of depreciation on the reducing balance method
@ 10 percent p.a. is Rs. 36,450/-. What was its original value?

a) Rs. 40,000/- b) Rs. 50,000/- c) Rs. 45,000/- d) Rs. 70,250/-

36,450
= Rs. 50,000
0.9 x 0.9 x 0.9

Ans : b

262. While doing valuation for a leasehold property, this is not necessary to be seen.

a) Lease deed b) Ground rent to be paid to lessor


c) Net income for the lessee d) Age of the lessor & the lessee

Ans : d

263. When calculating the depreciation by linear method, this aspect is not considered.

a) Replacement value b) Age of the building


c) Rate of depreciation d) Salvage value

Ans : d

264. While doing valuation of a commercial building by rent capitalisation method (income
approach), this aspect is not considered.

a) Rent collected b) Outgoes


c) Rate of return d) Age of the building

Ans : d
354

265. While doing valuation of a petrol bunk, this aspect is not considered.

a) Ground rent received form the petroleum dealer


b) Electricity charges paid to EB
c) Property tax paid to the corporation
d) Capacity of the tank

Ans : d

266. While determining the joint venture ratio for the development of an apartment building,
this is not considered.

a) Prevailing market rate of land


b) Construction cost of materials
c) Floor Space Index
d) Promoter’s profit

Ans : d

267. While executing a WILL, this is not necessary.

a) Signature of the person who writes a WILL


b) Signature of the witnesses
c) Date of the WILL
d) Registration of the WILL in the registration department

Ans : d

268. While determining the adequacy of the sum insured for the purpose of paying the
claim to the insurer, this aspect is not generally considered.

a) Age of the building b) Replacement rate


c) Life of the building d) Salvage value of the building

Ans : d

269. As per the SARFAESI Act, this property is not considered as a security to the bank.

a) Residential flat b) Commercial building


c) Residential home d) Agricultural land

Ans : d
355

270. Which is not considered as a marketable property?

a) Restaurant b) Hospital building


c) A cinema theatre d) A temple property

Ans : d

271. While calculating the depreciation of a 40 years RCC roofed load bearing structure,
this aspect is not seen.

a) Replacement rate of construction b) Age of the building


b) Economic life of the building d) Physical life of the building

Ans : d

272. Which of the following is not a duty and function of the valuer?

a) To verify ownership in document and occupancy on actual site


b) To identify plot number in document as well as on site
c) To be responsible for correctness of survey findings by his assistant
d) To investigate title to the property to be valued

Ans : c

273. While valuing assets in which the valuer is not conversant with its features ....................

a) It is advisable to engage the services of an expert and his report need


not be made a part of valuation report
b) It is advisable to engage the services of an expert and the signed report
of the expert be made a part of valuation report
c) He can privately seek the services of such expert and need not disclose
in the report
d) It is enough to mention in report about details of expert engaged.

Ans : b

274. Which of the following details are not required to be included in the valuation report for
present day value of an open plot of land?

a) Name of all the owners of the property and with details of share of each
coowner
356

b) Available comparable sale instances in the locality during relevant period


of valuation
c) Details of year of purchase of land and price paid while acquisition.
d) Analysis of data collection by giving details of weight ages adopted for
different factors and adjustment made in sale rate.

Ans : c

275. Which one of the following actions should be done by valuer if he has knowledge of
any prior valuer having been appointed before accepting the assignment?

a) Communicate in writing with prior valuer.


b) Reject the valuation assignment.
c) Carry out the valuation assignment and arrive at conclusions which are
within range of reasonableness of conclusions arrived by prior valuer.
d) Carry out the valuation assignment and arrive at independent conclusions
without knowledge of a prior valuer.

Ans : d

276. Which one of the following statement is the responsibility of valuer in case of value
dispute?

a) Prepare plan of property.


b) Prepare machine drawings.
c) Meet regulatory authorities for value discussions.
d) Substantiate his valuation report by providing relevant supporting
evidence.

Ans : d

277. Valuer should endeavour to ensure that he provides true and adequate information,
because

a) This is not required under any law


b) This makes a sensible valuation report
c) This misleads client
d) This misleads public

Ans : b
357

278. Statement that valuer does not have interest in property valued appears in one of
following section of report -

a) Declaration b) Conclusion
c) Assumptions and limiting conditions d) Summary

Ans : a

279. Opinion expressed on value for the property by the valuer as an expert witness in a
valuation dispute case is .................... to the court.

a) Binding b) Not binding


c) Of no value d) In form of advice

Ans : b

280. Acceptance of valuation report submitted by valuer in the court does not depend of
....................

a) Reasons given by the valuer in the valuation report


b) The valuer successfully withstanding the test of cross examination.
c) Age of the valuer
d) Substantiating the report by producing supporting instances as proof.

Ans : c

281. As an independent valuer, the valuer should not charge .................... fee.

a) Professional b) Success
c) Mandate d) Legal

Ans : b

282. A valuer should not use or divulge to other clients or any other party any confidential
information about the .................... company.

a) Subject b) Client
c) Public d) Listed

Ans : a
358

283. In the case of ...................., either outflow of resources to settle the obligation is not
probable or the amount expected to be paid to settle the liability cannot be measured
with sufficient reliability.

a) Liability b) Provision
c) Contingent liabilities d) Contingent assets

Ans : c

284. .................... refers to a situation where any person or a body corporate is unable to
fulfil its financial obligations (often occurring due to several factors such as a decrease
in cash flow, losses and other issues).

a) Insolvency b) Bankruptcy
c) Liquidation d) Acquisition

Ans : a

285. Deflation is ....................

a) Deficit budget
b) Reduction in taxation
c) Contraction in volume of money or credit that results in a decline of price
level
d) Increase in public expenditure

Ans : c

286. In case of a female intestate dying without issue but leaving her husband ....................

a) A husband will take all property


b) A husband will not take any of the property
c) A husband will take all property except property inherited from her father
or mother
d) A husband will only take 25 per cent of her property

Ans : c

287. Under sec 2(n) of RERA ‘common areas’ mean ....................


359

a) Open parking areas b) Terraces


c) Staircase d) Open parking space, terraces, and
staircase

Ans : d

288. Question of onus of proof when both parties lead evidence

a) TRUE b) FALSE
c) Will vary d) Need not be

Ans : c

289. X transfers Rs.500 to Y on condition that he shall execute a certain lease within three
months after V’s death, and, if he should neglect to do so to Z, if Y dies in X’s life time,

a) The disposition shall not take effect in favour of Z


b) The disposition in favour of Z takes effect
c) The disposition requires further conditions
d) The disposition in favour of X takes effect

Ans : b

290. Premature termination of lease is major .................... factor affecting the value of
leasehold property,

a) Physical b) Legal
c) Social d) Economic

Ans : d

291. A monopoly market will usually generate : ....................

a) Higher prices and lower output


b) Higher prices and higher output
c) Lower prices and lower output
d) Lower prices and higher output

Ans : b
360

292. Cost prior to remediation specifically when no option is available except waiting for
the sanction from the authority and keeping the part of the property under non-use, is
said to be ....................

a) Cost of utility b) Cost of disruption


c) Cost of control d) Cost to public liability

Ans : b

293. Which of the following represents getting back of capital invested in a property

a) Net income generation b) Gross income generation


c) Redemption of capital d) Accumulation of capital

Ans : c

294. Hypothetical building scheme is normally used for estimating value of ....................

a) Large size open land


b) Property with underutilized FSI on the plot
c) Industrial structure
d) Residential flats

Ans : b

295. According to NPV method projects that would be acceptable must have

a) Zero net present value b) Negative net present value


c) Positive net present value d) Internal rate of return in positive

Ans : b

296. Which one of the following is the largest employer in India?

a) Primary sector b) Secondary sector


c) Tertiary sector d) IT sector

Ans : d

297. Which of the following is recreational use of land under zoning regulation?
361

a) Playgrounds b) Parks in residential society


c) Flower gardens and fruit farms d) Hotels

Ans : b

298. Project’s expected monetary loss or gain by discounting all cash outflows, using
required rate of return is classified as?

a) Net present value b) Internal rate of return


c) Net discounted value d) Net future value

Ans : b

299. Which of the following factor is not relevant to the value estimation by considering the
highest and best use of property?

a) Reasonably probable future use b) Physically possible use


c) Financially feasible use d) Present inferior user of
property

Ans : d

300. Under sec 60 of Indian succession act, who can appoint a guardian or guardians for
his child during minority?

a) Father b) Mother
c) Uncle d) Friends

Ans : a

301. Hypothetically plotting scheme is mainly used to value ....................

a) Large open land b) Flats


c) Industries d) Residence

Ans : a

302. Depreciation fund/sinking fund method is designed to :

a) Only provide for depreciation of an asset


362

b) Provide for depreciation & also to accumulate the amount for its
replacement
c) Provide for the payment of some liability
d) To accumulate the amount of its replacement

Ans : b

303. Market value of an object depends upon the future ....................that can be derived out
of it.

a) Benefits b) Demand
c) Supply d) Loan

Ans : a

304. Under profit method of valuation income is generated from ....................

a) Only tangible assets b) Both tangible and intangible assets


c) Only intangible assets d) Only movable assets

Ans : b

305. These are kinds of rent except ....................

a) Differential rent b) Scarcity rent


c) Mobility rent d) Location rent

Ans : c

306. Inflation ....................

a) Always reduces the cost of living


b) Always reduces the standard of living
c) Reduces the price of products
d) Reduces the purchasing power of a rupee

Ans : d

307. Induce investment .................... with increase in income and .................... with decrease
in income
363

a) Decrease, increase b) Increase, decrease


c) Increase, remain same d) Decrease, remain same

Ans : b

308. Under discounted cash flow method, value of the property increase with ....................

a) Higher property tax b) Higher capitalization rate


c) Lower discount rate d) Reduced income flow

Ans : c

309. What is public sector?

a) Sector run by people b) Sector run by government


c) Sector run by co operation d) International sector

Ans : b

310. What do you mean by the value of an asset to a particular owner or the prospective
owner for individual investment or operational objectives?

a) Synergic value b) Equitable value


c) Investment value d) Market value

Ans : c

311. The three broad types of productive resources are :

a) Money profit and interest


b) Capital, labour and natural resources
c) Bond, stock shares and deposits
d) Technology, human capital and markets

Ans : b

312. Which of the following is considered as a special property

a) Residential land b) Industry


c) Commercial office space d) Sports Arena

Ans : d
364

313. The internal rate of return (IRR) is ....................

a) The amount which Re 1 invested today will become after some time at
a certain rate of interest
b) The actual return obtain from an investment
c) So calculated that future discounted receipts and discounted payments
always show profit
d) A rate fixed by the reserve bank of India above which lending cannot be
done by any bank

Ans : b

314. Which of the following is not relevant while estimating market value of property with
improvements?

a) Continuation of existing use


b) Cost of repair carried out few years back
c) Adaptation to another use
d) Demolition and redevelopment

Ans : b

315. Sec 60 under transfer of act, affirms a right of redemption

a) All mortgage b) In some particular mortgage


c) Not in a single mortgage d) In some pledge

Ans : b

316. Which of the following is not physical factor and affects the value of the property?

a) Annual maintenance b) Gross domestic products


c) Property location d) Neighbourhood properties

Ans : b

317. Economic obsolescence can be explained as ....................

a) Physical determination
b) Loss in value due to external factors
365

c) A loss in value due to poor design


d) Repairable damage

Ans : b

318. Which of the following project should an investor choose on the basis of Net present
value results?

a) Zero is the NPV of project


b) INR (-) 1000 Million is the NPV of project
c) INR (+) 2000 Million is the NPV of Project
d) INR (+) 2200 Million is the NPV of project

Ans : b

319. After the institution of a suit for partition by a member of the joint family

a) Subsequent birth in the family diminish his share


b) Subsequent death in the family augment his share
c) Subsequent birth and death in the family does not affect share
d) Subsequent renunciation of the family diminish his share

Ans : c

320. The value generated by the income is the characteristic of .................... property

a) Marketable investment property


b) Service property
c) Marketable non-investment property
d) Non marketable non-investment property

Ans : b

321. Saving function explain the relationship between .................... and ....................

a) Saving, investment b) Saving, consumption


c) Saving, income d) Investment, consumption

Ans : c
366

322. A widow who is willed the use of family home for the rest of her natural life, with provision
that title shall pass to children upon her death holds.

a) Easement interest b) Life interest


c) Freehold right d) Leasehold right

Ans : b

323. When the prices of plant and machinery are to be compared, we compute ....................

a) Value index b) Price index


c) Volume index d) Aggregative index

Ans : b

324. Sec 70 under transfer of property act, the natural accession is addition to the security
and becoming incorporated it are subject to ....................

a) Redemption b) Not redemption


c) Accession d) Accretion

Ans : c

325. Interest is paid because ....................

a) Capital is scarce b) Capital is productive


c) Capital is attractive d) Capital is surplus

Ans : b

326. In the notes of fixed assets of a company, closing WDV + Depreciation for the year is
equal to

a) Opening Gross Block b) Opening WDV


c) Closing WDV d) Closing Gross Block

Ans : d

327. In Indian Politics which one is Supreme?


367

a) The Parliament b) Religion


c) The Supreme Court d) The Constitution

Ans : d

328. Why a valuer, or his relative should not accept gifts or hospitality which undermines or
affects his independence as a valuer?

a) Avoid risk of influence on valuation conclusions


b) This is required as per valuation standards
c) This is required as per guidelines
d) This is in public interest

Ans : b

329. A flat was valued by ‘A’ at Rs. 21 Lakhs and then purchaser ‘B’ purchased from seller
‘C’ said flat for Rs. 20 Lakhs with the help of broker ‘D’. In a court case about correct
sale value, which of the following is not called a ‘witness of fact’?

a) Purchaser ‘B’ b) Seller ‘C’


c) Broker ‘D’ d) Valuer ‘A’

Ans : d

330. A building is erected on a leasehold land which has 30 years lease period. The valuer
adopting life of an RCC building on the plot at 30 years is called

a) Life due to legal constrains b) Obsolescence life


c) Physical life d) Economic life

Ans : a

331. Infrastructure works like augmentation of road networks & transport facilities will result
in increased ....................

a) Environmental deterioration b) Overall developments


c) Crime rate d) Withdrawal of capital

Ans : b
368

332. Which are two value ingredients of a marketable non-investment property?

a) Liquidity and marketability b) Liquidity and utility


c) Utility and Marketability d) Non marketability and Liquidity

Ans : c

333. Value of a property offered for immediate sale by its owner for urgent need of money
to meet with legal obligation is called ....................

a) Going concern value b) Distress value


c) Forced sale value d) Breakup value

Ans : b

334. Upon paying the amount of loss to the insured, the insurer steps in to the place of the
insured, taking over all his rights is called exercise of

a) Right of reinstatement b) Right of contribution


c) Right of subrogation d) Right to salvage

Ans : c

335. Under LARAR Act 2013 market value of acquired land is derived from ....................

a) Circle rates
b) Higher of the two: Average Rates under sale instances and Rates fixed
for stamp duty
c) Tate determined by the collector
d) The rate at which the land was initially purchased

Ans : b

336. Sale recorded at Registrar’s office is one of the sources from where ....................

a) Legal proof for market value of the property can be obtained


b) Sale instances of only leasehold properties can be collected
c) Cost of property can be collected
d) Price of only freehold properties can be collected

Ans : a
369

337. Which of the following statements best describes the ‘carrying value’ of an asset?

a) The cost of the asset less its residual value


b) Amount at which the asset is recognized in the balance sheet after
deduction any accumulated depreciation and accumulated impairment
losses
c) Net value of asset or value carried in books of account, whichever is
higher
d) The higher of the asset’s value in use and its recoverable amount.

Ans : b

338. Depreciated replacement cost of building to the new owner is called .................... to
him

a) Price b) Historic cost


c) Original cost d) Value

Ans : c

339. Sec 112 under transfer of property act for waiver of forfeiture but where rent is accepted
after the institution of a suit to eject the lessee on the ground of forfeiture, such
acceptance ....................

a) Is a forfeiture b) Is not forfeiture


c) Determination of lease d) Transfer of lease

Ans : b

340. Under RERA, at what stage does promoter can advertise his project?

a) After project is registered


b) Before project is registered
c) Either before & after project is registered
d) Neither before & after project is registered

Ans : a

341. Which of the following statement is true related to obligation of insured on notification
of a claim
370

a) To defend the insurer


b) To indemnify the insurer
c) Ensure third parties named in policy are notified
d) To investigate / settle potentially covered claim

Ans : b

342. Agreement without consideration is declared ....................

a) Void b) Valid
c) Free consent d) Enforceable

Ans : a

343. Inflation brings most benefits to which one of the following?

a) Government pensioners b) Creditors


c) Saving bank account holders d) Debtors

Ans : b

344. Cost approach is useful mainly for valuing .....................

a) Marketable properties
b) Non-marketable non-investment property
c) Properties which are ready for redevelopment
d) Going concern business

Ans : b

345. All the property value is created by the .................... of the future benefits the property
will provide.

a) Competition b) Contribution
c) Substitution d) Anticipation

Ans : b

346. Value is an estimate of .................... as it ought to be


371

a) Price + Profit b) Price


c) Cost d) Offer amount

Ans : b

347. Capital amount paid once in order to receive an annuity of Rs. 1 for a specified period
of time at the specified rate of interest is called ....................

a) Net amount b) Capital


c) Years purchase d) Capitalized value

Ans : d

348. Investment in real estate enjoys the benefits of ....................

a) Perfect competition b) Capital appreciation


c) Divisibility of holding d) Easy liquidity

Ans : b

349. As per RERA Act under sec 10 (3) if Government acquires multi-crop irrigated land
than ....................

a) Less value amount of land


b) No cultivable wasteland shall be developed for agricultural purposes
c) A less area o cultivable wasteland shall be developed for agricultural
purposes
d) Equivalent area of cultivable wasteland shall be developed for agricultural
purposes

Ans : d

350. The factor which is not to be considered when applying the residual method of valuation
is ....................

a) Anticipated price by sale of the finished property


b) Cost of development project
c) Profit motive of the developer
d) Social benefit of the project

Ans : d
372

351. Estimate of an amount for auction sale is known as .....................

a) Liquidation value b) Salvage value


c) Net present value d) Replacement value

Ans : a

352. Inadequate infrastructure in the locality results in .................... market value of property

a) Higher b) Lower
c) Stagnant d) 50 percent higher

Ans : b

353. No person shall construct any building with its plinth

a) Less than 40 C.Mts. b) Less than 45 C.Mts


c) Less than 55 C.Mts d) Less than 35 C.Mts

Ans : c

354. An ownership flat ‘A’ with 80 S.M. Area is in building in by lane. Similar flat ‘B’ in similar
building but located on main road is recently sold at Rs. 95,000 per S.M. If weightage
for location is considered at 20 percent, which of the following will be the fair sake
price of flat ‘A’?

a) Rs. 76,00,000 b) Rs. 91,20,000


c) Rs. 60,80,000 d) Rs. 50,00,000

Ans : c

355. As per IBC, 2016, an application against the decision of the liquidator rejecting the
claim of a creditor may be made to

a) The Insolvency and Bankruptcy Board of India


b) The National Company Law Tribunal
c) The committee of creditors
d) The Debt Recovery Tribunal

Ans : b
373

356. Which of the following economic activity is not in the teritary sector?

a) Banking b) Bee-keeping
c) Teaching d) Working in a call centre

Ans : b

357. In which of the following methods of depreciation, the depreciated replacement cost
of the building is directly obtained from the formula itself.

a) Written down value method b) Straight line method


c) Constant percentage method d) Sinking fund method

Ans : c

358. Which of the following denotes the dividend declared by the directors between two
annual general meetings?

a) Proposed dividend b) Final dividend


c) Interim dividend d) Unpaid dividend

Ans : c

359. A property fetches a leaseholder Rs. 40,000 per annum. The rent fixed to the paid to
the superior landlord is Rs. 17,000 per annum. If freeholder expects a return of 8 percent,
then the leaseholder should expect a rate as indicated below so that he makes a
reasonable profit.

a) 0.09 b) 0.07 c) 1.00 d) 0.01

Ans : a

360. Value of a property depends upon the future .................... that can be derived out of it.

a) Benefits b) Demand
c) Supply d) Cost

Ans : a

361. When there are only few sellers of the commodity, the market is called
374

a) Monopoly b) Duopoly
c) Oligopoly d) Monopsony

Ans : c

362. Which of the following investment is having constrains of liquidity?

a) Long term Govt. Securities b) Immovable properties


c) Shares & stocks d) Bank FDs

Ans : b

363. As per sec 60A under transfer of property act, Where a mortgagor is entitled to
redemption he may require the mortgage, instead of re-transferring the property, to
assign the mortgage debt and transfer the mortgaged property to such third person
as the mortgagor may direct than the mortgagee.

a) Shall not be bound to assign and transfer accordingly.


b) Shall be bound to assign and transfer accordingly.
c) Is bound if the mortgagor assents
d) Shall not be bound to assign

Ans : b

364. Supply and demand is considered as .................... aspect affecting the value of the
property.

a) Physical b) Social
c) Economic d) Legal

Ans : c

365. Whenever there us a provision to the effect ‘that the court shall presume a fact’ the
court ..................... It is compelled to take the fact as provided, i.e., it shall have to
presume the fact.

a) Can exercise its discretion b) Cannot exercise its discretion


c) May exercise its discretion d) Will exercise its discretion

Ans : c
375

366. Which of the following mathematical formula is used to find out Years Purchase for
annuity receivable in perpetuity?

a) 100/Rate of interest b) 1 / {1-(100/rate of interest) ^n}


c) ((100/rate of interest) ^n)-1 d) {((100/rate of interest) ^n)-1}/rate of
interest

Ans : a

367. .................... is a situation whereby a court of competent jurisdiction has declared a


person of other entity insolvent, having passed appropriate orders to resolve it and
protect the right the creditors

a) Bankruptcy b) Insolvency
c) Liquidation d) Acquisition

Ans : a

368. Which of the following investment finds hedge against inflation

a) Long term Govt securities b) Bank FD’s


c) Shares & stocks d) Immovable properties

Ans : d

369. The plots which is connected to main road through a passage is called ....................

a) Narrow plots b) Rectangular plots


c) Ribbon plots d) Tandem plots

Ans : d

370. Who is not a special purchaser of property among the following?

a) Foreign purchaser b) Relative of the seller


c) Owner of the adjoining plot d) Director of the company

Ans : a

371. Who can’t be registered as a valuer?


376

a) Company b) Individual
c) Partnership d) Society

Ans : d

372. Money is what Money does – who said it?

a) Smith b) Marshall
c) F.A.Walker d) Robbinson

Ans : c

373. The real GDP is measured in .................... prices & nominal GDP is measured in
.................... prices

a) Base year & Current year b) Current year


c) Base year d) Current Year & Base year

Ans : a

374. Which is the safest and earliest liquidity

a) 2% of stock market b) 10% of Return of Real estate


investments
c) 7% of bank FD d) 30% of promoter’s loan

Ans : c

375. Under transfer of property act, the immovable goods does not include :

a) Standing timber or grass.


b) Standing timber, jewellery and crop,
c) Standing timber, growing grass or crops.
d) Only grass.

Ans : c

376. When the Mortgager right to redeem accrues, the Mortgagee has a right to enforce
....................
377

a) His Security b) His interest


c) His cash d) His debt

Ans : d

377. In 99 year perpetuity, lease is to be extended for another 99 years

a) Details of age of lessor and lessee


b) Improvements and extra FSI used
c) Date of the year of commencement
d) Date of start first year

Ans : c

378. Section 54 of Easement act the grant of License may ....................

a) Express only b) Implied only


c) Express or Implied d) Oral only

Ans : c

379. Which of the following can be transferred under the Transfer of Property Act?

a) Easement along with the dominant heritage


b) Political pension
c) Succession
d) Stipends of Government civil pensioners

Ans : a

380. ‘A’ leases land to ‘B’ on condition that he shall walk a hundred miles in an hour. The
lease is ....................

a) Valid b) Void
c) Voidable d) Illegal

Ans : b

381. Which of following statement is true regarding Hindu succession (amendment) Act
2005.
378

a) Women can now act as Karta of joint Hindu family.


b) Women cannot act as Karta of joint Hindu family before or after passing
of the Hindu
c) Succession (amendment) Act 2005.
d) Women cannot become Karta of joint Hindu family.

Ans : a

382. When the result of a combination of two or more assets or interests where the combined
value is more than the sum of the separate values is known as -

a) Real investment value b) Synergistic value


c) Liquidation value d) Hope value

Ans : b

383. Onus of proof is a .................... in the evaluation of evidence

a) Fact b) Principle
c) Continuous process d) One time process

Ans : c

384. Which of the following is not the surrender of lease?

a) Premature termination of a lease.


b) Unilaterally terminated by a lessee
c) Terminated with the consent of the lessor
d) If it happens post expiry of the lease

Ans : d

385. Under which of the valuation approach, future cash flows from property will yield to
capitalized value of property?

a) Market approach b) Income approach


c) Cost approach d) Depreciated Replacement Cost
Method

Ans : b
379

386. Under Sec 58 of Indian Easements Act the grantor cannot render the property which is

a) Safe b) Unsafe
c) Dangerous d) Unsafe & dangerous

Ans : d

387. Annual Sinking fund to be set aside each year for recouping Rs 1 at the end of
6 years, at 5 percent rate of interest is represented by formula ....................

a) 0.05/ ((1+ 0.05)6-1} b) {(1+0.05)6+1} / 0.05


c) 5 / (1+ 5)6-1) d) 0.05 / {(1+0.05)6+1}

Ans : a

388. The important criteria for the selection of the best land in the town is ....................

a) More width of road b) Industrial zone


c) Prime Location and High FSI d) Residential Zone

Ans : c

389. Which of the following method of valuation is normally adopted to estimate value of
land in the locality where there are absolutely no instances of land sales available.

a) Profit method b) Sale comparison method


c) Land & building method d) Residue Technique or Development
method

Ans : d

390. Which of the following is not a natural attribute of a land, but artificially created by
man?

a) Benefits b) Location
c) Tenure d) Frontage

Ans : c

391. Costs due to extra supervisory personnel, additional testing, careful monitoring as
well as security is known as ....................
380

a) Cost to operation b) Cost to control


c) Cost to disruption d) Cost of construction

Ans : a

392. Which of the following distinguish Investment in Real estate vis-à-vis other forms of
investment.

a) Capital erosion b) Redemption of capital


c) Income generation d) Solvency

Ans : b

393. Which out of the following is not a factor that affects demand & supply or real estate?

a) Economic growth b) Interest rate


c) Increased population d) Number of cars in Household

Ans : d

394. Under sec 61 of Indian succession Act, A will or any part of a will, which has been
caused by fraud and which takes away the free agency of the testator, is ....................

a) Illegal b) Valid
c) Voidable d) Void

Ans : d

395. The building cost index is worked out by the CPWD on the basis of ....................

a) Estimates of building material and labour for the subsequent year as


estimated by CPWD
b) Rates of building materials and labour in Delhi vis-a-vis rates in city
in relevant state
c) Expected inflation rate as estimated by the finance ministry, Govt. of
India.
d) An index obtained by estimating demand/ supply of housing determined
by the Bank of India Reserve

Ans : b
381

396. Utility means existing and anticipated .................... due to ownership & use of property

a) Marketability b) Scarcity
c) Transferability d) Benefits

Ans : d

397. Which property does the corporation has a power to acquire

a) Movable
b) Immovable property
c) Both movable & immovable property
d) Neither movable & immovable property

Ans : a

398. The value of the rent Controlled properties will have .................... value than vacant
property.

a) 100 percent higher b) 50 percent higher


c) Exactly same d) Lower

Ans : d

399. In case of gift, if donee dies before acceptance, then,

a) Gift is valid b) Gift is void


c) Gift is reduced d) Court has to decide

Ans : b

400. In case of .................... of contract, the motive (intention) for the breach is immaterial.

a) Breach b) Failure
c) Recession d) Cancellation

Ans : b

401. Under sec 59 of Indian Succession Act, A person who is ordinarily insane may make
a will
382

a) Never b) Every time during insane


c) During interval when he d) His heir
is of sound mind

Ans : c

402. Gift to two or more donees, of whom one does not accept it is ....................

a) Valid as to interest which he would have taken had he accepted


b) Void as to interest which he would have taken had he accepted
c) Voidable at option of donor
d) Valid at option of heir.

Ans : b

403. The mortgager in Indian law who had parted with some right of ownership and the right
of redemption is a right which he exercises by virtue of his ....................

a) Pecuniary ownership b) Residuary ownership


c) Conditional ownership d) Possession

Ans : b

404. Which valuation methodology should be used to calculate the market value of vacant
building excluding land?

a) Sales comparable b) Depreciated replacement cost.


c) Income capitalization d) Residue Technique

Ans : b

405. Under adjustment grid model, for sale instance premises having .................... positive
weightage is considered on sale rate, while comparing with subject premises having
standard specification?

a) Standard specification b) Superior specification


c) Substandard specification d) Deluxe speculation

Ans : d
383

406. Accumulative rate of interest is considered at .................... rate than remunerative rate
because higher security for accumulation of capital is needed.

a) Lower b) Higher
c) Same d) Double

Ans : a

407. Which of the following represents the year purchase for Rs.1 with remunerative rate of
interest at 8% and annual sinking fund amount to be set aside for recouping Rs .1 is
0.021.

a) 1 / (0.08+0.021) b) 0.021/0.08
c) (0.08+0.021) / (0.021) d) 1/(0.08-0.021)

Ans : a

408. Which one of the following is not essential element of the basic concepts of valuation
of immovable property?

a) Scarcity b) Transferability
c) Safety of assets d) Utility

Ans : c

409. ‘A’ owes B Rs. 1000 payable on 1st December 2017 with interest. On 1st June 2017
A offers to pay the amount with interest up to 1st of June 2017.

a) It is a valid Tender b) It is not a valid Tender of


performance
c) It is a legal tender d) It is an Illegal Tender

Ans : a

410. Lessor has right to ....................

a) Receive profit rent from the property for the unexpired period of the lease
b) Receive lease rent for the unexpired period of lease
c) Receive lease rent for the economic life of the building.
d) Receive lease rent for the physical life of the building.

Ans : b
384

411. Which of the following is not a criterion in determining the highest and best use of
land?

a) Physically possible b) Legally permissible


c) Financially feasible d) Socially acceptable

Ans : d

412. Marriage value (also often referred to as synergic value) in valuation basically
means ....................

a) Extinction of lease holder’s interest in the property when the merger


takes place
b) Merger of right of co-owner of property with the co-owner
c) Increase in the value of the property due to merger of two different
interests held by to different persons.
d) Owner’s interest in the property which is extinguished at the time of
merger.

Ans : c

413. The important criteria for the selection of the best land in the town is

a) More width of road b) Industrial zone


c) Prime location and High FSI d) Residential zone

Ans : c

414. Which one of the following is not the characteristic of real estate market?

a) No free flow of information b) It is imperfect market


c) It is perfect investment market d) Heterogeneity

Ans : b

415. An agreement is voidable by the party whose consent is obtained by

a) Misrepresentation b) Fraud
c) Incomplete information d) Wrong information

Ans : b
385

416. Which of the following is not to be considered while estimating Market value of the
property?

a) Utility b) Benefits
c) Transferability d) Distress of buyer/ seller

Ans : d

417. Cost of creating a new building having identical utility and performing similar function
as being performed by the existing old asset is called as -

a) Replacement cost b) Reproduction cost


c) Original cost d) Historic cost

Ans : a

418. Economic life of a RCC roofed load bearing building is normal considered as
....................

a) 100 years b) 50-60 years


c) 40 years d) 25 years

Ans : b

419. Under what circumstances, tenant can change the user of the tenanted premises.

a) At the whims and fancies of the tenant


b) With consent of the landlord in writing, provided the user is permissible
in law.
c) After giving notice to the landlord
d) After permission from local planning authority.

Ans : b

420. What is common in Distress sale & forced sale?

a) Adequate marketing time b) Market value


c) Insufficient marketing time d) Private negotiations

Ans : c
386

421. Rent control Act, Transfer of property Act, development control rules and building
Bye-laws, and laws governing the land are .................... which affect the value of property.

a) Technical factors. b) Economic factors


c) Social factors d) Legal factors

Ans : d

422. Under sec 63 of the Indian easements act, the licensee’s has a right on revocation
which means.

a) He is entitled to a reasonable time to leave the property


b) He has no light to remove any goods from such property.
c) He is not entitled for a reasonable time to leave the property.
d) He has no right to keep and goods from such property.

Ans : a

423. Which of the following development control rules have effect on intensity of use of
land?

a) Zoning b) Floor space index


c) For fire-fighting d) Rules for minimum of open space

Ans : b

424. Where rent reserved in an occupational lease is less than the full rental value, it is
called ....................

a) Acknowledgement rent b) Head rent


c) Standard rent d) Virtual rent

Ans : a

425. Which of the following is not a method used to calculate depreciation?

a) Straight Line Method b) Declining Balance Method


c) Average-of-the year Digits d) Constant percentage method

Ans : c
387

426. Right to property is a .................... right.

a) Fundamental b) Statutory
c) Constitutional d) Human

Ans : a

427. Number of years for which a new asset is profitably used for its intended purpose is
known as ....................

a) Economic life b) Normal useful life


c) Age d) Physical life

Ans : a

428. Useful life of an asset is affected by which of the following?

a) Cost b) Replacement cost


c) Wear and tear d) Net Book Value

Ans : c

429. An investment pays Rs. 300 annually for five years, with the first payment occurring
today. The present value (PV) of the investment discounted at a 4% annual rate is
approximately ....................

a) Rs. 1,336 b) Rs. 1,389


c) Rs. 1,625 d) Rs. 1,925

Ans : b

430. Where a coparcenary consists of several branches and a partition takes place

a) Each branch takes as per capital


b) Each branch and member of each branch takes per capital
c) Each branch and member of each branch takes per stripes
d) Each branch takes per stripes and member of each branch takes per
capital

Ans : a
388

431. A valuer should co-operate and be available for .................... and investigation carried
out by the concerned authority

a) Inspection b) Scrutiny
c) Alliance d) Argumentation

Ans : b

432. Valuer should adopt .................... rate of capitalization if income flow is safe & secured.

a) Lower b) Higher
c) Same rate as lending rate of bank d) Interest rate of his own choice

Ans : a

433. Which of the following is not in relevant factor while estimating depreciated replacement
cost of a building?

a) Year of construction b) Size


c) Accessibility d) Specification

Ans : c

434. Not true with respect to valuation of Joint venture development of property

a) estimate market value of land offered for development of land owner


b) estimate cost of construction of building proposed on land.
c) required to consider which political party is ruling in the state.
d) consider value of financial benefit and profit @ future date that is
contingent upon performance and fulfillment of terms of contract.

Ans : c

435. In the following approach to valuation, an accrued depreciation must be counted to


determine market value of assets

a) Market approach b) Income approach


c) Cost approach d) Discounted cash flow method

Ans : c
389

436. Common in distress sale forced sale ....................

a) Adequate marketing time b) Market value


c) Insufficient market time d) Private negotiations

Ans : c

437. Economic principle is not required to be considered for conventional valuation of


property ....................

a) Demand b) Price
c) Utility d) Cost

Ans : d

438. Opinion expressed on value of the property by valuer as expert witness in a valuation
of dispute case is .................... to the court.

a) binding b) not binding


c) of no value d) inform of advice

Ans : b

439. Infrastructure works like augmentation of road networks & transport facilities will result
in increased ....................

a) Environmental deterioration b) Overall developments


c) Crime Rate d) Withdrawal of capital

Ans : b

440. Not a fact but an estimate

a) cost b) price
c) value d) Loss

Ans : c

441. Adverse changes to demand for the product or services produced by an asset will
result into ....................
390

a) Physical b) Functional
c) Economic d) Legal

Ans : c

442. Factor is not considered for weightage adjustment in sales comparison

a) Location b) Size
c) Accessibility d) payable by seller of comparable

Ans : d

443. Valuation of land always involves consideration on the principle of

a) HABU b) Competition
c) Surplus productivity d) Contribution

Ans : a

444. Sec 107 under transfer of act, a lease of immovable property from year to year, or for
any term exceeding one year or reserving a yearly rent, can be made only by

a) Ordinary instrument b) Registered instrument


c) Unregistered instrument d) Oral

Ans : b

445. Why should an entrepreneur do a feasibility study for a new venture?

a) To see if there are possible barriers to success


b) To identify possible sources of funds
c) To estimate the expected sales
d) To explore potential customers

Ans : a

446. Fully developed land with fully tenanted building occupied by the protected tenants by
.................... of valuation

a) Any method b) Cost approach


391

c) Market approach d) Income approach

Ans : d

447. Which is the most appropriate method of valuation for an income generating
commercial asset?

a) Land & Building method b) Any method


c) Direct comparison method d) Discounted cash flow method

Ans : d

448. Which of the following need not form part of the valuation report if fully tenanted property
is offered for sale by the user of the report?

a) Prevalent cost of construction in the locality


b) Total rent receivable from the property
c) Details about total life period for which the rental income flow will continue
d) Details of all outgoings and prevalent rate of capitalization in the market
for similar property in the locality.

Ans : a

449. The property falling in the locality lacking in the infrastructure will .................... with the
value of the property

a) Increase b) Decrease
c) Stabilize d) Have no relation with

Ans : b

* * *
392

This page is kept vacant intentionally.


393

PART - VII

MODEL QUESTION PAPER


PRESCRIBED BY IBBI
394

This page is kept vacant intentionally.


395
Part - VII

MODEL QUESTION PAPER FOR VALUATION EXAMINATION


Asset Class: LAND and BUILDING

1. The price elasticity of demand measures the change in the quantity demanded of a
service in relation to a change in its price when nothing but the ___________ changes.
a) price
b) cost
c) quality
d) value
Ans.(a)

2. Other things remaining same, the law of demand states __________.


a) the higher the price, the lower the quantity demanded
b) the higher the price, the higher the quantity demanded
c) the higher the price, the lower the quantity supplied
d) the higher the price, the higher the quantity demanded
Ans.(a)

3. A market which has only one seller selling a homogeneous product to many buyers, is
known as _______.
a) Monopoly
b) Oligopolistic
c) Perfect Competition
d) Monopolist Competition
Ans.(a)

4. Which of the following statements is not true about factors of production?


a) Land means natural resources.
b) Labour means human resources.
c) Capital means financial resources.
d) Entrepreneurship means pursuit of opportunity beyond resources controlled.
Ans.(c)

5. Which of the following is not true about Ricardian theory of rent?


a) Rent increases with increase in population.
b) Rent is surplus above cost.
c) Rent is unearned income.
d) Rent arises only in the short run.
Ans.(d)

6. Which of the following is not correct about functions of money?


a) It is a medium of exchange.
b) It is a measure of economic power.
c) It is a standard of deferred payments.
d) It is a store of value.
Ans.(b)

7. Which of the following is not a cause of demand-pull inflation?


a) Increase in wages and salaries
b) Increase in Government spending
396

c) Increase in interest rates


d) Increase in exports
Ans.(c)

8. If a person has an income of Rs.30000 and his consumption is Rs.10000, then his
propensity to save is____.
a) 1.33
b) 0.33
c) 0.67
d) 1.50
Ans.(c)

9. Which of the following is an example of secondary sector of an economy?


a) Mining
b) Forestry
c) Food Processing
d) Pisciculture
Ans.(c)

10. How are Gross Domestic Product (GDP) and Gross National Product (GNP) related?
a) GNP = GDP – NR + NP
b) GNP = GDP + NR - NP
c) GNP = GDP + EX - IM
d) GNP = GDP -EX + IM
Ans.(b)

11. In double entry system, accounts are primarily classified in to ___________.


a) Receiver account & Giver account
b) Income account & Expenses account
c) Real account & Nominal account
d) Personal Account & Impersonal account
Ans.(d)

12. Discount received is recorded on which side of a cash book?


a) Receipts
b) Payments
c) Income
d) Expense
Ans.(b)

13. For a real estate construction company, which of the following are not part of Profit
and Loss Statement?
a) Revenue from apartment sales
b) Interest paid to lenders
c) Cash deposited in bank
d) Depreciation expense
Ans.(c)

14. Proposed dividend is shown in the Balance Sheet of a company under the head _____.
a) provisions
397

b) reserves and surplus


c) current liabilities
d) other liabilities
Ans.(a)

15. Difference between variable cost per unit and selling price can be classified as______
margin per unit.
a) contribution
b) gross
c) net
d) profit
Ans.(a)

16. Right to property is a ___ right.


a) fundamental
b) statutory
c) constitutional
d) human
Ans.(c)

17. Which of the following is not a requirement of a contract?


a) Competent Parties
b) Free Consent
c) Legal Object
d) Adequate Consideration
Ans.(d)

18. Performance of contract is one of the methods to__________.


a) create a new contract
b) set-aside a contract
c) make a contract void
d) discharge a contract
Ans.(d)

19. In case of a ___________, the duty is one imposed by the law and is owed to the
community at large.
a) contingent contract
b) cecile agreement
c) government tender
d) tort
Ans.(d)

20. As per section 36 of the Insolvency and Bankruptcy Code, 2016, the liquidator shall
hold the liquidation estate ___________.
a) as an agent of debtor
b) as an agent of committee of creditors
c) as a fiduciary for the benefit of all the creditors
d) as a fiduciary for the benefit of all the stakeholders
Ans.(c)
398

21. Section 231 (2) of the Companies Act, 2013 empowers a tribunal to __________, if it
is satisfied that the compromise sanctioned under section 230 cannot be implemented
satisfactorily, and the company is unable to pay its debts as per the scheme.
a) wind up the company
b) restructure the debt
c) call for rearrangement
d) replace the management
Ans.(a)

22. In a right skewed frequency distribution, __________.


a) mean is typically greater than the median
b) mean is typically less than the median
c) mean is typically less than the mode
d) mean is equal to mode which is equal to median
Ans.(a)

23. When numbers are associated with weights, then obtained mean is said to be _______.
a) weighted arithmetic mean
b) harmonic mean
c) standard mean
d) geometric mean
Ans.(a)

24. If two events A and B are mutually exclusive, the probability of occurrence of either A
or B is__________.
a) difference between individual probabilities of A and B
b) sum of the individual probabilities of A and B
c) always 1
d) one minus sum of the probabilities of A and B
Ans.(b)

25. Gradual shifting of a time series over a long period of time is called as __________.
a) periodicity
b) cycle
c) regression
d) trend
Ans.(d)

26. Technology that permits safe, efficient, and inexpensive clean-up of contaminants in
property tends to minimise _________ in asset value.
a) decrease
b) increase
c) fluctuations
d) variations
Ans.(a)

27. Which planning provision is required around the battery limit of industry and for
industry having odour problem?
a) No development zone
b) Green belt
399

c) Special permission zone


d) Industrial regulation zone
Ans.(b)

28. Which of the following is not covered under the Indian Forest Act, 1927?
a) Reserved forest
b) Village forest
c) Protected forest
d) Prohibited forest
Ans.(d)

29. Which of the following legislations does not provide for pollution control?
a) The Water (Prevention and Control of Pollution) Act, 1974
b) The Air (Prevention and Control of Pollution) Act, 1974
c) The Environment (Protection) Act, 1986
d) The Technology (Transfer of Abuse) Act, 2007
Ans.(d)

30. A valuer shall act with objectivity in his professional dealings by ensuring that his
decisions are made ________.
a) without the presence of any bias
b) with coercion
c) with confidence
d) with undue influence of any party
Ans.(a)

31. A valuer, while respecting the confidentiality of information acquired during the course
of performing professional services, shall maintain proper working papers for a period of
______.
a) 3 years
b) 2 years or such shorter period as required in its contract for a specific valuation
c) 3 years or such longer period as required in its contract for a specific valuation
d) 2 years
Ans.(c)

32. Integrity of registered valuer is seriously affected by ____.


a) being honest
b) being straightforward
c) misrepresenting any facts or situations
d) ignoring public interest
Ans.(c)

33. Which of the following is not a duty of the valuer?


a) To verify ownership in document and occupancy on actual site
b) To identify plot number in document as well as on-site
c) To be responsible for correctness of survey findings by his assistant
d) To investigate title of the property to be valued
Ans.(d)
400

34. Under the Right to Fair Compensation and Transparency in the Land Acquisition and
Rehabilitation Act, 2013, market value is decided by the collector as per________.
a) circle rate/guidelines rates of last 5 years
b) the average of the sale price for similar type of land being acquired, ascertained from
the highest fifty per cent of the sale deeds registered during the preceding three years
in the nearest village or nearest vicinity of the land being acquired
c) purchase price of adjoining land
d) purchase price paid by the owner at the time of purchase of land which is to be acquired
Ans.(b)

35. Which of the following would have effect on development potential and values of
properties?
a) Floor space index
b) Ownership pattern
c) size and height of rooms
d) provision of utility services in a building
Ans.(a)

36. The approach to valuation of rented properties depends on_________.


a) nature and behaviour of landlords
b) category of tenants - tenant protected or not protected under the applicable rent act
c) rent paying capacity of the tenant
d) cordial and homely relationship between landlord and tenant
Ans.(b)

37. As per the Real Estate (Regulation and Development) Act, 2016, what percentage of
collections needs to be deposited by developers in Escrow accounts towards the cost of
construction including that of land?
a) 50%
b) 60%
c) 70%
d) 80%
Ans.(c)

38. As per the Transfer of Property Act, 1882, immovable property includes ______.
a) furniture
b) growing crops
c) grass
d) things permanently fastened to anything attached to the earth
Ans.(d)

39. Where, on a transfer of property, an interest therein is created in favour of a person to


take effect only on the happening of a specified event, such interest is called _________.

a) simple interest
b) diminishing interest
c) vested interest
d) contingent interest
Ans.(d)
401

40. A mortgage is a transfer of an interest in a specific immovable property for the purpose
of securing __________.
a) the payment of money advanced by way of loan
b) fully paid up debts
c) transfer of ownership of property
d) transfer of rights to sublease
Ans.(a)

41. A gives Rs. 5,00,000 to B on condition that he shall marry A’s daughter C. At the date
of transfer, C was dead. The transfer is void ______.
a) it is forbidden by the law
b) court regards it as immoral or opposed to public policy
c) it is impossible to fulfil the condition
d) it is immoral transfer
Ans.(c)

42. The Hindu Succession Act, 1956 does not apply to ______.
a) a follower of the Arya Samaj
b) a Sikh by religion
c) a child both of whose parents are Jains by religion
d) a person who is a convert to Christian
Ans.(d)

43. Which of the following items cannot be transferred under Inheritance/Succession laws
in India?
a) Personal movable property
b) Jewellery
c) Property not owned by self
d) Personal belongings
Ans.(c)

44. Value is an estimate of what ______ ought to be.


a) value
b) price
c) cost
d) worth
Ans.(b)

45. The value at the end of utility period of the asset without being dismantled is called
____ value.
a) salvage
b) realisable
c) scrap
d) junk
Ans.(a)

46. Real estate has some _______and, therefore, has a value.


a) profit
b) immobility
c) characteristics of non-marketable and non-investment property
402

d) use
Ans.(d)

47. Which of the following factor is not physical yet affects the valuation of the subject
property?
a) Damages to the building
b) Gross Domestic Product
c) Property location
d) Neighbourhood properties
Ans.(b)

48. Property is a ___ concept.


a) physical
b) legal
c) technical
d) social
Ans.(b)

49. The price that would tend to prevail in a free, open and competitive market on the basis
of equilibrium set by the forces of demand and supply is called ______.
a) value in exchange
b) value in use
c) optimum value
d) realisable value
Ans.(a)

50. What is the subject matter of valuation?


a) Interest in a property
b) Land and Building
c) Standard Rent
d) Profit Rent
Ans.(a)

51. A fund formed by setting aside an annual recurring amount for a given period of time
to recoup capital invested in a landed property is called ____.
a) sinking fund
b) annual amount of Rs. 1
c) annual value
d) investment value
Ans.(a)

52. A project requires an investment of Rs.10 lakh and has an NPV of Rs.16 lakh. What is
its profitability index?
a) 1.0
b) 1.6
c) 0.6
d) 3.2
Ans.(b)

53. Which one of the following best defines Annual sinking fund?
403

a) Annual sum required to be invested to amount to Rs. 1/- in specified years


b) Monthly sum required to be invested to amount to Rs. 10/- in specified years
c) Annual sum required to be invested to amount to Rs. 10/- in specified years
d) Annual sum required to be invested to amount to Rs. 100/- in specified years
Ans.(a)

54. A proposed development of Metrorail station in the city would _____.


a) decrease in supply of real estate in surrounding area
b) increase demand of real estate in surrounding area
c) decrease cost of construction of buildings in surrounding area
d) decrease demand of the real estate in surrounding area
Ans.(b)

55. Which one of the following is not a characteristic of real estate market?
a) No Free flow of information
b) It is imperfect market
c) It is perfect investment market
d) Heterogeneity
Ans.(c)

56. Which factor would negatively influence demand in real estate?


a) Low liquidity
b) Potential of capital appreciation
c) Regular income
d) Redevelopment potential of the property
Ans.(a)

57. Under rent capitalization method, value of the property increases with_______.
a) higher property tax
b) higher capitalization rate
c) lower capitalization rate
d) longer duration
Ans.(c)

58. In case the unexpired period of lease is too long then reversionary value would be___.
a) negative net present value
b) zero
c) negligible
d) less than zero
Ans.(c)

59. Which among the following is not a factor affecting market rent?
a) City
b) Location
c) Type of building
d) Turnover of the lessee
Ans.(d)

60. In relation to a lease agreement, the actual rent specified in the lease is called _____.
a) contractual rent
404

b) effective rent
c) negotiated rent
d) standard rent
Ans.(a)

61. Which of the following is not correct about ‘surrender of lease’?


a) Premature termination by lessee
b) Unilaterally terminated by a lessee
c) Terminated with the consent of the lessor
d) It can happen post expiry of the lease
Ans.(d)

62. Acceptance of project depends on __________.


a) positive net present value
b) negative net present value
c) zero net present value
d) zero internal rate of return
Ans.(a)

63. For which of the following, profit method of valuation is most appropriate?
a) Vacant Land
b) Petrol pump
c) Residential Home
d) School
Ans.(b)

64. The market demand curve shows ____________.


a) effect on market supply of a change in the demand for a good or service
b) marginal cost of producing and selling different quantities of a good
c) quantity of a good that consumers would like to purchase at different prices
d) effect of advertising expenditures on the market price of a good
Ans.(c)

65. Market approach basically operates on the_______.


a) principle of increasing and decreasing returns
b) principle of substitution
c) principle of conformity
d) principle of contribution
Ans.(b)

66. Which of the following is not a source from which sale instances of immovable property
in a particular locality can be collected?
a) Sales record at the registrar’s office
b) Advertisements in newspapers
c) Auction sale information from different authorities
d) Share market
Ans.(d)

67. The residual income from real estate available to land is representative of the principle
of_______.
405

a) surplus productivity
b) balance
c) increasing and decreasing returns
d) consistent use
Ans.(a)

68. Which of the valuation method is generally used for carrying out valuation of large
plots when sale instances of large size plots in the locality are not available?
a) Sales comparison method
b) Hypothetical plotting scheme
c) Hypothetical building scheme
d) Adopting rental instances
Ans.(b)

69. Quick estimation of costs of construction can be carried out by_________.


a) sum of digit method
b) detail estimation
c) plinth area rate
d) discounted cash flow method
Ans.(c)

70. Actual survival life of the building before it collapses is called the _____.
a) useful life
b) economic life
c) physical life
d) residual life
Ans.(c)

71. Asset which has become outdated mainly due to the planning and designing being
unsuitable for present day requirement of the user is an example of ___________.
a) technological obsolescence
b) economic obsolescence
c) functional obsolescence
d) both technological obsolescence as well as economic obsolescence
Ans.(c)

72. Depreciated Replacement Cost is market value of special purpose industrial plant
building subject to potential______.
a) profitability
b) cost
c) price
d) value
Ans.(a)

73. Which one of the following valuation methodology is most appropriate for valuing a
non-income generating residential bungalow?
a) Discounted Cash Flow Method
b) Sales Comparable & Depreciated Replacement Cost Method
c) Profit Method
d) Direct Capitalization Method
406

Ans.(b)

74. While carrying out valuation of property for bank finance, which of the following is not
to be taken into consideration?
a) Age of the building
b) Rent fetching capacity of the property
c) Economic obsolescence
d) Amount of loan
Ans.(d)

75. The income chargeable under head ‘capital gains’ is computed by deducting the
___________ from the full value of the consideration received as a result of transfer of the
capital asset __________.
a) expenditure incurred wholly and exclusively in connection with such transfer
b) expenditure incurred wholly and excessively after such transfer
c) indexed cost of repairing to be carried out
d) index cost of any improvements carried out after such transfer
Ans.(a)

76. Under the provision of the Securitisation and Reconstruction of Financial Assets and
Enforcement of Security Interest Act, 2002, enforcement of security interest means
__________.
a) sale of assets of the borrowing party by the bank
b) sale of charged assets by the secured creditor through the Debt Recovery Tribunal
c) sale of charged assets by the secured creditor without court intervention
d) getting bank's charge registered with central registry
Ans.(c)

77. If owner of plot A with house has a right of way over his neighbour’s plot B for
beneficial enjoyment of house, owner A is called ____.
a) co-owner
b) dominant owner
c) joint owner
d) servient owner
Ans.(b)

78. Which of the following transaction is an arm’s length transaction?


a) Transaction between parties at auction sale.
b) Transaction between parent and subsidiary company
c) Transaction between two old friends
d) Transaction between two brothers
Ans.(a)

79. Which return would you adopt as base while determining appropriate rate of the net
profit, unless it is otherwise found unsuitable?
a) The return from investments in stock and shares
b) The return from gilt-edge security
c) Interest paid in the saving account
d) The return from investment in gold
Ans.(b)
407

80. Which of the following judgements recognises the concept ‘Valuation is an art, not an
exact science. Mathematical certainly is not demanded, nor indeed is it possible.’.
a) K.P. Varghese vs ITO (1981) 131 ITR 597 (SC)
b) Gold Coast Trust Ltd. vs Humphray (1949) 17 ITR 19
c) Rustam C Cooper vs Union of India AIR 1970 SC 564
d) Hays Will Trust vs Hays and Others (1971) 1 WLR 758
Ans.(b)

81. Which of the following is expressed by a valuer while giving expert evidence in the
examination-in-chief in the court?
a) Opinions regarding values with reasoning
b) Evidence of facts
c) Answers without reasoning
d) Answers only in ‘yes’ or ‘no’
Ans.(a)

82. In context of the property insurance, which of the following is a human peril?
a) War
b) Age of the Property
c) Cyclone Facade
d) Upliftment
Ans.(a)

83. An insurance policy generally allows one to value the respective building and/or
contents by-____
a) Internal Rate of Return
b) Replacement Cost
c) Floor space index
d) Ground Coverage
Ans.(b)

84. A fire broke out in Hemant's factory and damaged half of the stock which was to be
shipped to a nearby cloth dealer. His fire insurance policy had the average clause in it. Actual
value of the stock: Rs.3,00,000, Sum insured for the stock: Rs.2,00,000, Loss incurred:
Rs.1,50,000 (As half the stock was destroyed). The claim amount will be Rs. ______.
a) Rs.1,00,000
b) Rs.3,00,000
c) Rs.2,00,000
d) Rs.1,50,000
Ans.(a)

85. When insurable amount is lower than ‘value at risk’, it is called ______.
a) over insured
b) fair insurable amount
c) fair premium for insurance
d) under insured
Ans.(d)
408

86. Upon paying the amount of loss to the insured, the insurer steps into the place of the
insured, taking over all his rights. It is called exercise of __________.
a) right of reinstatement
b) right of contribution
c) right of subrogation
d) right to salvage
Ans.(c)

87. Where the opinion of an expert is required for litigation in valuation, the report ____.
a) must rely on requirements imposed by the local authority where the property is located
b) rely on requirements imposed by the local authority where the Client is registered
c) rely on requirements imposed by the local authority of the registered office of the
lawyer
d) rely on requirements imposed by the local authority of the registered office of the reliant
party
Ans.(a)

88. While valuing assets where the valuer is not conversant with its features, __________.

a) it is advisable to engage the services of an expert and the signed report of the expert be
made a part of valuation report
b) it is advisable to engage the services of an expert and his report need not be made a part
of valuation report
c) he can privately seek the services of such expert and need not disclose in the report
d) it is enough to mention in report about details of expert engaged.
Ans.(a)

Attempt Questions 89 to 94 based upon the following case study.

A business man purchased a plot of 1000 Sq.mt. in a posh locality of a city in the year 1987 for a price
of Rs. 30,00,000. In the year 1988, he constructed a residential bungalow having 300 Sq.mt. built up
floor area at ground level and 100 Sq.mt. built up area at first floor level at the cost of Rs. 14,00,000.
Prevalent replacement cost of similar bungalow as on today is Rs. 30,000 per Sq.mt. Prevalent land
price in the locality at present is Rs. 60,000 per Sq.mt. Age of building is 30 years and the total life of
the building is 60 years.

89. What will be the depreciation amount of the bungalow by adopting straight line method
of depreciation and considering scrap value at 10 % ?

a) Rs.60,00,000
b) Rs.54,00,000
c) Rs.45,00,000
d) Rs.12,00,000
Ans . (b)

90. What will be the depreciation amount of the bungalow by adopting constant percentage
method of depreciation?

a) Rs.54,00,000
b) Rs.47,37,600
c) Rs.60,00,000
409

d) Rs.54,46,000
Ans. (b)

91. What will be the market value of the land at present?

a) Rs.240,00,000
b) Rs.600,00,000
c) Rs.480,00,000
d) Rs.410,00,000
Ans. (b)

92. What will be the total market value of the bungalow property for the bank loan purpose?

a) Rs.600,00,000
b) Rs.666,00,000
c) Rs.612,00,000
d) Rs.566,10,000
Ans (b)

93. What is the balance economic life of the building?

a) 60 years
b) 30 years
c) Zero
d) 45 years
Ans. (b)

94. Which of the following will not be considered for the estimation of present market value of
above property?

a) Deprecation
b) Current Replacement cost of the building
c) Economic obsolescence
d) Current land rate
Ans. (c)

****
410

This page is kept vacant intentionally.


411

PART - VIII

MOCK TESTS - 3 Nos.


412

This page is kept vacant intentionally.


413

Part - VIII

MOCK TEST NO. 1

1. A property was acquired on 01.04.1972. The property was sold on 01.04.2017. On


which date, the FMV is to be ascertained?

a) 01.04.1981 b) 01.04.2001
c) 01.04.1972 d) 01.04.2017

Ans : b

2. 99 years lease with renewal clause is called as

a) Long lease c) Short lease


c) Perpetual lease d) None of the above

Ans : c

3. A person seeks an income of Rs. 1,000 per annum from an investment. He wishes
this is to be an 8% return on his investment. What is the amount he has to invest?

a) Rs. 1,000/- b) Rs. 80/-


c) Rs. 12,500/- d) Rs. 10,000/-

Ans : c

4. If any property is to be valued for mortgage purpose to bank as collateral security,


the important factor to be considered is

a) Loan amount b) Value of the property


c) Location of the property d) Marketability & enforceability

Ans : d

5. Which are same?

a) Straight line method b) Linear method and


and linear method constant percentage method
414

c) Linear method and d) Sinking fund and


sinking fund method straight line method

Ans : b

r n
6. A=P(1- ) - this formula is to find the
100

a) Depreciated value by constant percentage method


b) Depreciated value by straight line method
c) Depreciated value by sinking fund method
d) Replacement value by straight line method

Ans : a

7. The meaning of cost of construction with reference to income tax valuation is

a) The value of the property purchased


b) The market value of land and depreciated value of building
c) The amount spent by the assessee in the construction of his new
building
d) The replacement value of the building

Ans : c

8. Land and building method is also known as

a) Depreciated cost method b) Detailed estimate method


c) Book value method d) Composite rate method

Ans : a

9. A corner plot is normally enjoys a higher rate by ............ percentage when


compared to the rate of single frontage plot

a) 10 to 15% b) 15 to 20%
c) 20 to 25% d) 25 to 30%

Ans : a
415

10. The open unobstructed view in front of the building is known as

a) Vista b) Elevation
c) Clear view d) Excellent

Ans : a

11. Belting theory method and Hypothetical plotting scheme (method) of valuing land
are corollaries of

a) Cost approach b) Market approach


c) Income approach d) Composite rate method

Ans : b

12. Loss in service value due to usage of an asset and passage of time - this is called
as

a) Appreciation b) Depreciation
c) Escalation d) Depletion

Ans : b

13. An asset is put into inferior usage of residence instead of commercial use - this is
called as

a) Physical depreciation b) Economic onsolescence


c) Functional obsolescence d) Technological obsolescence

Ans : b

14. A single screen theatre is obsolete - this is called as

a) Physical depreciation b) Economic onsolescence


c) Functional obsolescence d) Technological obsolescence

Ans : c

15. The method generally adopted by chartered accountants for preparation of


balance sheet of a company. This is called as
416

a) Direct appraisal method b) Written down value


c) Straight line method d) Constant percentage method

Ans : b

16. Workout N.P.V. of a building having 20 years of age and 60 years of total life. Its
replacement cost as on today is Rs. 4,30,000/-. Salvage value 10%. Adopt SLM

a) Rs. 3,11,000/- b) Rs. 4,11,000/-


c) Rs. 3,01,000/- d) Rs. 4,01,000/-

Ans : c

17. In this method, rate of depreciation is adopted as prescribed in Income tax Act -
This is called as

a) Statuatory depreciation method b) Linear method


c) Straight line method d) Sinking fund method

Ans : a

18. Vyagramukhi plot has .............. front width along the road and ............. width in the
rear side.

a) Wider, narrow b) Equal, equal


c) Narrow, wider d) None of the above

Ans : a

19. The mortgager delivers possession of the property to the mortgagee. The
mortgagee receives rent and profits from the property and retains the possession
till the full mortgage money is paid. This mortgage is called as

a) Simple mortgage b) Mortgage by conditional sale


c) Usufructuary mortgage d) English mortgage

Ans : c

20. Additional securities pledged to the bank in addition to the primary security are
called as
417

a) Primary security b) Collateral security


c) Current assets d) Stocks

Ans : b

21. It is an estimate of the price of the property would fetch in open market on ‘as is
where is basis’ in a short possible time is called as

a) Reserve price b) Market value


c) Forced sale value d) Auction value

Ans : c

22. Collapse of American Economy in 2008 - 2009 was due to

a) Lehman Brothers scam b) Joseph Edwin scam


c) Charles Brothers scam d) Winston (p) Ltd scam

Ans : a

23. .......................... possession is taken by the bank when the borrower is in


possession and occupation of the property or when the property is occupation of
third party like tenant, lessee. This is called as

a) Symbolic possession b) Physical possession


c) Actual possession d) Legal possession

Ans : a

24. Reverse mortgage scheme (RMS) was introduced in 2007 for the benefit of

a) Senior citizen over 60 years b) Young engineers


c) New entrepreneurs d) Honest business people

Ans : a

25. A right granted by a land owner to an owner of another property for non-exclusive
use of a portion of the land of a specific purpose or enjoyment of certain rights, is
called as
418

a) Leasehold right b) Easement right


c) Freehold right d) None of the above

Ans : b

26. According to this concept, the ownership of a particular property or a flat is held for
a specified period of time during a year. This is called as

a) Time share b) Short lease


c) Temporary accommodation d) Service apartment

Ans : a

27. Rented properties, hotels, cinemas, malls are termed as

a) Income fetching marketable properties


b) Non-income fetching marketable properties
c) Non-income fetching - non-marketable properties
d) None of the above

Ans : a

28. This approach is generally recommended for the non-income fetching marketable
properties.

a) Income approach b) Market approach


c) Cost approach d) Composite rate method

Ans : b

29. Sometimes property owners expect some likely changes in government and
expect the value of their property to rise in the near future. Such value is called as

a) Hope value b) Special value


c) Potential value d) Desired value

Ans : a

30. Temple property is not marketable, yet it has got value, we may call it as
419

a) Notional value b) Market value


c) True value d) Actual value

Ans : a

31. The assets which cannot be seen or touched but its effect can be notionally seen
and felt - They are called as

a) Intangible assets b) Tangible assets


c) Fixed assets d) None of the above

Ans : a

32. Reservation under different Acts, Height restriction rules near the airport area, safety
distance rules from High tension lines, railway tracks, highways, water courses are
some of the ............. factors affecting the value.

a) Economic factors b) Physical (technical) factors


c) Social factors d) Legal factors

Ans : d

33. The main requirement to write a valuation report

a) Technical and communication skill


b) Skill in the English language
c) To write a detailed report like a project report
d) Good presentation

Ans : a

34. Many a times, use of ‘cut’, ‘copy’ and ‘paste’ functions in computer make valuers
lazy about report writing which may result in enquiry by ................

a) Clients b) User
c) Investigation agency like d) Lawyer
CBI, Policy, etc.

Ans : c
420

35. While estimating the market value for mortgage, a valuer should consider himself
to be ..............

a) a lawyer b) a mortgage
c) a mortgagor d) both mortgagor and mortgagee

Ans : d

36. It means reduce in numbers of exhaust. It is the word applied to the consumption of
natural resources. Typical examples are natural gas, oil, coal, gravel

a) Depreciation b) Depletion
c) Exhaust d) None of the above

Ans : b

37. The Doctrine of Unearned Increase was enunciated because of a famous court
judgement. Select the correct judgement.

a) CIT vs. Smt. Ashima Sinha (1979) 116 ITR 26 (Calcutta), 1980 Tax 56(1) 19
(Calcutta).
b) Commissioner of Wealth Tax, New Delhi vs. Sri P.N. Sikand (1977) 107 ITR
922 (SC).
c) Controller of Estate Duty vs. Radha Devi Jalan (1968) 67 ITR 761, Calcutta
High Court.
d) C.W.T. vs. Venugopal Konar&Ors. (1977) 109 ITR 52, Madras High Court.

Ans :

38. The landmark judgement, Commissioner of Wealth Tax, New Delhi Vs. Sri P.N.
Sikand (1979) 107 ITR 922 (SC) states that:

a) only the lessor has the right to the increase in value


b) the increase in value of the leasehold interest of the property leased is to be
equally shared by both the lessor and lessee.
c) only the lessee has the right to the increase in value
d) the ratio of the lessor’s and lessee’s share is to be determined by negotiation

Ans :
421

39. The most common reason for value differing from price is that buyer/ seller is
uninformed as to what a property’s market value is but agrees on a contract at a
high or cheap price. In this case, the valuer has to .......................

a) to estimate the true market value


b) to estimate the true market price
c) not its market price
d) not its market value

Ans : b

40. Value in Use is often referred to as .......................

a) Objective value b) Subjective value


c) Exchange value d) Negotiated value

Ans : b

41. Real estate has some ....................... and, therefore, has a value.

a) Profit b) Immobility
c) Characteristics of d) Use
non-marketable and
non-investment property

Ans : d

42. The price that would tend to prevail in a free, open and competitive market on the
basis of equilibrium set by the forces of demand and supply is called .......................

a) Value in exchange b) Value in use


c) Optimum value d) Realizable value

Ans : a

43. Which one of the following is not a characteristic of real estate market?

a) No Free flow of information b) It is imperfect market


c) It is perfect investment market d) Heterogeneity

Ans : c
422

44. Fee simple means .......................

a) Freehold ownership b) Complete ownership


c) Leasehold ownership d) Most complete ownership subject
to the Government powers

Ans : d

45. In a condominium the common area rights like lifts, staircase and pavements are
called as
a) Co-Ownership b) Concurrent Ownership
c) Duplicate Ownership d) Contingent Ownership

Ans : b

46. The ....................... to value is most useful in determining insurable value, and cost
to construct a new structure or building

a) Cost Approach b) Market Approach


c) Income Approach d) None of the above

Ans : a

47. Which of the following would have effect on development potential and values of
properties?

a) Floor space index b) Ownership pattern


c) Size and height of rooms d) Provision of utility services in a
building

Ans : a

48. Value is an estimate of what ....................... ought to be.

a) Value b) Price
c) Cost d) Worth

Ans : b

49. Which one of the following valuation methodology is most appropriate for valuing a
non-income generating residential bungalow?
423

a) Discounted Cash Flow Method


b) Sales Comparable & Depreciated Replacement Cost Method
c) Profit Method
d) Direct Capitalization Method

Ans : b

50. Which of the following transaction is an arm’s length transaction?

a) Transaction between parties at auction sale.


b) Transaction between parent and subsidiary company
c) Transaction between two old friends
d) Transaction between two brothers

Ans : a

51. Since comparable sales are not identical to the subject property, adjustments may be
made for

a) Date of sale b) Location, style, amenities


c) Square footage, Site size d) All the above

Ans : d

52. If the comparable is superior to the subject in a factor or aspect, ....................... is


needed for that factor

a) Downward adjustment b) Upward adjustment


c) No adjustment d) Adhoc adjustment

Ans : a

53. Residual method is used for properties .......................

a) Ripe for development b) Redevelopment


c) For bare land only d) All of the above

Ans : d

54. A timber building on brick-walled pillars load bearing structure is an example of


424

a) Functional obsolescence b) Technological obsolescence


c) Economical obsolescence d) Legal obsolescence

Ans : b

55. An old residential building in a centrally commercial area in the heart of the city can be
termed as

a) Functional obsolescence b) Technical obsolescence


c) Economical obsolescence d) Legal obsolescence

Ans : c

56. Actual survival life of the building before it collapses is called the ........................

a) Useful life b) Economic life


c) Physical life d) Residual life

Ans : c

57. Asset which has become outdated mainly due to the planning and designing being
unsuitable for present day requirement of the user is an example of ........................

a) Technological obsolescence b) Economic obsolescence


c) Functional obsolescence d) Both technological
obsolescence as well as
economic obsolescence

Ans : c

58. The attributes can be the deciding factor to have precise market on the property value,
when comparing the sales instances and these attributes decides the market value of
a specific property and

a) will change from locality to locality


b) will change from time to time
c) will have no relation with time or locality
d) will change from locality to locality and time to time

Ans : d
425

59. In Evaluation grid the first principle attribute to be considered and has more weightage
in determining the market value

a) Size aspect b) Location aspect


c) Time aspect d) Age aspect

Ans : c

60. ....................... method is resorted to value a large sized plot for comparable in market
with smaller plots.

a) Belting method b) Hypothetical Plotting scheme


c) Hedonic Pricing Model d) Adjustment Grid Model

Ans : b

61. In the joint venture agreement, which are the factors affecting on the market value
aspects?

a) Building byelaws / development control rules


b) Lesser FSI
c) Open space and set back rules/ height restrictions.
d) All the above

Ans : d

62. DCF technique applies market-supported yields (or discount rates) ....................... to
arrive at a present value indication

a) To projected annual income


b) Lump sum reversion
c) to projected future cash flows - annual income figures and lump sum
reversion from the eventual sale of property
d) None of the above

Ans : c

63. The rate of interest for Capitalization is ....................... to the degree of security
426

a) not proportional b) inversely proportional


c) directly proportional d) not related

Ans : d

64. Two interest rates are considered for ....................... income

a) Perpetual b) Terminable
c) Annual d) None

Ans : a

65. As the unexpired period of lease increases, the capital value after reversion goes on
.......................

a) Increasing b) Decreasing
c) Without changing d) Negligible

Ans : d

66. Profit method used for trading properties where evidence of rates is slight, such as
hotels, restaurants and gas filling stations by adopting .......................

a) Three-year average of net income (as per profit and loss or income
statement) and capitalized using an appropriate yield
b) Five-year average of net income is capitalized using an appropriate
yield
c) Future income to be derived
d) None of the above

Ans : a

67. Under rent capitalization method, value of the property increases with .......................

a) Higher property tax b) Higher capitalization rate


c) Lower capitalization rate d) Longer duration

Ans : c

68. As per which Section of Transfer of Property Act are against those provisions of
Mohammedan law?
427

a) Section 13 of Transfer of Property Act


b) Section 14 of Transfer of Property Act
c) Section 13 and 14 of Transfer of Property Act
d) Section 129 of Transfer of Property Act

Ans : c

69. As per Transfer of Property Act Mohammedan gift is subject to .......................

a) No writing is necessary
b) Delivery of possession is essential
c) No writing is necessary, but delivery of possession is essential
d) Writing is necessary and delivery of possession is not essential

Ans : c

70. According to Transfer of Property Act, 1882 Instrument means

a) Non-testamentary instrument
b) Testamentary instrument
c) Both testamentary and non- testamentary instrument
d) None of the above.

Ans : a

71. Under the Transfer of Property Act, 1882, the term “attested” means

a) Attested by two or more witnesses


b) Attested by one witness only
c) Attested by two witnesses only
d) No condition prevails

Ans : a

72. Within the meaning of provisions of the Transfer of Property Act, 1882, the immovable
property does not include:

a) Standing timber or grass b) Standing timber, jewelry and crops


c) Standing timber, growing d) Only grass
crops or grass

Ans : c
428

73. Under section 16 of the Transfer of Property Act, 1882 where an interest created for
the benefit of a person or class of persons fails then:

a) Any interest created in the same transaction intended to take effect after
or upon failure of such prior interest also fails
b) Any interest created in the same transaction and intended to take effect
after or upon failure of such prior interest does not fail
c) Such failure does not affect
d) None of the above

Ans : a

74. According to the provisions of section 19 of the Transfer of Property Act, 1882

a) The vested interest is not defeated by the death of the transferee before
he obtains possession
b) Vested interest is defeated by the death of transferee before he obtains
possession
c) No such provision is made
d) None of the above.

Ans : a

75. Under the provisions of section 6 of the Transfer of Property Act, 1882, a right to future
maintenance

a) Can be transferred b) Cannot be transferred


c) No such provision is d) None of the above
made in the Act

Ans : b

76. Under the Transfer of Property Act, 1882, registered pertains to

a) Registration of property b) Registration of documents


c) Registration of parties d) None of the above

Ans : b

77. According to the provisions of the Transfer of Property Act, 1882, an easement cannot
be transferred apart from the dominant heritage
429

a) The statement is true b) The statement is false


c) The statement is partly true d) None of the above

Ans : a

78. Under the provisions of the Transfer of Property Act, 1882, right to sue

a) A mere right to sue can be transferred


b) A mere right to sue cannot be transferred
c) No such provision is made in the Act
d) None of the above

Ans : b

79. Sale for tangible immovable assets can be made by .......................

a) Rs. 100 or more to be done by a registered instrument


b) Less than Rs. 100 to be done by delivery of property
c) More than Rs. 100 to be done by delivery of property
d) Rs. 100 or less to be done by a registered instrument

Ans : a

80. Document is not necessary if value of immovable property is less than .......................

a) Rs.100 b) Rs. 1000


c) If by delivery d) If held in possession

Ans : a

81. In mortgage contract any condition that prevents the mortgagor from getting back his
property after the mortgage debt has been paid will be .......................

a) Invalid b) valid
c) void d) voidable

Ans : a

82. At any time after the principal money has become due the mortgagor has .......................
430

a) Right of redemption b) Has no rights


c) only by the Court decisions d) None of the above

Ans : a

83. Rights on redemption means

a) Right of Redemption is available from the date of execution - No time


specified
b) There must be demand and from the date mortgagor can redeem at
any time. - Debt payable on demand:
c) The money only due after specified date then from the date redemption
starts. - Term is fixed:
d) All the above

Ans : d

84. Which of the following is not correct about ‘surrender of lease’?

a) Premature termination by lessee


b) Unilaterally terminated by a lessee
c) Terminated with the consent of the lessor
d) It can happen post expiry of the lease

Ans : d

85. A lease of immovable property from year to year or exceeding can be by .......................

a) Oral agreement b) Unregistered instrument


c) Registered instrument d) Written statement

Ans : c

86. Head lessee subleases the property and he receive the rent and it is called as
.......................

a) Head rent b) Standard rent


c) Differential rent d) Monopoly rent

Ans : a
431

87. A gift to two or more donees of whom one does not accept it. Then the gift is
.......................

a) Void as to interest which he would have taken had he accepted


b) Valid as to interest which he would have taken had he accepted
c) Voidable at the option of the donor
d) Valid at the option of receiver

Ans : a

88. If owner of plot A with house has a right of way over his neighbour’s plot B for beneficial
enjoyment of house, owner A is called .......................

a) Co-owner b) Dominant owner


c) Joint owner d) Servient owner

Ans : b

89. An Easement is imposed on property, that property is called .......................

a) Co-ownership b) Dominant ownership


c) Joint ownership d) Servient ownership

Ans : d

90. Under Section 52 of the Easement Act, License is .......................

a) For exclusive possession


b) Does not create or transfer an interest in the land
c) Not transferable and revocable
d) All the above

Ans : d

91. Under Section 57of Easement Act, Grantor’s duty to ....................... defects

a) Disclose b) Not to disclose


c) Not necessary d) Duty of the licensee

Ans : a
432

92. Under Section 58 of Easement Act, the grant control render the property, which is?

a) Safe b) Unsafe
c) Danger d) Unsafe and danger

Ans : b

93. Under Section 61 of Easement Act, Revocation of the licensed property is .......................

a) Expressed or implied b) Expressedonly


c) Implied only d) None of the above

Ans : a

94. The Real Estate Act (RERA) makes it mandatory for commercial and residential real
estate projects, before launching the project, where with .................... has to register
with RERA.

a) land area is over 500 square metre or proposal for 8 building units
b) 500 square metre of land area only
c) proposal for 8 building units only
d) land area less than 500 sqm is also required to register

Ans : a

95. For ongoing projects which has not completed or have not received completion
certificate on the date of commencement of RERA Act will have to seek registration
...................

a) within 5 months b) within 3 months


c) not required d) or stop the work

Ans : b

96. Are commercial or community facilities that are provided within the real estate project
can be ...................?

a) sold separately
b) cannot be sold separately
c) can be with the promoter
433

d) common area includes commercial or community facilities, provided


within the real estate project are to be handed over to the Association

Ans : d

97. Under Section 14, can the promoter modify, amend sanctioned plan during execution?

a) the promoter can do major modifications, amend sanctioned plan duly


approved by the competent authority, after getting RERA approval and
obtaining prior written consent from at least 2/3rd no. of Allottees,
b) the promoter can modify, amend sanctioned plan during execution of a
real estate project after RERA approval
c) without obtaining prior written consent from at least 2/3rd no. of Allottees,
d) the promoter can modify, amend sanctioned plan after getting approval
from the local authority

Ans : a

98. What is the period for which the promoter is liable for any structural defects, Section
14(2)?

a) 5 years b) 4 years
c) 3 years d) 2 years

Ans : a

99. Under section 13, any promoter shall accept ................... as an advance payment,
from a buyer?

a) maximum of 10% of the cost of the apartment, as an advance payment,


from a buyer without first entering into a written agreement for sale and
register the same
b) 15% of the cost of the apartment, plot, or building, as an advance
payment, from a buyer
c) sum more than 10% of the cost of the apartment, plot, or building, as an
advance payment, from a buyer
d) sum less than 15% of the cost of the apartment, plot, or building, as an
advance payment, from a buyer

Ans : a
434

100. As per Section 4(2) (l) (D, it is obligatory for promoters to deposit in a separate account
to cover the cost of land and construction a sum of ................... the money collected
from buyers for a particular project

a) 70% of the money b) 60% of the money


c) 65% of the money d) Nothing specific

Ans : a

101. Whether the promoter can entertain a court proceedings and can get an injunction?

a) As per Section 79, no civil court shall have jurisdiction to entertain any
suit or proceeding in respect of any matter which RERA
b) Appellate Tribunal is empowered by or under this Act to determine and
no injunction shall be granted
c) Both (a) & (b) are correct
d) No such regulation

Ans : c

102. As per MORD Notification dated 9th February, 2016, in sub-section (1) of section 46
of the said Act, the limit is ................... for its own use

a) shall be 20 hectares in urban areas.


b) shall be 40 hectares in rural areas.
c) shall be 20 hectares in urban areas and 40 hectares in rural areas.
d) shall be 40 hectares in urban areas and 20 hectares in rural areas.

Ans : c

103. For a where a private company purchases land for rehabilitation and resettlement
under the Act would apply to land ...................

a) equal to or more than 50 acres in urban areas


b) equal to or more than 100 acres in rural areas
c) equal to or more than 50 acres in urban areas and 100 acres in rural
areas
d) equal to or more than 100 acres in urban areas and 50 acres in rural
areas.

Ans : c
435

104. Exercise :

A client wants to purchase a petrol bunk outlet situated on the main road in the center of town.
The main road has traffic of 300 PCU. For the land, the company pays the rent
Rs. 4,00,000/ per annum. Total income from sale of petrol and diesel and other items is Rs.
2,00,00,000/-. Property tax Rs. 50,000/6 months. Staff salary and other out goings are Rs.
60,000/ per month. Other expenses for running the business is Rs. 1,70,00,000/-. Rate of
capitalisation is 12%.

Question :

1. What is the total income for the owner?


2. What is the total expense for the owner?
3. What is the net profit?
4. What is the method to be used?
5. What is Y.P.?
6. What is the amount the client can pay to purchase the bunk?

Answers :

Given data :

Ground rent = Rs. 4,00,000 / annum


Income from sale of petrol = Rs. 2,00,00,000
Property tax = Rs. 50,000 / 6 months
Staff salary & other out goings = Rs. 60,000 / month
Other expenses = Rs. 1,70,00,000
Rate of capitalisation = 12%

Solution :

1.0. Income to the owner of the bunk

Ground rent = Rs. 4,00,000


Income from sale of petrol&diesel = Rs. 2,00,00,000
Total income for the owner = Rs. 2,04,00,000 (1)

2.0. Expenses for the owner

Property tax (50,000 x 2) = Rs. 1,00,000


436

Staff salary & other out goings = Rs. 7,20,000


(60,000 x 12)
Other expenses for running the = Rs. 1,70,00,000
business
Total expenses = Rs. 1,78,20,000 (2)

3.0. Net profit

Net income / Net profit = 2,04,00,000 - 1,78,20,000


Net profit = Rs. 25,80,000 (3)

100
4.0. Years purchase = = 8.5 (4)
12

5.0. Method = Profit method (5)

6.0. Value

Value of the bunk by capitalising = 25,80,000 x (100 / 12)


@ 12%
Capitalised value = Rs. 2,15,00,000/-

... The amount that can be paid for the = Rs. 2.15 crores (6)
purchase of the bunk

* * *
437

MOCK TEST NO. 2

Courtesy : IOV Coimbatore branch

1. Per-capita GNP is the total GNP divided by ____________

i) Total population
ii) Total number of persons earning
iii) Total male population
iv) Total Adult population
Ans : i
2. Propensity to consume is _________ of income used for consumption

i) Amount
ii) Percentage
iii) Total
iv) None of the above
Ans : ii
3. In India, different income classes (EWS/LIG/MIG/HIG) represent ________
distribution of national income

i) High
ii) Medium
iii) Unequal
iv) Equal
Ans : iii
4. National Product is summation of _______

i) Values of outputs of all Firms


ii) Production of all Firms
iii) Value of Government spending
iv) Value of Savings
Ans : i
5. In an open economy, part of output consumed outside the country is called
________

i) Import
ii) Export
iii) Domestic production
iv) All the above
438

Ans : ii
6. Industry sector is generally ___________ intensive

i) Labour
ii) Capital
iii) Land
iv) Profit
Ans : ii
7. Nominal GNP measures gross output of an economy at current prices; Real GNP
measures gross output of an economy at ________

i) Current year
ii) Year of independence
iii) 2000
iv) Base year
Ans : iv
8. Saving is __________

i) Part of income not spent


ii) Part of Expenditure
iii) Part of Investment
iv) Govt Securities
Ans : i

9. India’s Primary sector engages _______ workers than needed

i) Less
ii) More
iii) Moderate
iv) None of the above
Ans : ii
10. If a person has an income of Rs.30000 and his consumption is Rs.10000, then
his propensity to save is____.

i) 1
ii) 0.33
iii) 0.67
iv) 3
Ans : iii
11. WDV stands for
439

i) Written Down Value


ii) Write Down Value
iii) Written Depreciated Value
iv) Write Depreciated Value

Ans : i
12. Adjustment entries recorded in which book:

i) Journal
ii) Ledger
iii) Journal Proper
iv) Cash book
Ans : iii

13. Arrange the steps of accounting in sequential order -


(a) Trial Balance;
(b) Journal Entry;
(c) Balancing of Accounts;
(d) Ledger Posting.

i) abcd
ii) bdac
iii) cadb
iv) bdca
Ans : iv
14. At breakeven point

i) Total expenses + Total Revenue


ii) Total expenses = Total revenue
iii) Total Revenue - Total expenses
iv) Total Income = Total Output

Ans : ii
15. In double entry system, accounts are primarily classified in to ___________.

i) Cash and Property


ii) Tangible and Intangible
iii) Real and Nominal
iv) Personal Account & Impersonal account
Ans : iv
16. In criminal matter, an accused can be proceeded against under the law in force
at the time of -------------

i) Committing the offence


440

ii) Judgement
iii) Trail
iv) None of the above
Ans : i
17. Transfer of Property Act, 1882 requires registration of all such deeds which
purport to transfer real estate of value --------

i) over Rs. 10000/-


ii) over Rs. 5000/-
iii) over Rs. 100/-
iv) over Rs. 1/-
Ans : iii

18. An agreement without any consideration is not a contract, however amongst near
relatives, --------- is also plausible consideration in a contract but the instrument of
such a contract must necessarily be--------

i) love and affection, written


ii) love and affection, registered
iii) Friends and well wishers, registered
iv) Partners, registered
Ans : iii
19. The decisions of an arbitral tribunal must be --------------

i) Principal Arbiter
ii) By majority
iii) All the Arbiters
iv) Any one of the Arbiters
Ans : ii
20. State if true or false:
Under the SARFAESI Act, any Security Interest created over Agricultural
Land cannot be proceeded with.

i) True
ii) False
Ans : i
21. The disrepair for which a leaseholder or tenant is usually liable for mainly
commercial properties when he has agreed to return premises in good repair is
called ---------

i) All the below


ii) As is were is
iii) Good condition
iv) Dilapidation
441

Ans : iv
22. If A and B are mutually exclusive events, then P(A+B) is _____ .

i) P(A)-P(B)
ii) P(A)+P(B)
iii) P(A)=P(B)
iv) P(A) x P(B)
Ans : ii
23. Mode is ______________ .

i) Highest value
ii) Average data
iii) The middle most data
iv) the data with maximum frequency.

Ans : iv
24. If the sample size increases the sampling error __________ .

i) Increases
ii) Decreases
iii) No effect
iv) Equals
Ans : ii
25. For a left skewed data, the mean is _________ .

i) Half the median


ii) Greater than the median.
iii) Twice the median
iv) Less than the median.
Ans : iv
26. The industrial site distance should be at least

i) 5 km from seashore
ii) 500 m. from High Tide Line in coastal area
iii) 3 km from High Tide Line in coastal area
iv) Non of the above
Ans : ii
27. Which of the following will not have to obtain separate site clearance from the
Central Government?
442

i) Projects affecting reserve forest


ii) Oil exploration
iii) Mining of minerals
iv) Township projects
Ans : iv
28. The neighborhood of bus and car bodies manufacturing units are affected by

i) Sound pollution
ii) Air pollution
iii) Chemical pollution
iv) Water pollution
Ans : i
29. Which of the following is not a source of water pollution

i) Tannery waste
ii) Iron
iii) Domestic waste
iv) Dying waste
Ans : ii
30. Which of the following is correct?
(i) A valuer shall conduct the valuation independent of external influences. (ii) A
valuer shall maintain complete independence in his/its professional relationships.
(iii) A valuer shall conduct the valuation according to the direction of his client.

i) (ii) and (iii) only


ii) (i) and (iii) only
iii) (i) and (ii) only
iv) None of the above
Ans : iii
31. Which of the following is correct?
(i) A valuer shall provide all information and records as may be required by the
authority.
(ii) A valuer shall provide all information and records as may be required by the
Tribunal, Appellate Tribunal.
(iii) A valuer shall provide all information and records as may be required by the
registered valuers organisation with which he/it is registered.
(iv) A valuer shall provide all information and records as may be required by a
tender committee of a valuation tender.

i) (ii) and (iii) only


ii) (i) (ii) and (iii) only
iii) (i)(ii) and (iv) only
iv) (i) and (iv) only
443

Ans : ii
32. Where illegal activity is suspected, the valuer is advised to
(i) disclose the activity if there is a legal obligation to do so
(ii)suggest alternative, legal ways in which the client’s needs might be met
(iii) report to the Police
(iv) refuse to accept the assignment

i) (ii) and (iii) only


ii) (i) (ii) and (iii) only
iii) (i)(ii) and (iv) only
iv) (i) and (ii) only
Ans : iv
33. ---------- which is an amount equivalent to one hundred per centage of the market
value and is given in addition to value of land, building, trees

i) Solatium
ii) Compensation
iii) Grant
iv) Assignment
Ans : i
34. The owner of the land, for the beneficial enjoyment of which, the easement right
exists is the

i) Public passage
ii) Servient owner
iii) Dominant owner
iv) Licensed owner
Ans : iii

35. Any person who, is receiving, or is entitled to receive, the rent for any premises,
whether on his own account or for another as a trustee/ guardian or court receiver is
called a --------

i) Lessor
ii) Court receiver
iii) Occupier
iv) Landlord
Ans : iv
36. What is 'Carpet Area’ as per RERA

i) Plinth area of an apartment, including external walls, services shafts,


exclusive balcony/ veranda/ terrace area, but includes the area covered by the
internal partition walls of the apartment
ii) Carpeted floor area
iii) Net usable floor area of an apartment, excluding external walls, services
444

shafts, exclusive balcony/ veranda/ terrace area, but includes the area covered by
the internal partition walls of the apartment
iv) Super Built up area
Ans : iii
37. The preliminary notification for land acquisition under LARR is to be issued -------
Social Impact Assessment?

i) Latest within 12 months after the date of report


ii) Latest within 2 years after the date of report
iii) Latest within 6 months after the date of report
iv) Non of the above
Ans : iv

38. As per sec 54 of TPA Act Sale is a transfer of property in exchange for a --------
paid, only promised to be paid or partly paid and partly promised

i) Money
ii) Price
iii) Value
iv) Amount
Ans : ii
39. Transfer of Development Rights (TDR) is a concept in

i) Land management
ii) Money management
iii) Social welfare
iv) Non of the above

Ans : i
40. According to SARFAESI Act, before effecting sale of the immovable property,
the authorised officer shall obtain valuation of the property from

i) Registered valuer “34AB wealth tax act”


ii) Approved valuer
iii) Panel valuer
iv) Registered valuer “Companies act”
Ans : i
41. The Rent Control Act generally only applies on lease agreements of at least -----
--- months

i) 6 months
ii) 11 months
iii) 24 months
iv) 12 months
445

Ans : i
42. As per the Transfer of Property Act, 1882, immovable property includes _

i) furniture.
ii) growing crops.
iii) grass.
iv) things permanently fastened to anything attached to the earth.
Ans : iv
43. Under the Transfer of Property Act, 1882, the expression 'registered' pertains to

i) Registration of property
ii) Registration of documents
iii) Registration of parties
iv) Registration of charges

Ans : ii
44. What is real estate?

i) Physical land and building only


ii) Physical land and building with all developments, etc., over and under the
land with rights.
iii) Real interest of the property
iv) Physical land and building only with all developments, etc., over and under
the land.
Ans : iv
45. Cost represents the ________ side of a Transaction.

i) Exchange
ii) Supply
iii) Demand
iv) Bargaining
Ans : ii
46. Value of an object arises out of its _________ .

i) Manufacture
ii) Marketing
iii) Usefulness
iv) Cost
Ans : iii
47. What is common in Distress sale & Forced sale ?
446

i) Circumstances
ii) Value
iii) Insufficient marketing time
iv) private negotiations

Ans : iii
48. What is the name of the value of the property realized after auction sale?

i) Liquidation value
ii) Salvage value
iii) Net Present value
iv) Replacement value
Ans : i

49. Value of an object depends upon the future ______ that can be derived out of it.

a) Benefits
b) Demand
c) Supply
d) Cost
Ans : i
50. Construction of a building on the Land can be called

i) a Production
ii) a Conversion
iii) a Potential
iv) an Improvement
Ans : iv
51. Development Control Rules decides the _________ of the property

i) Security
ii) Risk
iii) Utility
iv) Transferability
Ans : iii
52. Development Control Rules decides the _________ of the property

i) Security
ii) Risk
iii) Utility
iv) Transferability

Ans : iii
447

53. Supply and Demand controls __________ of property

i) Transferability
ii) Scarcity
iii) Utility
iv) Non-transferability

Ans : iii
54. The Plots which is connected to the access road through a passage is called
_______

i) Narrow Plots
ii) Rectangular Plots
iii) Ribbon Plots
iv) Tandem Plots
Ans : iv
55. The open unobstructed view from then front of building is called ________

i) Frontage
ii) Access width
iii) Vista
iv) Wide view
Ans : iii
56. The study of Supply and demand is __________ Aspect

i) a Physical
ii) an Economic
iii) a Social
iv) a legal

Ans : ii
57. When the tenant will bear the burden of all outgoings, it will be termed as

i) Virtual rent
ii) Net rent
iii) Gross rent
iv) Exclusive rent
Ans : iv
58. The highest rent that is receivable for the property, by the landlord, in the open
market is called

i) Virtual rent
ii) Market rent.
iii) Gross rent
iv) Exclusive rent
448

Ans : ii
59. The NPV of a project generally ……………. as the rate of return increases.

i) decreases
ii) increases
iii) equal
iv) No effect
Ans : i
60. The difference between head rent and improved rent is called

i) Net rent
ii) Profit rent
iii) Gross rent
iv) Annual rent

Ans : ii
61. Rent fixed by court proceedings may be

i) Fair rent
ii) Standard Rent
iii) Statuary rent
iv) All the above
Ans : iv

62. Gymnasiums will come under which occupancy?

i) Residential
ii) Mercantile
iii) Special Residential
iv) Assembly
Ans : iv
63. What is the Years Purchase for Rs 1/- with a remunerative interest 8% and
Annual Sinking Fund to be set aside for recouping Rs 1/- is 0.021is

i) 1/(0.08 + 0.021)
ii) 0.021/0.08
iii) (0.08 + 0.021)/0.021
iv) None
Ans : i

64. Recess land and land locked lands:

i) They both generally abut on a private road.


ii) They both generally abut on a public road.
449

iii) They both generally do not abut on a public road.


iv) None of the above

Ans : iii
65. A multistoreyed building has several flats, almost all similar. Two recently sold
comparables were found. The valuer, using the market comparison approach, set
up the following table:
Flat under Valuation Comparable 1 Comparable 2
Date of Sale 1 ½ years ago 3 years ago
Rate per sq.ft. Rs. 18,500 Rs. 18,000
What would be the best estimate of the rate?

i) Rs. 19,000 per sq.ft..


ii) Rs. 17,500 per sq.ft.
iii) Rs. 17,000 per sq.ft..
iv) Rs. 18,000 per sq.ft.
Ans : i

66. Belting Theory is used for Valuation of

i) Buildings
ii) Plant & Machinery
iii) Land with less width and more depth
iv) Shares
Ans : iii

67. Which of the following is not a marketable property

i) Government property
ii) Shop
iii) House building
iv) Hotel building
Ans : iii
68. Market approach basically operates on the_______.

i) principle of increasing and decreasing returns


ii) principle of substitution
iii) principle of conformity
iv) principle of contribution
Ans : ii
69. Which of the following is not a source from which sale instances of immovable
property in a particular locality can be collected?

i) Sales record at the registrar’s office


450

ii) Advertisements in newspapers


iii) Auction sale information from different authorities
iv) Share market
Ans : iv
70. Economic obsolescence occurs due to

i) Wear and tare of the building


ii) Under-utilisation of buildings
iii) Out dated planning of building
iv) All the above
Ans : ii
71. A building is 40 years old. It has a total life span of 80 years. Current
replacement cost of the building is INR 40,00,000. The salvage value of the
materials of the building at the end of its life is 10% of its CRC. What is the
depreciation in % today?

i) 55%
ii) Rs 18,00,000
iii) Rs 45,000
iv) 45%
Ans : ii
72. The estimated of the age of a structure based on its utility and physical wear and
tear is called

i) Economical Age
ii) Total Age
iii) Physical Age
iv) Effective Age

Ans : iv
73. Which one of the following valuation methodology is most appropriate for valuing
a non-income generating residential bungalow?

i) Discounted Cash Flow Method


ii) Sales Comparable & Depreciated Replacement Cost Method
iii) Profit Method
iv) Direct Capitalization Method
Ans : ii
74. Owelty money is to be determined by a valuer for the purpose of

i) Settlement
ii) Land Acquisition
iii) Market sale
iv) Family Partition
451

Ans : iv
75. The base year for Capital Gain Tax calculation as on date is

i) 2001-2002
ii) 2000 -2001
iii) 1981-1982
iv) 1978-1979
Ans : i
76. While carrying out valuation of property for bank finance, which of the following is
not to be taken into consideration?

i) Age of the building


ii) Rent fetching capacity of the property
iii) Economic obsolescence
iv) Amount of loan
Ans : iv
77. For valuation of easement, the valuer is to estimate the value of the interest of

i) Dominant owner only


ii) servient owner only
iii) Dominant and servient owner
iv) None of the above
Ans : iii
78. For the purpose of Insurance, valuers generally provide

i) Reinstatement Value
ii) Historic Value
iii) Book Value
iv) Present Value
Ans : i
79. The difference between one honest valuation and another may range upto 15%

i) V.C. Ramchandran vs. CWT (1979) 126 ITR 157 Karnataka HC


ii) K.P. Varghese vs. ITO (1981) 131 ITR 597 (SC)
iii) Gold Coast Trust Ltd. vs. Humphray (1949) 17 ITR 19
iv) Subh Karan Choudhury vs. IAC (1979) 118 ITR 777 Kolkata HC (Special
Value/FMV)
Ans : ii
80. Valuation is not an exact science. Mathematical certainly is not demanded, nor
indeed is it possible

i) V.C. Ramchandran vs. CWT (1979) 126 ITR 157 Karnataka HC


452

ii) K.P. Varghese vs. ITO (1981) 131 ITR 597 (SC)
iii) Gold Coast Trust Ltd. vs. Humphray (1949) 17 ITR 19
iv) Subh Karan Choudhury vs. IAC (1979) 118 ITR 777 Kolkata HC (Special
Value/FMV)

Ans : iii
81. Fire Policy covers

i) Only fire, lightening, explosions & implosions


ii) Flood, storm & tempest, inundation
iii) Land slide, rock slide & impact by road or rail
iv) All of them said above

Ans : iv
82. Fire insurance policy in addition to the assessed loss, also pays

i) Loss minimisation expenses


ii) 1% of claim amount towards debris removal
iii) 3% claim amount towards Engineers/Architects Fee
iv) All the above
Ans : iv
83. Insurance policy is

i) A simple contract between two parties


ii) Assures for return of the premium on completion of the policy period
iii) An offer and acceptance document
iv) A saleable document by an insurer to the insured

Ans : iii
84. Duties of the insured

i) To make the premium payment through the bank, from where he had taken a
loan
ii) To disclose all material facts of the property to be insured
iii) To sought for a fire & burglary risk in the same policy as demanded by the
banker
iv) To inform a lower value of risk so as to pay a lower premium
Ans : ii
85. AOG Peril is

i) Loss due to war, invasions, act of foreign enemy hostilities


ii) Loss due to fire
iii) Loss due to earthquake
iv) Loss due to Lightening, STFI, subsidence, land slide and rock slide
453

Ans : iii
86. Which of the following is not required to be mentioned in a leasehold valuation
report

i) Postal address of the property


ii) Name of the title holder of the property
iii) Name of the Lessor
iv) Name of the heir of the Lessor
Ans : iv
87. Valuation reports under The Companies (Registered Valuers and Valuation)
Rules, 2017 shall be prepared following

i) Indian Valuation Standards


ii) International Valuation Standards
iii) Global Valuation Standards
iv) IBBI Standards
Ans : ii
454
455

MOCK TEST NO. 3

Courtesy : IOV Coimbatore branch

1. Sarkaria Commission was made in the year

i) 1980 ii) 1981


iii) 1982 iv) 1983

RBI : 1935, SBI : 1955, NABARD : 1982

2. Maximum limit of investment in small scale industries

i) 0 to 25 lacs ii) 25 – 50 lacs


iii) 50 – 1 cr iv) 25 – 5 cr

Tiny sectors : 0 - 25 lacs

3. Cost behaviour refers to

i) Whether a particular expense has to be incurred honestly


ii) Low costs reach to change in the level of activity
iii) Whether a cost incurred in a manufacturing trading or service company
iv) Classifying costs as either product or period costs

4. Indifference curves of two no’s

i) Positive curves ii) Negative curves


iii) Parallel iv) Perpendicular

Indifference curves : Convex to the originIndifference curves refers to Two products.

5. Demand pull inflation rises due to

i) Mismatch between demand and supply of commodities


ii) Increase in the price of precious metal
iii) Persistent rise in factored cost
iv) Combine phenomena of demand pull and cost push inflation
456

Increase in wages, Govt spending, Population. Decrease in production.

6. In Closed economy

i) GDP = GNP ii) GDP < GNP


iii) GDP > GNP iv) GDP = GNP G

NP = GDP + (Export - Import) NNP = GNP - Depreciation

7. Which of the following deposits gives highest rate of interest

i) Current deposit ii) Fixed deposit


iii) Recurring deposit iv) None of the above

8. This is capital

i) Money ii) Forest


iii) Machinery iv) Trademarks

The term capital is used in economics in various senses. In ordinary language and
sometimes in economics also capital is used in the sense of money. But when we talk
of capital as a factor of production, to confuse capital with money is quite wrong. Of
course, money is used to purchase various factors such as raw materials, machinery,
labour which help to produce goods, but money itself does not directly help in the
production of goods.

The money which is available for investment and productive purposes has been called
money capital or financial capital by some economists. But money capital is not the
real capital. The real capital consists of machinery, tools, tube well, factories; tractors,
etc., Which directly assist in the production of goods.

Similarly, government securities and bonds, shares and debentures of public limited
companies do not represent real capital. Securities, bonds, stocks, etc., Possessed
by individuals yield income to them but they cannot be called real capital because they
represent only titles of ownership rather than factors of production.

9. In the following which one gives short term loan?

i) SBI ii) Agricultural Bank, NABARD


iii) Commercial Bank iv) Industrial development bank
457

10. The real GDP is measured in .................... prices & The nominal GDP is measured in
.................... prices

i) Base year & Current year ii) Current year


iii) Base year iv) Current Year & Base year

11. A Machinery purchased for Rs. 18,000 / before two years. Its sold for Rs. 16,000/
considering 10% depreciation per annum. The machinery sold for

i) Rs. 2,000 Less ii) Rs. 1,600 Less


iii) Rs. 1,600 profit iv) No Loss and No Gain

Depreciated value : 18,000 - 18,000 x 0.2 = 14,400,


Profit = Sale value – Depreciated Value

12. The owner of the business used, goods of worth Rs. 200 for his personal use. In this
transaction which account will be credited

i) Drawing Account ii) Sales Account


(Credit what goes out)
iii) Purchase Account iv) Capital Account

13. A Capital reserve account is created when

i) A new fixed asset is purchased


ii) The asset becomes absolute due to invention of new technology
iii) A fixed asset is obtained from gift
iv) The asset is sold on profit

14. Outstanding Wages

i) Nominal Account ii) Real Account


iii) Personal Account iv) Profit Account

Illustration 7
Classify the following into personal, real and nominal accunts

(a) Capital (b) Building


(c) Carriage inwards (d) Cash
(e) Commissin received (f) Bank
458

(g) Purchases (h) Chandru


(i) Outstanding wages

Solution :

Sl.No. Items Classification

(a) Capital Personal account


(b) Building Real account
(c) Carriage inwards Nominal account
(d) Cash Real account
(e) Commission received Nominal account
(f) Bank Personal account
(g) Purchases Nominal account
(h) Chandru Personal account
(i) Outstanding wages Personal account

15. Average Fixed cost is obtained

i) TC/Q ii) TFC/Q


iii) TVC/Q iv) None

16. Public liability in a property

i) Reduce the value ii) Public helps in reducing the


toxic and contaminate
iii) Public takes the property iv) None of the above

17. As per SARFAESI Act 2002 within how many days borrower should repay the debit
from the date of notice

i) 30 days ii) 60 days


iii) 90 days iv) 180 days

18. Article 14 of the constitution of India protects the rights of the equality before the law to

i) A citizen of India only ii) All persons


iii) Children only iv) Adult citizens only
459

19. Depreciation is in which section of income tax act

i) Section 35 ii) Section 34


iii) Section 33 iv) Section 32

20. Who is the guardian of the girl child? Any Child

i) Father ii) Mother


iii) Uncle iv) Any one of the relatives

21. All agreements are contract as per Indian contract act 1872

i) Section 25 ii) Section 11


iii) Section 12 iv) Section 10

22. Harmonic mean is better than the other means ....................

i) Height and Length ii) Speed


iii) Binary values 0 – 1 iv) Ratios

data expressed as rates such as kilometers per hour, kilometres per litre, hour per
semester, tones per month

23. When three coins are tossed what is the probability of getting maximum no of heads

i) 1/24 ii) 1/16


iii) 1/12 iv) 1/8 (1/2*1/2*1/2)

24. A Time series Consists of

i) Two components ii) Three components

iii) Four components iv) Five components


(Secular, Seasonal,
Cyclical, Irregular)

25. In a Positive Skewed data:

i) Mean=Median=Mode ii) Mean>Median>Mode


iii) Mean<Median<Mode = iv) Mean ‘“ Median ‘“ Mode
Negative Skew.
460

26. For a old building valuation report to be prepared the building is defected by pollution.
Before preparing report consult the pollution consultant.

i) Not Required ii) Required


iii) Refer legal opinion iv) Option of the building owner

27. Which is not true? With respect to factory Act

i) 10 persons working in the preceding 12 months with power


ii) 20 persons working in the preceding 12 months without power
iii) Manager of a factory is a occupier
iv) Mining is a factory

28. Property not contaminated, located in area of contamination is ....................

i) Contaminated ii) Suspected


iii) Remediated iv) Adjacent properties

29. When was “Utilized of green revolution” applied in India

i) 1969 ii) 1961


iii) 1966 iv) 1991

30. Which of the following is correct?


(i) A valuer shall conduct the valuation independent of external influences.
(ii) A valuer shall maintain complete independence in his / its professional
relationships.
(iii) A valuer shall conduct the valuation according to the direction of his client.

i) (ii) and (iii) only ii) (i) and (iii) only


iii) (i) and (ii) only iv) None of the above

31. Which of the following is correct?


(i) A valuer shall provide all information and records as may be required by the
authority.
(ii) A valuer shall provide all information and records as may be required by the
Tribunal, Appellate Tribunal.
(iii) A valuer shall provide all information and records as may be required by the
registered valuers organisation with which he/it is registered.
(iv) A valuer shall provide all information and records as may be required by a
tender committee of a valuation tender.
461

i) (ii) and (iii) only ii) (i) (ii) and (iii) only
iii) (i) (ii) and (iv) only iv) (i) and (iv) only

32. Where illegal activity is suspected, the valuer is advised to


(i) disclose the activity if there is a legal obligation to do so
(ii) suggest alternative, legal ways in which the client’s needs might be met
(iii) report to the Police
(iv) refuse to accept the assignment

i) (ii) and (iii) only ii) (i) (ii) and (iii) only
iii) (i) (ii) and (iv) only iv) (i) and (ii) only

33. Which of the following is correct?


(i) A valuer shall maintain integrity by being straightforward in all professional
relationships.
(ii) A valuer shall maintain integrity by being forthright in all professional relationships.
(iii) A valuer shall maintain integrity by being honest in all professional relationships.

i) (i) and (ii) only ii) (ii) and (iii) only


iii) (i) & (iii) only iv) All

34. Under Transfer of property act Instruments means

i) Non-testamentary instrument ii) Testamentary instrument


iii) Both iv) None

35. Is the permissible for land lord to disconnect / cut off essential services being provided
to tenant

i) Yes ii) No
iii) Yes after order from court iv) Yes after notice to tenant

36. Where TDR is introduced first in India

i) Chennai ii) Delhi


iii) Calcutta iv) Mumbai

37. What is ‘Carpet Area’ as per RERA


462

i) Area of an apartment, including external walls, services shafts, exclusive


balcony / veranda / terrace area, but includes the area covered by the
internal partition walls of the apartment
ii) Carpeted floor area
iii) Net usable floor area of an apartment, excluding external walls,
services shafts, exclusive balcony / veranda / terrace area, but
includes the area covered by the internal partition walls of the
apartment
iv) Super Built up area

38. Land Acquisition authority

i) Tahsildar ii) VAO


iii) RDO iv) Collector

39. Lease with building – what is the name of lease

i) Occupational lease ii) Building lease


iii) Life time lease iv) Short time lease

40. If the owner’s right which is imperfect is made perfect on fulfillment of certain conditions
is called

i) Contingent ownership ii) Duplicate ownership


iii) Co ownership iv) Concurrent ownership

41. According to SARFAESI Act, before effecting sale of the immovable property, the
authorised officer shall obtain valuation of the property from

i) Registered valuer “34AB wealth tax act”


ii) Approved valuer
iii) Panel valuer
iv) Registered valuer “Companies act”

42. In case of operating lease which statement is true ?

i) Depreciation can’t be accounted by lessee


ii) All repairs carried out by lessee
iii) All taxes to be paid by lessee
iv) All buildings are constructed by lessee
463

43. How many toilets and urinals provided in the Cinema theatre?

i) Toilet 1 in 100 and Urinals 1 in 25


ii) Toilet 2 in 200 and Urinals 1 in 50
iii) Toilet 3 in 400 and Urinals 1 in 75
iv) Toilet 4 in 500 and Urinals 1 in 100

44. A value is also known as estimate of .................... and the future .................... that can
be obtained from the property.

i) Price & Income ii) Present worth & Benefits


iii) Value & Profit iv) None of the above

45. Petrol Pumps

i) Utility and Non Marketable ii) Utility and Marketable


iii) Marketable property iv) Income fetching and Marketable

46. Cost represents the .................... side of a Transaction.

i) Exchange ii) Supply


iii) Demand iv) Bargaining

47. Which is the safest and earliest liquidity


464

i) 2% of stock market ii) 10% of Return of Real estate


investments
iii) 7% of bank FD iv) 30% pf promotors loan

48. What is common in Distress sale & Forced sale?

i) Circumstances ii) Value


iii) Insufficient marketing time iv) Private negotiations

Distress sale – Non Legal, Forced Sale – Legal.

49. Which one correct with respect to condominium ownership

i) Beneficial ownership ii) Co ownership


iii) Time share ownership iv) Owning a portion of multiunit
building

50. Value of an object depends upon the future .................... that can be derived out of it.

i) Benefits ii) Demand


iii) Supply iv) Cost

51. In joint venture which factors decides the ratio of landlord and promotor for
redevelopment

i) Approved plan ii) Plot coverage


iii) Land cost & Building cost iv) FSI

52. Vertical interest is about

i) Legal concept of space above land surface


ii) Legal concept of space below land surface
iii) i & ii both
iv) None of the above

The vertical division of real property has its basis in the legal concept of land as a
volume of space above and below the land surface.

53. Supply and Demand controls .................... of property


465

i) Transferability ii) Scarcity


iii) Utility iv) Non-transferability

54. A disruption or loss of supply of labour or material will result in ....................?

i) Physical obsolescence ii) Economic obsolescence


iii) Functional obsolescence iv) Legal obsolescence

55. A fraction of a whole property is to valued. Such a building fraction is classified as

i) Investment property ii) Non marketable property


iii) Service property iv) None of the above

A building is to be valued for fire insurance purposes, the building is treated as a


fraction of a whole property. The building fraction is classified as Service Property

56. Dilapidation of building is

i) Physical obsolescence ii) Economic obsolescence


iii) Functional obsolescence iv) Legal obsolescence

57. The amount set aside for getting back the capital invested after the period for which
annuity will cease

i) Annual Income ii) Deferred Income


iii) Gross Income iv) Sinking fund

58. Project profitable if IRR .................... then discount rate.

i) Greater ii) Lesser


iii) Equals iv) None

IRR is a actual rate of return in the project. Discount rate is the expected rate of return
in the project.

59. Reversionary value means

i) The value on reversion of rent i.e. new rent agreement between landlord and
tenant
ii) An amount reverted after lease of the property
466

iii) A specified value estimated for payment of taxes such as capital gains
act or municipal act
iv) Present value of land which would revert to the lessor after expiry
of lease period.

60. Which of the following element is not key element of the income approach

i) Potential income stream ii) Outstanding loan


iii) Expenses iv) Capitalization rate

61. When it is expected that the property values will rise, which one is correct?

i) Increase the Capitalization rate


ii) Increase the remunerative rate
iii) down the remunerative rate
iv) down the capitalization rate

62. Years purchase .................... if number of years increases

i) Increases ii) Decreases


iii) Remains same iv) None

63. What is the Years Purchase for Rs 1/- with a remunerative interest 8% and Annual
Sinking Fund to be set aside for recouping Rs 1/- is 0.021is

i) 1/(0.08 + 0.021) ii) 0.021/0.08


iii) (0.08 + 0.021)/0.021 iv) None

64. Which is the following is not physical characteristics of land

i) Land is in destructible ii) Land has different soil strata


iii) Scarcity iv) Land is immovable

65. A multistoreyed building has several flats, almost all similar. Two recently sold
comparables were found. The valuer, using the market comparison approach, set up
the following table:
Flat under Valuation Comparable 1 Comparable 2
Date of Sale 1½ years ago 3 years ago
Rate per sq.ft. Rs. 18,500 Rs. 18,000
What would be the best estimate of the rate?
467

i) Rs. 19,000 per sq.ft.. ii) Rs. 17,500 per sq.ft.


iii) Rs. 17,000 per sq.ft.. iv) Rs. 18,000 per sq.ft.

66. Deterioration of environment is .................... proportional to market value

i) Inversely ii) Directly


iii) Not iv) None

67. Under adjustment grid model for sales instances premises having a positive weightage
is considered on sale rate while comparing with subject premises having standard
specification

i) Sub stand ii) Normal specification


iii) Superior iv) Deluxe

68. Market approach basically operates on the ....................

i) principle of increasing and decreasing returns


ii) principle of substitution
iii) principle of conformity
iv) principle of contribution

69. When the result of a combination of two or more assets or interests where the combined
value is more then the sum of the separate values is known as

i) Real investment value ii) Liquidation value


iii) Hope value iv) Synergistic value (As per IVS
Commonly termed as Marriage
Value/Merger Value)

70. In calculating cost index numbers for basic cost of construction of an RCC load bearing
building by CPWD which of the following items is not given any weightage

i) Bricks ii) Stone chips


iii) Skilled labours iv) Wooden furniture work in the
building

71. A building is 40 years old. It has a total life span of 80 years. Current replacement cost
of the building is INR 40,00,000. The salvage value of the materials of the building at
the end of its life is 10% of its CRC. What is the depreciation in % today?
468

i) 55% ii) 45%


iii) Rs 45,000 iv) Rs 18,00,000

72. The estimated of the age of a structure based on its utility and physical wear and tear
is called

i) Economical Age ii) Total Age


iii) Effective Age iv) Physical Age

73,. Percentage of deduction for residential rental income

i) 5% ii) 10%
iii) 20% iv) 30%

74. Owelty money is to be determined by a valuer for the purpose of

i) Settlement ii) Land Acquisition


iii) Market sale iv) Family Partition

75. The base year for Capital Gain Tax calculation is

i) 2001-2002 ii) 2000 -2001


iii) 1981-1982 iv) 1978-1979

76. While carrying out valuation of property for bank finance, which of the following is not
to be taken into consideration?

i) Age of the building ii) Rent fetching capacity of the


property
iii) Economic obsolescence iv) Amount of loan

77. For valuation of easement, the valuer is to estimate the value of the interest of

i) Dominant owner only ii) servient owner only


iii) Dominant and servient iv) Non of the above
owner

78. For the purpose of Insurance, valuers generally provide

i) Reinstatement Value ii) Historic Value


iii) Book Value iv) Present Value
469

79. A buyer who looked at seven very similar homes in a three year old subdivision made
an offer on the home with the lowest list price, the buyer was utilizing the principle of
....................

i) Substitution ii) Anticipation


iii) Contribution iv) Conformity

80. The difference between one honest valuation and another may range upto 15%

i) V.C. Ramchandran vs. CWT (1979) 126 ITR 157 Karnataka HC


ii) K.P. Varghese vs. ITO (1981) 131 ITR 597 (SC)
iii) Gold Coast Trust Ltd. vs. Humphray (1949) 17 ITR 19
iv) Subh Karan Choudhury vs. IAC (1979) 118 ITR 777 Kolkata HC (Special
Value/FMV)

81. Valuation is not an exact science. Mathematical certainly is not demanded, nor indeed
is it possible

i) V.C. Ramchandran vs. CWT (1979) 126 ITR 157 Karnataka HC


ii) K.P. Varghese vs. ITO (1981) 131 ITR 597 (SC)
iii) Gold Coast Trust Ltd. vs. Humphray (1949) 17 ITR 19
iv) Subh Karan Choudhury vs. IAC (1979) 118 ITR 777 Kolkata HC (Special
Value/FMV)

82. Escalation clauses added to fire policy allows automatic regular increase not exceeding
....................% in sum insured during policy period

i) 5% ii) 10%
iii) 25% iv) 50%

83. Which of the following perils is covered under fire at extra premium

i) Impact damage ii) Bush fire


iii) Subsidence iv) Forest fire

84. Minimum retention premium under fire declaration policy is derived at what % the
annual premium

i) 10% ii) 25%


iii) 50% iv) 75%
470

85. Contract of indemnity is

i) Bilateral ii) Tripartite


iii) Multi partite iv) Multilateral

86. Upon paying the amount of loss to the insured, the insurer steps into the place of the
insured, taking overall his rights. It is called exercise of ....................

i) Proximate Cause ii) Contribution


iii) Subrogation iv) Indemnity

87. Which of the following is not required to be mentioned in a leasehold valuation report

i) Postal address of the property


ii) Name of the title holder of the property
iii) Name of the Lessor
iv) Name of the heir of the Lessor

88. Valuation reports under The Companies (Registered Valuers and Valuation) Rules,
2017 shall be prepared following

i) Indian Valuation Standards


ii) International Valuation Standards
iii) Global Valuation Standards
iv) IBBI Standards

* * *

You might also like